Автор: Гончаренко С.У.  

Теги: физика   задачи по физике  

ISBN: 5-11-000114-6

Год: 1988

Текст
                    Задачник-практикум
Под редакцией канд. пед. наук С. У. Гончаренко
Допущено Министерством высшего и среднего специального образования УССР в качестве учебного пособия для слушателей подготовительных отделений вузов

КИЕВ ГОЛОВНОЕ ИЗДАТЕЛЬСТВО ИЗДАТЕЛЬСКОГО ОБЪЕДИНЕНИЯ «ВЫЩА ШКОЛАм 1988
Практикум
Воловик Павел Никитович
Гончаренко Семен Устинович Мавло Дмитрий Пантелеймонович Мойся Елена Григорьевна
ФИЗИКА
Задачник-практикум
Редактор Л. И. Попеначенко
Художественный редактор С. В. Анненков Технический редактор А. И. Омоховская Корректор Е. А. Каплан
ИБ № 11534
Сдано в набор 23.09.86. Подписано в печать 26.02.88. Формат 60X90/16. Бум. тип. № 2. Гарнитура литературная. Высокая печать. Печ. л. 22,5. Кр.-отт. 22,81. Уч.-изд. л. 25,37. Тираж 30 000 экз. Изд. № 7621. Зак. 7-253. Цена I р.
Головное издательство издательского объединения <Выща школа», 252054, Киев-54, ул. Гоголевская. 7
Книжная ф-ка им. М. В. Фрунзе, 310057, Харьков-57, ул. Донец-Захаржевского. 6/8
ББК 22.3
Ф50
УДК 53(076.1)
Рецензенты:
д-р физ.-мат. наук, проф. А. М. Федорченко, канд. физ.-мат. наук, доц. К. В. Корсак (Киевский государственный университет); канд. пед. наук, доц. В. В. Чижик (Винницкий политехнический институт)
Редакция литературы по математике и физике
Зав. редакцией Ю. Е. Кострица

Физика: Задачник-практикум / П. Н. Воловик, С. У. Гон-Ф50 чаренко, Д. П. Мавло, Е. Г. Мойся; Под ред. С. У. Гончаренко.— К- ' Выща шк. Головное изд-во, 1988.— 360 с.
ISBN 5—11—000114—6.
Практикум содержит свыше 1000 задач, предлагавшихся на вступительных экзаменах в ведущих вузах страны в последние годы. По каждому разделу программы вступительных экзаменов рассмотрены особенности решения задач, приведены подробные решения 5—15 типовых задач. При этом особое внимание уделено поиску способа решения задач и его обоснованию. К остальным задачам даны ответы.
Для слушателей подготовительных отделений вузов, а также учащихся старших классов общеобразовательных школ и средних специальных учебных заведений.
ISBN 5—11—000114—6.
© Издательское объединение
«Выща школа», 1988
ПРЕДИСЛОВИЕ
Знание физики предполагает умение применять физические законы в конкретных случаях при решении задач. Решение и анализ задачи позволяют понять и запомнить основные законы и формулы, создают представление об их характерных особенностях и границах применения. Задачи развивают навыки в использовании общих законов естествознания для решения конкретных вопросов, имеющих познавательное и практическое значение. Умение решать задачи является лучшим критерием оценки глубины изучения программного материала и его усвоения.
Однако именно ре пен ie задач вызывает на1б ольшие затруднения у школьников, изучающих физику. Из года в год на вступительных экзаменах в вузы констатируются слабые навыки и умения абитуриентов решать даже несложные задачи. Для решения задач, как правило, недостаточно формального знания физических законов. В некоторых случаях необходимо знание специальных методов и приемов, общих для решения определенных групп задач. В других случаях таких методов не существует. Тогда главным (кроме знания теории) становится способность аналитического мышления, т. е. умение рассуждать.
В средней школе, главным образом из-за острого дефицита учебного времени, отводимого на изучение теоретического материала, этим двум аспектам обучения решению задач на уроках не всегда уделяют должное внимание. На уроках физики в средней школе решается слишком мало задач для того, чтобы можно было выработать у учащихся прочные умения и навыки их решения. Поэтому, готовясь к поступлению в вуз, выпускники школы пытаются ликвидировать этот пробел в своей подготовке самостоятельно, на подготовительных отделениях вузов, на различных курсах.
В помощь слушателям подготовительных отделений, а также готовящимся самостоятельно к поступлению в вузы предлагается настоящее пособие. В нем приведены с решениями типовые конкурсные задачи, предлагавшиеся на вступительных экзаменах в вузы различного профиля. Особое внимание уделяется проблеме поиска решения и обоснованию выбранного способа решения.
Задачи подбирались таким образом, чтобы они соответствовали содержанию и структуре учебного материала тем программы вступительных экзаменов. Содержание задач, их структура и методы
з
решения, а также степень трудности различны. Это позволяет использовать их для групповых и индивидуальных занятий со слушателями, организации самостоятельной работы и составления контрольных работ. Достаточное внимание уделено графическим методам решения задач, а также векторному характеру ряда физических величин и проектированию векторов на координатные оси.
В отличие от аналогичных пособий, изданных за последнее время, в предлагаемом сборнике по каждому разделу курса физики рассматриваются подробно решения нескольких (от 5 до15) типовых задач, а затем предлагаются задачи для самостоятельного решения. Такой подход к построению сборника задач является более естественным для активного творческого изучения физики. Ведь путь к получению ответа на поставленный в задаче вопрос должен стать для учащегося индивидуальным и увлекательным научным поиском, и этот поиск нельзя заменить изучением готовых решений задач.
Задачник-практикум содержит задачи по всем разделам курса элементарной физики в соответствии с программой вступительных экзаменов в вузы. Для решения задач, предлагаемых этим изданием, требуются знания в объеме программы средней школы.
В конце книги помещены таблицы физических величин, необходимых для решения задач.
I. МЕХАНИКА
§ 1. Кинематика
Задача 1. Такси проехало по прямолинейному участку шоссе путь sx = 10 км, затем выехало на кольцевую дорогу, описало по ней четверть окружности радиусом R = 10 км, затем по перпендикулярной дороге проехало еще = 10 км. Определить весь пройденный такси путь s и численное значение перемещения г.
Решение. Изобразим траекторию движения такси (рис. 1).
Весь пройденный такси путь равен s = sx + у + sx = 2s1 + -у или, после подстановки данных, 8=37,5 км.
Численное значение перемещения показывает, на какое расстояние сместилось такси вследствие движения, и численно равно длине вектора А К, соединяющего начальную и конечную точки движения. Длину отрезка АК можно определить из прямоугольного треугольника АСК:
г=АК =
= l/(s1 + BC)2 + (s1+CD)2.
длине траектории, т. е.
Учитывая, что BC=CD=R, получим г = A/(=p<(s1 + P)2+(sx4~£!)2= = (sx + /?)/2 или, после подстановки данных, г = 28,2 км.
Задача 2. Автомобиль движется из пункта А в пункт В, расстояние между которыми s = 100 км. Первые sx = 40 км пути он ехал со скоростью их= 40 км/ч, а остальные s2 = 60 км со скоростью п2 = = 30 км/ч. Определить среднюю скорость автомобиля на всем пути. Построить графики зависимости скорости и пройденного автомобилем пути от времени.
Решение. По определению, средняя скорость равна отношению пройденного пути к полному времени движения тела, т. е. vc = у . В данной задаче vc =	%  Поскольку /х = ^- и t2 =
= ^ , то vc =	+	33,3 км/ч.
Os	c Sxt)2 + S2UX	’	1
Графики зависимостей и (/) и s (/) приведены на рис. 2.
б
Задача 3. От двух станций А и В, расстояние между которыми равно I, одновременно отправились в путь два поезда со скоростями щ и и2. Векторы скоростей образуют с отрезком АВ одинаковые углы а = 45° (рис. 3). Считая движение поездов равномерным и прямолинейным, определить минимальное расстояние между ними.
Решение. Задачу можно решить, воспользовавшись системой отсчета, связанной с одним из поездов, например первым В этой системе отсчета первый поезд будет неподвижным, а движение вто-рого поезда будет сложным: со скоростью v2 относительно Земли
и со скоростью”^ = —щ вместе с Землей относительно первого поезда (рис. 4). Скорость результирующего движения выразится вектором V, причем и = ш = j/"Ui + Минимальным расстоянием между поездами будет длина перпендикуляра АС, опущенного на направление вектора и. Из подобия прямоугольных треугольников получим
Задача 4. На рис. 5 {а и б) представлены графики, характеризующие движение пешехода. Опишите это движение, пользуясь обоими графиками. Постройте график vx(f).
6
Решение. Рис. 5, а представляет собой график изменения координаты х (/), а рис. 5, б — график пути s (/). Из первого графика следует, что пешеход вернулся в то место, откуда он начинал движение (координаты начальной и конечной точек совпадают). На втором графике по ординате точки С можно определить весь пройденный путь. Участки ОА и ВС соответствуют движению с постоянными и одинаковыми по модулю скоростями (рис. 5, б):
pl = tga.
На участке ОА вектор скорости совпадает с положительным направлением оси Ox (vx > 0). Уравнение движения имеет вид х = vt.
На участке ВС скорость противоположна направлению оси Ox (vx <0), и уравнение движения запишется следующим образом: х = х' — vt.
График проекции скорости vx (/) представлен на рис. 5, в.
Задача 5. Из летящего в горизонтальном направлении со скоростью и самолета выпустили в горизонтальном направлении вперед с интервалом времени т две ракеты со скоростью и. Пренебрегая сопротивлением воздуха, определить: а) уравнение траектории первой ракеты относительно Земли; б) уравнение траектории первой ракеты относительно самолета; в) как изменяется положение первой ракеты относительно второй (после запуска обеих ракет).
Решение. Выбор системы отсчета в данной задаче фактически задан ее условием. Направим ось Ох го
ризонтально, параллельно курсу самолета; ось Оу — вертикально вниз. Начало же системы координат каждый раз будем указы-
вать отдельно.
а)	Поместим начало системы координат в ту точку, в которой находится самолет в момент запуска первой ракеты (самолет и ракеты считаем материальными точками). Относительно этой неподвижней системы отсчета начальна^ горизонтальна^ скорость ракеты равна сумме скоростей ракеты относительно самолета и самого самолета относительно Земли, т. е. и + v. Тогда горизонтальная и вертикальная координаты ракеты в произвольный момент времени запишутся х = (и + v) t и У = \ gt2- Исключив из этих двух
уравнений время t, получим уравнение траектории ракеты относительно Земли
У 2 (и + у)2 •
7
Это уравнение параболы. Уравнение траектории второй ракеты относительно Земли будет таким же.
б)	В этом случае начало системы отсчета целесообразно связать с самолетом Относительно него начальная (горизонтальная) скорость ракеты равна и, поэтому горизонтальная координата ракеты будет изменяться по закону х — ut Вертикальная же координата опять будет изменяться по закону у = ~ gt2. Исключив из этих уравнений время t, получим уравнение траектории ракеты относительно движущегося самолета
Траекторией движения ракеты также будет парабола.
в)	Для описания движения первой ракеты относительно второй снова поместим начало координат в ту точку, где находился самолет в момент запуска первой ракеты. Запишем уравнения для координат хг и У1 первой ракеты и для соответствующих проекций vlx и vlu ее скорости:
*i =- (и + и) t\ уу = | gi2\ vlx = и + v; v[y = gt.
Время будем отсчитывать от момента запуска первой ракеты, но по смыслу задачи оно не может быть меньшим, чем интервал т между запусками ракет, т. е. t > т.
Запишем аналогичные уравнения для координат и проекций скоростей для второй ракеты:
х2 = ут + (« + у)(/ — т); у2 = у g[t — т)2; и2г= и 4- у; v2y= g(t—x}.
Пусть наблюдатель связан со второй ракетой, тогда относительно него координаты (или проекции скорости) первой ракеты равны разности соответствующих координат (или соответствующих проекций скоростей) первой и второй ракет в неподвижной системе отсчета:
х, от = Х1 — х2 = (и + у) t — vx — (и + и) (t — т) = их\ yim = yi—y2 = gt2 — ^g(t — x)2 = gtx — ^gx2;
У1 от. х = Уи — Узх = (« + у) — (и + у) = 0;
У1 от. у = viy — v2y = gt — g(t — x) = gx.
Таким образом, после запуска в горизонтальном направлении ракеты движутся с одинаковой скоростью и поэтому расстояние между ними х10т = их не изменяется с течением времени. По верти.-кали же первая ракета движется относительно второй с постоянной скоростью oioti/ = gx, а поэтому расстояние между ракетами изменяется со временем по закону «/ют = g/т — ggT2.
Задача 6. По прямолинейному участку шоссейной дороги движется со скоростью t>! = 4 м/с велосипедист. В определенный момент 3
времени он проезжает пост ГАИ. Спустя т — 3 мин этот пост проезжает мотоциклист со скоростью v2 — 19 м/с и сразу после поста начинает тормозить, так что далее движется равнозамедленно с ускорением, модуль которого а =0,15 м/с2. Через сколько времени после начала торможения и на каком расстоянии от поста ГАИ мотоциклист догонит велосипедиста?
Р е ш е н и е. За начало системы координат примем пункт ГАИ, а ось Ох направим в направлении движения велосипедиста и мотоциклиста. За начало отсчета времени примем момент, когда мимо поста ГАИ проезжал мотоциклист. Поскольку движение одномерное, то законы движения целесообразно записывать в проекциях на горизонтальную ось (ось Ох). Тогда для координат велосипедиста и мотоциклиста можно записать
= У1 (Т + t) И х2 = v2t — J at2.
В момент встречи координаты велосипедиста и мотоциклиста совпадают. Приравняв координаты х1=х2, получим квадратное уравнение относительно времени /:
at2— 2 (у2— цх) t + 2V]T = 0.
Решив уравнение, получим /1|2 = ~	±	1 —	или,
после подстановки числовых данных,	= 120 си t2 = 80 с.
Теперь необходимо выяснить, какое из двух значений корня удовлетворяет условию задачи. Для этого определим скорости мотоциклиста v'2 и v"2 в моменты времени и /2:
v’2 = v2 — at1 = 1 м/с и и2 = v2 — at2 = l м/с.
Скорость и2 меньше скорости велосипедиста, а и2 больше этой скорости. Очевидно, что условию задачи удовлетворяет только второе значение момента встречи t2 = 80 с — в этот момент мотоциклист, догоняя велосипедиста, должен иметь большую скорость, чем велосипедист. При этом расстояние от поста ГАИ до места встречи I = Vt (т + t2) = 1040 м.
Следует обратить внимание на то, что первое значение корня также имеет определенный физический смысл. Сначала скорость v2 превышала скорость vb в момент времени t2 = 80 с мотоциклист догоняет велосипедиста и затем обгоняет его. Со временем скорость мотоциклиста убывает, в определенный момент времени (t = 100 с) становится равной скорости велосипедиста, затем продолжает убывать и в момент времени /г= 120 с велосипедист догоняет мотоциклиста. Таким образом, корень /х тоже мог бы быть решением задачи, если бы в ней требовалось определить просто время встречи, а не время, за которое мотоциклист догонит велосипедиста.
Задача 7. Мяч бросили вертикально вверх со скоростью о0= = 50 м/с, а спустя т = 1 с из того же места, в том же направлении и с той же скоростью бросили второй мяч. Когда и где встретятся
9
мячи? Какова будет их скорость в момент встречи? Как со временем изменяется расстояние между мячами и скорость второго мяча относительно первого?
Решение. За начало системы координат примем точку бросания мячей, ось Ох направим вертикально вверх. Для наглядности законы движения тел можно изобразить графически (рис. 6). Решением задачи являются координаты точки пересечения графиков. В момент встречи /в координаты мячей равны, т. е.
Xi = х2, или v0/B — | gtl = v0 (tB — т) — 1 g(tB — т)3.
Решив это уравнение относительно tB, найдем
Рис. 6
tB =	| т ъ 5,6 с.
Подставляя это значение tB в уравнение для xt или х2, получаем
= h =	126,1 м.
В любой момент времени t скорости мячей
= Vo — gt и = v0 — g (/ — т).
В момент встречи мячей их скорости будут
fiB = vQ —	+|т) = —4,9 м/с и y2B = o0-g(-’-—1т) =
= 4,9 м/с.
Из того факта, что в момент встречи мячей |П1в| = |п2в| = gG = 4,9 м/с и ti = 0,5с,
вытекает, что мячи встречаются через полсекунды после момента достижения первым мячом максимальной высоты. Первый мяч падает, второй — поднимается вверх, причем оба мяча имеют одинаковые по модулю скорости, но направлены они навстречу друг другу.
В любой момент времени t расстояние между мячами
Дх = Xi— х2 = (—gx)t + иот+ |gT2.
Скорость второго мяча относительно первого
иот = v2 — Uj = gx = const.
Задача 8. Брошенный горизонтально на высоте ft = 2 м над Землей камень упал на расстоянии s = 8 м от точки бросания. Определить скорость падения камня на Землю.
Ю
Решение. Во время полета горизонтальная составляющая скорости иг не изменяется, а вертикальная возрастает от 0j[o]^2gh в момент удара о Землю. Конечная скорость камня
и = 1/”^ + Vb = 1/ Vr + 2gh.
В горизонтальном направлении камень пролетает расстояние /s2 — h2, и поэтому горизонтальная составляющая скорости
_	_	<(S2_A2M
Vr~ t V 2h 
Тогда конечная скорость
„ _	+	= 1/4<£±S « 13,6 м/с.
Задача 9. Ракету запустили под определенным углом а к поверхности Земли со скоростью va. На какой высоте скорость ракеты будет равна vr = ^и0? Сопротивлением воздуха пренебречь.
Р е ш е н и е. За начало системы координат примем точку запуска ракеты, одну ось координат направим вертикально вверх, вторую — горизонтально. За начало отсчета времени примем момент запуска ракеты. Запишем проекции скорости ракеты на координатные оси в произвольный момент времени
vx(t) = u0cos а и Vy(f) = uosina — gt.
Скорость ракеты и (t) в момент времени t будет
v (t) = У v2x (/) + Up (0 = У v0 — 2vogt sin a + g2t2.
Определим высоту h = y, на которой скорость ракеты и(1) станет равной | q,, т. е. и (/) =	и0, или ио — 2vogt sin a + gt2= ~ . Пере-
пишем это уравнение так: 4g2/2—8v0g sin at + 3i>o = 0. Решив его, получим
Zj = ^(2 sin a —1^4 sin2 a —3) и (2 sin a + ]/4 sin2a — 3).
Высоты, на которых находилась ракета в эти моменты времени ti и /2, определим, записав уравнение движения ракеты в проекции на ось Оуг
у= votsina — j-gt2.
Подставим в это уравнение значения и t2:
У (^i) =	sin a (2 sin a — V 4 sin2 a — 3) — g (8 sin2 a — 3 —
— 4sinaV4sin®a —3) и =
11
Моменты времени и t2 симметричны относительно момента вре-. О —	2v„ sin а
мени t3 =	% 2 = —, в который ракета достигла максималь-
ной высоты.
Задача имеет решение, если дискриминант квадратного уравнения
А = 4 sin2a — 3 > 0 или sin2a— = sin2a — sin260° >0.
4
Поскольку ракету запускают под острым углом к поверхности Земли, то окончательно получим: 60° с а < 90°.
Задачу можно решить проще, воспользовавшись законом сохранения энергии:
mv20 mv2	Зи2
Et = Е2, или — = nigh., откуда h = -g-.
Задача 10. На рис. 7, а приведен график ускорения тела. Построить соответствующие графики зависимости модуля скорости и
тело движется с разными ускорениями: = const Ha2=const. На графике модуля скорости (рис. 7, б) участок ОА представляет собой наклонную прямую (tgctj численно равен ускорению fli). Участок АВ параллелен оси времени t, т. е. скорость на этом участке постоянная, включая и сами точки А и В. Участок ВС аналогичен участку ОА, и здесь tg а2 численно равен ускорению а2. На графике пути (рис. 7, в) в точках А и В должно быть гладкое сопряжение прямого участка с участками парабол, так как скорость — величина однозначная и характеризуется тангенсом угла наклона касательной в каждой точке графика. Следовательно, в точках А и В должна быть одна единственная касательная tg а = — v = const для участка АВ.
12
Задача 11. На рис. 8,а приведен график пути некоторого движения. Дать качественное описание движения по этому графику. Возможно ли реально такое движение? Найти принципиальную ошибку, содержащуюся в этом графике. Какой вид должен иметь график скорости этого движения?
Решение. Поскольку график пути в интервале времени О с t fi изображается параболой, то движение здесь является равноускоренным. Вершина параболы находится в точке t = О, следовательно, начальная скорость равна нулю. Значит, скорость при этом равномерно изменялась от нуля до некоторого значения и е рафик изобразится отрезком прямой .соединяющей точки
О и 1 (рис. 8, б). Как видно из рис. 8, а, в интервале времени
< t < /2 скорость тела должна быть постоянной (путь прямо пропорционален времени). При этом ее значение у2 должно быть меньшим, чем Vj, так как наклон отрезка АВ меньше, чем наклон параболы в точке А. Значит, график скорости в этом интервале должен изобразиться горизонтальным отрезком, проходящим через точку 2 с ординатой v2 <v1. Из полученного графика скорости видно, что в момент времени tv скорость тела должна была бы мгновенно измениться от значения Ох до и2, но это невозможно, так как все тела обладают массой (инертностью), и поэтому изменение скорости на заданную величину может произойти только за конечный промежуток времени. К этому выводу можно прийти и непосредственно на основе анализа графика пути. В точке А можно провести две касательные к кривой, а поскольку тангенс угла наклона касательной определяет скорость тела, то это означает, что в этой точке произошло мгновенное изменение скорости тела, что невозможно. На графиках пути не может быть изломов. Переход от одного участка кривой к другому должен быть плавным.
Задача 12. Брошенный со скоростью v0 в горизонтальном направлении мячик влетает в щель, образованную двумя вертикально стоящими стенками. Расстояние между стенками х0. Мячик отражается поочередно от стенок. Определить расстояния между точками последовательных ударов мячика о стенки. Удар считать абсолютно упругим (значение скорости при ударе не изменяется).
Решение. Удары мячика о стенки абсолютно упругие, поэтому угол отражения мячика равен углу падения и при ударах изменяется только направление скорости. Нарисуем траекторию мячика в предположении, что противоположной стенки нет, и разделим эту траекторию вертикальными линиями, проведенными так, что расстояние между ними равно расстоянию между стенками х0 (рис. 9). Точки пересечения траектории мячика с этими линиями лежат на одном уровне с точками отражения мячика от стенок. Время полета мячика от одной стенки к другой определим из урав-нения х0 = vQxt, откуда t = —.
v0x
Уравнение движения мячика в проекции на ось Оу запишется y = ^gi2, где t = —, а поскольку х = их0 (где п= 1, 2, 3, ...), *	°0л
13
то t = — . Подставляя это значение t в выражение для коорди-у0х
наты у, получим
' 2
2^ ох
Расстояние между точками последовательных ударов о стенки
Рис. 9
&У = Уп— Уп-\ =
8Х« /п
—— (2п —
2цох
1).
Для вычисления расстояний между следами отражения мячика только на левой стенке в эту формулу следует подставлять значения
п — 0, 2, 4, 6, .... а для
определения расстояния между следами отражения на правой стенке вместо п надо подставлять значения п = 1, 3, 5, ...
Задача 13. Плоский обруч катится без проскальзывания по прямолинейной горизонтальной дороге с постоянной скоростью и0. Определить мгновенные скорости точек А, В, С, D и Е (рис. 10) относительно Земли.
Решение. Обруч катится без проскальзывания, а поэтому мгновенная скорость точки А равна нулю. Остальные точки принимают участие в двух движениях: равномерном движении
Рис. 11	по окружности со скоростью v„ = v0
(со скоростью движения точки О) и в поступательном движении тоже со скоростью и0 (рис. 11). Значения мгновенных скоростей равны геометрической сумме составляющих скоростей. Получим:
в точке Л: vA = ил — vg = 0;
14
в точке В: vB = пл 4- v0 = 2va;
в точке С: vc= У v2 + Vo — v0l^2;
в точке D: vD = ]/ил2 4- vl 4- 2цли0 cos a = 2a0 cos ;
в точке E: vE = У ил 4- v% — 2пли0 cos a = 2v0 sin ~ .
ЗАДАЧИ ДЛЯ САМОСТОЯТЕЛЬНОГО РЕШЕНИЯ
1.	Материальная точка движется равномерно и прямолинейно. На расстоянии 100 м от исходной точки она поворачивает и, пройдя в противоположном направлении 120 м, останавливается. Определить полное перемещение точки и пройденный ею путь.
2.	При выезде машины из гаража ее счетчик показывал 40 280 км. Машина проехала s1== 30 км по прямолинейному участку шоссе, затем сделала поворот, описав половину окружности по кольцевой дороге радиусом 7? = 20 км, и остановилась. Определить численное значение перемещения машины. Что показывал счетчик после поездки?
3.	Когда поезда движутся параллельными путями навстречу друг другу, то они проходят один мимо другого за время tlt а когда в одном направлении, то за время /2. Длины поездов соответственно равны Е и /2. Определить скорости движения поездов.
4.	Автомобиль, движущийся со скоростью 80 км/ч, обгоняет автомобиль, движущийся со скоростью и2=60км/ч. Какое расстояние прошел первый автомобиль за время обгона, если обгон начался на расстоянии s1= 50 м до второго автомобиля и закончился после того, как расстояние между автомобилями стало s2= 50 м? Размерами автомобилей пренебречь.
5.	Ежедневно в одно и то же время из морского порта А в порт В отправляется корабль и прибывает туда через 12 суток. Одновременно отправляется корабль из порта В и прибывает в А тоже через 12 суток. Сколько кораблей встретит в открытом море каждый корабль? Задачу решить графически.
6.	Автомобиль и велосипедист равномерно движутся навстречу друг другу со скоростями соответственно = 10 м/с и о2=5 м/с. Расстояние между ними в начальный момент времени было I *>= = 300 м. Графически и аналитически определить время и место их встречи. Изменится ли время и место встречи, если их скорость будет в два раза большей?
7.	Колонна военных грузовиков длиной I = 4,5 км движется по дороге со скоростью щ = 18 км/ч. Командир посылает из головы колонны в ее хвост мотоциклиста с приказом. Мотоциклист движется со скоростью v2 — 54 км/ч, не останавливается, вручает приказ и возвращается назад в голову колонны. Сколько времени длилась поездка мотоциклиста?
8.	Два велосипедиста выезжают одновременно навстречу друг другу из пунктов А и В, расстояние между которыми I = 54 км,
15
и через 2 ч встречаются. Не останавливаясь, они продолжают движение с той же скоростью, и второй прибывает в А на 54 мин раньше, чем первый в В. Определить скорости велосипедистов.
9.	Эскалатор метро опускает неподвижного пассажира за tr = = 50 с. Если же пассажир идет по движущемуся эскалатору в направлении движения, то он опускается за t2 = 30 с. За какое время ^’опустится пассажир по неподвижному эскалатору?
10.	Возле вертикальной стенки стоят два мальчика на расстоянии /j и /2 соответственно от нее и на расстоянии г друг от друга. Один мальчик громко произносит краткое слово. Сколько времени он должен произносить, чтобы второй мальчик услыхал совпадение его окончания с началом эха? Скорость звука в воздухе о.
11.	Два поезда движутся параллельными путями навстречу друг другу. Пассажир первого поезда видит, что второй поезд проходит мимо него в течение /1== 10 с. Пассажир второго поезда видит, что первый поезд проходит мимо него за t2 = 5 с. Определить отношение длины второго поезда к длине первого.
12.	По какому закону должно двигаться тело, чтобы для вычисления его средней скорости можно было пользоваться формулой
13.	Первую половину пути поезд двигался с постоянной скоростью, в 1,5 раза большей, чем вторую половину. Средняя скорость поезда на всем пути нс = 43,2 км/ч. Определить скорости поезда на каждой половине пути.
14.	Треть пути автомобиль двигался со скоростью v — 36 км/ч. Остальную часть пути I = 300 м автомобиль проехал за t = 60 с. Определить среднюю скорость движения автомобиля.
15.	Первую треть пути поезд проходит со скоростью Uj = 36 км/ч; вторую треть — со скоростью и2 = 27 км/ч и последний участок пути — со скоростью иа = 72 км/ч. Определить среднюю скорость движения поезда.
16.	Катер пересекает реку, ширина которой I = 380 м, а скорость течения о = 2,5 м/с. За какое минимальное время t катер пересечет реку, если скорость катера относительно воды и = 18 км/ч.
17.	Сколько времени понадобится катеру, чтобы дважды пересечь реку: один раз по кратчайшему пути, а второй раз — затратив для этого наименьшее время? Скорость течения реки и, ширина реки I, скорость катера относительно воды и, причем и > V.
18.	Каким курсом (по отношению к меридиану) должен лететь дирижабль, чтобы трасса его полета проходила с юга на север, если скорость дирижабля относительно воздуха = 100 км/ч, а скорость ветра, дующего с северо-востока на юго-запад, у2=8м/с. Определить также скорость дирижабля относительно Земли.
19.	Пилот должен пролететь прямо на восток от пункта А до пункта В и затем вернуться в пункт А, держа курс прямо на запад. Скорость самолета относительно воздуха постоянна и равна о, а скорость воздуха относительно Земли и. Расстояние между пунктами А и В равно I. Определить полное время полета при: а) безвет-
16
рии; б) западном (или восточном) ветре; в) северном (или южном) ветре.
20.	Рыбак на лодке переправляется через реку с параллельными берегами, ширина которой I. Скорость движения лодки в стоячей воде v. Скорость течения считается постоянной во всех точках реки и равной и, причем и > V. Под каким постоянным углом а к нормали к берегу должен рыбак направлять лодку, чтобы течение снесло ее как можно меньше?
21.	Вертолет летит по замкнутому контуру АВС, представляющему собой правильный треугольник. В каком случае время, затраченное на перелет, будет меньше: если ветер дует в направлении вектора АВ или если он дует в направлении вектора ВЛ?
22.	С плывущего вниз по реке корабля в момент прохождения мимо пристани А выбросили закрытую пробкой пустую бутылку. Через t = 1 ч корабль доплыл до пристани В, быстро развернулся и поплыл обратно с прежней относительно воды скоростью к пристани А. На каком расстоянии от пристани А корабль встретит бутылку, если скорость течения реки = 5 км/ч?
23.	Между двумя высокими перронами со скоростью движется платформа длиной I и такой ширины d, что на нее можно легко выпрыгнуть с обоих перронов. С точки А перрона на платформу прыгнул человек, пробежал со скоростью п2 вдоль диагонали платформы и в точке В выпрыгнул на второй перрон (рис. 12). Сколько времени бежал человек от Л кВ? Зависит ли это время от скорости движения платформы? На каком расстоянии от А лежит точка В', находящаяся напротив точки В?
24.	Один автомобиль движется с юга на север со скоростью у1== 80 км/ч, другой с запада на восток со скоростью и2= 60 км/ч. Определить скорость перемещения второго автомобиля относительно первого (ее модуль и направление).
25.	Из точки А движется прямолинейно и равномерно со скоростью Vi корабль. Наперерез курсу корабля из точки В отправляется катер с постоянной скоростью о2. Под каким углом к прямой АВ должен двигаться катер, чтобы встретиться с кораблем? Через
сколько времени произойдет встреча, если первоначальное расстояние между ними АВ = /? Линия АВ составляет угол а с перпендикуляром к курсу корабля.
Инв. №.
17

26.	Два автомобиля движутся к перекрестку по взаимно перпендикулярным дорогам со скоростями щ = 50 км/ч и v2 — 100 км/ч. До начала движения первая машина находилась на расстоянии Si =100 км от перекрестка, вторая — на расстоянии s2=50km. Через сколько времени после начала движения расстояние между машинами будет минимальным?
27.	Два мотоциклиста движутся с постоянными скоростями «1= 30 м/с и и2 = 20 м/с по взаимно перпендикулярным дорогам. В тот момент, когда расстояние между ними наименьшее, первый мотоциклист находился на расстоянии Si = 500 м от перекрестка. На каком расстоянии sa от перекрестка находился в этот момент второй мотоциклист?
28.	Два автомобиля движутся с одинаковой скоростью v — = 60 км/ч по прямым, пересекающимся в точке О, дорогам (рис. 13). Определить минимальное расстояние между автомобилями, если в начальный момент времени АО = 1 км, ВО = 2 км, угол между дорогами а = 60 °.
29.	Два корабля движутся относительно берегов со скоростями 1^=9 узлов и ц2= 12 узлов (1 узел = 0,514 м/с), направленными под углами а!=30° и а2=60° к меридиану. С какой скоростью второй корабль движется относительно первого?
30.	Два поезда прошли путь между станциями О и А за t = 2 ч. Один поезд, тронувшись с места, прошел весь путь равноускоренно, второй — половину пути шел со скоростью щ = 30 км/ч, а другую— у2=45 км/ч. Определить расстояние ОА между станциями, ускорение первого поезда и его конечную скорость. В какой момент времени скорости поездов были одинаковыми? Построить графики скоростей.
31.	Расстояние между станциями метрополитена / = 1,5 км. Первую половину его поезд проходит равноускоренно, а вторую— равнозамедленно; на станциях останавливается. Максимальная скорость на перегоне = 54 км/ч. Определить время движения поезда между станциями.
32.	Два тела проходят мимо друг друга, двигаясь по одной прямой в одну сторону. Первое тело движется с постоянной скоростью Ui, второе тело в момент встречи двигалось со скоростью о2 и ускорением а, причем ускорение направлено противоположно о2. Определить, в какой момент времени тела снова встретятся и где произойдет встреча. Как изменяется со временем расстояние между телами?
33.	От станции А отправляется с постоянной скоростью vt электропоезд. Одновременно от станции В, находящейся на расстоянии х0от А, отправляется другой поезд с начальной скоростью и2 и ускорением а, причем движется по той же прямой и в том же направлении, что и первый. Записать уравнения движения поездов и определить время их встречи, если У2. КйИОМу УСЛОВИЮ должно удовлетворять ускорение а, чтобы встреча могла состояться?
34.	Два автомобиля движутся в одном направлении с ускорением а = 0,4 м/с2. Второй автомобиль выехал на А/ = 20 с позже первого.
18	__
Через сколько времени от начала движения второго автомобиля расстояние между ними окажется As = 240 м?
35.	Расстояние между железнодорожными станциями s = 18 км поезд проходит за t — 20 мин. Первые /х= 5 мин он идет равноускоренно (без начальной скорости), азатем—равнозамедленно, пока не остановится. Определить ускорение поезда на пути разгона.
36.	Тело начинает двигаться без начальной скорости с ускорением а = 2,4 м/с2. Определить путь s,, пройденный телом за первые 5 с от начала движения, и путь As, пройденный за следующие 5 с. С какой средней скоростью двигалось тело эти 10 с?
37.	За t = 10 с тело прошло s = 18 м, при этом скорость его увеличилась в п = 5 раз. Считая движение равноускоренным, определить ускорение тела.
38.	Самолет для взлета должен приобрести скорость v = = 270 км/ч, пробежав взлетную полосу длиной I = 1 км. Сколько времени длится разбег? С каким ускорением? Какова средняя скорость самолета? Движение считать равноускоренным.
39.	Два велосипедиста выехали в одном направлении с интервалом т = 10 с. Первый в течение (, = 5с двигался с ускорением ат = = 2 м/с2, затем ехал с достигнутой скоростью, а после выезда второго велосипедиста — с ускорением а\ = — 0,25 м/с2. Второй велосипедист ехал с ускорением а2= 1 м/с2. Как только скорости велосипедистов сравнялись, оба продолжали ехать с постоянной скоростью. Построить графики скоростей велосипедистов и по графикам определить: 1) когда и на каком расстоянии от начала пути сравнялись скорости велосипедистов; 2) значение этой скорости; 3) путь, пройденный первым велосипедистом к моменту выезда второго.
40.	Движение материальной точки задано уравнением х = 4/ — — 0,05/2. Определить момент времени, в который скорость точки v = 0. Найти координату и ускорение в этот момент. Построить графики координаты, пути, скорости и ускорения этого движения.
41.	Прямолинейное движение тела вдоль оси Ох описывается зависимостью х = —6/5 + 30/, где х измеряется в метрах, /— в секундах. Найти зависимость скорости и ускорения тела от времени. Что произойдет с телом через т = 1 с от начала движения?
42.	Материальная точка перемещается прямолинейно вдоль оси Ох. По известному графику зависимости проекции vx скорости точки от времени (рис. 14) построить графики проекции ускорения ах и координаты х точки. Начальные условия: / = 0, х = 0.
43.	Частица движется вдоль оси Ох со скоростью 17 (рис. 15). Один под другим построить графики проекций ускорения ax(t) и перемещения гД/), а также пути $ (/), пройденного частицей. Найти среднее значение модуля скорости v за время движения от / = 0 до t = 2т.
44.	Частица движется в плоскости хОу с постоянным ускорением а — 10 м/с2, направление которого противоположно направлению оси Оу. Уравнение траектории частицы у ** kx — bx2, где
18
k=3 и b = 2 м-1. Определить скорость частицы в начале координат.
45.	На рис. 16 приведены графики скоростей двух тел, движущихся по одной прямой из одного и того же начального положения.
Известны моменты времени tx и t2. По истечении какого времени тела встретятся?
46.	На рис. 17 приведен график скорости точки, движущейся прямолинейно. На какое максимальное расстояние от начального положения отклоняется точка за время движения?
47.	Скорость некоторого тела представлена на графике (рис. 18). Построить графики пути и ускорения этого тела.
20
48.	Графики скоростей точки представлены на рис. 19. Построить графики путей и ускорений точки, если в начальный момент времени она находится в начале координат.
49.	График модуля скорости одного тела изображается дугой полуокружности АМВ (рис. 20). За время это тело прошло такой же путь, как и второе тело, движущееся с постоянной скоростью их=50 м/с. Определить начальную скорость о0 первого тела.
50.	Жонглер бросил вертикально вверх со скоростью и0 = = 19,6 м/с мяч. Какое расстояние пролетит мяч за t =4 с? Сопротивлением воздуха пренебречь.
51	. С высоты Нг= 10 м начинает падать камень. Одновременно с высоты Н2 = 5 м вертикально вверх бросают другой камень.
Рис. 19
Рис. 20
С какой начальной скоростью о0 брошен второй камень, если известно, что камни встретились на высоте h = 1 м над Землей.
52.	С крыши здания высотой Н = 16 м через одинаковые промежутки времени падают капли воды, причем первая ударяется о Землю в тот момент, когда пятая отделяется от крыши. Найти расстояние между соседними каплями в момент удара первой о Землю.
53.	С высоты h = 100 м над поверхностью Луны вертикально вниз брошено тело с начальной скоростью v0 = 2 м/с. Через сколько времени тело достигнет поверхности Луны? Ускорение свободного падения на Луне =1,6 м/с2.
54.	Тело на веревке поднимают с ускорением а — 2 м/с2 вертикально вверх. Через t = 5сверевка оборвалась. Сколько времени двигалось тело до Земли после того, как оборвалась веревка?
55.	Подвешенная на высоте Н над поверхностью Земли граната взрывается так, что осколки разлетаются симметрично во всех направлениях со скоростью и0. Через сколько времени на Землю упадет половина осколков; все осколки?
56.	Второе тело бросили от Земли вертикально вверх вслед за первым через т0 = 4 с с той же скоростью о0= 49 м/с, что и первое тело. Через какое время т после бросания второго тела и на какой высоте h оба тела столкнутся?
57.	Воздушный шар поднимается вертикально вверх с Земли с ускорением а = 2 м/с2. Через /х= 10 с от него отделился камень.
21
На какой высоте /г от поверхности Земли будет находиться камень через /2=4 с после отделения от шара?
58.	Тело, брошенное вертикально вверх, вернулось на Землю через t =3 с. Какова была начальная скорость тела? На какой максимальной высоте оно побывало?
59.	Дождевая капля в момент достижения поверхности Земли имеет скорость v = 15 м/с.Одна из капель падает в колодец глубиной h = 10 м. Через сколько времени человек, стоящий на Земле, услышал удар капли о поверхность воды, если скорость звука в воздухе щ = 340 м/с?
60.	Двигатели ракеты, запущенной вертикально вверх с поверхности Земли, работали t = 1 мин и сообщали ракете постоянное ускорение а = 3g. Какой максимальной высоты h достигла ракета?
61.	Воздушный шар поднимается вертикально вверх с постоянной скоростью 3 м/с. Спустя т = 5 с после старта шара вертикально вверх бросили камень с начальной скоростью о2 = 28 м/с. Сколько раз и на каких высотах камень встретится с шаром? При каком т камень и шар встретятся только один раз?
62.	Прямоугольная призма с углом а при вершине движется по горизонтальной плоскости с ускорением ах (рис. 21). С каким ускорением движется стержень, опирающийся о грань призмы, если он может перемещаться .только в вертикальном направлении?
63.	Прямоугольная призма с углом а при вершине движется по горизонтальной плоскости с ускорением ах. Определить ускорение движения стержня, опирающегося на наклонную плоскость призмы (рис. 22).
64.	С башни высотой h = 25 м горизонтально брошен камень со скоростью v0 = 15 м/с. Найти: а) сколько времени камень был в движении; б) на каком расстоянии от основания башни упал на Землю; в) с какой скоростью упал?
65.	На стержень длиной I = 0,9 м надета бусинка, которая может перемещаться по нему без трения. В начальный момент бусинка находилась на середине стержня. Стержень начал двигаться поступательно в горизонтальной плоскости с ускорением а = 0,6 м/с2 в направлении, составляющем угол а = 60° со стержнем. Через сколько времени бусинка упадет со стержня?
22
66.	Два тела брошены с одной и той же скоростью под углами а и 	а к горизонту. Определить отношение наибольших высот
подъема тел.
67.	Тело брошено горизонтально. Через t = 5 с угол а между направлением скорости и и ускорения а стал равным 45°. Опреде-
лить скорость и тела в этот момент.
68.	Нападающий футбольной команды посылает мяч с начальной скоростью 19,5 м/с под углом а = 45° к поверхности Земли. В тот же момент вратарь, находившийся в I = 55 м от места посылки мяча, бросается вперед, чтобы встретить мяч. С какой ско-
ростью он должен бежать для того, чтобы успеть схватить мяч, прежде чем он ударится о Землю?
Рис. 23
Рис. 24
69.	Баскетболист бросает мяч в кольцо. В момент броска мяч находится на высоте h = 2,05 м, а через t = 1 с падает в кольцо, расположенное на высоте Н = 3,05 м. С какого расстояния от кольца (по горизонтали) произведен бросок, если мяч был брошен под углом а = 60° к горизонту?
70.	Баскетболист бросает мяч в кольцо со скоростью v0= = 8 м/с под углом а = 60° к горизонту. С какой скоростью мяч попал в кольцо, если долетел до него за 1 с?
71.	Высота крытого манежа для игры в футбол зимой Н — Юм. На какое максимальное расстояние I можно послать мяч в этом зале, сообщив ему скорость v0 — 40 м/с? Какой угол а с полом зала должен в этом случае составлять вектор начальной скорости мяча? Мяч не должен ударяться о потолок зала.
72.	С самолета, летевшего на высоте h в горизонтальном направлении со скоростью и0, сбросили груз, который упал в точке А (рис. 23). Под каким углом пилот видел цель в момент выбрасывания груза?
73.	Пущенные в горизонтальном направлении на высоте h от поверхности Земли шарики должны пролететь сквозь окно в стене, находящейся на расстоянии х0 от места выброса шариков (рис. 24).
23
Какова должна быть скорость шариков, если нижний и верхний края окна находятся соответственно на высотах z/j и у2 от поверхности Земли?
74.	Две ступеньки, возвышающиеся над горизонтальной плоскостью, имеют одинаковую высоту h и находятся на расстоянии L одна от другой (рис. 25). На краю одной ступеньки лежит маленький шарик. Массивное тело налетает на шарик, соударяется с ним и сталкивает со ступеньки. Какую скорость должно иметь тело, что-
Рис. 26
Рис. 25
плоскости попал на вторую ступеньку? Все соударения абсолютно упругие.
75.	Мячик скатывается с верхушки лестницы, обладая горизонтальной начальной скоростью ц0 = 1,5 м/с. Высота и ширина каждой ступеньки одинаковы: h — d — 20,3 см. О какую по счету ступеньку мячик ударится впервые?
76.	Два камня брошены под различными углами к горизонту со скоростями vv и и2 (рис. 26, а). Не прибегая к численному расчету, объяснить, какой камень улетит дальше. Какой камень полетит дальше, если их бросить, как показано на рис. 26, б?
77.	Из брандспойта (шланг с металлическим наконечником), расположенного около поверхности Земли, вырывается струя воды
24
со скоростью t>0= 10 м/с. Брандспойт медленно вращают вокруг вертикальной оси, одновременно изменяя угол его наклона к горизонту. Определить максимальную площадь, которую можно полить брандспойтом.
78.	С вершины горы, угол наклона которой к горизонту а, выпустили в горизонтальном направлении снаряд со скоростью t»0. При каких значениях этой скорости снаряд попадает в люк блиндажа (рис. 27)?
79.	Самолет летит горизонтально со скоростью v = 1440 км/ч на высоте Н = 20 км. Когда он пролетает над зенитной установкой, в него выпускают снаряд. Какова должна быть минимальная начальная скорость о0 снаряда и угол а ее с горизонтом, чтобы он попал в самолет?
80.	Две стрелы выпущены одновременно из двух разных точек:
одна — в горизонтальном направлении с начальной скоростью va с башни высотой h, а вторая — от подножья башни (рис. 28). С ка
кой скоростью и и под каким углом а к горизонту надо выпустить
вторую стрелу, чтобы они столкнулись над поверхностью Земли?
81. Посланный футболистом под углом а к горизонту с начальной скоростью о0 мяч отражается от перекладины ворот. Где упадет мяч после отражения', если расстояние до ворот по горизонтали равно х0 и соударение мяча с перекладиной является абсолютно упругим? Какому условию должна удовлетворять начальная скорость мяча, чтобы он упал перед футболистом?
82. С аэростата, поднимающегося с ускорением а = 0,5 м/с2, через т = 4 с после его отрыва от Земли бросают камень со скоростью п0=5,5 м/с под углом а0= 30° к горизонту относительно аэростата (рис. 29). На каком расстоянии I от места старта аэростата упадет камень? Определить также время Tj нахождения камня в воздухе с момента его
сбрасывания.
83.	Два тела одновременно начинают двигаться из одной точки в поле силы тяжести с одинаковыми скоростями v0 = 2 м/с. Одна из скоростей направлена под углом а1= л/4, а другая — под углом а2= — л/4 к горизонту. Определить относительную скорость этих тел через t = 2 с после начала движения.
84.	Горизонтальный диск вращается вокруг своей оси, делая п = 5 об/мин. Человек идет вдоль радиуса диска с постоянной скоростью и = 1,5 м/с относительно диска. Как изменяется модуль скорости человека относительно Земли в зависимости от расстояния г от оси диска? Чему равен модуль этой скорости на расстоянии R = 3 м от оси диска?
25
85. На легкий шкив радиусом /? = 10 см намотана нить, к концу которой прикреплен груз. Груз начинает опускаться с ускорением а — 0,02 м/с2. Чему равна угловая скорость шкива в тот момент, когда груз опустится на h = 1 м?
86. Шарик, привязанный к нити длиной I — 30 см, равномерно вращают в вертикальной плоскости. Когда шарик проходит нижнее положение, нить обрывается и через t = 1 с шарик падает на Землю на расстоянии d — 9,3 м от оси вращения (по горизонтали). Какова частота вращения шарика на нити?
; 87. Пропеллер самолета радиусом R = 1,5 м вращается, делая п == 2000 об/мин, причем посадочная скорость самолета относительно Земли v = 161 км/ч. С какой скоростью движется точка,
находящаяся на конце пропеллера? Какую траекторию она опи-
сывает?
88.	Тело движется по окружности радиуса R = 10 см с постоянным тангенциальным ускорением ат = 0,05 м/с2. Через сколько времени от начала движения центростремительное ускорение превысит тангенциальное в п = 2 раза?
89.	С какой угловой скоростью нужно вращать барабан лебедки диаметром d = 20 см, чтобы равномерно поднять груз на высоту h = 60 м за t = 2 мин?
90.	Гладкий диск радиусом R, плоскость которого горизонтальна, вращается вокруг своей оси. От поверхности диска отрывается небольшое тело, которое без трения скользит по нему. На каком расстоянии от оси оторвалось тело, если за время, пока оно соскальзывало с диска, диск сделал полный оборот?
91.	Плоский обруч движется так, что в некоторый момент времени скорости концов диаметра АВ лежат в плоскости обруча и перпендикулярны к этому диаметру (рис. 30). Скорость точки А равна VA, а скорость точки В vB. Определить скорости концов диаметра CD, перпендикулярного к АВ, в этот же момент времени, считая, что эти скорости также лежат в плоскости обруча.
92.	По краю вращающейся с угловой скоростью со = 1 рад/с карусели радиусом R = 5 м шагает мальчик. Определить ускорение, с которым движется мальчик относительно Земли, если известно, что, поворачивая обратно и шагая по карусели с прежней скоростью, мальчик перестает перемещаться относительно Земли.
93.	Черный диск с одним узким белым сектором приводится во вращение. При освещении диска периодически вспыхивающей лампой видны три неподвижных белых сектора, расположенных через 120°. Найти скорость вращения диска по наблюдаемой картине при известной частоте вспышек освещающей лампы.
26
§ 2. Динамика
Задача 1. Брусок движется по горизонтальной плоскости под действием силы F, направленной под углом а > 0 к горизонту. Определить ускорение бруска, если его масса т, а коэффициент трения между бруском и плоскостью р.
Решение. Свяжем систему координат с плоскостью, по которой движется брусок (рис. 31). Нанесем все действующие на брусок —>	—>	—>
силы: тяжести mg, реакции опоры JV, трения FT и силу F. Запишем уравнение движения бруска в общем виде: та = mg + N + FT + + F и в проекциях на координатные оси: та — Feos а — FT;
mg — N — F sin а =0. Учитывая, что сила трения FT = pN и N — mg — Fsin а, первое уравнение запишем та = Feos а — — р mg + p mg sin а, откуда а = — (cos а + р sin а) — pg =
= (F (cos а + р sin а) — p/ngl.
Задача 2. Два бруска массами тг и тг связаны нерастяжимой нитью и находятся на горизонтальной поверхности. К брускам приложены силы Fj и F2, составляющие с горизонтом углы а = = const и const соответственно. Коэффициенты трения брусков о поверхность постоянны, одинаковы и равны р. Определить ускорение системы и натяжение нити. Система движется в направлении действия силы Fj.
Решение. Систему координат свяжем с поверхностью, по которой движутся бруски. Нанесем действующие на каждый брусок силы (рис. 32). Запишем для каждого бруска уравнения движения в векторном виде:
для первого бруска: т±а = mrg + Ni + T -f- FT1 -f- Fj
для второго бруска: maa — m2g + Na + T + Ft2 + F2.
27
Перепишем эти уравнения в проекциях на координатные оси; для первого тела:
mjtz = Fjcos а — Т — F-ri',	(1)
0=	Fxsin а — mig;	(2)
для второго тела:
m2a = Т — F.2cos (3 — F^;	(3)
0= /V2+ F2sin p — m2g.	(4)
Из уравнений (2) и (4) определим силы реакции опоры:
A^i= mxg — Fxsin а и N2 = m2g — /^sin р.
Поскольку силы трения /4= pjV, уравнения (1) и (3) принимают вид:
mta = Т7! cos а — Т — ц (mxg — Fx sin а);
тга = Т — F2 cos (3 — ц (тг£ — sin Р)-
Решив полученную систему, определим ускорение системы
Fi cos а — Aa cos р — ц (mxg + m2g — Fx sin а — F2 sin p)
a~	mx + m2
и силу натяжения нити
Т — Fx cos а — ц (mjg — Fx sin а) — Fx cos a — F2 cos p — ц (mxg + m2g — Fx sin а — F2 sin P) — tTl\	।	—	.
1	mx 4- m2
Задача 3. Тележку массой M = 20 кг, на которой лежит груз массой m = 10 кг, тянут с силой F, направленной горизонтально (рис. 33). Коэффициент трения между ।---------уг----1	грузом и тележкой р =0,1. Пренеб-
I	I—	7 регая трением между тележкой и опо-
гг, °	рой, найти ускорения тележки ах
''jof _	и груза а2, а также силу трения меж-
0	х	ду грузом и тележкой в двух случаях:
Рис. зз	1) F = 20 Н и 2) F = 60 Н.
Решение. Рассмотрим силы, действующие на оба тела. При этом, поскольку их ускорения направлены по горизонтали, будем учитывать лишь силы, действующие горизонтально, так как остальные — направленные по вертикали — заведомо уравновешиваются. ——> На тележку действуют сила F и сила со стороны груза FTl. Последняя направлена против скорости тележки относительно груза при трении скольжения или против силы F при трении покоя, т. е. в любом случае сила FT) направлена влево. На груз действует сила трения со стороны тележки FT2, направленная, согласно третьему закону Ньютона, вправо, причем по модулю FTi = FTs = FT-
28
Запишем уравнения движения тележки и грузов в проекции на горизонтальную ось:
F — Ft = Ма1г	(1)
Ft = та 2.	(2)
Эти два уравнения содержат три неизвестных. Чтобы получить еще одно уравнение, выясним характер силы трения между тележкой и грузом. Если тележка выскальзывает из-под груза, то между ними действует сила трения скольжения FT = цЛ/. Так как в данном случае сила N равна по модулю силе тяжести груза, то FT = [img. Если же тележка и груз движутся как одно целое, то между ними действует сила трения покоя ЕПок#= \irng. Однако в этом случае выполняется равенство «j = а2. Таким образом, в обоих возможных случаях получим систему трех уравнений.
Итак, необходимо выяснить характер сил трения, действующих между телами. Рассмотрим подробнее оба возможных варианта:
а)	тележка выскальзывает из-под груза. Между ними действует сила трения скольжения, которую найдем по формуле FT = umg = = 9,8 Н;
б)	тележка и груз движутся как одно целое. Тогда, обозначив ai==a2 = а, запишем систему уравнений (1), (2) в виде
F — FnoK = Ма и Гпок = та.
Решив эту систему, получим
Fao*~ FM^'
Формула (4) выражает пропорциональную зависимость между F и Епок- Однако значение РПОк имеет предел, равный силе Ет, которая уже найдена. Поэтому в действительности два тела будут двигаться как одно целое лишь при тех значениях F, при которых значение Епок. определяемое по (4), не превышает ее предельного значения. Проделав расчеты, получим:
1) если F = 20 Н, то Fn0K~ 6,6 Н;
2) если F = 60 Н, то Fn0K = 20 Н, что невозможно, ибо предельное значение Fn0K равно 9,8 Н. Значит, в этом случае между телами будет действовать трение скольжения.
Теперь легко ответить на все вопросы задачи:
1) F = 20 Н. Между телами действует сила трения покоя Fn0K = = 6,6 Н. Из формулы (3) находим а » 0,66 м/с2;
2) F = 60 Н. Между телами действует сила трения скольжения Ft =9,8 Н. Из (1) и (2) находим ускорения тел: а^ 2,5 м/с2 и а2 = = 0,98 м/с2.
Задача 4. Два одинаковых груза массой М — 4,5 кг каждый связаны нерастяжимой нитью, переброшенной через неподвижный невесомый блок. При этом в начальный момент времени грузы находятся на расстоянии h = 1 м друг от друга (по вертикали). Затем
29
на верхний груз кладут перегрузок массой т =0,1 кг. Определить силу давления перегрузка на груз и время, через которое оба груза поравняются. Массой нити, трением в оси блока и сопротивлением воздуха пренебречь.
Решение. Выберем начало координат и направление оси, как показано на рис. 34. На каждый из грузов действует сила тяжести Mg и сила натяжения нити Т. На перегрузок действует сила тяжести mg и сила реакции опоры R (нить на перегрузок не действует). Перегрузок, в свою очередь, действует на груз с силой N = —R.
Перепишем эти уравнения в проекции на координатную ось. При —>-	—>	__
этом учтем, что InJ = |а2| = а и |/?| = |М|. Получим: Т — Mg = = Ма; Т — N — Mg — — Мег, N — mg = — ma. Решив полученную систему уравнений, определим:
m
а — m + 2MS
(1)
2mM , ,, И N = отгл— g & 1 Н. 2М 4- т °
Из формулы (1) следует, что все тела данной системы движутся с постоянным ускорением. Поэтому координаты грузов в любой момент времени определим по известным уравнениям
Ух = | at2 и i/2 = h — j at2.
30
Когда грузы поравнялись, то уг = у2, или у at2 — h — at2, откуда
Задача 5. С каким ускорением движется система грузов (рис. 35) и какова сила натяжения нити, если m.i = 2 кг, т2 = 0,2 кг, а = 30°, р, = 0,2?
Решение. Согласно условию задачи, тела движутся ускоренно, однако направление ускорения неизвестно. Следовательно, прежде всего необходимо найти направление, а затем численное значение ускорения. На первое тело действуют: сила тяжести m^g, реакция опоры Nlt натяжение нити Т и сила трения Л, направление которой обусловлено направлением движения груза. Так как сила трения не может изменить направления движения тел, выбрав произвольно направление движения и ускорения, решают задачу без учета силы трения. Если в результате получают а > 0, то направление движения грузов выбрано верно, после чего задачу решают с учетом силы трения. Итак, составим динамическое уравнение движения для первого тела без учета силы трения: тха - mrg + N + Т.
Пусть ускорение а направлено вдоль оси Ох. Тогда в проекциях на ось Ох это уравнение запишется
гт^а = mig sin а — Т.	(1)
Второе тело движется ускоренно под действием силы тяжести tn2g и силы реакции нити Т. Запишем динамическое уравнение —>	—>-
движения для второго тела в векторной форме: пг2а = m2g + Т и в проекции на ось Ог:
tn2a=—m2g+T.	(2)
Решим систему уравнений (1), (2) относительно ускорения: mlg sin а — m2g 2 • 9,8 • 0,5 — 0,2 • 9,8	„
а = m, +т2	“	2^2	> U'
Следовательно, первый груз движется вниз по наклонной плоскости с ускорением. Сила трения А направлена, как показано на рис. 35. Перепишем динамические уравнения движения для каждого тела, учитывая, что на первое тело действует сила трения. Для первого тела
тла = mrg + N + Т + FT.
В проекциях на координатные оси Ох и Оу это уравнение запишется ^<2 = mjg sin а — Т — Ет,	(3)
0 — —mtg cos а + N.	(4)
31
Для второго тела
тга = т2ё + Т-
В проекции на ось Ог это уравнение запишется т2а — —m2g + Т.	(5)
В качестве дополнительного уравнения используем уравнение для силы трения
FT = 1lN.	(6)
Решаем совместно систему уравнений (3) — (6) относительно ускорения и силы реакции нити:
а = «igsin«-^2_g-wysa _ 2i2	2 и т = m ,	2(4 н
ffij -{-
Задача 6. Определить ускорения, с которыми движутся грузы tnt и т2 (рис. 36) и силу натяжения шнура. Трением и массами
блоков пренебречь. Шнур считать невесомым и нерастяжимым.
Решение. Пренебрежение трением и массами блоков приводит к постоянству силы натяжения
шнура. На груз mi действует сила тяжести rrtig и сила натяжения шнура Flt а на груз т2 — сила тяжести и силы натяжения двух шнуров 2FV Уравнения движения грузов запишутся
mig + Л = "Mi и m2g + 2FX = т2а2.
Предположим, что массы и т2 подобраны таким образом, что груз массой nil опускается. Перепишем эти уравнения в проекции на ось Оу.
tttig — Fi= miFi и /n2g — 2FX = — m2a2.
32
В силу нерастяжимости шнура при движении системы связанных грузов груз т2 проходит за определенное время расстояние, в два раза меньшее, чем груз mlt т. е. модули ускорений грузов связаны соотношением а2 = -^ а1. Тогда уравнения движения грузов будут m1a1 = m1g — F1 и 2FX — m2g = m2^, откуда	~ g
~	4772^ j 7/2^
и fl2 = 4т'~ф т S- Сила натяжения шнура	= тг (g — ах) =
3m, m, =----- 0.
4m, + m2 °
Задача 7. Как движется тело под действием силы, которая периодически изменяет свое направление на противоположное (рис. 37, а)? Считать, что начальные скорость и координата равны нулю.
Р е ш е н и е. На участке О — 1 сила постоянна по модулю и направлению; следовательно, движение тела равноускоренное: скорость изменяется пропорционально времени, а перемещение — квадрату времени. В таком случае график скорости (точнее — проекции vx на выбранное направление) представляет собой прямую, проходящую через начало координат (рис. 37, б), а график перемещения (точнее — координаты х) — параболу, вершина которой находится в начале координат (рис. 37, в).
На участке 1—2 сила тоже постоянная, но направлена в противоположную сторону. Скорость линейно падает со временем и в точке 2 обращается в нуль (время действия силы такое же, как и на первом участке; значит, такое же и изменение импульса тела). График координаты на этом участке— парабола, у которой вершина находится в точке 2. В точке 1 происходит плавный переход одной параболы в другую, так как разрыва скорости в этой точке нет.
Участки 2—3 и 3—4 рассматриваем аналогично. Как видно из полученных графиков, скорость нигде не меняет знак, лишь периодически обращается в нуль, а координата все время увеличивается. Другими словами, тело все время удаляется от первоначального положения, его движение не является периодическим.
Задача 8. Камень, привязанный к нити, другой конец которой мальчик держит в руке, движется в вертикальной плоскости по окружности радиусом R =50см со скоростью v =3м/с. Масса камня т =0,1 кг. Определить силу натяжения нити, когда камень находится в верхнем и нижнем положениях.
Р е ш е н и е. На камень действуют две силы — тяжести mg и реакции нити Q. Эти силы создают нормальное ускорение, которое характеризует изменение направления вектора скорости (рис. 38). Запишем уравнение движения камня в векторном виде для положений 1 и 2:
man = mg + Q1; man = mg + Qa.	(1)
2 7-253	33
Направим ось Ох вертикально вверх и перепишем уравнения системы (1) в проекциях на эту ось координат:
тап = mg и тап = Q2+ mg.
Эти уравнения содержат три неизвестных величины. Запишем до-полнительное уравнение ап = #-. Решив эти три уравнения как систему относительно силы реакции нити в нижнем и верхнем положениях, получим
«*2	/	п2\
<21 =	4- mg = т (g + *) « 2,8 Н
Сила натяжения нити равна силе реакции ее согласно третьему закону динамики: Fx =	= 2,8 Н — сила
натяжения нити в нижнем положении и /%= <?2= 0,8 Н — сила натяжения нити в верхнем положении. Из решения следует, что сила натяжения нити максимальная в нижнем положении.
Задача 9. Найти силу Рд, с которой автомобиль массой т = 104 кг, движущийся со скоростью v = 72 км/ч, давит на мост в одном из следующих случаев: а) горизонтальный мост; б) выпуклый мост; в) вогнутый мост. Радиус кривизны моста в последних
двух случаях R = 100 м. Для случаев б) и в) силу Рд определить для наивысшей и наинизшейточек моста. С какой наименьшей скоростью Уть, должен двигаться автомобиль для того, чтобы он не оказывал давления на выпуклый мост?
Решение. На автомобиль действуют силы: тяжести mg, нормальной реакции моста N и трения колес о поверхность моста
34
FT (рис. 39). Автомобиль движется под действием силы тяги F. Задача сводится к нахождению силы нормальной реакции N моста во всех случаях, поскольку по III закону Ньютона Рл — N. Рассмотрим каждый случай отдельно.
а)	Для движения автомобиля по горизонтальному мосту динамическое уравнение движения запишется
mg 4- N + FT + F = 0.	(1)
Систему координат свяжем с мостом и одну ось координат направим горизонтально в направлении движения автомобиля, другую — вертикально ввер£ рий9 ,)аТ огда уравнегеиЦ) в рюекциях на координатные оси запишется
F F — 0 т=. ;	(2)
N — mg = 0.	(3)
Из уравнения (3) N = mg = 9,8 • 104 Н. Таким образом, в этом случае сила давления автомобиля на середину моста численно равна силе его тяжести: Рл = mg =9,8 • 104 Н.
б)	Для случая движения автомобиля по выпуклому мосту (рис. 39, б) динамическое уравнение движения запишется
та = mg + N + FT + F.	(4)
Перепишем уравнение (4) в проекциях на оси Ох и Оу:
та = mg — А;	(5)
0 = F — FT.	(6)
Из уравнения (5) находим силу нормальной реакции моста
N = т (g — а).	(7)
Это уравнение является основным для нахождения силы реакции, однако оно содержит два неизвестных: N и а. Как дополнительное уравнение запишем формулу для центростремительного ускорения а = Подставив это значение а в уравнение (7), получим Г\
N = m(g —	= 5,8 • 104 Н.
\ RI
Таким образом, сила давления автомобиля на середину выпуклого моста меньше силы его тяжести.
Чтобы автомобиль не давил на мост, его скорость должна удовлетворять условию
N — 0, или m(g —	= 0, откуда и = Kg/? ~ 31,4 м/с = 113 км/ч.
Это и есть состояние невесомости, когда тело перестает давить на опору.
2*
35
в)	Как и во втором случае, записываем динамическое уравнение движения (4), вводим оси Ох и Оу, переписываем уравнение (5) в проекциях на эти оси для третьего случая (рис. 39, в):
та — N — mg-,	(8)
О = F - FT.	(9)
Из уравнения (8) находим силу нормальной реакции моста
W =т(а + g).	(Ю)
Это уравнение основное для определения силы нормальной реакции. В качестве дополнительного используем формулу а =	. Тог-
да W = m (g- + ^ = 13,8 • 10* Н.
Сила давления автомобиля на середину вогнутого моста больше силы тяжести автомобиля.
Задача 10. Велосипедист при повороте по кругу радиусом 7? наклоняется внутрь закругления так, что угол между плоскостью велосипедиста и Землей равен а. С какой скоростью должен ехать велосипедист, чтобы не упасть?
Решение. На данную систему тел (велосипедист и велоси-пед) действуют три силы:.тяжести mg, реакции опоры N , направленная перпендикулярно к плоскости опоры, и трения FT, направленная в каждый данный момент времени против возможного смещения колеса вбок. Свяжем систему отсчета с Землей и в данный момент ось Ох направим к центру круга, по которому едет велосипедист (рис. 40). Запишем динамическое уравнение движения системы
—>	—>- —*	—*
та - mg + W + Ег.
Перепишем это уравнение в проекциях на координатные оси с учетом того, что движение вдоль оси Оу отсутствует, а на ось Ох про-V2
ектируется центростремительное ускорение н=-п~:
у2
на ось Ох: т-^ = FT;
на ось Оу: О = N — mg.
Из условия равновесия велосипеда и велосипедиста вытекает, « >
что результирующая сила N + FT = Q направлена по оси велосипеда, так как в противном случае она создала бы вращающий момент относительно центра тяжести и велосипед опрокинулся бы, поэтому FT = N ctg а = mg ctg а. Следовательно, mg ctg а = , откуда
v = Kg7?ctga.
Из ответа следует, что для данного радиуса закругления и данного угла наклона скорость тела не зависит от его массы. Получен
36
ное решение справедливо для любого тела, движущегося по закруглению радиусом У?.
Задача 11. Стержень А00', изогнутый как показано на рис. 41, вращается с постоянной угловой скоростью со = 10 рад/с относительно оси 00'. На стержень надета бусинка, размеры которой очень малы. Определить, на каком максимальном расстоянии I от точки О бусинка может находиться в равновесии относительно стержня АО, если коэффициент трения покоя между бусинкой и стержнем п =0,1. Угол а =30°.
Решение. При вращении стержня на бусинку действуют три силы: тяжести mg, нормальной реакции опоры N, направлен
ная перпендикулярно к стержню, и трения покоя F?, направленная вдоль стержня к точке О (положению устойчивого равновесия). Под действием этих сил бусинка равномерно движется в горизонтальной плоскости по окружности (рис. 41) радиусом R = I sin а с центростремительным ускорением.
Динамическое уравнение движения бусинки
та = mg + N + FT.	(1)
Введем систему координат, направив оси Ох и Оу так, как показано на рис. 41. Перепишем уравнение (1) в проекциях на эти оси:
та = N cos а + FTsin а;	(2)
0 = mg — N sin а + FTcos а.	(3)
Запишем дополнительные уравнения: для центростремительного ускорения а = -% и силы трения покоя FT = Решая сов местно полученные уравнения, найдем
,	cos а 4- u sin а Л лс
I = g~r~----~ 0,46 м.
“со2 sin a (sin а — цсоза)
37
ЗАДАЧИ ДЛЯ САМОСТОЯТЕЛЬНОГО РЕШЕНИЯ
94.	На покоящееся тело массой т = 2 кг действует сила, направленная под углом а = 30° к горизонту. После начала движения тело за t = 5 с прошло s = 25 м Найти значение действующей силы, если коэффициент трения р = 0,02.
95.	Два бруска, массы которых т} — 0,3 кг и т2 = 0,4 кг, связаны невесомой нитью и лежат на горизонтальной поверхности. Первый груз тянут горизонтально направленной силой F =1,4 Н. Определить ускорение а, с которым движутся бруски, и натяжение нити Т. Трением пренебречь. Изменится ли натяжение нити, если ту же силу приложить ко второму телу?
96.	Тело массой т = 2 кг тянут по гладкой горизонтальной поверхности с помощью пружины, которая при движении растянулась на А/ = 1 см. Жесткость пружины k = = 400 Н/м. Определить ускорение движения тела.
97.	Два тела одинаковой массы т = 1 кг соединены невесомой пружиной, жесткость которой k = — 200 Н/м. Тела находятся на абсолютно гладкой горизонтальной поверхности. К одному из тел При-H. Определить удлинение
пружины при движении тел с постоянным одинаковым ускорением.
98.	На горизонтальной плоскости лежит доска массой mt^ = 9 кг, а на ней — кирпич массой т2 = 2 кг. Какую горизонтальную силу надо приложить к доске, чтобы кирпич начал сползать с нее, если доска движется по плоскости без трения, а коэффициент трения кирпича о доску р = 0,55?
99.	Груз массой т = 50 кг перемещается по горизонтальной плоскости под действием силы F = 500 Н, направленной под углом а = 30° к горизонту. Коэффициент трения груза о плоскость р = = 0,5. Определить ускорение движения груза.
100.	На горизонтальной плоскости лежат четыре кирпича массой т = 2 кг каждый. Сила F = 46 Н действует, как показано на рис. 42. Найти силу, действующую на каждый кирпич. Коэффициент трения кирпичей о плоскость р = 0,5. С какой силой F23 второй кирпич действует на третий?
101.	Локомотив развивает постоянную силу тяги F =3,5 X X 108Н. На горизонтальном участке пути I = 600 м скорость поезда возросла с щ = 10 м/с до и2=т20 м/с. Определить коэффициент трения, если масса поезда т = 10е кг.
102.	Автомобиль, все колеса которого ведущие, трогается с места. Зная, что коэффициент трения между покрышками колес
38
и дорогой [1 = 0,8, найти максимально возможное ускорение автомобиля.
103.	Три тела с массами т0, тг и т2 связаны между собой с помощью нитей и динамометров (рис. 43). К телу массой приложена сила Flt к телу массой т2 — сила F2. Определить показания динамометров, если F\ > F2. Массой динамометров пренебречь.
104.	От поезда массой М = 1000 т, двигавшегося равномерно, по горизонтальному пути, отцепилась часть вагонов массой т = = 100 т. По какому закону будет изменяться со временем расстояние s между частями состава (до остановки отцепившейся части состава)? Тяга локомотива остается постоянной, коэффициент трения (< = 0.005.
105.	График изменения силы F, действующей на тело в горизонтальном направлении, приведен на рис. 44. Ее максимальное
Рис. 45
’значение в два раза превышает силу трения скольжения. Построить график скорости тела, если ц0 = 0.
106.	Тележка массой М = 20 кг катится без трения по горизонтальной плоскости. На ней лежит брусок массой т = 2 кг. Коэффициент трения бруска о тележку р = 0,25. В одном случае к бруску приложена сила Е1== 2  10~3 Н; в другом F2 — 6 Н. Определить ускорение бруска и тележки в обоих случаях.
107.	Небольшой брусок лежит на краю горизонтальной доски длиной 1=2 м (рис. 45). Через какое время брусок соскользнет, если доска начнет двигаться по горизонтали с ускорением а = = 3 м/с2? Коэффициент трения между бруском и доской р = 0,2.
108.	Электровоз тянет состав из п одинаково нагруженных вагонов массой т каждый, сообщая ему ускорение а. Вагоны соединены между собой одинаковыми пружинными сцепками, причем жесткость каждой пружины k. Коэффициент трения между колесами вагонов и рельсами р. Определить силу тяги электровоза и изменение длины каждой пружины.
109.	К лежащему на горизонтальном столе бруску массой т = = 0,5 кг прикреплена одним концом вертикально расположенная пружина жесткостью k = 10 Н/м. Другой ее конец закреплен над бруском на высоте /0 = 0,1 м. В этом положении пружина не деформирована. При равномерном движении стола в горизонтальном направлении пружина отклоняется на угол а = 60 ° от вертикали. Найти коэффициент трения между бруском и столом.
ПО. Можно ли опустить с крыши предмет массой т = 5 кг с помощью веревки, прочность на разрыв которой Т =40 Н, так, чтобы веревка не оборвалась?
39
111.	Воздушный шар объемом V = 600 м3 находится в равновесии. Какое количество балласта надо выбросить за борт, чтобы шар начал подниматься с ускорением а = 0,1 м/с2? Плотность воздуха р =1,29 кг/м3.
112.	Две гири с массами тх— 7 кг и т2 = И кг висят на концах нити, перекинутой через неподвижный блок. Вначале гири находятся на одной высоте. Через какое время после начала движения гиря т, окажется на h = 10 см выше гири /н2? Массой блока и нити, а также сопротивлением воздуха пренебречь.
113.	На концах нити, перекинутой через неподвижный блок, висят на высоте Н = 2 м от пола два груза, массы которых тх = = 100 г и т2 = 200 г. В начальный момент грузы покоятся. Опре
Рис. 46
Рис. 48
делить натяжение нити при движении грузов и время, за которое груз т2 достигнет пола. Массу блока и нити не учитывать.
114.	Два груза одинаковой массы М = 400 г связаны невесомой нитью, перекинутой через блок массой т = 100 г, и находятся на одинаковом расстоянии от блока (рис. 46). На один из грузов кладут перегрузок массой tn. С какой скоростью v будет двигаться обод блока, когда расстояние между грузами будет h ~ 1 м, если на каждый из грузов массой М действует одинаковая средняя сила сопротивления Fc = 0,17 Н? Считать, что масса блока сосредоточена в его ободе. Силами сопротивления, действующими на блок, нить и перегрузок, пренебречь. Проскальзывание между блоком и нитью отсутствует.
115.	Через блок (рис. 47) перекинута нерастяжимая нить, на концах которой висят грузы с массами и т2, причем тх > т2. Блок начали поднимать вверх с ускорением аа относительно Земли. Полагая, что нить скользит по блоку без трения, найти силу натяжения Т нити и ускорение щ груза тх относительно Земли.
116.	Хоккейная шайба после удара клюшкой приобрела начальную скорость v = 5 м/с. Проскользнув по льду Sx— 10 м, она ударилась о борт, отскочила и, пройдя еще s2= 2,5 м, остановилась. Определить коэффициент трения шайбы о лед.
4Q
117.	К потолку лифта, масса которого т = 100 кг, подвешен груз. Сила F = 1500 Н заставляет лифт двигаться вверх. Груз находится на расстоянии I = 1 м от пола лифта. Нить внезапно обрывается. Сколько времени пройдет от момента разрыва нити до удара груза об пол?
118.	Тело массой т = 200 кг равномерно поднимают по наклонной плоскости, образующей угол а = 30° с горизонтом, прикладывая силу F — 1500 Н вдоль линии движения. С каким ускорением тело будет соскальзывать вдоль наклонной плоскости, если его отпустить?
119.	Какую горизонтальную силу F надо приложить к телу массой т = 10 кг, находящемуся на наклонной плоскости длиной I = 5 м и высотой h = 3 м, чтобы тело равномерно двигалось по ней? Трением пренебречь.
120.	За сколько времени t тело спустится с вершины наклонной плоскости высотой h = 2 м и углом при основании а = 45°, если предельный угол, при котором тело может покоиться на этой плоскости, 0 = 30°?
121.	По наклонной плоскости пускают снизу вверх тело, которое в течение = 2 с проходит расстояние I = 16 м, после чего соскальзывает вниз. Сколько времени длится соскальзывание тела вниз? Каков коэффициент трения между поверхностью и телом? Угол наклона плоскости к горизонту а = 30°.
122.	Брусок массой т находится на плоскости, угол наклона которой а может изменяться от 0 до 90°. Построить график зависимости силы трения бруска о плоскость от угла наклона плоскости к горизонту. Явлением застоя пренебречь. Коэффициент трения р.
123.	Груз массой т = 20 кг перемещается вверх по наклонной плоскости с углом наклона а = 30° и коэффициентом трения р = = 0,05. К грузу параллельно основанию приложена сила F = — 500 Н. Найти ускорение, с которым перемещается груз.
124.	На наклонную плоскость с углом наклона а к горизонту кладут плиту массой М с укрепленными на ней динамометрами (рис. 48). Затем на плиту кладут брусок массой т и дают возможность плите скользить вниз по наклонной плоскости. Что будут показывать динамометры, если плита скользит: а) с трением (коэффициент трения р); б) без трения.
125.	Автомобиль массой т = 2 т поднимается в гору с уклоном 0,20. На участке пути s = 32 м скорость автомобиля возросла от = 21,6 км/ч до и2= 36 км/ч. Считая движение автомобиля равноускоренным, определить силу тяги двигателя. Коэффициент сопротивления движению р=0,02.
126.	Найти силу тяги, развиваемую мотором автомобиля, движущегося в гору с ускорением а = 1 м/с2. Уклон горы равен 0,05. Масса автомобиля т = 900 кг, коэффициент трения колес о дорогу р =0,1.
127.	Два тела массами тг и т2 связаны невесомой и нерастяжимой нитью (рис. 49). Определить ускорения тел и силу натяжения
41
нити. Массой блоков и трением (в осях и тела массой тг по плоскости) пренебречь.
128.	На гладкой наклонной плоскости, образующей угол а = = 30° с горизонтом, лежит тело массой т}= 2 кг. Тело связано невесомой нерастяжимой нитью, переброшенной через неподвижный блок, с телом массой т2 — 3 кг (рис. 50). Определить ускорение
и силу натяжения нити при движении тел.
129. С каким ускорением будут скользить по наклонной плоскости два жестко скрепленных между собой тела с массами и /и2? Коэффициенты трения между телами и наклонной плоскостью равны соответственно pt и р2. Угол, образуемый плоскостью с горизон-
Рис. 49
том, а.
130.	Два тела массами т1= 4 кг и т2 — = 8 кг, связанные нитью, скользят по наклонной плоскости, образующей с горизонтом угол
Рис. 51
а = 30°. Коэффициенты трения между наклонной плоскостью и первым телом |л1= 0,1, а между наклонной плоскостью и вторым телом р2=0,2. Какова сила натяжения нити, если тело массой движется впереди?
131.	На гладком горизонтальном столе лежит брусок массой М = 300 г (рис. 51). На краю стола укреплен невесомый блок, способный вращаться без трения. Брусок связан нитью, перекинутой через блок, с телом массой т = 100 г.-Начальная скорость бруска направлена от блока и равна v0= 4,9 м/с. Определить через t = 4 с после начала движения бруска: а) скорость v бруска; б) расстояние s бруска от его начального положения.
132.	С какими ускорениями движутся тела (рис. 52)? Какова сила натяжения нити? Коэффициент трения между телом массой тг и горизонтальной плоскостью равен р. Массами блоков и трением в их осях пренебречь Нить считать нерастяжимой.
133.	Брусок массой т положили на пружинные весы, укрепленные на подставке Л в виде трехгранной призмы (рис. 53). Подставка может скользить по наклонной плоскости, образующей угол ас горизонтом. Какую силу показывают весы, если подставка скользит без трения? Каким должен быть коэффициент трения р между бруском и чашкой весов, чтобы брусок не соскользнул с нее?
42
134.	Клин с углом при основании а = 45° может скользить вдоль горизонтальной плоскости. На клине находится брусок (рис. 54). С каким ускорением а должен двигаться клин в горизонтальном направлении (в плоскости рисунка), чтобы брусок относительно клина находился в покое, если коэффициент трения между поверхностями клина и бруска р. =0,1?
135.	На наклонную плоскость с углом наклона к горизонту а кладут трехгранную призму массой М так, что верхняя грань призмы горизонтальная. На эту грань кладут брусок массой т. Коэффициент трения скольжения между соприкасающимися поверх
ностями |л. Определить ускорение движения бруска и призмы при любых значениях ц, т и М.
136.	На наклонную плоскость с углом наклона к горизонту ct положили прямоугольный клин массой М так, что одна грань клина горизонтальная, другая — вертикальная (рис. 55). Тела массами тг и т2 связаны невесомой нерастяжимой нитью, переброшенной через укрепленный на вершине клина невесомый блок так, что при натянутой нити тело массой т2 скользит по вертикальной грани клина. Определить ускорения движения тел. Трением пренебречь.
137.	Какой должна быть масса mlt чтобы брусок массой т не скользил по призме массой М (рис. 56)? Известно, что М = 50 кг, т = 5 кг, а. = 30° и коэффициент трения бруска о призму р = 0,2.
43
138.	С каким ускорением а должна двигаться тележка (рис. 57), чтобы положение грузов тг и т2 оставалось неизменным? Коэффициент трения ц = 0,3.
139.	Два тела с массами mj=2 кг и т2 = 1 кг подвешены с по-
мощью невесомых и нерастяжимых нитей и невесомого блока (рис. 58). Нить перерезают в месте, отмеченном косым крестом. Через
сколько времени после этого
скорость тела тг станет равной v = = 4,9 м/с.? Трением пренебречь. Считать, что блок движению тел не мешает.
140.	В определенный момент времени на ось верхнего блока на
Рис. 58
чинает действовать сила F, направленная вертикально вверх (рис. 59). Найти зависимость проекции v относительной скорости грузов от времени.
141.	Определить ускорения грузов массой тг и т2 в системе (рис. 60). Нить считать невесомой и нерастяжимой. Трение в осях блоков отсутствует.
142.	На покоящееся тело массой т= 5 кг начинает действовать сила F, значение которой убывает со временем по линейному закону до нуля (рис. 61). Какую скорость приобретает тело?
44
143.	Масса груза т1 = I кг, углы а = 30°, 0 = 64° (рис. 62). Какой должна быть масса т2, чтобы грузы двигались с ускорением а = 4,9 м/с2? Нить нерастяжимая. Трением пренебречь.
144.	Автомобиль массой т идет по выпуклому мосту, радиус кривизны которого R, со скоростью V. С какой силой F давит автомобиль на мост в точке, направление на которую из центра кривизны моста составляет с направлением на вершину моста угол а?
145.	Трамвай, масса которого т — 19,6 т, идет по выпуклому мосту со скоростью v = 32,4 км/ч. Радиус кривизны моста R =
= 30 м. С какой силой F давит трам- . вай на от его
Z///ZZ/7/////////Z.
мост на расстоянии s = 15,7 м средины?
П^2
4
146.	Шарик массой т = 0,2 кг, подвешенный на нити длиной I <= 20 см, равномерно вращается в вертикальной плоскости. Нить выдерживает максимальное натяжение F =12,2 Н. При какой частоте вращения нить оборвется?
147.	Самолет описывает «мертвую петлю» радиусом R = 200 м в вертикальной плоскости со скоростью v = 360 км/ч. С какой силой прижимается летчик к сиденью в наивысшей и наинизшей точках петли, если его масса т — 70 кг?
148.	Самолет делает «мертвую петлю», радиус которой R = 1 км, с постоянной скоростью. При этом максимальная сила давления пилота на кресло превышает минимальную в п = 3 раза. Определить скорость самолета.
149.	Тело массой т = 1 кг скользит по вертикальному круговому желобу (рис. 63). С какой силой давит тело на желоб в наинизшей точке В, если оно отпущено из точки А, находящейся на горизонтальной оси желоба, без начальной скорости?
45
150.	Тело массой tn = 1 кг, подвешенное на нити, отклонили до горизонтального положения нити и отпустили. Чему равно натяжение нити в момент, когда она образует с вертикалью угол а = = 30°?
151.	Грузик массой т = 30 г прикреплен к концу невесомого стержня, который равномерно вращается в вертикальной плоскости вокруг другого конца, делая п = 5 об/с. Длина стержня I = 30 см. Каково натяжение стержня, когда грузик проходит верхнюю и нижнюю точки своей траектории?
152.	Шар массой т = 1 кг подвешен на нерастяжимой нити. Нить отклонили от вертикального положения на угол а -- 60°. Определить натяжение нити в момент, когда шар проходит положение равновесия.	>у
Рис. 64
Рис. 65
153.	Груз массой т = 100 кг подвешен к горизонтальной балке на двух тросах одинаковой длины так, что угол между тросами а = 120°. Внезапно один трос обрывается. На какую предельную силу натяжения должен быть рассчитан другой трос, чтобы он не оборвался?
154.	На концах невесомого стержня длиной 2 I, который может свободно вращаться вокруг горизонтальной оси, проходящей через его середину, укреплены два груза с массами т1= 0,6 кг и т2 = = 2m,. В первоначальном положении стержень располагался горизонтально, а затем был отпущен без начальной скорости. С какой силой первый груз действует на стержень в тот момент, когда стержень оказывается в вертикальном положении?
155.	Невесомый жесткий стержень может свободно вращаться в вертикальной плоскости вокруг точки О (рис. 64). На стержне на расстояниях 1г и /2 от точки О закреплены небольшие по размерам грузы массой nii и т2 соответственно. Стержень отклонили от вертикали на угол а и отпустили. Каково будет натяжение стержня в точке А, находящейся между грузами, в момент прохождения стержнем положения равновесия?
156.	Невесомый стержень может свободно вращаться в вертикальной плоскости вокруг горизонтальной оси, проходящей через точку О (рис. 65). Массы и т2, закрепленные на концах стержня,
46
находятся, соответственно, на расстояниях и /2 от оси. В началь-
ном положении стержень отклонен на угол а от вертикального положения. После освобождения стержень совершает колебания. Определить линейную скорость массы т2 в момент прохождения положения равновесия.
157.	Маленький шарик подвешен на нерастяжимой невесомой нити. В начальный момент нить составляет угол <р0= 60° с вертикалью, а скорость шарика равна нулю. Определить, какой угол ф с верти-
калью составляет нить в тот момент, когда вертикальная проекция
вектора скорости шарика максимальна.
158.	На горизонтальном диске, который может вращаться во
круг вертикальной оси, лежит шайба массой т — 100 г, прикрепленная горизонтальной пружиной к оси. Если частота вращения
диска не превышает пг = 2 с-1, то пружина находится в нерастянутом состоянии. Если же частоту вращения диска медленно увеличить до п2=5 с-1, то длина пружины увеличится вдвое. Определить коэффициент жесткости k пружины.
159.	Человек стоит на краю круглой горизонтальной платформы радиусом R = = 4 м, которая вращается вокруг вертикальной оси, проходящей через центр платформы. При какой угловой скорости вра-
щения платформы человек не сможет удержаться на ней? Коэффициент трения ц = = 0,2.
160.	Горизонтальный диск радиусом R = 10 м вращается вокруг своей оси, делая п = 2 об/мин. Вдоль края диска навстречу вращению едет мотоциклист со скоростью v = 30 км/ч относительно диска. При каком коэффициенте трения между шинами мотоцикла и диском мотоцикл не будет соскальзывать с диска?
161.	На каком расстоянии от центра диска, делающего п = = 120 об/мин, нужно поместить тело, чтобы оно не соскальзывало с него? Коэффициент трения между диском и телом р. =0,2.
162.	На колесе радиусом R = 10 см имеется плоская часть длиной а = 1 см (рис. 66). При каком коэффициенте трения ц колесо будет скользить, а не катиться по гладкой горизонтальной поверхности, если его плавно тянуть за ось вращения?
163.	На движущейся тележке на нерастяжимой и невесомой нити длиной I = 80 см висит шарик. Тележка сначала двигалась с ускорением аг = 2 м/с2, а затем с ускорением аг~— 1 м/с2. Определить расстояние s между равновесными положениями шарика в обоих случаях.
164.	Шарик, привязанный нитью к подвесу, описывает в горизонтальной плоскости окружность с постоянной скоростью. Определить скорость и период вращения шарика по окружности, если длина нити I — 1 м, а угол, который она образует с вертикалью, составляет а = 45°.
47
165.	Грузик, подвешенный на нити длиной I = 1 м, вращается с постоянной угловой скоростью так, что ось вращения вертикальна и проходит через точку подвеса, а нить описывает конус с плоским углом раствора при вершине 2а = л/3 (конический маятник). Определить угловую скорость вращения грузика.
166.	Маленький шарик подвешен на длинной невесомой и нерастяжимой нити. Шарик вращается по окружности в горизонтальной плоскости. Период вращения 7\, а угол отклонения нити от вертикали аР Чему будет равен период вращения, если вся система начнет опускаться вниз с ускорением а < g? Угол отклонения нити при этом а2.
167.	На резиновой нити, длина которой в нерастянутом состоянии 10, подвешен шарик массой т, который равномерно вращается в горизонтальной плоскости вокруг вертикальной оси, проходящей через точку подвеса; при этом нить отклоняется на угол а. Деформация нити упругая, жесткость k. Найти угловую скорость вращения (О.
168.	В вагоне, движущемся по закруглению радиуса /? = 120 м с постоянной скоростью v = 25 м/с, на нерастяжимой нити висит груз. Определить, на какой угол отклоняется нить от вертикали.
169.	Вагон идет по закруглению радиуса R = 800 м со скоростью и = 72 км/ч. Расстояние между рельсами I = 1,68 м. Определить, на сколько должен быть выше внешний рельс по сравнению с внутренним, чтобы вагон не перевернулся.
170.	Какого наименьшего радиуса R поворот может сделать мотоциклист, движущийся со скоростью и = 21 м/с при коэффициенте трения скольжения резины о землю ц = 0,3? Под каким углом к горизонту мотоциклист должен наклониться?
171.	С какой максимальной скоростью может проехать мотоцикл по закруглению дороги радиуса R = 80 м, если коэффициент трения между шинами мотоцикла и асфальтом р = 0,5?
172.	Мотоциклист совершает вираж с линейной скоростью v = = 72 км/ч по окружности радиуса R =71 м. Определить угол отклонения мотоциклиста от вертикали.
173.	На горизонтальном столе лежит тонкий диск массой М = = 500 г и радиусом R = 15 см (рис. 67). В центре диска укреплен тонкий невесомый вертикальный стержень длиной I = 40 см. К верхнему концу стержня на невесомой нерастяжимой нити подвешен шарик массой т=300 г. Длина нити меньше длины стержня. Шарик описывает окружность в горизонтальной плоскости вокруг стержня. Какой максимальный угол а может при этом составить нить со стержнем, чтобы диск не отрывался от стола? Считать, что вследствие трения диск не может скользить по столу.
174.	Шарик массой т подвешен на нити, привязанной к бруску Л такой же массы т (рис. 68). Шарик толкнули так, что он начал описывать окружность радиуса R в горизонтальной плоскости. Каким должен быть коэффициент трения бруска о плоскость, если период обращения шарика, при котором брусок А начинает двигаться, равен 71?
48
175.	Два одинаковых шарика массой т каждый подвешены так, как показано на рис. 69, а затем приведены во вращение. 1) В какую сторону будет двигаться груз массой 2m? 2) Нарушится ли равновесие, если шарики неподвижны, а вращается груз массой 2m?
176.	Полусферическая чашка радиусом R вращается вокруг вертикальной оси с постоянной угловой скоростью со (рис. 70). В чашке находится шарик массой т, вращающийся вместе с нею. Найти угол <р, определяющий равновесное положение шарика в чашке.
49
177.	Проволока состоит из двух дуг (рис. 71), радиусы которых 7?i= 1 м и 7?2= 27?!. Дуги расположены в одной вертикальной плоскости так, что их центры Ot и О2 лежат на одной вертикальной прямой с точкой А сопряжения этих дуг. На проволоку надета бусинка В, которая может скользить по ней без трения. Бусинку помещают на небольшое по сравнению с 7?j расстояние от точки А и отпускают. Найти время I, которое потребуется бусинке, чтобы достигнуть наибольшей высоты по другую сторону точки А.
178.	Брусок может сползти с некоторой высоты по пути 7 или 2 (рис. 72). В каком случае скорость бруска в конце пути будет больше, если движение происходит: а) без трения; б) с трением?
§ 3. Закон всемирного тяготения
Задача 1. Вычислить первую космическую скорость искусственного спутника Земли, движущегося по круговой орбите на высоте Н =600 км.
Решение. Если пренебречь силами притяжения, действующими на спутник со стороны небесных тел, и не учитывать сопротивления среды, то можно считать, что на спутник при его равномерном движении по круговой орбите действует только сила земного тя-
уравнение движения в
готения F =	.Направлена
она по радиусу орбиты к центру Земли (рис. 73). Под действием этой силы изменяется направление вектора линейной скорости движения спутника, т. е. она вызывает центростремительное ускорение. Запишем динамическое уравнение —л-  >-
движения спутника та = F. Направим координатную ось Ох к центру вращения и перепишем
проекции на эту ось: та = F.
Учитывая, что а =
и'1
——т;, это уравнение запишем + п
так:
?(^+77?’ 0TKyAa v = ]/ 7«7’57 KM/C'
Задача 2. Спутник Земли движется по круговой орбите радиуса /?!• Его хотят перевести на эллиптическую орбиту с максимальным удалением от центра Земли /?2 и минимальным7?г На сколько для этого необходимо изменить скорость спутника? Каким будет период обращения?
Решение. Рассмотрим сначала спутник на круговой орбите. Законы сохранения не позволяют найти скорость спутника, так как 50
на круговой орбите все состояния спутника равноправны. Поэтому применим законы Ньютона и уравнения кинематики точки:
2
r	г тМ vo гг, 2л7?1
F — та, где г = у , a = -п- , Т — —± r R$ Ri v0
(и0— скорость спутника на круговой орбите). Тогда и0 и
ио

При движении спутника по эллиптической орбите, наоборот, за-
коны Ньютона оказываются неудобными для решения, так как сила тяготения в разных точках орбиты различна. Применим закон сохранения механической энергии и второй закон Кеплера:
2 mVl
тМ mv2 тМ п ~ 7^ = 1--------=
где — скорость спутника на эллиптической орбите в перигее. Отсюда
у _ 1/	_ v 1/о я*
1-	v (R1+R2)R1	°" Ri + Rz'
Искомое увеличение скорости равно Ди = — v0. Период обращения можно найти с помощью третьего закона Кеплера
£* = (/?! -f- Rt)»
2»r{	*
n f р
где	С—2 — большая полуось эллиптической орбиты.
Итак, окончательно:
Аи =	“О’ 7’2 = л(/?1 + 7?2)
г i\i \ Г Ki J-	' Zy/H
Задача 3. В медном шаре с радиусом 7? = 50 см сделана сферическая полость с радиусом ^-7?, поверхность которой касается поверхности шара. Найти, с какой силой F этот шар с полостью притягивает маленький шарик с массой т = 20 г, находящийся на расстоянии d = 90 см от центра медного шара по прямой, соединяющей центры шаров.
Решение. Силу взаимного притяжения между шаром с полостью и шариком, находящимся на расстоянии d> R, можно определить как разность двух сил притяжения: между полным шаром и шариком и медным шаром в объеме полости и шариком.
а)	Рассмотрим случай, когда шарик массой т находится со стороны полости (рис. 74). Если бы шар с массой М был сплошным,
«г. тМ он притягивал бы шарик, масса которого т, с силой F = у
Для шарэ'-о полостью сила притяжения будет меньше на величину
£1
Ft, которая представляет собой силу, с которой действовал бы медный шар того же объема, что и полость. Так как радиус полости равен то масса такого шара была бы равна Mt = р4л(4)3=^.
Поскольку расстояние между центром полости и шариком равно d — l^R, то медный шар в объеме полости действовал бы на шарик, масса которого т, с силой
Таким образом, в этом случае сила притяжения Ft между шаром с полостью и шариком
тМ
~7da — 8d7? + 2/?2'
/ 1 \а
8da(d — /?1
4 лрК3ут о
л? 68,6 • 10-10 Н.
б)	Если шарик находится с противоположной к полости стороны, то сила притяжения между шаром в объеме полости и ша-
4 npR3ym
риком будет Fs = у—----г2
8U+4* '
\ “ /
а сила притяжения между шаром с полостью и шариком будет
е4 = е-е3 =
.. r7d2 + 8d/?+ 27?2 == утМ -------!--
2
•7d2 + 8d/? + 2/?2' _ 8da (d + ± /г)" _
Задача 4. Космическая станция запущена на Луну. На каком расстоянии от центра Земли станция будет притягиваться Землей и Луной с одинаковой силой? Считать, что масса М Земли больше массы т Луны в 81 раз, а расстояние между их центрами равно 60 земным радиусам.
Р е ш е н и е. По условию в какой-то точке прямой, соединяющей центры Луны и Земли, станция притягивается Землей и Луной с одинаковой силой, т. е. F„ = F3.Обозначим расстояние от центра Земли к этой точке через х. Тогда расстояние этой же точки от центра Луны равно 607? — х. Учитывая данное в условии соотношение масс
52
Земли и Луны (М = 81/п) и обозначив массу станции через запишем условие равенства сил:
т, • 81/п	т,т	с,,,™ г.	«
V— = ? (60)? -X)2. или 81 (607? — х)2 = х2.
Решив это уравнение, получим х1 = 547? и х2 = 67,57?. Это значит, что на прямой, проходящей через центры Земли и Луны, есть две точки, в которых на станцию со стороны Земли и Луны дей
ствуют одинаковые силы. Корень хх означает, что одна точка лежит между Землей и Луной на расстоянии 547? от центра Земли (или
67? от центра Луны). Корень х2 означает, что Земля и Луна дей
ствуют с одинаковой силой на станцию в точке .лежащей за Луной на
расстоянии 7,57? от ее центра. Но если в первой точке силы притяжения направлены в противоположные стороны и уравновешивают друг друга, то
во второй точке эти силы направлены в одну сторону и не уравновешивают друг друга.
Задача 5. В бесконечной однородной среде плотностью ра находится шарик А объемом Va из вещества с плотностью рл и пузырек В воздуха объемом Vв. Расстояние между их центрами 7? значительно больше радиусов шарика и пузырька. Будут ли они притягиваться друг к другу или отталкиваться? Рассмотреть
случаи: рл > р0; рв > р0 и рд <р0.
Решение. Предположим сначала, что в бесконечной однородной среде плотностью р0 находится только шарик А массой М и плотностью рл > Ро- Тогда на этот шарик со всех сторон действуют
одинаковые силы притяжения со стороны элементарных масс т плотностью р0, окружающих шарик. Эти силы уравновешивают друг друга и шарик остается в покое (рис. 75, а). Если с правой стороны шарика А поместить пустой пузырек В объемом Vb (рис. 75, б), то действующая на шарик А с правой стороны (т. е. со стороны, с которой находится пузырек В) сила будет меньше ранее действовавшей силы Еп на значение силы притяжения к шарику А массы вещества объемом Vb и плотностью р0. Тогда на шарик А о правой стороны действует сила Е'п, равная
f п = Р п — у
Рл^л^вРо R2
где paVa = Ма— масса шарика А, а р0 Уд — масса вещества, удаленного из пузырька В. Равнодействующая сил, действующих
53
на пузырек В, равна Fp = F„— F'n. Учитывая, что Fn — F„, получим
Р РоРл
РР = У---•
Шарик А будет отталкиваться от пустого пузырька В влево, или, иначе говоря, пузырек В будет отталкиваться от шарика А вправо.
Если шарик В заполнить веществом плотностью рв (рис. 75, в), то сила взаимодействия между шариком и пузырьком F” превысит Fn на значение силы тяготения между массами pAVA — МА и рв1/в. Тогда сила взаимодействия между шариком и пузырьком
j-t и	тз . .	г-. ^'4 V 8
Fn = F„ + УР^Рд gz “
р РдРо^л^в , РлРвКд^в
Fn—V---------Ь V-------
На шарик А действует результирующая сила
РоРл^л^в РаРеУа^в Ра^а , ,,	,
---да-----= У —rT (РоРв — рвУв).
Fp = F л Fn у	у
Из этой формулы следует, что шарик и пузырек отталкиваются, когда рй<ро, и притягиваются, когда рв>р0.
Задача 6. Искусственный спутник, используемый в системе телесвязи, запущен в плоскости земного экватора так, что все время находится в зените одной и той же точки земного шара. Во сколько раз радиус орбиты спутника больше радиуса Земли R3 = 6370 км?
Решение. Спутник движется вокруг Земли под действием силы тяготения, поэтому можно записать
тМ 9П 4л2 n	М 4л2 п
у-^2- =	= ОТ yj /?, ИЛИ y -^2=j^ к.
По условию задачи, спутник постоянно «висит» над одной и той же точкой Земли (стационарный спутник), поэтому период его обращения равен периоду обращения Земли вокруг своей оси, т. е. одним суткам. Левую часть записанного равенства можно записать так:
М/ЯзУ
Л2
4л2	^3
Таким образом, R = gl-^j . Умножим обе части на , тогда
/ R\3 аТг	R gT* ~ д
Щ =	"л" «, = V “6’6-
Задачу можно также решить, воспользовавшись третьим законом
54
ЗАДАЧИ ДЛЯ САМОСТОЯТЕЛЬНОГО РЕШЕНИЯ
179.	Определить минимальный период обращения спутника нейтронной звезды, плотность вещества которой р — 1017 кг/м3.
180.	На какую высоту от поверхности Земли поднялся космический корабль, если приборы отметили уменьшение ускорения свободного падения до g1=4,9 м/с2?
181.	Радиус Марса в п = 1,9 раза меньше радиуса Земли, а масса меньше массы Земли в k = 9 раз. Определить ускорение свободного падения на его поверхности.
182.	На какой высоте Н от поверхности Земли ускорение свободного падения уменьшается в k = 4 раза по сравнению с его значением на поверхности Земли?
183.	Радиус астероида 7? = 10 км, плотность его вещества р = 5,5 • 103 кг/м3. Определить ускорение свободного падения на поверхности астероида.
184.	На какой глубине h. от поверхности Земли ускорение свободного падения gi= 4 м/с2? Учесть, что действие силы тяготения со стороны сферического слоя на находящуюся внутри слоя материальную частицу равно нулю. Считать Землю однородным шаром.
185.	Определить ускорение свободного падения g на полюсе планеты, если на ее экваторе вес тела на т] = 20 % меньше, чем на полюсе. Длительность суток на планете t = 12 ч, ее радиус 7? = = 104 км.
186.	Искусственный спутник Земли запущен с экватора и вращается по круговой орбите в направлении ее вращения. Найти отношение радиуса орбиты спутника к радиусу Земли, при котором спутник периодически проходит над дочкой запуска точно через двое суток.
187.	Определить радиус планеты 7?, у которой на экваторе вес тела на t] = 20 % меньше, чем на полюсе. Масса планеты М = = 6 • 1024 кг, сутки на ней составляют Т = 24 ч.
188.	Шароподобная планета со средней плотностью вещества р = 1,2 • 103 кг/м3 делает п = 3,5 оборота вокруг оси за сутки. Во сколько раз неподвижное относительно планеты тело весит на экваторе меньше, чем на полюсе?
189.	При какой продолжительности суток тела на экваторе Земли весили бы в два раза меньше, чем на полюсе?
190.	Период обращения Луны вокруг Земли Т =27 суток. Считая орбиту Луны окружностью, определить ее радиус г.
191.	Период обращения искусственного спутника Земли Т = = 2 ч. Считая орбиту спутника круговой, найти, на какой высоте над поверхностью Земли он движется.
192.	Вычислить первую космическую скорость при старте ракеты с поверхности Юпитера, если известно, что третий спутник Юпитера — Ганимед — вращается практически по круговой орбите радиуса г = 1,07 • 10е км с периодом обращения Т =7,15 суток. Радиус Юпитера по = 7 • 104 км.
55
193.	В одной из книгЖюль Верна описывается, что в некоторый момент времени обитатели снаряда, пущенного на Луну, перестали ощущать наличие силы тяжести. Когда это могло бы произойти?
194.	Радиус Луны приблизительно в 3,8 раза меньше радиуса Земли, а масса в 81 раз меньше массы Земли. Во сколько раз нужно изменить начальную скорость бросания, чтобы подбросить тело на Луне на такую же высоту, как и на Земле?
195.	Искусственный спутник Земли запущен с экватора и вращается по круговой орбите в плоскости экватора в направлении осевого вращения Земли. Радиус орбиты спутника в п = 3 раза больше радиуса Земли. Через какое время спутник в первый раз пройдет над точкой запуска?
196.	Определить массу М Солнца, зная среднюю линейную скорость Земли на орбите и = 30 км/с и радиус орбиты Земли R = = 1,5 • 108 км.
197.	Радиус планеты, обращающейся вокруг звезды Сириус, в семь раз превышает радиус Земли, а средняя плотность в 1000 раз больше средней плотности Земли. Определить ускорение свободного падения на поверхности этой планеты.
198.	Период обращения Юпитера Т-ю вокруг Солнца в п — 12 раз превышает период обращения Земли. Определить радиус орбиты Юпитера Л?ю, зная радиус земной орбиты А?з= 150 • 10е км. Считать орбиты круговыми.
199.	Определить среднюю плотность планеты р, продолжительность суток на которой t = 6 ч, если на ее экваторе динамометр показывает вес на 10 % меньший, чем на полюсе.
200.	Средние расстояния Земли и Венеры от Солнца равны соответственно Из = 150  10е км и A*b = 108 • 10е км. Определить отношение линейных скоростей планет при их обращении вокруг Солнца. Орбиты считать круговыми.
201.	Во сколько раз период обращения искусственного спутника по круговой орбите на высоте h от поверхности Земли превышает период его обращения на околоземной орбите (И = 0)?
202.	Определить силу тяжести, действующую на тело массой m = 12 кг, поднятое над Землей на расстояние, равное трети земного радиуса.
203.	На какой высоте над поверхностью Земли сила тяжести тела будет в два раза меньше, чем на ее поверхности?
204.	Земля движется по орбите вокруг Солнца с линейной скоростью v — 30 км/с. На сколько должна возрасти эта скорость, чтобы Земля покинула Солнечную систему? Влиянием других планет Солнечной системы пренебречь.
205.	Спутник, масса которого m — 2 • 10® кг, из-за торможения о верхние слои атмосферы переходит с орбиты, расположенной на высоте /ц =210 км над поверхностью Земли, ИЗ Орбиту С ВЫСОТОЙ /i2 = 200 км. Какую энергию теряет при этом спутник? В обоих случаях орбиту спутника считать круговой.
206.	Космический корабль находится на расстоянии h = = 20 000 км от поверхности Земли и в системе координат, связан
56
ной с Землей, имеет скорость ох= 6 км/с, направленную вдоль радиуса от центра Земли. Двигатели не работают. Упадет ли корабль на Землю или улетит в космическое пространство? Влиянием Солнца, Луны и планет пренебречь. Что произойдет, если при тех же условиях скорость корабля и2 = 5 км/с или ц3=4 км/с?
207.	Астронавты, высадившиеся на Луну, должны возвратиться на базовый космический корабль, который летает по круговой орбите на высоте, равной радиусу Луны 7? л = 1740 км. Какую начальную скорость необходимо сообщить лунной кабине, чтобы стыковка с базовым кораблем стала возможной без дополнительной коррекции значения скорости кабины? Ускорение свободного падения на поверхности Луны gn = 1,7 м/с2.
208.	С поверхности планеты радиуса 7? вертикально вверх стартует ракета с первой космической скоростью. Планета не имеет атмосферы. На какую высоту поднимется ракета?
209.	Пусть по радиусу Земли от ее полюса до центра прорыта узкая шахта. В эту шахту с поверхности падает тело, достигая центра Земли. Какую скорость будет иметь тело в этот момент, если Землю считать однородным шаром и сопротивлением воздуха пренебречь.
210.	В прорытую вдоль оси, проходящей через оба полюса Земли, узкую шахту влетает с первой космической скоростью их ракета. Какую скорость будет она иметь в момент достижения центра Земли? С какой скоростью вылетит с противоположного конца шахты? Сопротивлением воздуха пренебречь.
211.	С высоты h = 6370 км над отверстием шахты, прорытой вдоль оси, проходящей через полюсы Земли, свободно падает тело. Предполагая, что движение происходит без сопротивления, определить: а) какую скорость будет иметь тело в центре Земли; б) с какой скоростью оно вылетит из шахты на противоположной стороне Земли.
212.	В прорытый насквозь земного шара параллельно его диаметру туннель запускают ракету с первой космической скоростью. С какой скоростью вылетит ракета из туннеля? Пренебречь сопротивлением движения, а также влиянием вращения Земли вокруг своей оси.
213.	Для защиты от вредного излучения и метеоритов космическую станцию построили в центре остывшей планеты, радиус которой R, а масса М. С какой скоростью у0 должны стартовать ракеты с центра планеты по вырытым в ней туннелям (рис. 76), чтобы преодолеть силу тяготения планеты? Ускорение свободного падения на поверхности планеты g. Планету считать однородным шаром.
214.	Две звезды с массами тх и т2 движутся так, что расстояние I между их центрами остается неизменным. Определить характер движения звезд (траекторию, радиус орбиты, период обращения), пренебрегая их взаимодействием с другими небесными телами.
215.	Две звезды с массами т1 и т2 равномерно вращаются по концентрическим окружностям вокруг центра, причем расстояние
57
между ними всегда постоянно и равно I (рис. 77).Определить радиусы орбит и периоды обращения звезд.
216.	Две звезды одинаковой массы т движутся по окружности радиуса г, оставаясь одна против другой. Пренебрегая влиянием других небесных тел, найти скорость движения этих звезд.
217.	Определить угловую скорость вращения двухпланетной системы. Массы планет Л4г и Л12, расстояние между их центрами R. Найти также ускорения, с которыми движутся планеты.
218.	Два спутника движутся вокруг Земли по круговым орбитам в одной плоскости с линейными скоростями ^=7,8 км/с и v2 = 7,7 км/с. Определить интервал времени т, через который оба спутника, вращающиеся вокруг Земли в одном направлении, периодически сближаются на минимальное расстояние.
219.	Две звезды одинаковой массы М движутся по одной орбите вокруг общего центра масс, расстояние между ними2Д. Как зависит линейная скорость движения звезд и период их обращения по орбите от расстояния между ними?
§ 4. Статика
Задача 1. Лестница опирается на вертикальную стену и горизонтальный пол (рис. 78). Центр тяжести лестницы находится на середине ее длины. Коэффициенты трения в точках А и В соответственно равны ц! =0,5 и ц2 = 0,4. Определить наименьший угол наклона лестницы к горизонту, при котором она может оставаться в равновесии.
Ре ш е н и е. На лестницу действуют силы: тяжести mg и реакции со стороны пола и стены, равные соответственно Ди = Na + + Fta и Rb = Nd + FTB- Под действием этих сил лестница находится в состоянии равновесия. Обозначим длину лестницы через 2/. Чтобы лестница была в равновесии, необходимо выполнение двух условий равновесия: 1) должна равняться нулю геометрическая сумма всех сил, действующих на лестницу; 2) должна равняться нулю алгебраическая сумма моментов сил относительно оси, проходящей
58
через любую точку. Приравнивая к нулю суммы проекций сил на оси Ох и Оу и сумму моментов сил относительно точки А, получим следующие три уравнения: Nb—FTx=0;
Na + Ftb — mg = 0 и mgZcosa — N в  2/sina — fTlB • 2/cosa = 0.
Так как угол a — наименьший угол наклона лестницы, при котором она находится на границе между покоем и скольжением, то силы трения будут максимальными, и это дает еще два уравнения: F^a = — л и F?b = [l%Nb.
Решая эти пять уравнений совместно, получаем
Эту же задачу можно решить другим способом — графически. Сна-—>	*
чала построим линии действия полных сил реакций Ra и Rb в точках А и В (рис. 79). Для этого отложим от нормали к полу в точке у?
А угол <Pi = arctg-r^ = arctgp,! (угол трения) в направлении про-™ А
тив часовой стрелки (направление возможного вращения под действием силы Ra). Такой угол составляет с нормалью к полу в точ-ке А линия действия силы Ra- Аналогичным образом получим линию действия силы Rb-
Так как лестница находится в равновесии под действием трех сил: mg, Ra и Rb, то линии действия этих сил пересекаются в одной точке С. Точку пересечения линий действия сил mg и Nb обозначим D. Из ABDC: — tg<p2 - ц2. Но BD = Zcosa, CD = CE — — DE —СЕ —ВО. Из АСЕА находим СЕ = EAclg^ = Zcosa х X ctgcpx = 1 Zcosa.
59
Из ЛВОА: ВО — 2/sina. Следовательно, CD = /cosa — 2/sina.
Подставив значения CD и BD в (1), получим
—	/cosa— 2/sina	j
—	г------------ и,, или-----2tga = a,,
/cosa	r2’	6	12
Отсюда a = arctg kZ-EiE2.
Задача 2. Массивный цилиндрический каток необходимо под-нять на ступеньку высотой I (рис. 80). Определить наименьшую силу Fmin, которую необходимо для этого приложить к центру катка в горизонтальном направлении, если масса катка т, а радиус г (г > /).
Решение. На каток действуют силы F, mg и реакция опоры N. Рассмотрим моменты этих сил относительно точки О. Сила F (плечо hA и сила mg (плечо h2) создают моменты, которые стремятся вращать каток в разные стороны. Реакция опоры N не создает момента относительно точки О, так как проходит через нее. Для того чтобы каток поднялся на ступеньку, необходимо, чтобы момент сил, вращающих каток вокруг точки О по часовой стрелке, был больше момента сил, вращающих его против часовой стрелки:
Fh2 > mgh.2, где h2 = г — I.
Из ЪОО'К /г2 = /г2-(г-/)2 = //(2r-Z). Тогда Fmln = _ mg // (2r — 1}
г — I
Задача 3. Однородный стержень АВ массой т = 0,5 кг, закрепленный в шарнире А, удерживается в равновесии горизонтальной проволокой ВС. К концу стержня подвешен груз массой М = 3 кг. Определить натяжение проволоки Г, если стержень образует с вертикалью <а =45° (рис. 81).
Решение. Составим уравнение моментов сил относительно оси, проходящей через точку А (рис. 82) перпендикулярно к плос-60
кости чертежа! T/cos а —	+ A4gZ)sin а = О, где I — длина
стержня. Отсюда натяжение проволоки
Т=^(1+2^)^а~32 Н-
Задача 4. К горизонтальной балке длиной I = 1 м и массой
т = 2 кг, опирающейся на концы, подвешены три груза на таких расстояниях от левого конца: т1= 1 кг на расстоянии /х= 20 см, т2= 2 кг на расстоянии 12 — 40 см и т3 =5 кг на расстоянии /3==
= 70 см. Определить силы давления на опоры.
Решение. Задачу можно решить, воспользовавшись как первым, так и вторым условием равновесия тела. Воспользуемся вторым условием (условием моментов) как более лаконичным. Прини-
тд а
Рис. 84
мая балку за однородный стержень длиной I, назовем приложенные к нему силы (рис. 83). Все силы лежат в одной плоскости. Под действием приложенных сил балка находится в равновесии. Рассмотрим моменты приложенных сил относительно точек А и В.Через эти точки проходят линии действия неизвестных сил, а поэтому моменты этих сил равны нулю (плечи этих сил равны нулю).
Запишем условие равновесия балки (условие моментов) относительно точки А:
mi^i + та^2 +
+ tn3l3-j-m -g-
+ m2gl2 + m3gl3 + mg-% , откуда R2 =--------,------g«
At 53,9 H.
Условие равновесия тела (условие моментов) относительно точки В:
RJ = т^(1 — /х) + m2g(l — /2) + m3g (I — l3) + mg^
61
откуда
«1 (l — li) —12) + m3 (/ — Z3) 4- m 4
*i =---------------}--------------£^44,1 H.
Ri и 7?2 — силы реакции опор, равные, по третьему закону динамики, силам давления балки на опоры.
Задача 5. К двум одинаковым пружинам, соединенным один раз последовательно, а другой — параллельно, подвешивают один и тот же груз массой т. Найти удлинение пружин в обоих случаях, если жесткость каждой пружины k.
Решение. Рассмотрим случай параллельного соединения пружин (рис. 84). На тело действует сила тяжести и силы со стороны
Рис. 85	Рис. 86
[Fj | = | F21 = F. Система находится в равновесии, т. е. Fr + F2 + + mg = 0. Проектируя на ось Оу, получим: mg = 2F, т. е. F = ^ . Сила F подчиняется закону Гука, т. е. F = kx = г~, от-mg куда х =
При последовательном соединении пружин (рис.84) на пружину 2 действуют сила тяжести и сила Fr со стороны пружины 1, причем F1= mg = kxlt так как пружина 2 находится в равновесии. Поскольку тело также находится в равновесии, то F2-—mg = kx2, откуда fj , f2 2/?ig
X = X, + X2 =	+ -г =	.
1,2 k я k
Задача 6. Однородный цилиндр А массой m и радиусом г опирается на гладкую поверхность цилиндра В радиуса R и удерживается в равновесии при помощи нити CD, длина которой I, закрепленной в верхней точке цилиндра В (рис. 85). Определить силу натяжения нити и силу реакции цилиндрической поверхности.
Р е ш е н и е. На цилиндр А действуют силы (рис. 86, а): тяжести mg, нормальной реакции W поверхности цилиндра В, натяжения
62
нити Т. Цилиндр А находится в равновесии, следовательно, линии действия этих сил пересекаются в одной точке. Так как линии ~ —>	—>
действия сил mg и N пересекаются в точке О', то и линия действия
силы Т должна проходить через эту точку, т. е. должна быть направлена вдоль радиуса г. Построим треугольник, сторонами которого являются силы mg, N иТ (рис. 86, б). Этот треугольник подобен треугольнику ДСОО'. Из подобия треугольников имеем:
о4/)ИЛИг1_ = ^ = ^_. отсюда Т ~l-F~mg и
Г _ mg
О'С ~ СО
Задача 7. На передней стенке ящика шкафа имеются две симметрично расположенные ручки. Расстояние между ручками I, ширина и длина ящика d. При каком коэффициенте
Рис. 88
трения между ящиком и шкафом ящик нельзя вытащить, приложив любую по значению силу F только к одной ручке?
Решение. Под действием силы F, приложенной к одной ручке, ящик перекосится и в точках А и В (рис. 87) будет давить на шкаф. Соответственно со стороны шкафа в этих точках возникнут силы реакции, действующие на ящик. Каждую из сил реакций
—*-	—1
можно представить как сумму нормальной ДГд (или А'д) и касатель-—>	—*-
ной FtA (или FtB) составляющих. Равновесие ящика будет выполняться при условии равенства нулю суммы сил, действующих на него, и равенства нулю суммы моментов относительно какой-либо точки, например точки А. Тогда
Na-Nb = 0-, F.A +F-,B-F = 0 и Ft±l-F7Bd-NBd = 0.
63
Поскольку Na ~ Nb = N, то F7A — Ftb = [iN и полученные уравнения можно переписать в виде
2pN = F и 2u/V — pNd — Nd = O,
d
откуда p == y.
Рис. 89
Отсюда при Ц>у при любой силе F вытащить ящик невозможно.
Задача 8. Степенной полиспаст состоит из двух неподвижных и четырех подвижных блоков (рис. 88). К первому блоку подвешен груз массой тг. Какой груз надо подвесить к свободному концу нити, чтобы система была в равновесии? Определить силы, действующие в точках закрепления нитей А, В, С, D, Е и F. Массой блоков и нити пренебречь Нить считать нерастяжимой.
Решение. На ось блока 1 действует сила m^g, поэтому нити, на которых он подвешен, растягиваются силой m^g. Такая же сила ~ mtg действует на точку А. На блок 2 вниз действует сила -- mjg, поэтому нити, на которых подвешен этот блок, растягива
ются силой у m^g. Такая же сила ~ n^g действует и на точку В. Аналогично на блок 3 вниз действует сила tnrg, а поэтому ни-ти, на которых подвешен этот блок, растягиваются силой g m^g. Поскольку блоки 3, 4, 5 и 6 связаны одной нитью, то в любой ' точке эта нить растягивается силой g- m1g . Отсюда следует, что такая же сила mxg действует на точку F. На блок 4 вниз действует сила n^g. Поэтому такая же сила ^m1g действует на точку D. Аналогично определяем, что на точку Е действует та-1 -* кая же сила 4 m1g.
Итак, система находится в равновесии, если к свободному концу „ I нити подвешен груз массой
Задача 9. Определить положение центра тяжести однородного тонкого диска радиусом /?, из которого вырезано отверстие радиуса г < Центр выреза находится на расстоянии —/? от центра диска.
64
Решение. Для определения центра тяжести диска свырезом предположим, что из диска симметрично первому вырезали отверстие такого же радиуса г (рис. 89). Центр тяжести диска с двумя симметричными вырезами будет лежать в его геометрическом центре О. Будем считать, что изображенная на рис. 89 фигура состоит из диска с двумя вырезами и наложенного на него слева диска радиусом г. Из соображений симметрии ясно, что центр тяжести фигуры должен лежать левее точки О, в какой-то точке С. Относительно этой точки моменты сил тяжести диска с двумя вырезами и диска радиуса г равны. Сила тяжести диска с двумя вырезами пропорциональна площади л (Л?2— 2г2), а диска радиуса г — площади яг2. Тогда условие равенства моментов
1	г2/?
л (У?2 — 2г2) х = лг2 у R, откуда х =	•
ЗАДАЧИ ДЛЯ САМОСТОЯТЕЛЬНОГО РЕШЕНИЯ
220.	Лестница опирается на вертикальную стену и горизонтальный пол. Коэффициент трения между лестницей и стеной р.1 = 0,5, а между полом и лестницей р2 = 0,4. Определить наименьший угол наклона лестницы, при котором она еще может оставаться в равновесии.
221.	К стене прислонена лестница массой т. Центр ее тяжести находится на расстоянии 1/3 длины от ее верхнего конца. Какую горизонтальную силу F нужно приложить к середине лестницы, чтобы верхний конец ее не оказывал давления на стену? Угол между лестницей и стеной а.
222.	Тяжелая однородная лестница стоит на шероховатом полу и опирается на гладкую стену. Покажите, как направлена сила реакции пола.
223.	Человек массой т взбирается по лестнице массой М, прислоненной к вертикальной стене. Коэффициент трения между лестницей и стеной pj, а между лестницей и полом — р2. Определить, при каком угле между лестницей и стеной человек сможет взобраться наверх.
224.	Стержень АВ опирается на шероховатый пол и гладкий выступ С (рис. 90). Расстояние АС = 0,75 АВ. При каком коэффициенте трения стержень составит угол а = 45° с полом в положении равновесия?
225.	На какую максимальную высоту может подняться человек массой иг по лестнице массой М и длиной I, приставленной к гладкой стене? Угол между лестницей и полом ос, коэффициент трения о пол ц.
226.	Однородная балка массой т = 60 кг и длиной I = 4 м опирается о гладкий пол и выступ В, находящийся на высоте h — 3 м над полом (рис. 91). Балка образует угол а = 30° с вертикалью и удерживается веревкой АС, протянутой у самого пола. Вычислить натяжение веревки, реакцию пола и реакцию выступа В.
3 7-253
65
Рис. 96
Рис. 97
Рис. 98
227.	Стержень длиной I и массой т одним концом упирается в вертикальную стену, а другой его конец удерживается с помощью нити, длина которой равна длине стержня (рис. 92). При каких углах а стержень будет находиться в равновесии, если коэффициент трения между стержнем и стеной ц = 0,3?
228.	Однородный стержень АВ массой т = 100 кг опирается одним концом на гладкий горизонтальный пол, а другим — на гладкую плоскость, наклоненную под углом а = 30° к горизонту (рис. 93). Конец В стержня поддерживается веревкой, перекинутой через блок С и несущей груз Q. Часть веревки ВС параллельна наклонной плоскости. Пренебрегая трением на блоке, определить массу груза Q и реакции пола и наклонной плоскости /?2-
229.	Однородный стержень АВ массой т опирается на две неподвижные гладкие плоскости, составляющие с горизонтом углы аир (рис. 94). Определить реакции в точках А и В (Л\ и N2) и угол ф при равновесии.
230.	Однородная палочка АВ, концы которой могут скользить без трения по горизонтальной плоскости ОА и вертикальной стенке ОВ, удерживается в положении равновесия нитью ОС (рис. 95). Палочка наклонена к горизонтальной плоскости под углом а, а нить — под углом р. Найти натяжение нити Т, если масса палочки т. При каких положениях точки С равновесие возможно и при каких невозможно?
231.	К верхнему краю доски длиной L и массой М прибит брусок, длина которого I и масса т (рис. 96). Доска закреплена в точке О и прислонена к стенке под углом а к основанию. При какой горизонтальной силе F, приложенной на высоте h, равновесие доски не нарушится, если стенку убрать?
232.	Расстояние между колесами мостового крана массой М = 2 • 103 кг L = 26 м (рис. 97). Трос подъемной электролебедки, с помощью которого поднимают груз массой т — 103 кг, находится на расстоянии I = 10 м от одного из колес. С какими силами кран давит на рельсы, если лебедка поднимает груз с ускорением а = 9,81 м/с2?
233.	Груз массой т поддерживается двумя шнурами, перекинутыми через блоки (рис. 98). На концах шнуров висят грузы массой /«J и т2. Определить углы аир при равновесии.
234.	Электрическая лампа массой т = 2 кг подвешена к потолку на шнуре АВ, а затем оттянута к стене веревкой ВС (рис. 99). Определить натяжение 7\ шпура АВ и Т2 веревки ВС, если известно, что угол а = 60°, а угол р = 135°. Массой шнура и веревки пренебречь.
235.	К веревке АВ в точке В привязаны груз массой т2 и шнур BCD, перекинутый через блок С (рис. 100). К другому концу шнура привязан груз D массой = 10 кг. Определить натяжение веревки АВ и массу т2 груза, если углы, составленные веревкой и шнуром с вертикалью, равны а = 45° и р = 60° соответственно. Трением пренебречь.
3*
67
236.	К потолку на нити длиной I подвешен однородный шар радиусом R и массой М. К этой же точке на длинной нити подвесили тело массой т. Какой угол а образует с вертикалью нить, на которой подвешен шар?
237.	Однородный стержень АВ прикреплен к вертикальной стене посредством шарнира А и удерживается под углом ср == 60° к вертикали при помощи веревки ВС, образующей с ним угол ^<р (рис. 101). Определить реакцию R шарнира, если известно, что масса стержня т = 2 кг.
238.	Однородный стержень А В упирается верхним концом А в гладкую стенку. К нижнему концу В привязана нерастяжимая нить ВС, прикрепленная к стене в точке С (рис. 102). Стержень образует со стеной угол а, а нить — угол 0. Определить соотношение углов аир при равновесии.
239.	Конец однородного стержня АС массой tnY прикреплен к кронштейну с помощью шарнира А (рис. 103). Какой массы груз следует подвесить к переброшенной через блок нити, чтобы отрезок нити ВС был горизонтальным, а угол наклона стержня к кронштейну был равен а?
240.	Однородный металлический стержень изогнули в виде буквы Г так, что его части имеют длину а = 10 см и b = 20 см. Стержень подвесили на нити за точку изгиба. Определить, какой угол а образует длинный конец стержня с вертикалью.
241.	Верхний конец однородного стержня свободно поворачивается вокруг неподвижной горизонтальной оси О (рис. 104). Нижним концом стержень опирается на брусок, лежащий на гладком столе. Масса стержня т = 1,5 кг, угол между стержнем и бруском а = 30°. Если брусок потянуть вправо с силой F > 2,45 Н, он сдвигается с места. С какой силой давит стержень на брусок во время его движения?
242.	Тело массой т = 1 кг одной нитью связано с неподвижной точкой О, а второй нитью, перекинутой через невесомый блок В (рис. 105), с телом массой М = 2 кг. В некоторый момент времени 2.ОАВ прямой, а отрезок ОА составляет угол а = 30° с горизонтом. Определить натяжение нити АВ в этот момент времени.
243.	На цилиндр массой т навита нерастяжимая невесомая нить (рис. 106). С какой силой F можно тянуть за нить под углом а к горизонту, чтобы цилиндр, вращаясь вокруг своей оси, не перемещался по плоскости? Коэффициент трения между цилиндром и плоскостью J1.
244.	Катушка с нитками лежит на наклонной плоскости, составляющей угол а с горизонтом (рис. 107). Свободный конец нити закреплен у верхнего конца наклонной плоскости так, что нить параллельна наклонной плоскости. Внешний радиус катушки R, радиус намотки ниток г. При каком минимальном коэффициенте трения pmin катушки о плоскость система будет в равновесии?
245.	Катушка висит, касаясь стенки (рис. 108), причем нить составляет со стенкой угол а = 30°. Радиус намотки ниток г — 1 см, 68
//|Х\	т2
Рис. 101	Рис. 102	Рис. ЮЗ
Рис. 104	Рис- 105
Рис. 107	Рис. 108
Рис. 109
250. При
радиус ее щечек R = 10 см. Определить минимальное значение коэффициента трения р между стенкой и катушкой.
246.	Катушка массой т с намотанной на нее нитью лежит па шероховатой горизонтальной плоскости. За нить тянут с силой F. Как должна быть направлена эта сила и каково должно быть ее значение, чтобы катушка двигалась поступательно, не вращаясь вокруг своей оси? Коэффициент трения между катушкой и плоскостью ц. Радиус щечек катушки /?, радиус намотки нити г.
247.	Однородный стержень массой М = 2 кг и длиной I = 1 м подвешен за концы на двух вертикальных пружинах, коэффициенты жесткости которых ^=40 Н/м п /г2= 60 Н/м соответственно. На каком расстоянии х от первой пружины надо подвесить груз массой т = 1 кг, чтобы стержень был горизонтальным? Считать удлинения пружин одинаковыми.
248.	На двух наклонных плоскостях, образующих с горизонтом углы аир, лежит шар массой т. Определить силы давления шара на каждую из плоскостей.
249.	Двугранный угол образован гладкими плоскостями, составляющими с горизонтом углы а± = = 30° и а2 = 90° (рис. 109). Внутри угла находится шар массой т = 0,173 кг. Определить силу давления шара на вертикальную плоскость.
жом максимальном угле а наклонной плоскости
стоящий на ней цилиндр не опрокинется, если высота цилиндра h = 25 см, а диаметр d = 15 см?
251.	Человек перемещает куб с длиной ребра а = 1,8 м вверх по наклонной плоскости, составляющей угол а = 45° с геризонтом, прикладывая силу F перпендикулярно к грани куба и обеспечивая его равномерное поступательное движение. На каком расстоянии х от наклонной плоскости он может прикладывать силу, чтобы основание куба не отрывалось от наклонной плоскости? Коэффициент трения между поверхностями куба и наклонной плоскости р = 0,2.
252.	К концам горизонтального стержня длиной I — 0,8 м и массой т = 2 кг подвешены два груза: слева массой mt =1 кг, справа — т2 = 3 кг. На каком расстоянии со стороны большей массы следует подпереть стержень, чтобы он остался в равновесии?
253.	Два шара массой т1= 4 кг и т2 = б кг скреплены стержнем, масса которого т3 = 2 кг. Определить положение общего центра масс, если радиус первого шара А\=6 см, второго R2 — 8 см> длина стержня I = 30 см.
254.	На какое расстояние х сместится центр тяжести однородного диска, радиус которого R = 40 см, если в нем вырезать круглое отверстие радиусом г = 10 см? Расстояние между центрами диска и отверстия Z = 15 см.
255.	Система (рис. 110) состоит из четырех шарнирно соединенных одинаковых невесомых стержней и невесомой пружины. В не-нагруженном состоянии (т — 0) стержни образуют квадрат и дли
70
на пружины I — 9,8 см. При подвешивании к нижнему шарниру груза массой т = 0,5 кг острый угол между стержнями а = 60 . Определить коэффициент жесткости пружины k.
256.	Конструкция из пяти одинаковых стержней закреплена в точке А (рис. 111). К точке D подвешен груз массой т = 1,7 кг. Определить, с какой силой растянуты (или сжаты) все стержни. Массой стержней пренебречь.
257.	10 одинаковых стержней длиной I и массой т каждый шарнирно соединены между собой (рис. 112). Какие две горизонтально направленные силы F надо приложить к концам нижних стержней, чтобы стержни образовывали угол а с горизонтальной
плоскостью? Трением нижних концов по плоскости и в шарнирах пренебречь.
258.	Две одинаковых трубы подвешены на одинаковой длины канатах (рис. 113). Сверху на эти трубы кладут трубу такого же радиуса и массы. Определить угол (3, если известно, что угол между канатами равен 2а. Пренебречь силами трения.
§ 5. Закон сохранения импульса
Задача 1. По гладкой горизонтальной поверхности движется тележка с песком массой /И со скоростью v. В нее стреляют из ру жья, причем пуля массой т, летящая со скоростью и, застревает в тележке. Какой станет скорость тележки после попадания в нее пули в случае, когда: а) скорость пули направлена горизонтально, так же как скорость тележки; б) скорость направлена вертикально вниз?
Р е ш е н и е. Выясним возможность применения закона сохранения импульса. На систему тележка с песком — пуля извне действуют сила тяжести и сила реакции поверхности. Следовательно, система не является замкнутой. Однако обе рассмотренные силы действуют по вертикали, в то время как в горизонтальном направле
нии никакие силы на систему не действуют (трением колес тележки о поверхность пренебрегаем). Поэтому проекция импульса системы на горизонтальное направление (направление движения тележки) есть величина постоянная. Рассматривая все движения относительно поверхности Земли, в случае а) для проекции импульса можно записать ти-\- Mv = (т + М) ик, откуда конечная скорость тележ-
ти 4- Mv п
ки Vk ~ т _р~д4 • После попадания пули скорость тележки увеличилась.
В случае б) начальный импульс пули не дает проекции на горизонтальное направление и закон сохранения импульса запишется
Mv = (т + М) ук, откуда щ	,
т. е. конечная скорость тележки уменьшилась.
Задача 2. На железнодорожной платформе, движущейся по инерции со скоростью V, укреплено орудие, ствол которого направлен в сторону движения платформы и приподнят над горизонтом на угол а. Орудие произвело выстрел, в результате чего скорость платформы уменьшилась в п раз. Найти скорость и снаряда (относительно Земли) при вылете из ствола. Масса снаряда т, масса платформы с орудием (без снаряда) М.
Решение. На систему платформа с орудием — снаряд извне действуют две силы: тяжести (m + M)g и реакции N рельсов. До выстрела эти силы уравновешивались, так как система двигалась равномерно. Во время выстрела сила взаимодействия между платформой и рельсами возрастает вследствие явления отдачи, поэтому равновесие сил, приложенных к системе, нарушается: N > (т + + M)g. Следовательно, во время выстрела система не является замкнутой, ее импульс изменяется. Однако следует учесть, что обе рассмотренные силы действуют по вертикали, в то время как в горизонтальном направлении никакие силы на систему не действуют (трением колес платформы о рельсы пренебрегаем). Это означает, что горизонтальная составляющая системы не должна измениться. Рассматривая все движения относительно Земли, получим
, п — 1 т + М-----
шщсозр + М — = (т + M)v, откуда vc = v—1 OT„"sp— ,	(1)
где vccos р — проекция на ось Ох скорости vc снаряда относительно Земли (рис. 114).
Чтобы связать скорость vc с искомой скоростью и, будем рассматривать движение снаряда относительно Земли как сложное, состоящее из двух: со скоростью и относительно орудия и со скоростью вместе с орудием относительно Земли. Тогда vc = и ф-
4- . Перепишем это уравнение в проекции на ось Ox: vccosp =
72
= ucosa + Заменив в (1) величину чим и =	+ у.
nzncosa
Задача 3. Тело массой т. — 1 кг с постоянной по модулю скоростью г» = ление и модуль | Ар | вектора изменения периода. Чему равно изменение модуля импульса Ар тела?
Решение. Задачу удобно решать графически (рис. 115). На рисунке изображены векторы импульса тела в начальный рх и конечный р2 моменты времени и вектор Ар изменения импульса
vccosp ее значением, полу-
двигалось по окружности 10 м/с. Определить направ-импульса тела за четверть
Рис. 114
Рис. 115
за четверть периода. Из рисунка видно, что вектор Др направлен под углом a == 45° к вектору рь а его модуль
I Ар I = i Р2 — Pi I = I Pi I И 2 = tn | v j ]/2 « 14 кг • м/с.
Очевидно также, что в силу равномерности движения изменение модуля импульса Др = j рг I — I р21 = 0.
В этой задаче четко проявляется векторный характер импульса, вследствие чего модуль изменения импульса (как и любого вектора) совсем не то же самое, что изменение его модуля. Импульс тела может изменяться даже в том случае, когда его модуль остается постоянным.
Задача 4. Лодка длиной L = 3 м стоит в неподвижной воде. Человек, находящийся в лодке, переходит с носа на корму. На какое расстояние s сдвинется при этом лодка относительно берега, если масса человека т = 60 кг, масса лодки М = 180 кг? Сопротивлением воды движению лодки пренебречь.
Решение. Действие на систему лодка — человек внешних сил (тяжести и реакции воды) скомпенсировано, в горизонтальном направлении на систему силы не действуют (сопротивлением воды пренебрегаем), поэтому эту систему в проекции на горизонтальное направление можно считать замкнутой и применять закон сохранения импульса. Пусть скорость человека относительно лодки V,
73
скорость лодки относительно воды «.тогда скорость человека относительно воды и + и. По закону сохранения импульса,
т (и + и) + Ми = 0. откуда ~	•	(1)
Пусть человек переходит с носа на корму лодки за время т, т. е. L = vt, за то же время лодка перемещается на s — их. Тогда
и	s „	и .,,	s	т
— = р . Подставив это значение — в (1), получим р — —
Рис. 116	Рис. 118
откуда s = —L ~-0$ — —0,75 м. Знак минус показывает, что перемещения человека и лодки противоположны.
Задачу можно решить и иначе, воспользовавшись тем, что если система замкнутая и ее импульс не изменяется, то не изменяется и скорость центра масс системы (центра тяжести). Пусть первоначально расстояние между центром масс системы Ос и центром масс лодки Ол равно х (рис. 116). Тогда Mx = т	откуда
х = 2(тТлц ' Когда человек перейдет с кормы лодки на ее середину, то очевидно положение его центра масс должно совпадать с положением центра масс системы. Следовательно, и положение центра масс лодки должно также совпадать с положением центра масс системы, т. е. лодка должна переместиться на расстояние х. На такое же расстояние переместится лодка при переходе человека на нос. Следовательно, полное перемещение лодки
l = 2x = —^L.
т + м
74
Задача 5. На конце соломинки массой М и небольшой длины I, лежащей на гладком столе, сидит кузнечик, масса которого т. С какой минимальной скоростью он должен прыгнуть, чтобы попасть на другой конец соломинки? Трением между столом и соломинкой пренебречь.
Решение. Чтобы скорость кузнечика была минимальной, он должен прыгнуть под углом 45° к горизонту. При этом vr == vR (рис. 117). Оттолкнувшись от соломинки, кузнечик сообщит ей импульс, проекция которого на горизонтальное направление Mvc равна mvr. В результате соломинка переместится на расстояние sr= vct в направлении, противоположном vc. Кузнечик за это же время пролетит в горизонтальном направлении расстояние s2 = = vTt (s2 < /). Чтобы кузнечик попал на конец соломинки, необходимо выполнение следующего условия: Sj + s2 = I.
2v
Учитывая, что полет кузнечика продолжается / =—в, а скорость т	т 2°в , 2г’в ,
СОЛОМИНКИ Vc = Vr, получим -^vr — + vr — = I. Поскольку v г2 т 4- М ,	। / М ,
= = ТО g-^- = Z’ 0ТКУда v = V
Задача 6. Ракету запустили со скоростью v под углом а к горизонту. В высшей точке траектории произошло разделение ступеней ракеты и первая ступень упала на Землю на расстоянии х0 от места запуска. Определить дальность полета второй ступени ракеты, если масса второй ступени т2 в k раз меньше массы первой ступени.
г-,	т-т	т, ,	k
Решение. По условию задачи, — = fe, тогда тг = т г и /7^2	л! "р 1
т2 =	 В высшей точке траектории импульс полной ракеты
р[х — tnvx — mucosa. Импульс первой ступени до разделения был ,	k	„	„	т
р2х тли[х = mwix, второй ступени р2х = m2v2x = v2x.
Поскольку на ракету в высшей точке траектории по касательной к ней (в горизонтальном направлении) не действуют внешние силы, то импульс системы сохраняется:
pix + Pq'x = Pix, или v2x + mvix = mucosa.
Для определения дальности полета второй ступени надо знать ее скорость v2x. Сначала определим скорость первой ступени vlx по заданной дальности полета х0 (рис. 118):
х0 =	+ vlxt, где t =	, и тогда vlx = g-f- — vx.
8	s	VU
Подставив это значение в закон сохранения импульса, получим
U2, = (2fe -f- 1) VX -
7&
„	О V,Vu
Дальность полета второй ступени равна сумме выражении
(дальность перед разделением ступеней) и v2xt, т. е. х =	+ (2&+
„ „	Va sin2a ,
_|_ 1)	— kx0, или х = —-— (fe + 1) — kx0.
Задача 7. Снаряд разрывается в верхней части траектории на два осколка равной массы. Скорость снаряда непосредственно перед
разрывом равна v, а скорость одного из осколков сразу же после разрыва иг = 2и и направлена вертикально вверх. Определить мо-
Рис. 119
дуль и направление вектора скорости v2 другого осколка в момент разрыва.
Решение. На снаряд действует внешняя сила — сила тяжести. Но так как при разрыве снаряда возникают большие внутренние силы и время их действия очень мало, тоимпуль- >
сом силы тяжести mg\t по сравнению
Рис. 120
с импульсом внутренних сил можно пренебречь и считать систему на время разрыва замкнутой. Запишем закон сохранения импульса:
mv = ~ mvt + mv2.
Перепишем это уравнение в проекциях на координатные оси (рис. 119): 2vx = vlx v2x и 2Vy =	t>2^-	По условию задачи, viy =
= у, = 2о; Vy = 0; vx = v, поэтому v2x = 2v и v2y = —2v. Отсюда v2 = |/ vjx +	= 2 I 2 v и tga = |	| = 1, a a = 45°.
Таким образом, второй осколок отлетит со скоростью v2 яй 2,8о вниз под углом а =45° к горизонту. Чтобы изобразить вектор v2 на рисунке, нужно восставить перпендикуляры к координатным осям из точек 1 и 2.
Задача 8. Пучок молекул падает на стенку и отражается от нее по закону абсолютно упругого удара. Найти давление р этого пучка на стенку, если направление скорости движения молекул со
76
ставляет угол 0 с нормалью к ней. Известны масса т, скорость v и концентрация п молекул в пучке. Рассмотреть случаи: а) стенка неподвижна; б) стенка движется в направлении нормали к ней со скоростью и.
Решение. Изменение импульса стенки Др, равное импульсу действующей на нее силы (Др = FAt), можно выразить через изменение импульса одной молекулы Д (та) и число N ударившихся о стенку молекул за время Д/: F At = Д (mv) N. Эго выражение позволяет найти давление, оказываемое пучком молекул:
где 5 — площадь стенки, «обстреливаемая» молекулами.
а)	Если стенка неподвижна (рис. 120), то в проекциях на оси координат
Д (tnv)x = 2tnvx = 2macos 0.	(2)
Для нахождения У нужно учесть, что за 1 с на стенку попадут только те молекулы, которые находятся от нее на расстоянии I = vx = = v cos 0, поэтому
N = /wcos 0S At.	(3)
Подставив (2) и (3) в выражение (1), получим
2mucos0 Mt'osfl SAZ n
p =---°——----------= 2m/w2cos-0.
б)	Если стенка движется навстречу молекулам со скоростью и, то Д (mu)x = 2m (ucos 0 + u); I = (acos 0 + u), откуда p = = 2mn (tzcos 0 + u)2.
в)	Если стенка перемещается в ту же сторону, что и молекулы, давление на нее р = 2тп (ncos 0 — w)2.
ЗАДАЧИ ДЛЯ САМОСТОЯТЕЛЬНОГО РЕШЕНИЯ
259.	Мяч массой m = 150 г ударяется о гладкую стенку под углом а = 30° к ней и отскакивает без потери скорости. Найти среднюю силу F, действующую на мяч со стороны стенки, если скорость мяча v = 10 м/с, а продолжительность удара At =0,1 с.
260.	Струя воды сечением S = 6 см2 ударяет о стенку под углом а = 60° к нормали и отскакивает от нее без потери скорости. Найти силу, действующую на стенку, если скорость течения воды в струе v = 12 м/с.
261.	С какой средней силой F давит на плечо ручной пулемет при стрельбе, если масса пули m — 10 г, скорость при вылете v = = 800 м/с и скорострельность пулемета п = 600 выстрелов в минуту.
77
262.	Шарик массой т — 20 г падает на стальную плиту со скоростью vL = 5 м/с и отскакивает от нее в прямо противоположном направлении со скоростью и2 = 4 м/с. Определить изменение импульса Ар шарика и модуль F средней силы, действующей на шарик во время удара, если соударение длилось т = 10~2 с.
263.	С какой силой давит на землю кобра, когда, готовясь к прыжку, поднимается вертикально со скоростью и? Масса змеи М, ее длина L.
264.	Ракета массой М = 6 • 103 кг установлена для запуска по вертикали. Если скорость истечения газов из сопла ракеты v — = 1000 м/с, то какое количество газа должно быть выброшено за 1 с, чтобы обеспечить тягу, достаточную: а) для преодоления силы тяжести ракеты; б) для сообщения ракете начального ускорения а = 19,6 м/с2?
265.	Тело массой т = 2 кг двигалось по окружности, причем в некоторой точке А имело скорость Va = 4 м/с, а пройдя четверть окружности, в точке В vB = 3 м/с. Определить модуль вектора изменения импульса тела.
266.	В лодке массой mj=240 кг стоит человек массой т2 = = 60 кг. Лодка плывет со скоростью Vi~- 2 м/с. Человек прыгает с лодки в горизонтальном направлении со скоростью v = 4 м/с относительно лодки. Найти скорость движения лодки после прыжка человека вперед по движению лодки.
267.	Человек массой /Mj = 80 кг переходит с носа лодки на корму. Длина лодки I = 5 м. Чему равна масса лодки, если она за время этого перехода переместилась в стоячей воде в обратном направлении на s = 2 м? Начальная скорость лодки относительно воды равна пулю. Трением пренебречь.
268.	Лодка длиной I и массой М стоит в спокойной воде. На ее концах сидят два рыбака, массы которых т± и т2. На сколько сместится центр тяжести лодки, если рыбаки поменяются местами?
269.	На носу лодки длиной I стоит рыбак, держа на высоте h рыбину массой т. Масса лодки вместе с рыбаком М. Рыбак бросает рыбину вдоль лодки. Какую горизонтальную скорость он должен сообщить рыбине, чтобы попасть в корму лодки? Сопротивление воды не учитывать.
270.	Баржа стоит неподвижно в стоячей воде. По ней с носа на корму перемещается автопогрузчик массой т. Длина баржи L, масса М. Определить перемещение Аг баржи относительно берега. Сопротивление воды ничтожно мало.
271.	Охотник стреляет с легкой надувной лодки. Какую скорость приобретает лодка в момент выстрела, если масса охотника с лодкой М = 80 кг, масса дроби т = 35 г, средняя начальная скорость Дроби Uj= 320 м/с? Ствол ружья ВО Время выстрела направлен под углом а = 60° к горизонту.
272.	Платформа массой М — 900 кг движется по инерции со скоростью Vi е» 1 м/с вместе с человеком, масса которого т *= ₽ 70 кг, Человек прыгает с платформы назад со скоростью 78 1
— 5 м/с относительно платформы. С какой скоростью v продолжает движение платформа?
273.	Тележка массой т1 = 120 кг движется по инерции со скоростью Vj= 6 м/с. С тележки соскакивает человек массой т2 = = 80 кг под углом а = 30° к направлению ее движения. Скорость тележки уменьшается при этом до и2 =4 м/с. Какова была скорость прыжка?
274.	Два пассажира одинаковой массы т = 70 кг находятся на платформе, стоящей неподвижно на рельсах. Масса платформы М = 280 кг. Каждый пассажир начинает бежать с одинаковой относительно платформы скоростью и = 6 м/с. Какую скорость приобретет платформа, если они спрыгнут в одну сторону одновременно и последовательно? А если в разные стороны?
275.	Тележка с песком катится со скоростью 1 м/с по горизонтальной поверхности без трения. Навстречу ей летит шар массой т = 2 кг с горизонтальной скоростью v2 = 7 м/с и застревает в песке. С какой скоростью и в какую сторону покатится тележка после встречи с шаром? Масса тележки М = 10 кг.
276.	Плот массой М = 600 кг и длиной I = 12 м самоплавом переправляют по реке со скоростью v = 1 м/с. Одновременно с одного конца плота на другой переходят два человека, причем один массой тг = 40 кг идет по течению реки, другой массой т2 = = 100 кг — ему навстречу. Оба движутся со скоростью и = 0,8 м/с относительно плота. Определить смещение s плота относительно Земли к тому моменту, когда сплавщики поменяются местами.
277.	Надувной плот массой /И отплывает от берега со скоростью и, направленной под углом а к берегу. С берега на плот прыгает человек массой т, развивая во время прыжка скорость и, направленную перпендикулярно к берегу. Определить скорость движения плота с человеком.
278.	Две платформы /Wj и /И2 движутся по инерции параллельно друг другу со скоростями щ и и2, причем щ > v2. С первой платформы на вторую перепрыгнуло N человек массой т каждый. Прыжок был сделан в направлении, перпендикулярном к направлению движения. Через некоторое время эти же N человек прыгают со второй платформы на первую. Какова была скорость платформы после первого и второго прыжков?
279.	На одном конце неподвижной длинной тележки массой mt= 20 кг стоит человек массой т2 = 60 кг. С какой скоростью будет двигаться тележка, если человек пойдет вдоль нее со скоростью v = 2 м/с относительно тележки?
280.	Человек массой т = 60 кг бежит со скоростью и= 8 км/ч. Догнав тележку, движущуюся со скоростью = 2,9 км/ч, он вскакивает на нее. Какова будет скорость тележки после этого, если ее масса щ1 = 80 кг?
281.	Человек стоит на краю тележки, движущейся горизонтально со скоростью v0 = 5 м/с. Затем он прыгает по ходу движения тележки под углом а = 30° к горизонту, в результате чего скорость
79
тележки уменьшается до и — 4 м/с. Определить скорость человека относительно Земли в момент прыжка, если его масса в п = 3 раза меньше массы тележки.
282.	Вагон массой ту = 3 • 104 кг, движущийся по горизонтальному пути со скоростью цх = 1,5 м/с, автоматически на ходу сцепляется с неподвижным вагоном массой т2 = 2 • 104 кг. С какой скоростью будут двигаться вагоны после сцепления?
283.	С лодки выбирается канат, поданный на баркас. Расстояние между ними I = 55 м. Определить пути, пройденные лодкой и баркасом до их встречи. Масса лодки т — 300 кг, баркаса М. = = 1200 кг.
284.	Двухступенчатую ракету массой т запускают со скоростью г0 под углом а к горизонту. В высшей точке траектории происходит разделение ступеней и одна из них падает как раз под местом разделения ступеней. С какой скоростью продолжает движение вторая ступень и на каком расстоянии от места запуска ракеты она упадет на Землю, если ее масса 0,1 т?
285.	В высшей точке траектории снаряд разрывается на два осколка одинаковой массы. Один из них полетел вперед по горизонтали со скоростью Uj= 800 м/с, второй — назад со скоростью и2 = = 300 м/с. Определить скорость снаряда в момент взрыва.
286.	Снаряд массой т — 40 кг, летевший в горизонтальном направлении со скоростью v = 600 м/с, разрывается на две части с массами тх= 30 кг и т2 — 10 кг. Большая часть стала двигаться в прежнем направлении со скоростью щ =900 м/с. Определить значение и направление скорости меньшей части снаряда.
287.	Ракету массой М запустили со скоростью и0 под углом а к горизонту. В высшей точке траектории двигатель ракеты выбросил порцию газа массой т со скоростью и относительно ракеты, причем т М. Определить дальность полета ракеты.
288.	Ракета взлетает вертикально вверх, выбрасывая газы последовательно двумя равными порциями. Скорость истечения газов относительно ракеты постоянна и равна и. Каким должен бьпь промежуток времени между сгоранием порций, чтобы ракета достигла наибольшей высоты? Считать, что сгорание топлига происходит мгновенно.
289.	После взрыва ракеты, летящей вертикально, образовались три осколка одинаковой массы, которые упали на Землю одновременно. Расстояние от места старта до места падения двух из них равны Si = 3 км и s2= 4 км, причем линии, соединяющие места их падения с местом старта, образуют между собой прямой угол. Каково расстояние $3 от места старта до места падения третьего осколка?
290.	Пучок молекул с массой т = 5 • 10“2в кг каждая, движущихся в вертикальной плоскости со скоростью v — 500 м/с, падает под углом а = 30° на боковую стенку сосуда и испытывает упругое соударение. Определить изменение импульса каждой молекулы за время удара. Чему равна средняя сила давления пучка на стенку, если ежесекундно происходит IV = 1022 соударений?
80
291.	Теннисный мяч ударяют ракеткой у самой поверхности Земли, сообщая ему начальную скорость v — 20 м/с, направленную под углом а — 45° к горизонту. Мяч летит к вертикальной стене и испытывает с ней абсолютно упругое соударение. Стена находится от места удара на расстоянии / = 15 м. На каком расстоянии х от места удара мяч упадет на Землю?
292.	Два свинцовых шара катятся без трения по горизонтальному столу во взаимно перпендикулярных направлениях и неупруго сталкиваются. Определить значение и направление скорости шаров после удара, если их скорости до удара их=4 м/с и и2=1,5 м/с, а отношение масс = п — 2.
293.	Два тела с массами /пх = 0,3 кг и т2 = ОД кг движутся по горизонтальной плоскости со взаимно перпендикулярными скоростями ух = 10 м/с и v2 — 20 м/с. Тела при соударении слипаются
Рис. 121	Рис. 122
и движутся как одно целое. Определить скорость системы после соударения.
294.	Два шара массами т1 = 2 кг и т2 = 1 кг движутся со скоростями щ = 5 м/с и и2= 4 м/с навстречу друг другу. Определить скорость первого шара после абсолютно упругого соударения. Удар считать центральным.
295.	В заднюю стенку башни танка, идущего со скоростью их =
—>•
= 72 км/ч, ударяется горизонтально летящая со скоростью у0 = = 750 м/с пуля и упруго отскакивает от нее (рис. 121). С какой скоростью полетит отскочившая пуля? Стенка наклонена к вертикали под углом <р = 30°.
296.	Гладкий абсолютно упругий бильярдный шар ударяется о такой же неподвижный шар. В момент удара линия центров шаров образует угол а = 20°с направлением скорости первого шара. На какой угол 0 изменится направление скорости первого шара по сравнению с первоначальным?
297.	Два одинаковых шара массой tn каждый покоятся, касаясь друг друга. Третий шар налетает на них, двигаясь по прямой, касающейся обоих шаров (рис. 122). Удар происходит без потерь энергии. Найти массу налетающего шара, если после удара он остановился. Радиусы всех шаров одинаковы.
298.	Ядро массой tn, летящее со скоростью v, распадается на две части одинаковой массы, причем один из осколков летит со скоростью их под углом а к направлению полета до его распада. Найти скорость и направление полета второго осколка ядра.
81
§ 6. Работа и энергия. Закон сохранения энергии
Задача 1. Человек перемещает ящик массой т =163 кг по горизонтальной поверхности на расстояние s = 10 м с постоянной скоростью, прикладывая силу под углом а = 30° к горизонту. С какой силой должен действовать человек на ящик и какую он выполняет работу по перемещению ящика, если коэффициент трения ящика о плоскость р =0,1? Рассмотреть случаи: а) человек толкает ящик; б) человек тянет ящик, прикладывая силу под тем же углом.
Ре ше н и е. а) Когда человек толкает ящик,то положительную работу выполняет лишь горизонтальная составляющая Feos а силы F, приложенной к ящику. Вертикальная же составляющая F sin а лишь увеличивает силу давления ящика на пол, вследствие чего сила трения возрастает до значения FT = [i(mg + F sin а). Ящик движется равномерно при условии F cos а — FT =0, или Feos а = = (тё + F sin °0р> откуда
F =-------196 H.
cosa — jtsina
Работа этой силы на пути s = 10 м составляет
At = Fseosa == ^g5Cosa « 1666 Дж.
1	cosa— fisina
б) Во втором случае вертикальная составляющая силы F направлена вертикально вверх и поэтому сила трения уменьшается до значения FT = ц (mg — Fsin а). Поскольку ящик движется равномерно, то F cos a = р (mg — Fsin а), откуда
F =-----.. .. ~ 174 H.
cosa + psma
Работа этой силы A2 = Fseosa — .	~ 1480 Дж.
1	cosa -f- psi na
Задача 2. Сани съезжают с горы, составляющей с горизонтом угол а. Высота горы h. Пройдя путь s по горизонтальной плоскости, они поднимаются в гору с углом наклона 0 к горизонту (рис. 123). Определить, на какой высоте /ц сани остановятся, если коэффициент трения на всем пути одинаков и равен р.
Р е ш е н и е. Рассмотрим начальное и конечное положение саней. Потенциальная энергия на вершине первой горы mgh больше потенциальной энергии на второй горе mgh^ на значение работы, совершенной против сил трения на всех участках пути. Поскольку тело покоилось в начальной и конечной точках, то изменения кинетической энергии в процессе движения не влияют на окончательный результат. Закон сохранения энергии в этом случае запишется
mgh = А-)- Л2+ Л3+ mgh^
где Л1( Л2 и А з — работы по преодолению сил трения на склоне первой горы (Л О, на горизонтальном участке (Ла) и при подъеме
82
на вторую гору (Л3). Следует отметить, что силы трения на этих участках неодинаковы; F1= timgcos a, F2 = [img и Fa = pmgcos (J. Подставляя эти значения в предыдущую формулу и выражая длины наклонных участков через соответствующие высоты h, и углы а, |3, запишем закон сохранения энергии: mgh = pmgcosct 4^ + pmgs+ 4- umgcosB + mgh,. Отсюда искомая высота h, —	.Л?-8/--,,,-S
1 r e ' sinp ' в 1	«	1 ц (J _р ctgp)
Задача 3. К лежащему на горизонтальной поверхности бруску массой т = 12 кг прикреплена пружина жесткостью k = 300 Н/м (рис. 124). Коэффициент трения между бруском и поверхностью ji = = 0,4. Вначале пружина не деформирована. Затем, приложив к свободному концу пружины силу F, направленную под углом а =
Рис. 124
как двигался брусок. Будем
скорость бруска увеличива-
= 30° к горизонту, медленно переместили брусок на расстояние s = 0,4 м. Какая работа при этом была совершена? Считать, что колебаний не происходит.
Решение. Совершенная работа А состоит из работы Лъ затраченной на растяжение пружины, и работы <42, затраченной k(L}2 \ k ) F2 на перемещение бруска. Работа AL =	с учетом за-
кона Гука, а работа Л 2 =--= Fscoso'.
Значение силы F зависит от того, считать, что движение происходило с постоянной скоростью. Очевидно, что на начальном этапе, когда лась от нуля до некоторого установившегося значения, наше допущение несправедливо. Однако, так как установившаяся скорость мала, этот этап длился очень недолго, и его вклад в работу не существен.
Согласно второму закону Ньютона, сумма всех действующих на брусок сил при равномерном движении равна нулю: F -J- mg + + А/ 4- Fj = 0, или, в проекциях на горизонтальную и вертикальную оси координат: Fcosa — Кт = 0 и N + Ksinct — mg = 0, где FT — u,N. Отсюда F =-----— Окончательно
cosa + psi na
A « Ai + Aa - + Fscosa « —+	« 19 Дж.
ла
Задача 4. Санки, движущиеся по горизонтальному льду со скоростью v, въезжают на асфальт. Считая, что длина полозьев санок равна I, а коэффициент трения их об асфальт равен ц, определить путь s, пройденный санками по асфальту, если известно, что s > I. Массу санок считать равномерно распределенной по длине полозьев. Трением санок о лед пренебречь.
Решение. Когда санки въезжают на асфальт, происходит постепенное увеличение силы давления N полозьев на асфальт от нуля до максимального значения, равного силе тяжести mg санок. В связи с этим возрастает и сила трения FT — p,N, действующая на санки со стороны асфальта. Поскольку санки движутся под действием переменной силы, воспользуемся для решения понятиями работы и энергии. Работа силы трения, действующей на санки,
определяется изменением их кине-тическои энергии от Ej = — до Е2 = 0, т. е.
и
Рис. 125
Рис. 126
Работу Ат можно вычислить и иначе. Разобьем весь путь, пройденный санками, на два участка (рис. 125): s = I + sr На пути Sj сила трения постоянна и равна ц/ng, поэтому совершаемая ею работа Аг = ц/ngSi. На пути I на сани действует переменная сила трения Ft = цЛ/, она возрастает линейно от нуля до конечного значения ц/ng, соответствующего полному выезду санок на асфальт. Тогда на всем пути I движения по асфальту можно считать, что на санки действует средняя сила трения Fc = ^mg. Работа этой силы
А2 = у ц/ng/. Полная работа сил трения
Ат = Xj + А2 = ц/ngsj + j pmgl = ц/ng (у I + s^.	(2)
Приравнивая правые части равенств (1) и (2), найдем «i = —
Тогда весь путь, пройденный санками, s = / + Sj = 2^g— •
Задача 5. Льдина площадью поперечного сечения S и толщиной Н плавает в воде. Какую работу надо совершить, чтобы полностью погрузить льдину в воду?
Решение. Определим толщину выступающей над водой .части льдины, обозначенную х (рис. 126). На льдину массой М действу
«4
ют сила тяжести Mg = pnSHg и сила Архимеда FA = рп(Н— — x)Sg. В равновесии Mg 4- FA = 0, или в проекции на ось Оу
PxSHg = рв (Н — х) gS, откуда х = Рв- Н.
•В
При погружении льдины необходимо преодолеть добавочную силу Архимеда Ал, связанную с погружением выступающей части. Эта сила переменная, она возрастает линейно от нуля до конечного значения FK, соответствующего полному погружению льдины. Тогда на всем пути перемещения х можно считать, что действует средняя сила Ас = т, FK = pBSxg — (рв — рл) SHg, а работа этой силы
z!'b
Задача 6. Летящая в горизонтальном направлении со скоростью у, = 5 м/с пуля попадает в деревянный брусок и застревает в нем. Брусок лежал на горизонтальной поверхности. С какой скоростью будет двигаться брусок с застрявшей в нем пулей? Масса пули — 0,5 кг, бруска т2— 2 кг. Какое расстояние пройдет брусок, если коэффициент трения равен р. =0,1? Какая часть кинетической энергии пули превратится во внутреннюю энергию при ударе?
Решение. В системе брусок — пуля во время удара действуют только внутренние силы, поэтому можно записать закон сохранения импульса
+ т2-0 = (т1 ф- т2) v2, откуда г>2 =	~	= 1 м/с.
Во время движения бруска на него действует сила трения Ат = = р(щг+ m^g. Работа этой силы на пути s равна начальному значению кинетической энергии бруска:
2	2
1	2	v2
и (т. + m2) gs = 2	0ТКУда s =	~
fa 0,5 м.
Поскольку начальная кинетическая энергия пули = — т^, а начальная кинетическая энергия бруска Е2 = (mx ф- т2)»2 =
= 2'(^~фГот j ’ то ПРИ УДаре во внутреннюю превратится часть кинетической энергии
ДЕ = Ег - Е2 = п . = 5 Дж,
или в процентах; k = — • 100 % = 80 %.
85
Задача 7. На столе лежит веревка, свешивающаяся на одну четверть своей длины за край стола. Длина веревки / = 1 м. Веревка начинает скользить. Определить скорость веревки в момент соскальзывания со стола ее конца, если коэффициент трения между веревкой и столом ц = 0,32.
Решение. При движении веревки начальная потенциальная энергия системы частично превращается в кинетическую и расходуется на выполнение работы по преодолению силы трения. Будем отсчитывать потенциальную энергию от пола, на котором стоит стол . Тогда в случае, когда веревка неподвижно лежит на столе, ее потенциальная энергия
Я	1	7	41
т rngl 4- -4- mg = & mgl’
где т—масса единицы длины веревки.
В момент соскальзывания конца веревки со стола ее механическая энергия равна Ем = mg^ -|- ^mv2. Однако в процессе движения веревки часть энергии расходуется на выполнение работы по прео-з
делению сил трения. Эта работа равна FTs = где W — сила
нормального давления, равная силе тяжести части веревки, лежа-
з
щей на столе. Значение М в процессе движения изменяется от mg до нуля, по поскольку это изменение происходит по линейному закону с изменением длины веревки, лежащей на столе, то можно
3	9
взять среднее значение Л/с = mg. Тогда F7s = р & nigl.
 Согласно закону сохранения и превращения энергии,
mgl = 4 mg I + | mv2 + p g92 rngl, откуда v =	|/(15 — 9ц) gl «
« 2,7 м/с.
Задача 8. Грузик массой m, подвешенный на резиновой нити, отклоняют на 90°, не натягивая нити, и отпускают. Когда нить проходит через вертикальное положение, ее относительное удлинение (по сравнению с горизонтальным положением) равно 25 %. Определить натяжение нити в этот момент.
Р е ш е н и е. Натяжение нити при прохождении через вертикальное положение будет максимальным, значит, максимальную длину будет иметь и пить, поэтому радиус кривизны грузика в этот момент равен длине нити. Обозначим длину нити в ненатянутом состоянии (в горизонтальном положении) через /, а в момент прохождения грузиком положения равновесия через За нулевой уровень отсчета потенциальной энергии грузика принимаем положение равновесия. Из закона сохранения энергии
, mv'2 , /г (А/)2	,п
= —+ -\-*	(О
66
Уравнение движения грузика в момент прохождения положения равновесия
mt/2	.	,, mv2 ,
-г- — — mg + N, или N = —|- mg, ‘1	ч
где N — сила упругости (реакция) нити. По третьему закону Ньютона, искомая сила натяжения нити численно равна реакции нити (но направлена в противоположную сторону и приложена не к шарику, а к нити). Таким образом,
п т1,г ,
F = — 4- mg.	(2)
Жесткость нити, по определению,
ь _ Z____
Л/ Ц-Г
(3)
—	/лх 1.	то2 4-игр/.
Подставляем в выражение (3) значение силы из (2): k — - ---—.
G (‘1 — </
Подставляя это значение жесткости в выражение (1), находим о2: , тиг .........................
mgli = -y
(rtw- 4- mtrl,) (It—I)2
_ gG (G + 0
Подставим
21х—1 
значение и2 в выражение (2):
(4)
p __ « gh (G 4- 0 4	/sx
1 -t; ni-i" + ms-	w
Так как относительное удлинение известно, то	—- = 0,25, отку-
да Zj = 1,25 Z. Подставляем это значение 1г в выражение (5):
с	1,25/ + I	п ,
F = mg	+ W = 2>5 тё-
Задача 9. Гиря массой т = 0,5 кг падает с некоторой высоты на плиту массой М = I кг, укрепленную на пружине жесткостью fe —980 Н/м. Определить значение максимального сжатия пружины, если в момент удара гиря имела скорость vr — 5 м/с. Удар считать неупругим.
Решение. Система гиря — плита— пружина— Земля является замкнутой и поскольку при движении гири и сжатии пружины трения практически нет, полная механическая энергия этой системы будет сохраняться. При падении гири на плиту происходит неуиругий удар, в результате чего механическая энергия полностью или частично превращается во внутреннюю. Рассмотрим два состояния системы (рис. 127). За начальное состояние примем момент после неупругого удара (/), за конечное — момент максимального сжатия пружины (2). Потенциальную энергию силы тяжести будем отсчитывать от положения, соответствующего максимальному сжатию пружины, потенциальную энергию силы упругости — от положения,
87
соответствующего состоянию пружины с плитои в момент падения гири на плиту.
В начальном положении энергия системы Ег состоит из потенциальной и кинетической энергии гири и плиты:
В конечном положении у гири нет кинетической энергии, зато сжатая пружина обладает энергией упругой деформации. Теперь полная механическая энергия системы
с kx2
Е2 = ^~-
Приравняем по закону сохранения энергии правые части этих уравнений и получим квадратное уравнение относительно х:
kx2 ,	. ... и (т + М) v2 п
—	+	—	\	= 0.
Этого уравнения недостаточно для определения сжатия пружины х, поскольку оно содержит еще одно неизвестное — скорость v гири и плиты после неупругого удара. Воспользуемся для ее определения законом сохранения импульса
= (т + М) v, откуда v = -т^ 
Учитывая, что х = И, квадратное уравнение можно записать так kx2 — 2(т + М) ех--= 0.
к 1 ,s m + М
Решив уравнение, найдем
(« + М) g zx: l/(m + M)2g2 + Х = ------------------------------х_-
Отрицательный корень уравнения должен быть отброшен: х < 0 означает растяжение пружины, тогда как на самом деле она сжимается. Подставив числовые значения величин, получим х да « 8,2 см.
Задача 10. Тело соскальзывает по наклонному желобу, переходящему в «мертвую петлю» радиусом г. Какой должна быть наименьшая высота ската, чтобы тело сделало полную петлю? С какой силой тело давит на желоб в точке, радиус-вектор которой составляет угол а с вертикалью?
Решение. На тело в произвольный момент времени его дви-жения вверх по петле действуют две силы: тяжести mg и давле
88
ния N петли, направленная по радиусу к центру окружности. Вто-рой закон динамики для тела: mg -|- N — та.
Направим координатные оси Ох и Оу по радиусу и по касательной к окружности. Учитывая, чтоа = -^-, перепишем векторное уравнение в проекции на ось Ox: mg cos а + N = у- . Поскольку mv2 при движении вверх по петле значение mg cos а возрастает, а —
убывает, то значение /V — —---mg cos а должно и подавно убы-
вать. При обращении М в нуль тело оторвется от петли. Приняв N — 0, получим v = ^grcosa. Для верхней точки петли cosa = 1, и поэтому минимальная скорость и, при которой тело пройдет верхнюю точку петли и при этом не будет давить на желоб, v = gr.
Наименьшую высоту ската находим из закона сохранения энергии: mgh =	+ mg  2г. Подставив в это выражение значение
v = V gr, найдем h — 2,5 г.
Скорость тела v1 в точке, радиус-вектор которой составляет угол а с вертикалью, найдем из закона сохранения энергии: mgh = X X mv\ + tngr(\ 4- cosa). Учитывая, что h = 2,5г, получим = = Sr (3 — 2 cos a).
Силу давления тела на желоб в этой точке, равную, по третьему закону динамики, реакции опоры, определим из- уравнения движения тела:
mvt
N2 + mg cos a —	, откуда AZ2 = 3mg (1 — cos a).
Задача 11. На шар массой М, первоначально покоившийся, налетает шарик массой т со скоростью v0. Найти скорости шаров после соударения. Определить наибольшую энергию, которую налетающий шарик может передать покоящемуся. Удар центральный и абсолютно упругий. Радиусы шаров равны.
Решение. Поскольку удар центральный, то движение шаров одномерное и скорости их направлены по одной прямой. Система двух шаров не замкнутая, но поскольку действующие на шары силы (тяжести и реакции опоры) уравновешиваются, а в горизонтальном направлении силы не действуют, то проекция импульса системы на горизонтальное направление есть величина постоянная. Предположим, что после удара оба шара двигались в положительном направлении оси Ох, т. е. в ту же сторону, в которую двигался первоначально шарик т. Тогда для проекций импульсов получим mv0= — mvr + Му2. При абсолютно упругом ударе сохраняется полная кинетическая энергия — 1. mv2 Mv2. Решив совместно эти
т — М	2m
два уравнения, получим vl = ^77ио и = ^±~mvo-
89
Кинетическая энергия первого шара после соударения
р	[т— М\3
~ 2 — 2 \т + м) ‘
Энергия, приобретенная вторым шаром (потерянная первым),
Р р Р ,nvo т-о 1т - М\ "Ч 4тМ Ел — 2	2 \тМ/ ~ 2 (т + Л1)2 ‘
Исследуем полученное решение. При Л1 = т налетающий шар после удара остановится, второй же приобретет скорость и0 и энер-mvo „	г-
гию —2- . Подставив условие М = т в выражение для с2, видим, что наибольшая энергия передается именно в этом случае. При М < т скорости Vj и v2 будут иметь один знак, т. е. оба шара будут двигаться в направлении налетающего шара. В случае М > > т налетающий шар т начнет двигаться назад, отдав второму шару М только часть своей энергии. При М -> оо происходит удар, как о неподвижную стенку: и2=0; vt= — v0.
Задача 12. На горизонтальном столе лежат три шарика с массами тг, пг2 и тъ так, что их центры находятся на одной прямой. Первому шарику сообщают некоторую скорость и, и происходят последовательные абсолютно упругие центральные столкновения шариков. Какой должна быть масса т2 второго шарика, чтобы скорость третьего шарика была максимальной?
Решеви е. Запишем законы сохранения импульса и энергии для центрального упругого соударения первых двух шариков:
mvv = mlvl + m2v2 и ?	□ т№ 
Из этих уравнений определим скорость второго шарика после 2т, v соударения: v2 = ——
Для центрального упругого соударения второго и третьего шариков законы сохранения импульса и энергии запишутся в виде
m2v2 = m2u2 + mava и - m2v2 = j m2zz2 + mav3.
Из этих уравнений va = m 2^_2^ . Подставив значение v2, получим
_ 4m1m2v
3 ~ (mr + m.2) (m.2 + m3) ‘
Исходя из того, что произведение положительных сомножителей, сумма которых является постоянной величиной, имеет максимальное значение при равенстве сомножителей, можно записать
ль т,	। г---
---г— =------, откуда т, = / т.та.
Таким образом, при tn2=Vтута скорость va будет иметь максимальное значение.
90
Задача 13. Автомобилист неожиданно замечает препятствие — длинную канаву, расположенную поперек его пути. Что он должен сделать: тормозить или попробовать свернуть в сторону, если доро-га позволяет это сделать?
Решение. При торможении кинетическая энергия автомобиля затрачивается на работу сил торможения: А = Fs. Таким образом, = Fs. Путь торможения s =	.
Если автомобилист задумает повернуть, ему придется двигаться по дуге, минимальный радиус которой опять-таки диктуется силой трения. Ведь именно она создает центростремительную силу, поворачивающую машину. Если силы трения не хватит для поворота по дуге, то машину просто занесет. Значит, в этом случае F = г\
Радиус R будет в два раза больше длины тормозного пути: R = ^-. Следовательно, если препятствие близко, выгоднее и безопаснее тормозить.
ЗАДАЧИ ДЛЯ САМОСТОЯТЕЛЬНОГО РЕШЕНИЯ
299.	Тело массой т = 100 кг поднимают постоянной силой на высоту h = 15 м в течение t = 10 с. Определить работу этой силы. Начальная скорость тела равна нулю.
300.	На горизонтальном участке пути s = 2 км скорость электро поезда возросла от щ = 54 км/ч до в2 = 72 км/ч. Определить выполненную при этом работу и среднюю мощность, развиваемую двигателями электровоза на этом пути, если масса поезда пг = 8  105 кг, коэффициент трения р. —. 0,005.
301.	Санки съезжают с горы высотой Н и углом наклона а и движутся далее по горизонтальному участку. Коэффициент трения на всем пути одинаков и равен р. Определить, какое расстояние s пройдут санки по горизонтальному участку до полной остановки.
302.	Санки массой т = 60 кг спускаются с горы длиной I = = 100 м и высотой h ~ 10 м, преодолевая силу сопротивления
= 20 Н. Какой будет скорость сапок в конце горы?
303.	Вертолет, масса которого с грузом т= 6 • 103 кг, за t = = 2,5 мин набрал высоту h = 2250 м. Определить работу двигателя за это время, считая подъем вертолета равноускоренным.
304.	Определить кинетическую энергию Е тела массой т = 200 г, брошенного горизонтально со скоростью и0 = 25 м/с, в конце 5-й секунды движения.
305.	Тело брошено вертикально вверх со скоростью и0 — 49 м/с. На какой высоте его кинетическая энергия будет равна потенциальной?
306.	Сваю массой М ~ 1000 кг забивают копром, масса которого т = 400 кг. Копер свободно падает с высоты Н — 5 м, и при каждом ударе свая погружается на h = 5 см. Определить силу сопротивления грунта, считая ее постоянной.
91
307.	Падающим с высоты ft = 1,2 м грузом забивают сваю, которая от удара уходит в землю на s = 2 см. Определить среднюю силу удара и его продолжительность, если масса груза /И = 5 х X 102 кг, а масса сваи намного меньше массы груза.
308.	Шарик, подвешенный на легком резиновом жгутике, растягивает его на А/ = 5 см. Каким будет максимальное растяжение *тах, если шарик поднять и затем отпустить с высоты ft = 20 см, отсчитанной от нижнего конца нерастянутого жгутика? Деформацию жгутика считать упругой.
309.	Насос заполняет бассейн водой за t = 20 с. Найти мощность двигателя насоса, если емкость бассейна V =100 м3, его центр находится на высоте ft = 18 м над поверхностью воды в озере, а к. п. д. насоса т] =90 %.
310.	Подъемный кран должен в течение t = 8 ч поднять т = = 3000 т строительных материалов на ft = 12 м. Какую мощность потребляет двигатель крана, если его к. п. д. т] = 60 %?
311.	Из резервуара откачивают воду с помощью насоса, потребляющего N = 2,5 кВт. Насос соединен с гладким шлангом, наконечник которого расположен на уровне воды в резервуаре так, что струя воды выходит вертикально. Расход воды составляет Q = = ПО м3/ч. Какой высоты достигает струя воды? К. п. д. насоса т] =60 %. Влиянием обратного потока воды пренебречь.
312.	Самолет массой т = 15 т летит горизонтально на высоте ft = 1200 м со скоростью и=100 м/с, затем переходит на снижение при выключенном двигателе. Пройдя планирующим полетом I = = 8 км, самолет достигает дорожки аэродрома со скоростью ut = = 30 м/с. Определить среднюю силу сопротивления во время планирующего полета.
313.	Самолет массой tn = 5  103 кг летел горизонтально с постоянной скоростью = 360 км/ч. Затем поднялся на ft = 2 км выше. При этом его скорость уменьшилась до 200 км/ч. Найти работу, затраченную двигателем на подъем самолета.
314.	Автомобиль массой т — 1000 кг трогается с места и, двигаясь равноускоренно, проходит s = 20 м за t = 2 с. Какую мощность должен развить двигатель автомобиля к концу второй секунды? Какова средняя мощность на данном пути? К. п. д. двигателя г| = 50 %.
315.	Тело движется вверх по наклонной плоскости с направленной вдоль плоскости начальной скоростью ц0 = 20 м/с. На какую высоту поднимается тело, если коэффициент трения ц =0,2, а угол наклона плоскости а = 30°?
316.	Груз массой т = 103 кг опускают вниз на упругом тросе с постоянной скоростью и = 10 м/с. Какова будет максимальная сила Fmax натяжения троса, если внезапно остановить его верхний конец? Жесткость троса k = 10е Н/м. Массой троса пренебречь.
317.	Пуля, летевшая горизонтально со скоростью v = 500 м/с, попадает в шар, подвешенный на очень легком жестком стержне, и застревает в нем. Расстояние от точки подвеса стержня до центра шара I = 2 м. Стержень с шаром вследствие удара пули отклонился
92
на угол а = 30°. Определить отношение массы шара М к массе пули т.
318.	Пуля массой т. = 10 г, летевшая горизонтально со скоростью v — 500 м/с, ударилась в свободно подвешенный деревянный брусок массой М = 10 кг и застряла в нем, углубившись на I = = 20 см. Найти среднюю силу сопротивления дерева движению пули.
319.	Пуля, летящая горизонтально со скоростью v0 = 510 м/с, попадает в ящик, лежащий на горизонтальной поверхности на расстоянии d = 0,5 м от стены дома, и, пробив ящик, вылетает в том же направлении со скоростью v = 10 м/с. Ящик начинает двигаться по направлению к стене. Ударится ли он о стену? Коэффициент трения между ящиком и поверхностью р. =0,1, масса ящика М = = 10 кг, масса пули т — 10 г.
320.	Пуля, летящая со скоростью и0, пробивает несколько одинаковых досок, расположенных на некотором расстоянии друг от друга. В какой по счету доске пуля застрянет, если ее скорость после прохождения первой доски Ui=O,8uo?
321.	Нож, брошенный горизонтально со скоростью попадает в деревянную мишень, подвешенную на веревке, и застревает в ней. Определить высоту, на которую поднимается мишень после попадания ножа, если масса ножа т1( а масса мишени т2.
322.	Конькобежец, стоя на льду, бросил вперед гирю массой т1~-5 кг, сообщив ей скорость yt= 12 м/с (относительно льда), и вследствие отдачи покатился назад. Определить работу, которую совершил конькобежец при бросании гири, если его масса т2 = = 60 кг.
323.	Конькобежец массой М = 60 кг, стоя на коньках на льду, бросает в горизонтальном направлении камень массой т. = 2 кг со скоростью v =9 м/с. На какое расстояние откатится конькобежец, если коэффициент трения коньков о лед ц = 0,02?
324.	Два конькобежца с массами т1= 40 кг и т2 = 60 кг стоят на льду друг против друга, держась за легкий шнур и слегка натянув его. Затем один из них начинает укорачивать шнур. Какую работу А он совершит к тому моменту, когда будет выбирать шнур со скоростью v = 2 м/с? Трением о лед пренебречь. Укорачивание шнура происходит при постоянном натяжении.
325.	Человек, стоящий на гладкой поверхности льда, бросает камень массой т = 3 кг в горизонтальном направлении с высоты Н = 1,8 м. Камень падает на лед на расстоянии s = 9 м от места бросания. Определить работу А, которую совершает человек при бросании камня. Масса человека М = 60 кг.
326.	Из пушки, свободно соскальзывающей с гладкой наклонной плоскости и прошедшей путь I, производится выстрел в горизонтальном направлении. При какой скорости v вылета снаряда пушка остановится после выстрела? Масса пушки М, снаряда т, угол наклона плоскости к горизонту а.
327.	Пружина детского пистолета имеет в свободном состоянии длину 1а = 15 см. Жесткость пружины k = 103 Н/м. Какова макси
93
мальная высота подъема шарика массой m = 10 г, если им зарядить пистолет, сжав пружину до /1= 5 см? Пистолет расположен вертикально.
328.	Тяжелый состав массой М и легкая дрезина массой т движутся параллельными путями с одинаковой скоростью. Дрезина или состав проедет большее расстояние до полной остановки, если одновременно выключить двигатели? Коэффициент трения в обоих случаях считать равным р.
329.	Автомобиль движется по шоссе под гору, угол наклона которой к горизонту а, с постоянной скоростью vr. При той же мощности двигателя автомобиль спускается с этой горы со скоростью v2. С какой скоростью а3 будет двигаться он при той же мощности по горизонтальному участку шоссе?
330.	Два одинаковых бака объемом V и высотой Н каждый стоят один на другом. С нижнего полного бака воду перекачивают насосом в верхний пустой. Какова должна быть минимальная мощность двигателя насоса, чтобы эта работа была выполнена за время /?
331.	Вагонетку массой т = 3 • 103 кг поднимают по рельсам в гору, наклон которой а = 30°, с ускорением а = 0,2 м/с2. Коэффициент трения [х =0,1. Какую работу совершила сила тяги на пути I = 50 м?
332.	Автомобиль массой т = 103 кг съезжает с горы при выключенном двигателе с постоянной скоростью v = 54 км/ч. Наклон горы составляет h = 4 м на каждые I = 100 м пути. Какую мсщ-ность должен развивать двигатель автомобиля, чтобы в гору с тем же уклоном он двигался с той же скоростью?
333.	Какую минимальную работу надо совершить, чтобы поднять вагонетку массой т = 200 кг по эстакаде длиной I — 10 м и высотой h = 2 м при коэффициенте трения р — 0,05? Каков коэффициент полезного действия подъемника?
334.	Поезд массой М = 5 • 106 кг движется с постоянной скоростью по горизонтальному участку пути. В некоторый момент от поезда оторвался последний вагон массой т = 2 • 104 кг. На каком расстоянии друг от друга остановятся поезд и вагон, если машинист выключил двигатели электровоза после прохождения им расстояния s = 240 м? Коэффициент трения р = 0,01.
335.	Двигатель лебедки шахтного подъемника развивает постоянную мощность. При подъеме груза гщ = 103 кг подъемник движется со скоростью 2,5 м/с, а при подъеме груза т2= 1,5 X X 103 кг его скорость v2 = 2 м/с. Определить массу тп клетки подъемника. Трением пренебречь.
336.	Санки скатываются с горки высотой Н = 15 м, угол наклона которой к горизонту а = 30°. Коэффициент трения санок о поверхность горы линейно увеличивается вдоль пути от pj = 0 у вершины горы до р2= 0,4 у подножия. Какую скорость будут иметь сапки у подножия горы?
337.	Воду откачивают из шахты глубиной Н и площадью поперечного сечения 5. Поверхность воды находится на глубине /г.
94
Шланг, с помощью которого насос откачивает воду, имеет поперечное сечение So. Какой должна быть мощность двигателя насоса, чтобы воду можно было откачать за время /?
338.	Двигатель мощностью М приводит в действие насос, с помощью которого перекачивается масса т воды. Какую мощность должен развивать двигатель, чтобы за то же время он мог перекачать вдвое большее количество воды?
339. Невесомая упругая нить длиной I = 1 м закреплена верхним концом в точке А (рис. 128). Из точки А без начальной скорости падает небольшая муфта массой т = 50 г. Дойдя до упора В, муфта растягивает нить. Найти жесткость нити k, если к моменту остановки муфты нить удлинилась на А/ = Q 2 м Трением пренебречь.
Рис. 131
Рис. 128	Рис. 129
340.	На чашку весов, подвешенную па резиновом шнуре, свободно падает с высоты h (считая от уровня чашки) пластилиновый шарик, масса которого т, и прилипает к ней. Определить максимальное удлинение шнура, если жесткость резины k.
341.	Маленькое тело массой т свободно падает с высоты й, попадает в середину однородной доски массой М и мгновенно прилипает к ней (рис. 129). Доска лежит горизонтально на пружине, жесткость которой k. Определить максимальное сжатие пружины. Массой пружины пренебречь.
342.	Тело массой М налетает на две последовательно соединенные пружины, жесткости которых и й2 (рис. 130). Максимальная энергия деформации пружины 2 оказалась равной Е. Определить начальную скорость тела о.
343.	Деревянный поршень прикреплен к цилиндру с помощью невесомой пружины жесткостью k (рис. 131). При движении поршня между ним и цилиндром возникает сила трепня F. Пуля, летящая со скоростью v вдоль оси цилиндра, попадает в поршень и застревает в нем. На сколько при этом сместится поршень? Масса пули т, поршня Л4. Цилиндр закреплен.
95
Рис. 132
= 3 кг подвешены на
344.	В закрепленную вертикальную трубку вставлена невесомая пружина, верхний конец которой прикреплен к подвижному поршню массой М (рис. 132). Нижний конец пружины упирается в дно трубки. Пружина сжата до длины I и удерживается в сжатом состоянии с помощью защелки. На поршень положили шарик массой т. На какую высоту подскочит шарик, если освободить пружину, сдвинув защелку? Пружина в недеформированном состоянии имеет длину L. Жесткость пружины k. Трением пренебречь.
345.	Два тела а массами т1= 0,2 кг и т2=0,1 кг, связанные резиновым жгутом, движутся по гладкой горизонтальной поверхности под действием горизонтальной силы F = 6 Н, приложенной к первому телу. Найти потенциальную энергию деформации жгута, если известно, что под действием силы Fo = 1 Н он растягивается на х0 = 5 см. Движение считать установившимся; массой резинового жгута пренебречь; трение не учитывать.
346.	Два упругих шарика, массы которых т1= 100 г и т2= 300 г, подвешены на одинаковых нитях длиной I = 50 см. Первый шарик отклонили от положения равновесия на а = 90° и отпустили. На какую высоту поднимется второй шарик после удара?
347.	Два тела массами = 2кгит,= нитях длиной Z = 2 м так, что они касают
ся друг друга. Тело т2 отклонили на угол а = 45° и отпустили. Какое количество энергии превратится во внутреннюю при абсолютно неупругом столкновении тел?
348.	Молекула, летящая со скоростью v = 300 м/с перпендикулярно к поверхности движущегося тяжелого поршня, испытывает с ним упругое столкновение. Поршень движется в ту же сторону, А
что и молекула, со скоростью и = 30 м/с. Какую часть -g- энергии молекула потеряет при соударении?
349.	Частица 1 упруго сталкивается с неподвижной частицей 2. После столкновения обе частицы начинают двигаться симметрично относительно начального направления движения частицы 1. Определить отношение масс частиц, если угол между направлениями их движения а.
350.	Частица массой т2 налетает на неподвижную частицу, масса которой > mJ. Определить, на какой максимальный угол атах от своего первоначального направления может отклониться частица с массой т2.
351.	Шар массой т налетает со скоростью v на неподвижный шар, масса которого /И, по линии, соединяющей их центры. После удара скорость первого шара становится вдвое меньше первоначальной. Найти отношение суммарной кинетической энергии обоих шаров после удара к первоначальной кинетической энергии налетающего шара.
96
352.	Висевшая на высоте Н сосулька массой М подтаяла и начала падать. Когда она оказалась на высоте у//, в нее попал снежок массой т, который мгновенно прилип к ней. В момент попадания в сосульку снежок двигался горизонтально со скоростью и0. Найти скорость, с которой сосулька упадет на землю.
353.	Шарик массой т = 100 г подвешен на нерастяжимой нити длиной I = 1 м. Определить энергию Е маятника и скорость v шарика при прохождении положения равновесия, если наибольший угол отклонения маятник® т вертикали 45=
354.	Цирковой артист, разогнавшись на мотоцикле по горизонтальному желобу, выезжает в вертикально расположенную петлю радивой) . пределить минимальную скорость t>0, с кбторои он должен въехать в петлю, чтобы благополучно закончить номер. Перед въездом в петлю артист выключает двигатель.
355.	С какой наименьшей высоты должен скатываться велосипедист, не вращая педалей, чтобы проехать по дорожке, имеющей форму «мертвой петли» радиусом R = 4 м, не отрываясь от дорожки в верхней точке петли?
356.	Тело скользит без трения из наивысшей точки по поверхности неподвижного шара радиусом R. На какой высоте И тело оторвется от поверхности шара?
357.	Небольшое тело соскальзывает по наклонной поверхности, переходящей в петлю, с высоты Н = 1,2 м. Найти работу силы трения, если известно, что сила давления тела на петлю в верхней точке равна нулю, масса тела т = 10 г, радиус петли R = 0,4 м.
358.	Преграда массой 44 = 10 кг, имеющая цилиндрический вырез радиусом R = 0,2 м, расположена на горизонтальной плоскости (рис. 133). Тело массой т = 1 кг с начальной горизонтальной скоростью и0 =3 м/с поднимается по цилиндрической поверхности. Определить скорость тела на высоте R. Трением пренебречь.
359.	Небольшой шайбе А сообщили в горизонтальном направлении скорость и0= 10 м/с (рис. 134). Шайба, поднявшись по закруглению ВС радиусом R = 2 м, взлетела и упала в точке D. Угол а = 60°. Найти расстояние CD, пренебрегая трением и сопротивлением воздуха.
360.	Небольшое тело соскальзываете гладкой наклонной плоскости, длина которой I = 16 см, а угол наклона а = 30°. Затем падает на гладкую горизонтальную плоскость, находящуюся на расстоянии h = 20 см от нижнего конца наклонной плоскости. На какую наибольшую высоту Н от горизонтальной плоскости поднимется тело после удара об нее, если этот удар абсолютно упругий?
361.	На гладкой горизонтальной плоскости на некотором расстоянии друг от друга стоят две одинаковые гладкие горки массой М и высотой Н (рис. 135). На вершине одной из них находится небольшое тело массой т. После легкого толчка оно соскальзывает с этой горки и поднимается на другую. На какую максимальную высоту h поднимется тело по второй горке?
4 7-253
97
362.	У основания наклонной плоскости, высота которой Н= = 1,2 м и угол наклона к горизонту а — 45°, закреплена массивная стальная плита. С вершины наклонной плоскости начинает скользить шайба. На какую высоту поднимется она после упругого удара о плиту, если коэффициент трения между шайбой и наклонной плоскостью р =0,2?
363.	Гладкая горка массой М находится на гладком горизонтальном полу (рис. 136). На горку положили и отпустили без толч-
ка шайбу массой т. Отношение масс п =	=0,6. Указанные
на рисунке размеры: И = 1,3 м и h = 0,5 м. Каким будет расстояние от шайбы до горки в момент падения шайбы на пол?
364.	Жесткий невесомый стержень (рис. 137) может вращаться без трения в вертикальной плоскости вокруг оси, проходящей через точку О. В середине стержня А и на конце В закреплены два шарика, массы которых 4/и и т. Стержень приводят в горизонтальное положение и отпускают. Определить натяжение стержня на участках О А и АВ в момент прохождения положения равновесия.
365.	Стальная пуля массой т пробивает подвешенный на тонкой нити свинцовый шар, масса которого М, в результате чего
9В
скорость пули уменьшается вдвое. Какая часть k кинетической энергии пули пошла на нагревание?
366.	Два тела прикреплены к концам нерастяжимой нити, перекинутой через легкий блок (рис. 138). Первое лежит на шероховатой поверхности, второе висит на нити. Если телам сообщить некоторую скорость, толкнув второе тело вниз, то оно опустится на расстояние /ij= 20 см. Если же сообщить ту же скорость, толкнув первое тело влево, то второе тело поднимется на высоту h2 = 10 см. Определить коэффициент трения между первым телом и поверхно-
mi с
стью, если отношение масс тел —- = 5.
т,
367.	Куб массой т = 2 кг и объемом V = 103 см3 находится в озере на глубине Н = 5 м. Какая работа совершается при подъеме куба на высоту h = 5 м над уровнем озера? Поверхностными явлениями пренебречь.
368.	Однородная прямая призма, площадь основания которой 5 = 1 м2 и высота h = 0,4 м, плавает на поверхности воды так, что в воде находится половина ее объема. Найти наименьшую работу, необходимую для полного погружения призмы в воду.
369.	Для растяжения недеформированной пружины на х1 = = 5 см нужно совершить работу Xr Чтобы растянуть ее еще на Ах, необходимо дополнительно совершить работу Л2. Найти Ах, если = п = 8.
Л
§ 7. Механика жидкостей и газов
Задача 1. В сообщающиеся сосуды налита ртуть, а поверх нее
в одно колено налит керосин до высоты hY = 20 см, в другое — масло до высоты h2 = 48 см. Определить разность уровней ртути в обо-
их коленах сосуда (рис. 139). Плотность масла рм = 920 кг/м3 и керосина рк = 800 кг/м8.
Решение. В сообщающихся сосудах в обоих коленах равновесие жидкости обеспечивается равенством давлений на любом горизонтальном уровне. На уровне раздела Ох—О2 существует одинаковое давление в левом и правом коленах: р = р1П- 4~ Pv
Поскольку атмосферное давление одинаково в левом и правом коленах, то приравняем гидростатические давления:
= h^g + /igPpg,
Рис. 139
откуда h3 = -Рм 2 р Pl</tl = 0,02 м.
Задача 2. В цилиндрический сосуд налиты вода и ртуть в равных по массе количествах. Общая высота двух слоев жидкости Н = = 29 см. Определить давление жидкости на дно сосуда.
4*
99
Решение. Давление жидкости На дно сосуда состоит из давления, создаваемого столбами ртути h± и воды /г2, т. е.
Р = (РЛ + рЛ)£-
Это уравнение содержит три неизвестных: р, /гг и/г2. Запишем еще два дополнительные уравнения, воспользовавшись условиями задачи: mY = т2 и И =	+ h2. Но т{ = pLgVt = Pigh]S и т2 — р2 х
X g^2 = P2gh2S- Тогда Pihi = Pih2’ h2 + h2 = H, откуда Лх = ~ Ии h2 = H-^—.
Pl + P2 2 Pl + Pi
Подставив эти значения ht и h2 в уравнение для р, получим = 2р^ н 5 2	10з Па
Pi + Ра
Задача 3. Аэростат массой т = 500 кг и объемом V = 600 м3 поднимается вертикально вверх. Считая это движение равноускоренным, определить, на какую высоту поднимается аэростат за t = = 10 с. Плотность воздуха р = 1,3 кг/м3.
Р е ш е н и е. На аэростат действует сила тяжести mg, направленная вертикально вниз, и архимедова сила pgV, направленная вертикально вверх. Направим координатную ось Ох вертикально вверх и запишем уравнение движения аэростата в проекции на эту ось:
ma = pgV — mg, откуда а = ———— .
Высоту подъема аэростата найдем по формуле пути для случая равноускоренного движения:
h = 1 at2 =	‘2	275 м.
2	2т
Задача 4. Полый стеклянный шар радиусом г = 0,1 м и массой mj = 0,5 кг плавает на воде. Какой массы свинцовый груз нужно подвесить к шару, чтобы он погрузился в воду ровно наполовину? Плотность свинца р — 11,3 • 103 кг/м3.
Р е ш е н и е. На стеклянный шар и подвешенный к нему свинцовый груз действует сила тяжести (/тцЧ- m2)g, направленная вертикально вниз, и архимедова сила pBg + У2), где V\— объем всего шара, V2—объем свинцового груза. Условие равновесия шара с грузом
(mj + m2)g = pBg (Ц ^1+
Но К2 = —2 и = 4 лг3- Подставив эти значения в условие рав-"	Р-2	“
новесия, после преобразований получим
р
100
Задача 5. Во время профилактического ремонта дио лодки-плоскодонки оклеили слоем пластика толщиной d — 3 см. После этого высота надводной части лодки уменьшилась на h = 1,8 см. Определить плотность пластика р.
Решение. Запишем условие плавания лодки в начальном состоянии, до ремонта:
mg = pBlSg,
где т — масса лодки, ра — плотность воды, I — глубина погруженной части лодки, S — площадь дна. После наклеивания пластика снаружи условие плавания запишется
mg + pdSg = р„(/ + d + h)gS.
Из полученных уравнений находим плотность пластика
р — р/ + Л р . Ю'1 кг/л? .
Если наклеить пластик на дно внутри лодки, то условие равновесия будет mg + pdgS = рв(/ + h)gS и р = рв-| ж 0,6  103 кг/м3.
Задача 6. В сосуде с водой плавает кусок льда. Как изменится уровень воды, когда лед растает?
Р е ш е н и е. На кусок льда действуют две силы: сила тяжести и архимедова сила. Обозначив плотность льда и воды через рл и рв, весь объем льда — Ул, объем его погруженной части через Уп, запишем условие равновесия рлУл^ = pBVng, откуда объем погруженной части V,, = — Ул. Учитывая, что объем льда Ул = — , полу-
Рв	Рл
v Рл"2 т
чим окончательно Vn =------— — 
РвРл Рв
Объем погруженной части льда равен объему вытесненной им воды. Если же лед растает, то объем, который займет образовав
шаяся вода, будет VB = — . Сравнивая объем вытесненной льдом Р в
воды и объем воды, образовавшейся после таяния льда, находим, что они равны: V„ = VB. Отсюда следует, что уровень воды в сосу
де не изменится.
Задача 7. Для транспортировки стальных труб морем их заваривают с двух концов так, чтобы они были водонепроницаемыми. Определить, при каком наименьшем внутреннем диаметре труба массой т = 3,9 • 103 кг и длиной I = 5 м не утонет. Плотность морской воды р, — 1,03  103 кг/м3, стали р2 = 7,8 • 103 кг/м3.
Решение. Труба полностью погружена в морскую воду и плавает в состоянии равновесия. Условие равновесия
mg = Pigl7, откуда V =	.
Р1
Объем трубы можно представить как сумму объемов материала Ver, из которого изготовлена труба, и внутренней полости Vn: V =
101
= VCT + V„. Объем материала можно вычислить, зная массу трубы и плотность стали: Уст =	. Объем полости V„ — IS = I ~. Под-
г2	4
ставив эти значения Кст и V„, получим — = — + / — , откуда ________________ Р1 Ра ’
rf=/^(Pg-Pi)^08M.
V Л/Р1Р2
Задача S. Тело массой т = 2 кг и объемом V = 10"3 м3 находится в озере на глубине h = 5 м . Какую работу нужно совершить, чтобы поднять тело на высоту Н = 6 м над поверхностью воды? Равна ли эта работа изменению потенциальной энергии? Объяснить
результат.
Решение. На тело, находящееся в воде, действует сила тяжести mg и архимедова сила pgV- Поэтому для поднятия тела со
Рис. 140
дна озера на его поверхность к телу надо приложить внешнюю силу = mg — pg К — — g(m — p V). Работа этой силы A t = F\h = = (m — pV)gh.
В воздухе на тело также будет действовать архимедова сила, но так как плотность воздуха значительно меньше плотности тела, архимедовой силой можно пренебречь. Поэтому для поднятия тела на высоту Н над уровнем воды необходимо совершить работу А2 = = mgH. Работа по поднятию тела со дна озера на высоту Н над его поверхностью будет равна сумме работ Аг и Л2:
А = Дi+ А2 = (т — pV)gk + mgH — mg (h + Fl) — — pghV « 166,6 Дж.
Для того чтобы ответить на вопрос, равна ли эта работа изменению потенциальной энергии тела, необходимо подсчитать это изменение,-
АЕП — mg (h + И) = 215,6 Дж.
Отсюда следует, что А < Д£п. Причину различия между А и ДЕП можно объяснить так. Изменение потенциальной энергии тела равно работе всех сил, действующих против силы тяжести. При решении же задачи мы не учитывали работу архимедовой силы. Поэтому полученная работа А меньше изменения потенциальной энергии на значение работы А3 = pghV = 49 Дж, совершаемой архимедовой силой.
Задача 9. Однородный деревянный стержень может вращаться без трения вокруг горизонтальной оси, проходящей через один из его концов. Под стержнем расположили сосуд с водой так, что стержень оказался частично погруженным в воду (рис. 140). Какая часть стержня находится в воде, если плотность дерева pj= 0,8 X X 103 кг/м3, а плотность воды р2 = 103 кг/м3?
Решение. Условием равновесия тела с неподвижной осью вращения является равенство нулю алгебраической суммы момен-
Ю2
тов сил, действующих на это тело. Относительно оси вращения —>
моменты создаются двумя силами: тяжести mg, приложенной к центру тяжести стержня, и архимедовой силой, приложенной к центру тяжести погруженной в воду части стержня.
Обозначим через I длину стержня, через S — площадь его поперечного сечения, через х — часть длины стержня, погруженную в воду. Тогда сила тяжести стержня mg = pj/gS, а архимедова си-ла F^ = p2gxlS. Плечи этих сил равны Zcosa и / ( 1----cosa
соответственно.
Условие равновесия стержня относительно оси вращения mg cos a — p2xlgSl 1 — j cos a = 0.
После преобразования получим квадратное уравнение
х2 — 2х +	= 0.
Р2
Корни этого уравнения равны: Xi, 2 =• 1 ± 1/ 1 — — ;	= 1 »45 и
х2 = 0,55. Очевидно, что доля длины стержня не может превышать единицы, поэтому первый корень лишен физического смысла. Таким образом, в воде находится 0,55 длины стержня.
ЗАДАЧИ ДЛЯ САМОСТОЯТЕЛЬНОГО РЕШЕНИЯ
370.	В сообщающиеся сосуды налили ртуть, а затем в один из сосудов — воду, в другой — бензин. Верхние уровни воды и бензина совпали. Какова разность уровней ртути в сосудах, если высота столбика воды h = 21,5 см?
371.	В цилиндрический сосуд налита ртуть и поверх нее — масло. Масса масла в 2 раза меньше массы ртути. Сосуд заполнен до высоты h — 30 см. Определить давление на дно сосуда, если плотность масла р2=900 кг/м3.
372.	Гидростат глубинной бомбы установлен на давление воды р = 5  Ю6 Па. На какой глубине взорвется эта бомба?
373.	Сосуд кубической формы с ребром а — 10 м до краев наполнен водой. Определить силу давления воды на боковую поверхность сосуда при нормальном атмосферном давлении.
374.	В сосуд, имеющий форму прямого кругового усеченного конуса с радиусом дна R = 10 см, налита вода так, что ее уровень находится на высоте h — 10 см от дна. Определить силу F давления воды на боковую поверхность сосуда, если образующая конуса составляет угол a = 45° с его высотой.
375.	Канал шириной d — 10 м и глубиной h = 5 м наполнен водой и перегорожен плотиной. Определить силу давления воды на плотину.
376.	В сосуд с водой вставлена трубка, площадь поперечного сечения которой S — 2 см2. В трубку налили масло массой т =
103
= 72 г (плотность масла рм = 900 кг/м3). Найти разность уровней масла и воды.
377.	Полый свинцовый шар плавает в ртути так, что его объема находится в ртути. Чему равен объем воздушной полости внутри шара, если его радиус R = 3 см?
378.	Стеклянный шар, внутри которого есть полость, плавает на поверхности воды, погрузившись в нее на половину своего объе-
I/ .
ма. Какую часть объема шара (-у-) занимает полость? Плотность стекла рс, = 2500 кг/м3.
379.	Полый металлический шар, внешний и внутренний диаметры которого соответственно dL = 11 см и d2 = 10 см, плавает на поверхности жидкости. Какой массы т груз нужно положить внутрь шара, чтобы он плавал внутри жидкости? Плотность металла Pi = = 2700 кг/м3, жидкости р2= 800 кг/м3.
380.	Полый стеклянный шар радиусом г ~ 0,1 м и массой тг = = 0,5 кг плавает в воде. Какой массы свинцовый груз нужно подвесить к шару, чтобы он погрузился в воду наполовину?
381.	Будет ли плавать в воде полый кубик из железа, если длина ребра кубика а = 20 см, а толщина стенок b = 0,5 см?
382.	Парафиновый шар, начиненный свинцовой дробью, плавает на поверхности воды так, что п = 0,1 часть его объема находится над водой. Во сколько раз объем свинца Vc меньше объема парафина Кп?
383.	Один конец нити закреплен на дне, а второй прикреплен к пробковому поплавку, погруженному в воду на 3/4 объема. Определить натяжение нити, если масса поплавка т = 2 кг и плотность пробки рп = 250 кг/м3.
384.	Дно лодки-плоскодонки оклеили снаружи пластиком (толщина пластика d =3 см, плотность р — 1,6 • 103 кг/м3). На сколько изменилась минимальная глубина водоема, по которому лодка может пройти, не задевая дна?
385.	Медный шар с внутренней полостью весит в воздухе mx = = 270 г, в воде т2= 225 г. Определить объем внутренней полости шара.
386.	Кусок металла, представляющий собой сплав меди и серебра, в воздухе весит т1 = 245,6 г, а при погружении в воду т2 = = 221,6 г. Сколько серебра и сколько меди в этом куске сплава?
387.	В цилиндрическом сосуде с водой плавает кусок льда, внутри которого вмерзла цинковая пластинка. После полного таяния льда уровень воды в сосуде снизился на Лй ~ 3 см. Определить массу цинковой пластинки. Внутренний диаметр сосуда d ~ 30 см.
388.	Прямоугольный понтон, масса которого М == 700 кг, имеет размеры: длину I = 5 м, ширину d = Зм и высоту h = 0,7м. Найти осадку без нагрузки и предельную грузоподъемность при высоте бортов над ватерлинией — 0,2 м.
389.	Небольшое легкое тело свободно падает с некоторой высоты на поверхность жидкости с известной плотностью р0. Найти плот
104
ность р тела, если максимальная глубина его погружения в жидкость вдвое меньше высоты падения. Сопротивлением воздуха и жидкости пренебречь. Что собой представляет движение тела при таких условиях?
390.	С аэростата сбросили два шарика одинакового радиуса Я — 1 см : один — алюминиевый, другой-—железный. Шарики соединены длинной тонкой нитью. Найти натяжение нити Т после того, как из-за сопротивления воздуха движение шариков установится, т. е. они приобретут постоянные скорости.
391.	Стеклянный шарик массой ш = 100 г, находящийся у поверхности глицерина, погружается на глубину Н = 1 м. Найти изменение потенциальной энергии шарика Д£п- Плотность глицерина Pi= 1260 кг/м3, стекла р2 =2,4 • 103 кг/ма.
392.	На дне водоема стоит бетонная колонна диаметром d = = 0,4 м и высотой ft = 2м. Определить силу, с которой колонна давит на дно, если глубина водоема И = 3 м, атмосферное давление р = 105 Па.
393.	Сможет ли удержать человека плоская льдина, имеющая высоту ft = 0,3 м и горизонтальную поверхность площадью 5 = = 1 м2, если масса человека m = 60 кг? Плотность льда р = = 900 кг/м3. Если удержит, то какова высота выступающей над водой части льдины?
394.	Пароход, войдя в гавань, выгрузил часть груза; при этом его осадка уменьшилась на h = 0,6 м. Сколько груза оставил пароход в гавани, если площадь сечения парохода на уровне ватерлинии S = 5400 м2?
395.	Стеклянный стакан, имеющий наружный диаметр диа d = = 4 см и высоту h = 10 см, плавает в воде, погрузившись в нее до половины. Сколько воды нужно налить в стакан, чтобы он полностью погрузился в воду?
396.	Льдина постоянной толщины плавает в воде, выдаваясь над поверхностью на h = 2 см. Какова масса льдины, если ее площадь 5 = 150 см2? Плотность льда рл = 920 кг/м3.
397.	В море плавает льдина, часть которой объемом V = 195 м3 находится над водой. Определить объем всей льдины и ее подводной части. Плотность льда 0,9 • 103 кг/м3, морской воды р„ = 1,03 х X 103 кг/м3.
398.	Каким может быть наибольший объем Кл льдины, плавающей в воде, если известно, что алюминиевый брусок объемом V = = 0,1 м3, примерзший к льдине, заставляет ее тонуть?
399.	Бревно длиной I = 3,5 м и диаметром d = 0,3 м плавает в воде. Какой массы М человек может стоять на бревне, не замочив ног? Плотность дерева р = 700 кг/м3.
400.	В бассейн с водой погружен и находится в неподвижном состоянии опрокинутый вверх дном цилиндрический сосуд высотой h — 1 м (рис. 141). Сосуд полностью заполнен маслом с плотностью рм — 900 кг/м3. Найти давление в сосуде непосредственно под его дном в точке А, если нижний открытый конец цилиндра нахо
105
дится на глубине И = 3 м от поверхности воды в бассейне. Атмосферное давление р = 105 Па.
401.	Уровень воды в резервуаре водонапорной башни находится на высоте Н — 30 м от поверхности водоема. Определить давление в водонапорной трубе на высоте h = 20 м. Атмосферное давление р0 = 105 Па.
402.	В длинной вертикальной трубе с гладкими стенками, опущенной одним концом в воду, на высоте h = 10 м плавает легкий поршень (рис. 142). Площадь поршня 5 = 100 см2. Какую работу
следует совершить, чтобы поршень медленно поднять до высоты Н = 20 м? Атмосферное давление pQ= 105 Па.
403.	Гидравлический подъемник поднимает автомашину массой т — 2000 кг. Сколько ходов совершает малый поршень за t = = 1 мин, если за один ход он опускается на h = 25 см? Мощность двигателя подъемника N = 250 Вт, к. п. д. г) =75 %. Площади поршней = 100 см2 и S2— 2000 см2.
404.	Цилиндрическую гирю, подвешенную к динамометру, опускают в воду (рис. 143), пока уровень воды в сосуде не изменится на A/i = 8 см. Показание динамометра при этом изменяется на АГ = = 0,5 Н. Определить площадь поперечного сечения сосуда.
405.	Каким должен быть минимальный объем баллона, наполненного водородом, чтобы он мог поднять человека массой т = = 70 кг на высоту h = 100 м за t = 30 с? Масса корзины = = 20 кг.
406.	К концу однородной палочки, масса которой М = 4,4 г, подвешен на нити алюминиевый шарик радиусом R = 0,5 см
106
(рис. 144). Палочку кладут на край стакана с водой, добиваясь такого положения равновесия, при котором погруженной в воду оказывается половина шарика. Определить, в каком отношении делится длина палочки точкой опоры.
407.	Под чашкой рычажных весов на тонкой нити подвешено тело. Масса уравновешивающих гирек тг = 50 г. Когда тело опустили в воду, потребовались уравновешивающие гирьки массой ш2 = 30 г. Определить плотность вещества р тела.
408.	Тонкий однородный цилиндрический стержень верхним концом прикреплен к шарниру. Нижняя часть стержня погружена в воду, причем равновесие достигается тогда, когда стержень расположен наклонно к поверхности воды и в воде находится половина стержня. Какова плотность материала, из которого сделан стержень?
II. МОЛЕКУЛЯРНАЯ ФИЗИКА. ТЕПЛОВЫЕ ЯВЛЕНИЯ
§ 8. Основы молекулярно-кинетической теории. Законы идеального газа
Задача 1. Вычислить массу одной молекулы: азота (N2), аммиака (NH3), ацетилена (С2Н2) и ацетона (С3НвО).
Решение. Масса молекулы любого вещества равна т0 = 1,66 х X 10'27Л4г — 1,66  10~24 Л4, где Л4Г — относительная молекулярная масса, а М — молярная масса вещества. Поскольку молярная масса азота Mn2 = 28 • 1СГ3 кг/моль, аммиака = 17 • 10'3 кг/моль, ацетилена Л4с,н, = 26 • 10“3 кг/моль и ацетона Л4с,н,о = 58 х X 10-3 кг/моль, то массы молекул этих веществ соответственно равны = 4,66 • IO’20 кг; mNH1 = 2,83 • 10“2в кг; шс,н, = 4,32  Ю'28 кг;
Я1с,н,о = 9,66  10"26 кг.
Задача 2. Сколько атомов содержится в 1 г: а) гелия; б) углерода; в) фтора; г) полония?
Решение. В теле массой т содержится v = молей веще-
ства, где Л4 — молярная масса. С другой стороны, число молей в
N теле равно v = тт-
где N— число молекул в теле, Мд = 6,02 х
X 1023 моль-1 — постоянная Авогадро. Тогда число атомов (молекул)
в теле массой т равно М = ~Мд. Таким образом, в 1 г содер
жится атомов:
а) 1,5 • 1023; б) Ас = 5,01 • 1022; в) MF = 3,17  1022; г) 7УРо = 2,87 • 1021.
Задача 3. Где больше содержится атомов: в стакане воды или стакане ртути? Молярная масса воды Afj= 18 • 10 3 кг/моль, ртути М2= 201 • 10~3 кг/моль.
Решение. В теле массой т находится А — д молекул.
Тогда отношение числа молекул в стакане ртути к числу молекул
107
в стакане воды
N2 _ m2NAM± =	p.jAl,
/Vi m1NAM2 P1VM2	P1M2^1’^
Поскольку молекула воды состоит из трех атомов, а ртути — из одного, то в стакане воды атомов содержится примерно в 2,5 раза больше.
Задача 4. Камеру установки для термоядерных реакций откачивают до разрежения порядка 10“п Па. Сколько молекул п0 содержится в 1 см3 при таком давлении и температуре tQ = 27 °C?
Решение. Количество молекул в моле вещества равно числу Авогадро Ад. Согласно уравнению состояния pV = vRT, число мо-pV
лей в объеме V равно v — рр  Тогда число молекул в единице
,	na pna о л ,пз _3
объема п0= v-y- = ж 2,4 • 10J см J.
Задача 5. Кристаллическая решетка железа при комнатной температуре— кубическая объемно центрированная. Это означает, что элементарной ячейкой является куб, во всех вершинах которого, а также в центре — на пересечении пространственных диагоналей — находятся атомы железа. Сколько атомов приходится на объем, равный объему одной элементарной ячейки в кристалле железа? Определить минимальное расстояние между атомами железа в кристалле, зная его молярную массу М — 56 • 10-3 кг/моль и плотность р = 7870 кг/мэ.
Решение. Каждый из атомов, находящихся в вершинах куба со стороной а, принадлежит одновременно восьми ячейкам. Поэтому на объем, равный объему одной элементарной ячейки, приходится 2 атома (один из них находится в центре куба). Этот объем равен
I/ = а3 =	 Минимальное расстояние между атомами
Задача 6. Какое количество кислорода выпустили из баллона емкостью V = 10 л, если при этом показания манометра на баллоне изменились от 14 • 105 до 7 • 105 Па, а температура понизилась от /1 = 27 °C до /2 = 7 °C?
Решение. Масса выпущенного из баллона газа Ат равна разности между начальной массой т1 кислорода в баллоне и его конечной массой tn2. Ат =	— т2. Поскольку условия, при ко-
торых кислород находится в баллоне, не слишком отличаются от нормальных, газ можно считать идеальным. Уравнение состояния идеального газа для начального и конечного состояний газа в баллоне
P,V = ”J КГ, и p,V = S откуда
108
Чтобы найти давления газа в баллоне рг и р2, нужно прибавить к показаниям манометра значение атмосферного давления, равное 105 Па. Тогда Am « 82 г.
Задача 7. Какова разница в массе воздуха, заполняющего помещение объемом V = 50 м3 зимой и летом, если летом температура поднимается до = 40 °C, а зимой падает до t2 = 0 °C? Атмосферное давление нормальное (ра= Ю5 Па). Молярная масса воздуха М = 29 • 10'3 кг/моль.
Решение. Массу воздуха в объеме V зимой при t2 = 0 °C можно найти из уравнения состояния т2 = —. Масса воздуха в том же объеме летом при 1\ = 40 °C т1 =	. Разность масс
Ат — т2 — т1 =
_ pVM ,4 _ 1\
R \Т3 tJ ~ 8,2 кг'
Задача 8. Посредине откачанной и запаянной с обоих концов горизонтальной трубки длиной L ~ 1 м находится столбик ртути высотой h = 20 см. Если трубку поставить вертикально, то столбик ртути сместится на I = 10 см. До какого давления была откачана
L-h
2 ~  h
a	S
Рис. 145
трубка?
Решение. При горизонтальном положении трубки (рис. 145,а) в обоих ее концах газ занимает одинаковый объем Vr——?—
где S — площадь поперечного сечения трубки, и находится под одинаковым давлением рг. Когда трубку поставили вертикально (рис. 145, б), объем воздуха в верхней части трубки увеличился и стал Va = (-7^ + I'j S при давлении р2. В нижней части трубки объем газа будет К3 =	— i'jS, а давление р3. Давление воздуха в
трубке изменяется изотермически.
Согласно закону Бойля—Мариотта:
для верхней части трубки ргУг = p2V2; pY (—— I 5 = р2 ----F
+ z)
S или
Pi (L—h) = p3(L — h -I- 2Z);
(1)
для нижней части трубки
PiVt = p3V3; р^ (A - h) = p3(L -h- 21).	(2)
109
С другой стороны, столбик ртути находится в равновесии, если н любом его сечении силы, действующие сверху и снизу, равны. Выберем для простоты сечение у нижнего края столбика. Тогда
p3S = p3S + pghS.
Исключив из уравнений (1), (2) и (3) р2 и р3, найдем
=	= 5 • 10* Па.	(9)
Задача 9. Стеклянная трубка длиной L = 125 см расположена вертикально запаянным концом вниз. Доходящий до открытого верхнего края трубки столбик ртути высотой h отделяет находящийся в ней воздух от атмосферы. Трубку осторожно переворачивают открытым концом вниз, при этом часть ртути выливается. Найти высоту оставшегося в трубке столбика ртути, если атмосферное давление /?а = Ю5 Па. Решили задачу при a) h = 60 см; б) 1г = 40 см.
Решение. Согласно закону Бойля—Мариотта, для столбика воздуха, заключенного в трубке, р^ = p3V3. В исходном состоянии давление воздуха в трубке р,= ра+ pgh, или Pi= pg (Н + h). Объем, занимаемый воздухом в этом случае, Р1= (L — h)S.
Обозначим через х высоту столбика ртути, оставшейся в трубке после переворачивания. Тогда р2 = Р» — Р£х — Р£ (Н — х) и V2=(L-x)S.
Подставив значения давлений и объемов в выражение закона Бойля—Мариотта, получим уравнение для нахождения величины х:
Pg(H + h)(L — h)S = pg(H — x)(L — x)S, или x2— (H-]-L)x + + h(H +h — L) = O.
„	Я + дГ, , i/",	4Д (// + ft - L) 1
Корни этого уравнения Xi,2 = ——[1 ± у 1--------(н~+ 1 у—J’
а)	При h = 60 см получаем x} 197 см и x2 x 3 см. Очевидно, что первый корень не удовлетворяет условию задачи — высота оставшегося в трубке столбика ртути не может превышать его первоначальную высоту (тем более не может превышать длину всей трубки). Второй корень отвечает поставленному в задаче вопросу.
б)	При h = 40 см корни принимают значения xt« 202 см и х2 = = —2 см. Первый корень не удовлетворяет условию задачи по тем же причинам, что и в предыдущем случае. Второй корень также не подходит, так как высота столбика ртути не может быть отрицательной. Таким образом, при h =40 см задача не имеет решения. Физически это означает, что ртуть вытекает из трубки не частично, а полностью.
Задача 10. На рис. 146, а в координатах р, V изображен замкнутый газовый процесс (цикл). Изобразить этот процесс в координатах V, Т и р, Т.
Решен ие. Рассмотрим процессы, происходящие с газом. На рисунке линия 1—2 соответствует изохорическому процессу (V =
НО
= const) повышения давления от до р2, причем ясно, что температура также повышается. Линия 2—3 изображает изотермическое расширение при Т = const от У3 до У3, причем давление уменьшается от р2 до pt= ps. Линия 3—1 соответствует изобарическому (р = const) сжатию от Va до — Vlt причем 7\ < Т3.
На рис. 146, б изображен цикл в координатах V, Т. Линия 1—2 соответствует изохорическому процессу, причем температура растет от 7\ до Т2 (Т2 > Т\). Линия 2—3 изображает изотермическое расширение от V3 до Vs при Т — const, линия 3—1 — изобарический процесс. Продолжение этой линии должно пройти через начало координат.
На рис. 146, в изображен цикл в координатах р, Т. Линия 1— 2 — изохорический процесс; продолжение этой линии должно
пройти через начало координат, причем рг < р2. Линия 2—3 — изотермическое расширение, при котором давление уменьшается от р2 до Pi — Рз- Линия 3—1 — изобарический процесс, температура уменьшается от Т3 до 7\.
Задача 11. С данным количеством идеального газа проводят круговой процесс (цикл), изображенный на диаграмме (рис. 147) в виде окружности. На каком участке цикла объем газа увеличивался и на каком уменьшался?
Решение. Согласно уравнению Менделеева — Клапейрона, каждой точке координатной плоскости (р, К) соответствует однозначно определенный объем данного количества газа. Для определения характера изменения объема газа в данном круговом процессе достаточно «рассечь» его семейством изохор, как показано на рисунке. Так как наклон изохор тем больше, чем меньше объем газа (это следует из уравнения Менделеева — Клапейрона и условия V = const для изохорного процесса), то очевидно, что в состоянии 1 объем газа минимален, а в состоянии 2 — максимален. Таким образом, на участке цикла 1 -> 2 объем газа увеличивается, а на участке 2 -> 1 — уменьшается.
Задача 12. В цилиндре длиной L и площадью поперечного сечения 5, разделенном тонкой подвижной перегородкой, находятся два газа — в одной части кислород массой тг при температуре Т\, в другой — водород массой т2 при температуре Т2. Определить,
ш
в каком отношении перегородка делит цилиндр при названных условиях. Определить также давление, которое установится при разрыве перегородки и последующем нагревании смеси до температуры Т.
Решение. В цилиндре перегородка будет перемещаться до тех пор, пока давление с обеих сторон не станет одинаковым, т. е. Pi= р2. Из уравнения состояния
m, R7\	m., RT„
Объемы, занимаемые газами: V2 = (L— x}S и V2—xS. С учетом равенства давлений получим
ftZj RT,	/л2 RT2
ЛГ4 (L — x)S — ~xS~ •
Отсюда легко найти отношение, вкотором перегородка делит цилиндр,
L — х__mlM2T1
х	'
После разрыва перегородки давление в цилиндре будет создаваться смесью кислорода и водорода: р = р\ + р2, где р\ и р2 — парциальные давления кислорода и водорода. Парциальное давление кислорода, находящегося в объеме V\4- V2= LS при температуре Та, р{ — ^‘^3 . Парциальное давление водорода, находяще-
, m.,RT3 т
гося при тех же условиях, р2 = Л , /  1огда давление, создавае-IVl
мое кислородом и водородом в объеме LS при температуре Т3,
р ls mJ-
Задача 13. Определить молекулярную формулу некоторого углеводорода, если его плотность в газообразном состоянии р = = 0,64 кг/м3 при температуре t°= 27 °C и давлении р = 106 Па.
Решение. Пусть СХНУ — молекулярная формула углеводорода. Тогда его молярная масса М= 12%+;/. Согласно уравнению Менделеева—Клапейрона, М — ^г- Учитывая, что = р, полу-чим Л4 = р — » 16. Подставляя это значение М в уравнение М = = 12/ + у и решая его подбором, находим х = 1 и у = 4. Таким образом, искомая формула — СН4 (метан).
Задача 14. Одинаковые по массе количества водорода и гелия поместили в сосуд объемом который отделен от пустого сосуда объемом V2 полупроницаемой перегородкой, свободно пропускающей молекулы водорода и не пропускающей гелий. После установления равновесия давление в первом сосуде упало в п = 2 раза. Опре-V
делить отношение -рЛ. Температура постоянна.
112
Решение. Условием равновесия является равенство парциальных давлений водорода в первом и втором сосудах. Пусть р — начальное давление в первом сосуде. Оно состоит из парциальных давлений гелия и водорода. Поскольку массы и температуры газов одинаковы, отношение давлений гелия и водорода обратно отношению их молярных масс. Таким образом,
,	₽Не All 1	1	2
РНе + РН2 = Р, Т =М =2’ 0ТКУДа ^Не = з Р, РН2 = 3 Р-
Чтобы суммарное давление гелия и оставшейся части водорода в первом сосуде было вдвое меньше начального, парциальное дав-1 п ление водорода должно уменьшиться до значения рн£ = -^р- Поскольку давление водорода обратно пропорционально объему, кото-
Рис. 148
рый он занимает, получаем
Рщ V, + и Уг _ о
₽Н2	V1
Задача 15. Из сосуда объемом V, давление в котором р, откачивают воздух при помощи поршне
вого насоса с рабочей камерой объемом V. Сколько качаний должен сделать поршневой насос для того, чтобы давление в сосуде упало в k раз? Температура воздуха не изменяется.
Решение. После каждого хода поршня часть массы воздуха удаляется из сосуда и «-кратное увеличение объема воздуха на и относится к разной массе газа. До того, как поршень сделал первое «качание», для воздуха в сосуде можно записать рУ = RT. Когда поршень насоса переместится в крайнее правое положение (рнс. 148), объем газа равен V + и, давление р1( причем Pi (У + и) — откуда рг = р v  При движении поршня в крайнее левое положение клапан А закрывается, в сосуде остается меньшая масса воздуха, а из объема цилиндра часть воздуха удаляется. Для оставшейся массы воздуха при втором ходе поршня вправо запишем: prV = р2(К + и), откуда давление воздуха в со-
V / V V д суде после второго «качания» р2 = рг ц = р 1р . Аналогично найдем, что после п качаний давление станет равным рп =
( V V г,	1	. 1	/ V \п
= р • По условию задачи, рп = у р, поэтому у =	•
Логарифмируя этот результат, получим
_ 1g (1/fe)	__ Igfe
'lg(V/(V + 0 1g ((V + o)/V) ‘
М3
Задача 16. Воздушный шар, представляющий собой баллон постоянного объема и имеющий в нижней части отверстие для сообщения с атмосферой, наполнен гелием и поднимается до максимальной высоты hx. Как изменится максимальная высота подъема шара, если гелий нагреть до температуры 7\ и поддерживать ее постоянной во время полета? Температуру атмосферы считать постоянной и равной То, а давление — изменяющимся по закону р ~ р0 (1 — ah), где а — постоянная величина, h — высота подъема, ра— давление у поверхности Земли.
Решение. В первом случае, когда шар поднимается до высоты hlt
m^g + pnegV = pBgV,	(1)
где m06 и V — масса и объем оболочки, рце и рв — плотности гелия и воздуха. Из уравнения Менделеева — Клапейрона следует, что _ Л,вР1
РИе — RTo И Рв — RTo •
Здесь Pi— давление атмосферы на высоте й/, гелий внутри шара имеет температуру окружающего воздуха То. Поскольку = = р0(1 — ah^, т0 выражение (1) принимает вид
МНеРо (> — «М) У __ Л,в₽0 (> — «А1) V т°б	RT~B ~ RT0 ’
РоО —а’т-д v (Л^в — МНе) откуда mos --------------------------•
(2)
Во втором случае, когда гелий нагрет до температуры Тх,
moag + рщ£И = PogK.	(3)
С учетом того, что
pHe =	’ Рв = И Р2 = Ро(1 ~ aft2)-
л/1	к/о
перепишем выражение (3):
, МНер„ (1 — afli) v __ мъР<> 0 — стйа) У
mo6 + RT^	RTo ,
откуда
_ Ро (1 — «М v Iм* _	(4)
m°6 — R \Т„ Ti Г	( ’
Приравняв правые части (2) и (4), получим выражение для Л2:
^MUe(Ti-TQ)+hiTi (Мв-МНе)
Ла=	MBTi-MtkTa
откуда
.,	,	, Л4Не (Т, — Го) (1 — a/!j)
- «2 —	- а (MJi - MHsT0)
114
ЗАДАЧИ ДЛЯ САМОСТОЯТЕЛЬНОГО РЕШЕНИЯ
409.	Сколько молекул находится в сосуде объемом V = 1 см3 при	10,°C, если давление в сосуде р = 1,3 • 10~9 Па?
410.	Какое количество вещества содержится в одном стакане воды? Масса воды в стакане т = 0,2 кг. Каково число молекул воды в стакане?
411.	Сколько молей газа содержится в сосуде объемом V = 10 л при /°= 27 °C, если давление газа р — 106 Па?
412.	В баллоне объемом V = 0,04 м3 под давлением р = 1,5 X X 10s Па при Т = 300 К находится газ. Сколько молекул газа находится в баллоне?
413.	Сколько молекул водорода содержится в объеме V = 1 м3 при нормальных условиях? Какова масса одной молекулы водорода?
414.	Сколько молекул воздуха выходит J-из комнаты объемом Ко при повышении	1 777
температуры от 7\ до 7\? Атмосферное дав-	л %%
ление р0,	_
415.	В вакуумной камере ускорителя	
заряженных частиц давление газа р =	““з-
= 10"в Па. Сколько молекул газа содер-	_ ь
a q	Q	п
жится в 1 см3 при указанном давлении
и температуре 1°= 27 °C?	рИс. 149
416.	Сухой атмосферный воздух состоит
из кислорода, азота и аргона (доля остальных газов мала). Определить массу и число молекул этих газов в объеме V = 1 м3 атмосферного воздуха при нормальных условиях. Парциальные давления газов соответственно равны р} = 2,1  104Па, рг= 7,8 х X 104 Па; р3= 103 Па.
417.	Чему равно атмосферное давление ра, если при длине ртутного столбика h = 12,5 см в тонкой трубке (рис. 149) длина столбика воздуха в первом положении (а) равна 1г = 7 см, а во втором (б) — 12 = 5 см?
418.	Узкую цилиндрическую запаянную с одного конца трубку длиной I = 90 см погружают открытым концом в сосуд с ртутью на глубину h = 75 см. Определить разность уровней ртути в трубке и вне ее, если атмосферное давление ра — 9,97  104 Па.
419.	Стеклянную трубку длиной I = 1м наполовину погружают в ртуть, затем закрывают сверху и вынимают. Какой длины h столбик ртути останется в трубке? Атмосферное давление р0 = 10,1 X X 104 Па.
420.	В узкой трубке, один конец которой запаян, находится столбик ртути длиной Z = 15 см, запирающий небольшой объем воздуха. Когда трубка стоит вертикально отверстием вверх, то объем воздуха под столбиком ртути Уг= 1 см3. Каким будет объем этого воздуха при горизонтальном положении трубки? Атмосферное давление ра — 9,97  104 Па; температура воздуха постоянная.
115
421.	Цилиндрическая пробирка длиной Z = 30 см, содержащая воздух при Т = 300 К, полностью погружена открытым концом в глицерин. При этом воздух занимает половину объема пробирки. Пробирку вынимают из глицерина так, что она открытым концом едва касается поверхности жидкости. Как и на сколько кельвинов следует изменить температуру воздуха в пробирке, чтобы он по-прежнему заполнял половину ее объема? Внешнее давление рй = = 105 Па.
422.	Цилиндрическая трубка с запаянным верхним концом вертикально опускается в ртуть так, что запаянный конец совпадает с поверхностью ртути в сосуде. При этом высота воздушного столба в трубке h. Определить длину трубки I. Атмосферное давление р, температуру считать постоянной.
423.	В запаянной с левого конца £7-образной трубке высота столбика воздуха /0 = 300 мм, а избыточная высота столба ртути в правом колене h0= 110 мм. В правое колено долили столько ртути, что ее уровень в левом колене поднялся на I = 10 мм. На сколько поднялся уровень ртути в правом колене? Атмосферное давление ра = Ю5 Па. Температура постоянна.
424.	В трубке, запаянной с одного конца, находится столбик ртути длиной I = 0,3 см. Трубку вращают в горизонтальной плоскости вокруг оси, проходящей через ее закрытый конец. При какой угловой скорости вращения ртуть достигнет открытого конца, если в неподвижной горизонтальной трубке она находится на расстоянии d = 64 см от закрытого конца? Длина трубки b = 80 см. Внешнее давление р0 = 105 Па. Температура постоянна. Капиллярными эффектами пренебречь.
425.	Цилиндрический сосуд длиной 1г = 0,2 м, заполненный газом при t°\ = 36 °C, полностью погрузили вверх дном в жидкость, плотность которой р = 800 кг/м3. Жидкость внутри сосуда поднялась до половины его высоты. При какой температуре газа уровень жидкости внутри сосуда сравняется со свободным уровнем жидкости, если сосуд приподнять из жидкости на /2 — 0,12 м? Атмосферное давление р =9,4 • 104 Па.
426.	В стеклянной вертикально расположенной трубке длиной I = 1 м, заполненной газом при t°= 40 °C и запаянной с верхнего конца, находится столбик ртути длиной /j= 10 см (столбик находится вблизи нижнего конца трубки). На сколько сместится столбик ртути, если трубку перевернуть и опустить в тающий лед? Атмосферное давление р = 9,5 • 104 Па.
427.	Узкую цилиндрическую запаянную с одного конца трубку длиной I = 45 см погружают открытым концом в сосуд с ртутью на глубину h = 40 см. Какой будет высота столбика ртути в трубке? Атмосферное давление р0 = 105 Па.
428.	Электрическую лампу при изготовлении наполняют азотом под давлением 5  104 Па и при = 15 °C. Какова температура газа в горящей лампе, если давление в ней повысилось до р2 = = 1,1 • 106 Па?
116
429.	В кабине космического корабля «Восток-2» были созданы атмосферные условия, близкие к нормальным. Температура в кабИ' не во время полета колебалась в пределах 10—22 °C. На сколько процентов при этом изменялось давление?
430.	Когда объем газа уменьшили на а = 10 %, а температуру увеличили на А/° = 16 °C, его давление возросло на /3 = 20 %. Определить начальную температуру газа.
431.	На рис. 150 приведены две изобары. Одна соответствует давлению ръ другая — давлению р2. Какое давление больше? Масса газа постоянная.
432. Сосуд разделен перегородкой на две части с объемами Kj
и К2 . В первой находится газ под давлением р1; а во второй — другой газ под давлением р2. Какое давление установится в сосуде
после удаления перегородки? Температура поддерживается постоянной.
433.	Когда газ, объем которого оставался неизменным, нагрели на АТ = 30 К, его давление увеличилось на 10 %. Какова начальная температура газа?
434.	При нагревании некоторой массы газа на АТ = 1 К при постоянном давлении , - 1 объем этой массы газа увеличился на п =350 часть первоначального объема. Найти началь
ную температуру газа.
435.	Стальной баллон наполнен азотом при t° = 12 °C. Давление азота р = 15 • 10° Па. Найти его плотность при этих условиях. При какой температуре давление возрастет до р2 — 18  10е Па? Расширением стенок баллона пренебречь.
436.	Баллон содержал сжатый газ при tr = 27 °C и давлении р1= 4 • 10е Па. Каким будет давление р2 в баллоне, если из него ,0 выпустить половину массы газа, а температуру снизить до /2 = = 12 °C?
437.	Цилиндрический сосуд длиной I = 1 м делится на две части подвижным поршнем. Каково будет равновесное положение поршня, если в одной части сосуда находится некоторая масса кислорода, а в другой — такая же масса водорода?
438.	Два баллона, содержащие один и тот же газ, соединили трубкой с краном. В первом баллоне объемом К1=3 л давление газа р1= 8 • 10s Па, во втором объемом V2 = 5 л р2 = 6 • 105 Па. Какое давление установится в баллонах, если открыть кран? Температура постоянная. Объемом трубки можно пренебречь.
439.	Со дна водоема глубиной И — 80 м поднимается вверх шарообразный пузырек воздуха. На какой глубине h радиус этого пузырька увеличится в k = 2 раза? Атмосферное давление рй — = 106 Па. Температуру считать постоянной.
440.	Резиновая камера содержит воздух при = 27 °C и нормальном давлении. На какую глубину нужно опустить камеру
117
в воду, чтобы ее объем уменьшился вдвое? Температура воды t\ = = 4 °C.
441.	Тонкий резиновый шар радиусом гг = 2 см наполнен воздухом при Zj = 20 °C и нормальном атмосферном давлении р0 = 106 Па. Каким станет радиус шара, если его опустить в воду с температурой t°2 = 4 °C на глубину h = 20 м? Натяжением резины пренебречь.
442.	Полый шар с жесткой оболочкой, масса которой т = Юг, наполнен водородом. Объем водорода V = 10 л. Температура водорода и окружающего шар воздуха t°— 0 °C. Найти давление водорода в шаре, если подъемная сила шара равна нулю. Атмосферное давление р№ = 10s Па.
443.	Газ находится под поршнем в горизонтальном цилиндрическом сосуде. Поршень может передвигаться в цилиндре без трения. Атмосферное давление р0 = 105 Па. Объем газа V =50 л. С какой силой надо подействовать на поршень, чтобы объем газа уменьшился до К, = 10 л? Площадь поршня 10 см2. Сжатие газа изотермическое.
444.	По газопроводу течет углекислый газ при давлении р = = 5 • 105 Па и температуре t0° = 17 °C. Какова скорость движения газа, если за т = 5 мин через поперечное сечение трубы проходит т = 2,5 кг углекислого газа? Площадь поперечного сечения трубы 5=6 см2.
445.	Из баллона объемом V = 10“3 м3, наполненного водородом, происходит утечка газа. Вначале давление в баллоне было р,= = 1,5  107 Па, а температура 1° = 17 °C. Через некоторое время давление упало до р2 = 1>2 • Ю7 Па, а температура повысилась до /"=27 °C. Найти массу вытекшего газа.
446.	Два одинаковых сосуда соединены трубкой, объемом которой можно пренебречь. Во сколько раз изменится давление в системе, если один сосуд нагреть до температуры Т, а другой оставить при первоначальной температуре То?
447.	В двух сосудах находится один и тот же газ. Объемы сосудов и К2, давление и температура газа в сосудах р}, Т} и р2, Т2 соответственно. После соединения сосудов в них установилась температура Т. Найти давление в соединенных сосудах.
448.	На рис. 151 изображен график процесса, проводимого с идеальным газом. Объем газа постоянен. Найти точки, где масса газа максимальна и минимальна.
449.	На рис. 152 дан график изменения состояния идеального газа в координатах (V, Т). Представить этот процесс на графиках в координатах (р, V) и (р, Т). Масса газа постоянная.
450.	Как изменялся объем газа в зависимости от температуры в процессе, изображенном на рис. 153?
451.	Как изменялся объем данной массы газа в зависимости от температуры в процессе, изображенном на рис. 154?
452.	Построить график зависимости плотности водорода от температуры при изобарном нагревании. Газ находится под давлением р = Ю5 Па.
на
453.	График изменения состояния одного моля идеального газа в координатах (р, Т) имеет вид полуокружности (рис. 155), причем точка А имеет координаты рд и Та и лежит на середине отрезка ОВ. Определить объемы VA и Vb в состояниях А и В.
454.	На рис. 156, а точки А, В, С изображают в координатах (р, Т) три состояния одной и той же массы газа. Как изменялся объем газа при переходе из состояния А в состояние В и затем в С?
119
Аналогично на рис. 156, б точки А, В, С изображают состояние газа на (V, Т)-диаграмме. Как изменялось давление газа при переходе от А до Б и до С?
455.	С идеальным газом проводят процесс 1 — 2 — 3 — 4 (рис. 157). В некотором состоянии 3 на отрезке, изображающем процесс 2 — 3 — 4, давление газа такое же, как и в состоянии 1. Известно, что V-i— 1 ма, V2 =- 4 ма, Т\ = 100 К, 7\ = 300 К- Каков объем газа в состоянии 3?
Рис. 160	Рис. 161
456.	На рис. 158, а и б приведены замкнутые циклы некоторой массы газа. Изобразить эти процессы на (р, К)-диаграмме на фоне семейства изотерм.
457.	На рис. 159 изображен замкнутый цикл для идеального газа. Кривая 3 — 1 - изотерма. Изобразить этот цикл в координатах (Т, V) и (р, Т).
458.	На рис. 160 показан замкнутый цикл некоторой массы газа на (р, 7’)-диаграмме. Изобразить этот цикл на (р, 1/)-диаграмме на фоне семейства изотерм.
459.	На рис. 161 изображен замкнутый цикл некоторой массы газа на (р, К)-диаграмме. Изобразить этот цикл на (р, ^-диаграмме.
120
460.	Некоторая масса молекулярного водорода занимает объем У1== 1 м3 при 7\ = 250 К ир1=2 • 105 Па. Какое давление будет иметь та же масса водорода при Т2 = 5000 К в объеме V2= 10 м3, если при столь высокой температуре молекулы водорода полностью диссоциируют на атомы?
461.	Вертикальный прямой цилиндр закрыт сверху тяжелым поршнем. В цилиндре находится кислород, масса которого m = — 2 • 10 2 кг. После нагревания на ДТ = 100 К поршень площадью Д = Ю-2 м2 поднялся на h. — 0,14 м. Определить массу т} поршня, если внешнее давление р0 = 10s Па во время процесса не изменялось. Трение поршня о стенки цилиндра не учитывать.
462.	Масса банлона объемом V, содержащего газ при давлении р и температуре Т, равна tn. После откачивания баллона при температуре Т до давления рг его масса стала тг. Определить плотность газа при нормальных условиях (р0 = Ю5 Па, То = 273 К).
463.	Из баллона объемом V\= 200 л, содержащего гелий при 7\=273 К под давлением р1=2 • 10е Па, израсходовали часть газа, занявшего при нормальных условиях объем 1/2= 1000 л. При повторном измерении давления в баллоне получено р2 = 1,4 • 10*’Па. При какой температуре проведено это измерение?
464.	В баллоне объемом V = 10 л содержится водород при t° = 20 °C под давлением р = 107 Па. Какое количество водорода было выпущено из баллона, если при полном сгорании оставшегося образовалось m = 50 г воды?
465.	В баллоне объемом V = 10 л находится гелий под давлением р, = 10е Па и при = 27 °C. После того как из баллона было взято m = 10 г гелия, температура в баллоне понизилась до t2= = 17 °C. Определить давление гелия, оставшегося в баллоне.
466.	В бомбе объемом V = 10 л содержится смесь водорода и кислорода в равных количествах (масса каждого m = 2 г). Весь кислород, соединяясь с частью водорода, образует воду. Каково давление оставшегося водорода при f= 17 °C?
467.	В теплоизолированный сосуд объемом V = 22,4 дм3, содержащий rtj = 1 моль водорода при 7\ = 200 К, добавляют т2= 4 г водорода. После установления равновесия давление в сосуде оказалось р =3 • 10s Па. Определить первоначальную температуру добавленного водорода.
468.	Плотность смеси азота и водорода при /°=47 °C и давлении р = 2 • 105 Па равна р = 0,3 кг/м3. Найти концентрацию молекул азота и водорода в смеси.
469.	Сосуд объемом V = 10“3 м3 содержит тх — 10 г азота и т2 _ 10 г водорода. Каково давление смеси газов при Т — = 300 К?
470.	Два сосуда соединены тонкой трубкой, содержащей капельку ртути, которая исключает перемешивание газов, находящихся в сосудах (рис. 162). В первом сосуде заключено т1= 2 г гелия, во втором — т2 — 4 г неона. Температура в обоих сосудах Тг. Во сколько раз увеличится давление в системе, если гелий нагреть до
121
Тг—2Т\, не меняя температуры неона? Расширения сосудов не учитывать.
471.	Из баллона выпустили Am = 2 г газа, в результате чего давление в нем упало на п =10 %. Определить емкость V баллона, если плотность газа в начальный момент составляла р = 0,2 кг/м3. Температура газа остается постоянной.
472.	Расположенный горизонтально закрытый цилиндр разделен на две части подвижной перегородкой. В одной части объемом Уг = 220 см8 имеется П1 = 2моля идеального газа при t° = — 53 °C. Сколько молей того же газа находится в другой части объемом К2 = = 300 см3, где температура /“=—13 °C? Перегородка находится в равновесии.
473.	Цилиндр разделен подвижным поршнем на две равные части объемом Vo= 1,3 л каждая. В одной части находится азот, а в другой — воздух. Темпе-eO ратура газов То = 300 К- На какое расстояние сместится поршень , если один газ нагреть на АТ = 50 К, а другой оставить неизменным?
Рис 162	Площадь поперечного сече-
ния цилиндра А = 100 см2.
474.	Сосуды с объемами V± = 200 см3 и V2= 100 см3, наполненные кислородом при t°= 27 °C под давлением р0 = 105 Па, соединены трубкой, внутри которой находится теплоизолирующая пористая перегородка, обеспечивающая одинаковость давлений в сосудах. Затем первый сосуд нагревается до 1° = 100 °C, а второй — охлаждается до /“ = 0 °C. Пренебрегая тепловым расширением сосудов, определить установившееся в системе давление.
475.	Пружина, прикрепленная верхним концом к крышке откачанного вертикального цилиндра, удерживает невесомый поршень
о
у самого дна цилиндра. В каком отношении р- поршень будет делить v н
объем цилиндра, если по обе стороны поршня ввести одинаковую массу одного и того же газа? Известно, что расстояние между дном и поршнем возрастет вдвое, если пространство над поршнем от газа освободить.
476.	Атмосферное давление на пике Ленина (высота 7134 м) р1= 3,8  104 Па. Определить плотность воздуха рх на вершине при /“ = —10 °C, если при нормальных условиях (t° = 0 °C и р0 = = 105 Па) р0 = 1,29 кг/м3.
477.	Лазерные трубки объемом Ко = 60 см3 должны заполняться смесью гелия и неона в молярном отношении 5: 1 при общем давлении = 800 Па. Имеются баллоны этих газов, каждый объемом V — 2 л. Давление в баллоне гелия рг = 6,65 • 103 Па, неона — р2= 2,7 • 104 Па. Какое количество трубок можно заполнить?
478.	Три баллона емкостью = 4 • 10"3 м3, К2 = 8 • 10~3 м3
122
и V3 = 10 2 м3 содержат соответственно водород под давлением р, = = 5  106 Па, кислород под давлением р2 = 1,5 • 106 Па и азот под давлением р3 = 3,4 • 106 Па. Какое давление установится в сосудах, если их соединить трубкой ничтожно малого объема? Температуру считать постоянной.
479.	В вертикально стоящем цилиндрическом сосуде со свободно движущимся (без трения) поршнем массой тп и площадью <$ находится некоторое количество идеального газа. При нагревании газа на АТ его объем увеличивается на Д1Л Найти массу т газа, если атмосферное давление постоянно и равно р0. Молярная масса газа М.
480.	Горизонтальный цилиндр с воздухом закрыт подвижным поршнем. Под действием горизонтально направленной силы поршень приблизился к основанию цилиндра на h = 30 см. Считая
Рис. 163
Рис. 164
процесс изотермическим, определить значение приложенной силы. Начальный объем воздуха в цилиндре Vo = 80 см3, а площадь основания цилиндра 3=2 см2. Атмосферное давление р0 = 106 Па.
481.	Определить плотность смеси, состоящей из mt=4 г водорода ит2= 32 г кислорода при /°= 7 °C и давлении р =9,3 • 104Па.
482.	Посредине цилиндра, герметически закрытого с обоих концов и закрепленного под углом а = 30 °к горизонту, находится поршень массой т = 1 кг (рис. 163). Площадь поршня S = 10 см2. Давление воздуха под поршнем и над ним р = 1,5 • 104 Па. С каким начальным ускорением будет двигаться поршень, если его предварительно медленно передвинуть, увеличив объем под ним в п =1,5 раза, а затем отпустить? Трением пренебречь.
483.	В вертикальном закрытом сверху и снизу цилиндре находится движущийся с ничтожным трением поршень. Над поршнем и под поршнем находятся одинаковые массы одного и того же газа при 7\ = 300 К. Сила тяжести поршня уравновешивается разностью сил давлений газа, если объем нижней части цилиндра в п = 3 раза меньше объема верхней части. Каково будет соотношение объемов nlt если температура повысится до Г2=400 К?
484.	Герметичный цилиндр разделен газонепроницаемым поршнем на равные части. В одной половине находится идеальный газ с молярной массой в другой — Л4а. Массы газов, их объемы и температуры одинаковы. Поршень освободили. На какую долю -у своего первоначального значения окажется увеличенным объем од
123
ной из частей цилиндра (или уменьшенным объем другой части) после установления равновесия при той же температуре?
485.	Закрытый цилиндр объемом V — 1 л разделен на две части подвижным невесомым поршнем. В одной части находится газ при ty = О °C, в другой — такая же масса данного газа при = = 100°С. Определить объемы Kj и частей цилиндра при равновесии поршня. Поршень и стенки сосуда теплонепроницаемы.
486.	Два одинаковых сосуда соединены тонкой трубкой с краном. В первом сосуде находится некоторое количество идеального газа при 1° = 27 °C, а во втором — иное количество того же газа при = 227 °C и том же давлении. Во сколько раз изменится давление газа, если открыть кран и довести температуру содержимого сосудов до /з =127 °C?
487.	Перед тактом сжатия давление в цилиндре двигателя внутреннего сгорания pY= 8 • 104 Па, а температура = 50 °C. Определить температуру смеси в конце такта сжатия, если при этом объем ее уменьшился в п = 5 раз, а давление увеличилось до р2 = = 7  10Б Па.
488.	В двух частях цилиндра, разделенных поршнем А, находятся разные массы тг и т2 воздуха при одной и той же температуре (рис. 164). Правый конец цилиндра закрыт подвижным поршнем В. На сколько сместится поршень А, если поршень В передвинуть вправо на b = 4 см? При равновесии в цилиндре устанавливается первоначальная температура. Отношение масс = 3.
489.	С какой максимальной силой прижимается к телу человека медицинская банка, если диаметр ее отверстия 0=4 см? В момент прикладывания банки к телу воздух в ней прогрет до t° = 80 °C, а температура окружающего воздуха = 20 °C. Атмосферное давление р0 = 106 Па. Изменением объема воздуха в банке (из-за втягивания кожи) пренебречь.
490.	Процесс в идеальном газе идет сначала так, что давление и объем связаны равенством pVV = В. Когда температура газа достигает значения Т, процесс продолжается при ином характере зависимости давления от объема: р = D/V2. Найти упомянутую температуру Т, если константы В и D, а также количество вещества п — m/М заданы.
491.	Идеальный газ находится при = 27 °C. Найти температуру этого газа, если в результате расширения, происходящего по закону pK3/i = const, объем газа увеличился в 4 раза.
492.	Газ занимает объем К = 2 л и находится под давлением р = 5 • 105 Па. Вычислить суммарную кинетическую энергию поступательного движения его молекул.
493.	Камеру объемом V = 10 л наполнили сухим воздухом при нормальных условиях и, закрыв после введения т 3 г воды, нагрели до t°= 100 °C. Определить давление р в камере.
124
494.	Цилиндрический сосуд высотой I = 40 см разделен на две части невесомым тонким поршнем, скользящим без трения. Поршень находится на высоте h = 26,7 см над дном цилиндра. Под поршнем находится водород, а над ним — газ с неизвестной молекулярной массой Мх. Определить Мх газа, если его масса равна массе водорода.
495.	В цилиндре под поршнем, масса которого М = 20 кг, находится т = 1 г гелия при Т = 400 К (рис. 165). Удлинение пружины А составляет х = 20 см, а энергия ее деформации Е = 60 Дж. Определить высоту поршня над дном цилиндра. Давлением газа вне цилиндра пренебречь.
496.	Длинный стержень с закрытым коротким цилиндром на конце вращается с угловой скоростью со = 20 рад/с в горнзонталь-
Рис. 165
ной плоскости вокруг
(рис. 166). В цилиндре, площадь поперечного сечения которого S = = 100 см2, находится воздух, разделенный поршнем массы т = = 490 г. При вращении расстояние от центра поршня до точки О равно г = 1,36 м. Найти отношение объемов воздуха в цилинд-
*2
ре при вращении, если известно, что в отсутствие вращения объемы одинаковы и давление воздуха в них р = 1,1 • 105 Па.
оси, проходящей через конец стержня О
497.	В чашечный ртутный барометр попал пузырек воздуха, вследствие чего барометр показывает давление меньше истинного. При давлении рх = 1,02 • 105 Па высота столбика ртути — = 748 мм, причем длина пустой части трубки I = 80 мм. Каково атмосферное давление р.2, если ртуть в этом барометре стоит на высоте h2 = 734 мм?
498.	В пространство над ртутным столбиком в трубке барометра попало немного воздуха. Когда барометр показывал давление рг = = 9,95 • 104 Па, длина цилиндрической части, занятой воздухом, была I = 60 мм. Трубку осторожно выдвинули из сосуда со ртутью еще на Д/г = 21 мм, после чего высота столбика ртути в барометре стала h2 — 751 мм. Считая температуру постоянной, определить атмосферное давление ря в момент опыта.
499.	Сколько ходов должен сделать поршень откачивающего насоса, чтобы откачать воздух из сосуда объемом V от атмосферного
125
давления р0 до давления р = 10-4 р0, если емкость насоса 1?0? Температуру считать неизменной.
500.	Давление воздуха в сосуде было р0 = 1,01 • 10Б Па. После трех ходов откачивающего поршневого насоса давление упало до р — 2 • 103 Па. Определить отношение объема сосуда к объему цилиндра поршневого насоса. Температуру воздуха в процессе откачки считать постоянной.
501.	При каждом ходе поршневой компрессор захватывает Ко = = 10 дм3 воздуха из атмосферы при нормальных условиях и нагнетает его в резервуар объемом V = 10 м3. Температура в резервуаре поддерживается Т = 323 К. Сколько ходов должен сделать поршень компрессора, чтобы повысить давление в резервуаре до р = Юр0, где р0 — нормальное атмосферное давление?
§ 9. Внутренняя энергия и механическая работа
Задача 1. Кусок льда массой гп = 2 кг при температуре /° = = — 20 °C нагрели, сообщив ему количество теплоты Q = 10е Дж. Определить температуру вещества после нагревания.
Решение. Особенностью таких задач является то, что неизвестно, в каком агрегатном состоянии будет находиться лед после нагревания. Поэтому сначала нужно выяснить этот вопрос, а затем уже использовать уравнение теплового баланса. Найдем количество теплоты необходимое для того, чтобы нагреть ш = 2 кг льда от —20 °C до 0 °C:
(Qj = Cntnt0 = 8,4 • 104 Дж.
Мы видим, что Qi < Q (<? = 10е Дж), поэтому весь лед нагреется до 0 °C и затем начнёт плавиться. Найдем количество теплоты Q2, необходимое, чтобы полностью расплавить лед, нагретый до 0°С:
Q2 = Xm = 6,8 • 10б Дж.
Общее количество теплоты, необходимое для нагревания всего льда до 0°C и его последующего полного плавления,
<?i + (?2= 7,64 • 10бДж.
Это значение также меньше количества теплоты Q = 10° Дж, сообщенного льду. Следовательно, весь лед расплавится и образовавшаяся вода будет затем нагреваться. Определим количество теплоты (?з, необходимое для нагревания 2 кг воды от 0 °C до t° = 100 °C: Q3 = cBmt° = 8,4 • 105 Дж.
Общее количество теплоты, необходимое для того, чтобы перевести 2 кг льда при Г = — 20 °C в воду при t° = 100 °C, равно
Qi+ Q2+ Qa = 1,6 • 10е Дж.
Qi+ Qa+ <2з > Q> значит, сообщенного льду количества теплоты
126
Q = 10е Дж достаточно для того, чтобы перевести лед в воду при некоторой температуре t°. Запишем уравнение теплового баланса:
n	n Q — cnmt° — Хт
Q = слт1" 4- 7.гп 4- свт/°, откуда	» 28,1 °C.
спт
Задача 2. В калориметр налито т1= 2 кг воды, имеющей температуру = 5 °C, и положен кусок льда массой т2 = 5 кг, имеющий температуру /2 = — 40 °C. Определить температуру 0° и объем V содержимого калориметра после установления теплового равновесия. Теплоемкостью калориметра и теплообменом с окружающей средой пренебречь. Плотность воды при О °C Pj= 103 кг/м3, льда — р2== 920 кг/м3.
Решение. Возможны четыре случая: 1) весь лед растает, и температура смеси будет 0°= 0 °C; 2) часть льда растает, и температура смеси будет 0° = 0°С; 3) вся вода замерзнет, и температура смеси будет 0° < 0°С; 4) часть воды замерзнет, и температура смеси будет 0°= 0 °C.
При охлаждении до 0° вода отдает количество теплоты Qx = = Cytiiy (Z®—0°) =4,2 • 104Дж, в то время как при нагревании до 0° лед поглощает количество теплоты Q2 = с2т2 (0°— /J) = = 4,2 • 105 Дж. Поскольку Q2 > Qlt то могут осуществляться лишь случаи 3 и 4. Если замерзнет вся вода, то она отдаст еще количество теплоты Q3= тД == 6,6 • 105 Дж. Так как Qj+ Q3 > Q2, т. е. теплового равновесия нет, то возможен лишь случай 4. Следовательно, температура смеси 0°= О °C.
Обозначим через тх массу замерзшей воды и составим уравнение теплового баланса схтх (tx — 0°) 4- rnxk = с2т2(6°— t°2), откуда
Таким образом, массы находящихся в калориметре при температуре 0°= 0 °C воды и льда будут соответственно равны	— тх и т2 4-
4- тх. Их объемы будут Vx = mi - -— и V2 = t а общий Р1	г2
объем V = Vx 4- К ==	? 4-	~ 7,54 . ИГ3 м3.
12 Pi	р2
Задача 3. Лыжник скатывается с горы, крутизна которой а, и проходит по горизонтальной поверхности до остановки расстояние s. Определить, сколько снега он при этом расплавит, если его масса т, коэффициент трения лыж о снежную поверхность р, а начальная температура снега 0 °C. Считать, что вся теплота идет на плавление снега.
Решение. Поскольку теплота, пошедшая на плавление снега, равна изменению потенциальной энергии лыжника, то mgh = где тс— масса растаявшего снега. Следовательно, тс =
127
_	. Для нахождения h воспользуемся тем, что изменение по-
тенциальной энергии лыжника равно работе, которую он произво-дит по преодолению силы трения на всем пути (рис. 167):
mgh = [tmgl cos а + umgs, или h = р/ cos а + ps.
Из Д АВС следует, что Zcosa = /ictga. Подставив это значение в предыдущее выражение для h, получим h = и/гctga + ps, откуда h = —• Следовательно, mc ~ ^ctga) ‘
Задача 4. В теплоизолированном сосуде содержится смесь воды тг = = 500 г и льда т2 =54,5 г при темпе
Рис. 168
ратуре /“ = 0 °C. В сосуд вводится сухой насыщенный пар массой та = 6,6 г при температуре /° = 100 °C. Какой будет температура 6° после установления теплового равновесия?
Решение. Максимальное количество выделяемой теплоты (когда весь пар сконденсируется, образовавшаяся при этом вода охладится до 0 °C)
Qi + Q.-i = гтя + ст3 ((° — i°) = 1,8 • 104 Дж.
Это равно количеству теплоты, поглощаемой льдом в том случае, когда он весь расплавится, т. е. Q3= Zm2= 1,8 • 104Дж. Таким образом, в сосуде будет только вода при температуре 0° = 0 °C.
Задача б. Автомобиль массой М = 1200 кг на горизонтальном пути развивает скорость vT = 72 км/ч, расходуя при этом т = 80 г бензина на 1 км пути. Какую скорость разовьет автомобиль при тех же условиях на пути с подъемом h = 3,5 м на I = 100 м пути? К- п. д. двигателя t] = 30 %. Удельная теплота сгорания бензина q = 4,5 • 107 Дж/кг.
Решение. При сгорании бензина выделяется энергия Е = = mq. Двигатель выполняет полезную работу Ап = [tMgs. Зная г], можно определить р:
П =	' ЮО % =	‘ ЮО %, откуда И =	
128
При движении автомобиля по горизонтальному пути двигатель развивает мощность Л\ =	а при движении по уклону мощ-
ность = (Mg sin а + pMgcos а) v2. Поскольку мощность двигателя— величина постоянная, то можно записать pMgUj = Mgv2 х X (sin а 4- Н cos а), откуда
°2 ~ ~—тг1----~14 м/с = 50 км/ч>
2 sina + pcosa Mgsh + r\mq V — h'1
Задача 6. Масса т идеального газа, находящегося при температуре Т, охлаждается изохорически так, что давление падает в п раз. Затем газ расширяется при постоянном давлении. В конечном состоянии его температура равна первоначальной. Определить совершенную газом работу. Молярная масса газа М.
Рис. 170
Решение. График изменения состояния газа в координатах (р, V) изображен на рис. 168. На этом рисунке пунктирными линиями проведены изотермы. Поскольку температура газа в конечном состоянии равна первоначальной, то точки 1 и 3 лежат на одной изотерме. Газ участвует в двух процессах: изохорически (V = = const) охлаждается от Т до 7\ (/ -> 2) и изобарически (р — = const) расширяется от до (2 -> 5). При изохорическом охлаждении газ не совершает работы. Работа газа при изобарическом расширении А = р2 (Г2 — Vj). Запишем уравнение Менделеева — Клапейрона для состояний газа 2 и 3:
= и py^RT,, откуда Л = ^/?(Т-7\).
Температуру газа Тг найдем из уравнения Шарля для изохорического процесса (1 ->2): - = L = п, откуда 7\ = — . Тогда работа 11	п
та, совершаемая газом,
/И уру 1
М п
Задача 7. Четыре моля газа совершают процесс, изображенный на рис. 169. На каком участке работа газа максимальна?
5 7-253
129
Решение. Начертим график данного процесса в координатах (р, V) (рис. 170). Для этого из уравнения Менделеева — Клапейрона найдем объем газа в точках 1, 2, 3, 4:
V, =	о,1 м3; V, = 2V, = 0,2 м3, IZ, = К, = 0,2 м3
1 Mpt ’	’	1	132
^ = ^=0’5 М3-
Из рис. 170 следует, что на участке 2	3 работа не соверша-
ется (процесс изохорический): Л2_3 =0. На участке 3 -> 4 (при изобарическом процессе) работа равна площади соответствующего
прямоугольника:
Лч_4 =Рз(К4- К3) = 1,5 • 10* Дж.
Оценим теперь работу при изотермическом процессе 1 -> 2. Площадь под
Рис. 172
участком / -> 2 изотермы меньше площади трапеции 1 — 2 — V2 — Эта последняя равна 1,5 • 104 Дж, т. е. равна площади прямоугольника, соответствующего участку 3 -> 4. Таким образом, ^1-2 "С Л3_4 И ^2-3 = 0.
Следовательно, работа газа максимальна на участке 3 -> 4.
Задача 8. В результате изобарического процесса температура углекислого газа увеличилась на АТ1 = 100 К. Определить совершенную при этом газом работу, если его масса т = 2,2 кг, а давление р = 105 Па.
Решена е. Работа газа в изобарическом процессе А = р (К2— — 1/j). Запишем уравнение Менделеева — Клапейрона для конечного и начального состояний газа:
pV2 = ^T2 и pV^RTi, откуда V2 - К4 = g (7’2 - 7\).
Тогда работа А = R(T2 — 7\) = RAT яз 41,6 • 103 Дж.
Задача 9. Над одним молем идеального газа совершают тепловой Процесс, изображенный на рис. 171, Как изменялась температура газа на участках I 2, 2 -> 3 и 3	/? На каких участках
газ получал, а на каких отдавал теплоту?
Решение. Для участка 1-^2 зависимость давления от объема линейная: р = kV, где k —постоянный коэффициент. Из уравнено
ния Менделеева — Клапейрона в этом случае получаем kV2 = RT. Это означает, что температура газа возрастала пропорционально квадрату объема и его внутренняя энергия увеличилась: At/ > 0. Кроме того, газ совершал положительную работу по расширению: А > 0. Из первого закона термодинамики следует, что количество теплоты Q^2 =	+ А > 0, т. е. на участке 1 —> 2 газ получал
теплоту.
На участке 2^3 объем не изменялся (V2 = V3 = const), т. е. работа газом не совершалась: А — 0. Давление газа падало: р3 < < р2; значит, Т3 < Т2 и At/ < 0. Поэтому на участке 2 -+ 3 газ отдавал теплоту: Q2_3 < 0.
На участке 3	1 оставалось постоянным давление газа (р1 =
= р3 = const), объем газа уменьшался от К3 до Vj и температура газа падала от Т3 до 7\. Следовательно, А < 0 и At/ < 0. В итоге Q3_j <0 — газ отдавал теплоту.
Задача 10. Состояние идеального газа изменяется по замкнутому циклу, состоящему из двух изохор и двух изобар (рис. 172). Температуры газа в точках 1 и 3 равны соответственно 7\ и Т3. Определить работу, совершаемую одним молем газа, если известно, что точки 2 и 4 лежат на одной изотерме.
Р е ш е н и е. На участках 1 ->• 2 и 3 ->• 4 работа не совершается, так как объем газа не изменяется (процесс изохорический). На изобаре 2—^3 газ совершает работу Аг = р2 (Р3— К2). Эта работа положительна, поскольку газ расширяется. На изобаре 4 -> 1 совершаемая газом работа отрицательна (газ сжимается) и равна А2 = ~ РУУ— l7])- Полная работа за цикл
А = р2(К3- К2) - p4(V4-	= (р2- р4)(К3- К2),
или, с учетом равенств р4 = р^\ р2~ р3 и К2 = Vlr
А = (р3— Pi) (К3— К4) = p3V3— PiVз~~ Рз^1+ РУ1-
Искомая работа численно равна площади фигуры, ограниченной графиком циклического процесса в координатах (р, V).
Уравнение Менделеева — Клапейрона для одного моля газа дает
PjVx = RTj и p3V3 = RT3.
Из условия, что точки 2 и 4 лежат на одной изотерме, получаем РУг = РУь, или p3Vx = рУ3.
В соответствии с этим
(Рз^)2 = (рУУрУз> = (рУ^рУ^ R2t\t2, откуда Pal/j = рУ3 = R/ЛЛ-
Работа, совершаемая газом,
А = RT3 - 2 R У'ТУ3 ф R7\ = R (УТ3 - У7\)2.
5*
131
ЗАДАЧИ ДЛЯ САМОСТОЯТЕЛЬНОГО РЕШЕНИЯ
502.	В сосуде находилось пг1 = 500 г воды и такое же количество льда при 0 °C. Сколько водяного пара при =100 °C было впущено в воду, если весь лед расстаял и температура в сосуде установилась 0°=ЗО °C?
503.	В стеклянный стакан массой «! = 120 г при /? = 15 °C налили т2 = 200 г воды при = 80 °C. Какое количество теплоты будет передано стакану? Удельная теплоемкость стекла с2 — = 840 Дж/ (кг • К).
504.	В латунном калориметре массой т1= 100 г находится й,= 5г льда при /? =— 10 °C. В калориметр вливают та = 30 г расплавленного свинца при t°2 = 327 °C. Что будет находиться в калориметре после установления теплового равновесия?
505.	В калориметр теплоемкостью С = 63 Дж/К налили т1 — = 250 кг масла при t°] — 12 °C. После опускания в масло медного тела массой т2 = 500 г при = 100°С установилась температура 0°=33°С. Какова удельная теплоемкость с2 масла по данным опыта?
506.	В калориметр теплоемкостью С = 2100 Дж/К, содержащий т1= 500 г воды при t° =40 °C, выливают ш2= 20 г расплавленного свинца при t2 = 327 °C. Считая, что вся вода нагревается до кипения, а затем часть ее обращается в пар, определить массу т испарившейся воды.
507.	Какая температура установится в латунном калориметре массой mt = 160 г, содержащем /?г2=400 г воды при /? = 25 °C, после того как расплавится помещенный в воду кусок льда массой т3 = 50 г, взятый при t2 — 0 °C?
508.	В медном калориметре, масса которого т1= 200 г, содержится т2 — 150 г воды при = 18 °C. Определить конечную температуру воды, если в калориметр опустили железный цилиндр массой т3= 100 г, нагретый до 1° =50 °C.
509.	В стеклянный стакан, масса которого т1= 120 г и температура = 20 °C, влили т2 = 200 г кипятка ($ = 100 °C). Через т = 5 мин температура воды и стакана равнялась = 40 °C. Считая, что потери теплоты происходят равномерно, найти количество теплоты q, теряемое в секунду. Удельная теплоемкость стекла с1=8,3 • 10а Дж/(кг • К).
510.	Нагретое до /? = 100 °C тело опустили в сосуд с водой, и при этом температура воды повысилась от t2 = 20 °C до /3 = — 30 °C. Какой станет температура воды если в нее одновременно с первым опустить еще одно такое же тело, нагретое до /5 = 50 °C?
511.	В калориметр, масса которого «1=400 г и температура t°i = 50 °C, влили т2 = 100 г воды при t2 — 10 °C. Через т = = 10 мин в калориметре установилась температура /3 = 20 °C. Вы
132
числить среднюю скорость потерь теплоты, обусловленных несовершенством теплоизоляции калориметра. Удельная теплоемкость вещества калориметра q=400 Дж/(кг • К).
512.	В сосуд, содержащий т1= 500 г воды при t° = 15 °C, бросили т2 = 50 г мокрого снега. Температура воды в сосуде понизилась на Д/° = 5 °C. Сколько воды было в снеге? Потерями теплоты пренебречь.
513.	Для определения удельной теплоемкости ms — 0,5 кг сплава при = 90 °C опустили в сосуд, содержащий т2 = 0,4 кг воды при = 20 °C. Вода нагрелась до t°K = 30 °C. Чтобы учесть передачу теплоты окружающей среде, повторяют опыт, бросая то же количество сплава, нагретого до той же температуры, в тот же сосуд, но содержащий т3 = 0,1 кг воды при = 10 °C. Вода нагрелась до = 20 °C. Определить удельную теплоемкость сплава, если масса сосуда т = 30 г, удельная теплоемкость вещества сосуда с = 400 Дж/ (кг • К.)
514.	В смесь льда с водой опустили нагреватель мощностью Р = 700 Вт и включили его в электрическую сеть. Спустя /0 = = 5,5 мин температура начала расти со скоростью а = 10 К/мин. Определить массу льда тл и воды тв, находившихся первоначально в смеси.
515.	Кастрюле с водой сообщили количество теплоты Q = = 1,82 • 10° Дж; при этом часть воды выкипела. Определить долю выкипевшей воды. Масса и удельная теплоемкость вещества кастрюли равны m1= 1 кг и сг = 400 Дж/(кг  К); масса воды т2 = = 5 кг. Начальная температура воды /° = 20°С.
516.	Определить мощность М нагревателя в установке для дистилляции воды, если в нее поступает вода при to = 20 °C и за т = = 1 мин установка дает т = 100 г дистиллированной воды при /1 = 100 °C.
517.	Сколько льда при t° = — 20 °C необходимо бросить в т = = 10 г переохлажденной до t02 = —5 °C воды, чтобы вся смесь замерзла?
518.	К перегретой до t\ = 110 °C воде долили воду при = = 100 °C. Какая часть воды испарится в результате установления теплового равновесия?
519.	При изготовлении дроби капли расплавленного свинца падают в воду с температурой /° = 17 °C. Определить массу свинца, которая доведет т0 = 1 кг воды до температуры кипения t°K = 100 °C.
520.	Кусок свинца массой т = 1 кг расплавился наполовину при сообщении ему Q = 54,5  103 Дж теплоты. Какова была начальная температура свинца /°?
521.	Сколько времени потребуется на приготовление льда в комнатном холодильнике, если на охлаждение взятого количества воды от = 15 °C до t°2 — 0 °C потребовалось т = 20 мин?
133
522.	Холодильник за тх = 20 мин работы охлаждает V — 1,5 л воды, помещенной в морозильную камеру, от $ = 16 °C до = = 4 °C. Найти массу льда, образовавшегося в камере, если процесс охлаждения был продолжен еще на т2— 60 мин при том же режиме.
523.	В калориметр с т = 400 г воды поместили электрическую лампочку мощностью Л/ = 40 Вт. Сколько времени должна быть включена в сеть лампочка, чтобы температура воды поднялась па Д/° = 30°? Теплоемкость калориметра и лампочки С = 105 Дж/кг.
524.	На нагреватель поставили открытый сосуд с водой. Через т1=40 мин после начала кипения в сосуд добавили воду, масса которой равна массе выкипевшей за это время воды. При неизменных условиях нагрева вода в сосуде снова закипела спустя время т2 = = 3 мин. Какова была первоначальная температура добавленной воды?
525.	Автомобиль мощностью N = 76,5 кВт, имеющий к. п. д. т] = 32 %,при движении с постоянной скоростью па пути s = = 120 м расходует т = 64 г бензина. Определить скорость движения автомобиля.
526.	Какое количество керосина нужно сжечь в нагревателе с к. п. д. т] = 40 % для превращения в пар при нормальном атмосферном давлении т =0,4 кг воды, взятой при /°=20°С?
527.	Найти расход бензина автомобилем «Запорожец)' на пути s = 1 км при скорости v = 60 км/ч. Мощность двигателя N = = 16 кВт, к. п. д. т] = 30 %.
528.	В баллоне емкостью V = 60 л находится метан (СН4) при tf = 0 °C и давлении р = 8 • 10° Па. Баллон подключили к газовой горелке с к. п. д. р = 80 % и, сжигая метан, нагрели воду от tQ — 0 °C до кипения. Если после этого нагреть баллон до = = 80 °C, то в нем снова установится начальное давление. Сколько воды нагрели? Удельная теплота сгорания метана q = = 41,8 • 10° Дж/кг, молярная масса М = 16 • 10"3 кг/моль.
529.	Какое количество бензина израсходовали двигатели самолета, пролетевшего I = 500 км со средней скоростью v = 250 км/ч, если средняя мощность его двигателей N — 2000 кВт? К. п. д. т) = = 25 %.
530.	Для нагревания некоторого количества воды от 0 °C до температуры кипения на электрическом нагревателе потребовалось ту = 15 мин. После этого потребовалось т2= 1 ч 20 мин для превращения всей этой воды в пар при тех же условиях. Определить по этим данным удельную теплоту парообразования воды.
531.	При соблюдении некоторых предосторожностей можно нагреть воду при нормальном атмосферном давлении до температуры выше t°K = 100 °C без ее закипания (перегретая вода). Пробирку, содержащую т = 100 г воды при Г = 109 °C и нормальном атмосферном давлении, слегка встряхивают, отчего происходит бурное вскипание воды. Найти массу тх выкипевшей воды.
532.	Паровой молот массой т1=9 т падает с высоты h — 2 м на стальную болванку массой т2 = 220 кг. Сколько раз он должен
134
упасть, чтобы температура болванки поднялась на Д/*= 50 °C? На нагревание болванки идет 50 % теплоты, полученной при ударах.
533.	С какой скоростью и должна лететь свинцовая пуля, чтобы при ударе о преграду она расплавилась, если до удара температура пули была t° = 100 °C? При ударе во внутреннюю энергию превращается ц = 60 % кинетической энергии пули.
534.	Свинцовая пуля, летевшая со скоростью v — 300 м/с, ударяется о стальную плиту и останавливается. На сколько градусов Д/° нагрелась при этом пуля, если а = 40 % ее кинетической энергии пошло в момент удара на нагревание плиты и окружающего воздуха?
535.	Свинцовая пуля пробивает деревянную стенку, причем скорость в момент удара была 400 м/с, а в момент вылета р2 = = 100 м/с. Какая часть пули расплавилась, если считать, что на ее нагревание расходуется ц = 60 % кинетической энергии? Температура пули в момент удара = 50 °C.
536.	Пуля массой т = 10 г пробивает подвешенное на длинной нити тело массой М = 1 кг, причем скорость ее в момент удара была vL = 1000 м/с, а в момент вылета v2 = 800 м/с. Определить, на сколько нагрелась пуля, если на ее нагревание пошло г] = 30 % количества теплоты, выделившейся при движении пули внутри тела.
537.	Свинцовая пуля, летящая горизонтально со скоростью п0= 500 м/с, пробивает доску на высоте h = 2 м над поверхностью Земли, не изменяя направления своей скорости. На каком расстоянии от доски пуля упадет на Землю, если при движении через доску она нагрелась на Д/°= 200 °C? Считать, что все выделившееся при движении через доску количество теплоты пошло на нагревание пули.
538.	Тело массой tn = 1 кг движется без трения по горизонтальной поверхности со скоростью щ = 10 м/с. Пуля массой т0 — = 10 г, летящая горизонтально со скоростью ц2 = 100 м/с в том же направлении, попадает в тело и застревает в нем. Какое количество механической энергии превратилось при ударе во внутреннюю?
539.	Пластилиновый шар бросают со скоростью vQ = 10 м/с под углом а = 60 ° к горизонту по направлению к вертикальной стенке, расположенной на расстоянии I = 6,3 м от точки бросания. Шар прилипает к стенке. Считая, что вся кинетическая энергия шара пошла на его нагревание, найти приращение температуры шара. Удельная теплоемкость пластилина с = 2,5 • 103 Дж/(кг  К).
540.	Два одинаковых ледяных метеорита летят навстречу друг другу с равными скоростями и при ударе обращаются в пар. Какова должна быть при этом минимальная скорость метеоритов, если их температура перед столкновением t° =—20 °C?
541.	Найти количество теплоты, которое выделилось при абсолютно неупругом соударении двух шаров, двигавшихся навстречу друг другу. Масса первого шара «^	0,4 кг, его скорость vt =
= 3 м/с. Масса второго шара т2 = 0,2 кг, скорость о2= 12 м/с.
542.	Сани скатываются с горы, составляющей с горизонтом угол а = 45°, и, пройдя по горизонтальной плоскости / — 50 м, оста
135
навливаются. Масса саней mt=120 кг, коэффициент трения полозьев о снег ц = 0,2. Сколько снега растает вследствие трения полозьев о снег, если считать, что все выделившееся количество теплоты идет на плавление снега? Начальная температура снега /°=0°С.
543.	Газ массой т — 3 кг, находящийся в цилиндре под подвижным поршнем, изобарно нагрели на ДТ == 100 К- Какую работу совершил газ, если его плотность при р0 = 10s Па и То= 273 К Ро = 1,29 кг/м3?
544.	На сколько градусов нужно нагреть V = 1 м3 газа, чтобы, находясь при t°— 15 °C и давлении р = 2,5 • 105 Па, он при изобарическом нагревании совершил работу расширения А = 5 • 103Дж?
545.	Воздух массой т — 2 кг находится в цилиндре, закрытом подвижным поршнем. Какая работа будет совершена при изобарном нагревании воздуха на Д71 = 100 К?
546.	В цилиндрическом сосуде под невесомым поршнем находится газ при Г = 20 °C. Газ занимает объем V = 1 л. При нагревании газа на Д/°= 10 °C поршень поднимается. Найти работу А, совершенную газом. Атмосферное давление pQ = 105 Па, трение поршня о стенки сосуда отсутствует.
547.	В вертикальном цилиндре под поршнем с площадью поперечного сечения Д' = 20 см2 находится столб газа высотой h = 60 см при f = 27 °C. Поршень может перемещаться без трения. Масса поршня т = 10 кг. Цилиндр нагрели на ДТ = 50 К. Определить работу А, совершенную газом. Атмосферное давление р0 = = 105 Па.
548.	В сосуде находится одноатомный газ при t° — 17 °C и р = = 105 Па. Объем сосуда V = 3 л. Газ изохорически нагревают на Д/°=Ю0 °C. Определить изменение внутренней энергии газа. Какое количество теплоты было передано газу в этом процессе?
549.	Определить изменение внутренней энергии п = 0,5 моль газа при нагревании его при постоянном давлении от = 27 °C до t°2 — 47 °C, если газу сообщено количество теплоты Q = = 290 Дж.
550.	В вертикальном цилиндре объемом V = 200 см3 под поршнем находится газ при Т — 300 К- Масса поршня т = 50 кг, его площадь S = 50 см2. Для повышения температуры газа на ДТ — 100 К ему было сообщено количество теплоты Q = 46,5 Дж. Найти изменение внутренней энергии газа. Атмосферное давление р0 = 105 Па. Трение не учитывать.
551.	Азот при изобарном расширении выполнил работу А — — 103 Дж. Какое количество теплоты было подведено к нему, если для нагревания 1 кг азота на 1 К в закрытом сосуде требуется q = 740 Дж?
552.	Моль идеального газа нагревается при постоянном давлении, а затем при постоянном объеме переводится в состояние с температурой, равной начальной температуре То = 300 К. Оказалось, что в итоге газу передано количество теплоты Q = 5 • 103 Дж. Во сколько раз изменился объем, занимаемый газом?
136
553.	Масса т кислорода совершает замкнутый цикл (рис. 173). В начальном состоянии (точка А) давление было pj и температура Тц затем газ изобарически расширился и его температура повысилась доТ^. Вычислить работу, совершаемую газом за цикл ABCDA, если максимальное давление в этом процессе было р2. Найти количество теплоты, сообщенное газу.
554.	Каковы первоначальный объем и температура гелия, находящегося под поршнем в цилиндре, если при охлаждении до /° = — — 23 °C груз массой М = 16 кг, лежащий на поршне, совершает
<Тг)
Рис. 173
Рис. 174
и
работу Л = 40 Дж? Площадь поршня 5 = 200 см2, атмосферное давление нормальное, масса гелия т = 5 г.
555.	Масса т азота совершает круговой процесс ABCDA (рис. 174). В ходе процесса объем газа изменяется от в точках А и D до V2 в точках В и С. Вычислить совершаемую газом работу, если в процессе DA температура азота изменяется от Т2 в точке D до 7\ в точке А. Какое количество теплоты Q было сообщено газу в процессе ABCDA?
556.	В цилиндре тепловой машины находится 1 моль идеального газа. Найти работу А газа за один цикл (рис. 175).
557.	Масса т водорода совершает процесс ABCD (рис. 176). В состоянии В газ имел температуру Т\ и объем Vi, а в состоянии С — объем V2. Вычислить выполненную над газом работу.
558.	Какая работа может быть совершена за один цикл (рис. 177), если наименьшая температура газа t° = 0 °C, а наибольшая =« = 127 °C? Объем газа при i? равен К, = 5 л, при /з = 6 л, а
137
при нормальных условиях данное количество газа занимает объем Уо = 10 л.
559.	Какое количество теплоты Q отдает водород в процессе ABCD, описанном в задаче 557, если в состояниях А и D температура водорода была соответственно Ts и Т4?
560.	Для процесса с идеальным одноатомным газом (рис. 178), представляющего собой линейную зависимость давления от объема, найти: а) работу, совершенную газом при расширении от состоя-
ния 1 до состояния 2; б) изменение внутренней энергии; в) количество теплоты, полученное газом.
561.	Вычислить работу, выполняемую при расширении массы m гелия из состояния А в состояние D (рис. 179). В состояниях А, В, С и D газ имел температуру соответственно 7\, Т2, Тя и Т4.
562.	Какое количество теплоты Q отдает гелий в процессе ABCD, рассмотренном в задаче 561?
563.	Идеальный газ переводят изотермически из состояния 1 в состояние 2 (V, < VJ, затем изобарнов состояние Зи возвращают изохорно в состояние 1. Положительную или отрицательную работу совершает газ в каждом из процессов и целиком за цикл? Имеет ли место теплообмен между газом и окружающей средой?
564.	Определить изменение внутренней энергии массы m гелия в процессе ABCD, описанном в задаче 561.
138
565.	При расширении газа его давление росло по закону р = =	+ ах1/+ а2И2, где и а2 — размерные постоянные коэффи-
циенты; plt и К2 — заданы (рис. 180). Какую работу совершил газ при расширении?
566.	Определить работу А, совершаемую газом в замкнутом цикле АВС (рис. 181), имеющем на (р, ^(-диаграмме вид полуокружности.
567.	Определить работу А, совершаемую газом при замкнутом цикле АВС (рис. 182), если параметры состояний В и А равны соответственно К1( И2 и рг, р2.
568.	Идеальная тепловая машина, для которой окружающий воздух при нормальных условиях является холодильником, поднимает груз массой т = 400 кг. Рабочее тело машины получает от нагревателя с температурой f = = 200 °C количество теплоты Q —	____________я
— 8  104 Дж. На какую макси- 2	/
Ь и
Ъ и
Рис. 181
Рис. 182
мальную высоту поднимет груз тепловая машина? Трением пренебречь.
569.	Определить температуру Т2 холодильника тепловой машины, которая работает при температуре нагревателя Tj и, выполняя работу А, передает холодильнику количество теплоты Q.
570.	Чтобы принять ванну, необходимо нагреть I/ = 200 л воды от Zj =7 °C до t°2 = 47 °C. Если такое количество теплоты сообщить идеальной тепловой машине, работающей при температуре нагревателя t.2 и холодильника то с помощью этой машины можно поднять груз массой m = 4,2 • 104 кг на высоту И = 10 м. Определить по этим данным удельную теплоемкость воды.
571.	К. п. д. тепловой машины г] = 40 %. Каким станет к. п. д. машины, если количество теплоты, потребляемое за цикл, увеличится на 20 %, а количество теплоты, отдаваемое холодильнику, уменьшится на 10 %?
572.	Тепловая машина за некоторое время потребляет количество теплоты Qi и передает холодильнику количество теплоты Q2. Определить максимальную температуру холодильника Т2, если температура нагревателя Т\.
573.	Температура газов, образующихся при сгорании топлива в цилиндрах двигателя автомобиля, /J = 800°C; температура выхлопных газов t°2 = 80 °C. Расход горючего на s = 100 км при скорости
139
v = 90 км/ч равен V = 10-2 м3; теплота сгорания топлива q = 3,2X х 1010 Дж/м3. Какую мощность мог бы развить двигатель, если бы он представлял собой идеальную тепловую машину, работающую с максимально возможным коэффициентом полезного действия?
§10. Упругие и тепловые свойства твердых тел и жидкостей
Задача 1. Каким должен быть предельный диаметр d стального троса, с помощью которого нужно поднимать груз массой т = 6 т при шестикратном запасе прочности? Предел прочности стали стпр = 6,86 • 108 Па.
Решение. Обозначив предел прочности через стлр, можно за-°пр 4ти	,	-| / 24mg о с
писать	, откуда d = I/ —- т 2,6 см.
6 nd2 ’ J	V лопр ’
Задача 2. Трубчатая чугунная колонна длиной 1 = 4 м с наружным диаметром D = 300 мм сжата силой F = 10е Н. Найти толщину стенок и изменение длины колонны, если допустимое давление Ри = 3 • 107 Па, а модуль Юнга Е = 1,2 • 10п Па.
Решение. Поскольку давление в поперечном сечении колонны должно быть меньше допустимого или равно ему, можно записать F	F
< рл, откуда S > —, где S — площадь кольца. Обозначив на-д	РД
ружный и внутренний диаметры колонны соответственно через D и d, запишем S = Д-(О2— d2). Тогда d= D2 —	« 21,8 см.
Толщина стенки h= D- ~2 d = 41 мм. Удлинение колонны найдем по закону Гука: у = g- , где Р = -у —действительное давление, Fl 41F
откуда А/ =	= яЕ(Рт_^ « 1 “м.
Задача 3. Стальной стержень длиной I и площадью поперечного сечения S, концы которого закреплены, нагревается на t°. Какую работу совершает стержень?
Р е ш е н и е. По закону Гука, упругая сила, возникающая при нагревании стержня,
F = ES j = ES$t°.
Если предоставить стержню возможность расширения, то длина его увеличится на А/ = l$f, а сила будет уменьшаться от F до 0. Среднее ее значение можно принять за ур. Работа, которую может совершить при нагревании стержень,
A = ^FAl = ^ESl^t°2.
Задача4.При изготовлении некоторого физического прибора оказалось необходимым обеспечить постоянство разности длин же-140
лезного и медного цилиндров при любых изменениях температуры. Какой длины должны быть эти цилиндры при О °C, чтобы разность их длин при всех температурах была А/ = 10 см? Коэффициент линейного расширения железа 1,2  10'а К-1, меди 02 = 1,7 X X IO’6 К-1.
Решение. Стержни при нагревании на Добудут иметь длины 11== 101(1 + PjAC) и 12 = 102(1 + р2АС). Разность этих длин А/ = = 1г— 12 = /а1+ 1п1^Ма — 1а2— /02₽2^°- Но, по условию, /2 — — /j — 101— /02 = А/. Следовательно, /оААЛ— Z02(32A/° = °- или Pi^oi = (Мог- Поскольку Zol = Z02 + Д/, то предыдущее уравнение примет вид
Рх (Zoa + AZ) = P2Z02, откуда /02 = р—1:^ А/ = 24 см.
Тогда /01 = Z02 + AZ, или Z01 = А/ = 34 см.
Задача 5. Сообщающиеся сосуды заполнены жидкостью,
имеющей температуру i\. При нагревании жидкости в одном из сосудов до температуры t° уровень жидкости в этом сосуде установился на высоте Л2, а в другом на высоте Zij (рис. 183). Найти коэф-
фициент объемного расширения жидкости.
Решение. При нагревании плотность жидкости изменяется обратно пропорционально объему. Таким образом, если при 0 °C плотность жидкости была р0, то при Z? ее плотность будет р! — —и при /2 р2 = ——;. Условие гидростатического рав-новесия жидкости
Р<А
1 + <
——т,, откуда 1 4-
— Zlt ftjC — ft2C
Задача 6. Стальной стержень (рис. 184), состоящий из двух частей равной длины Zt= Z2= 0,5 м, с площадями поперечных сечений 51= 252 = 20 см2, снизу закреплен на фундаменте, а верхним концом не доходит на Д/о = 0,3 мм до неподвижного упора. Найти
141
силу, с которой стержень будет воздействовать на опоры, если его температура повысится на Д/° = 50 °C.
Решение. Повышение температуры вызывает удлинение стержня Д/р а сжатие его реакциями опор F — укорочение Д/2. Разность этих двух деформаций по модулю равна Д/о= Д/х— Д/2. Значения Д/t и Д/2 соответственно равны
Д/х = (ЗД/° (/. + /2) = 2рД^’/1; Д/2 =	01/2 + 1- а212 =	.
С	С	F-* О j
3FI
Таким образом, 2рД 1"1Х —	= Д/о. Отсюда сила, действующая
на опоры,
Р___ (2рД/и0 Д^р) ESl _ g . [Q4 pj
ЗАДАЧИ ДЛЯ САМОСТОЯТЕЛЬНОГО РЕШЕНИЯ
574. Свод Ледового дворца спорта в Гренобле, масса которого т = 107 кг, опирается на 4 железобетонные круглые колонны. Найти напряжение, испытываемое железобетоном, и деформацию сжатия одной колонны, если диаметр каждой d = 1,5 м, а высота h = 5 м.
575. На сила F = 4
Лг Ct
а О
Рис. 185
железобетонную колонну высотой h = 10 м действует • 10” Н. Найти деформацию колонны, если площадь поперечного сечения колонны, занятая бетоном, Si- = 0,09 м2, площадь, занятая стальной арматурой, 5СТ = 0,015g, а модуль упругости бетона Ев — = 0,1 ЕСт Массой колонны пренебречь.
576. На железной и медной проволоках одинаковой длины и поперечного сечения подвесили легкий стержень BD длиной I = 80 см (рис. 185). На каком расстоянии х от точки В надо подвесить груз массой т = 2 кг, чтобы стержень остался горизонтальным?
577. Медную проволоку сечением 5, длиной Ц и стальную такого же сечения и длиной /2 соединили
между собой концами, так что общая длина проволоки I =	/2.'
Какую силу F надо приложить к этой проволоке, чтобы увеличить ее длину на q %? Модуль Юнга для меди Еи для стали Е2.
578. Определить потенциальную энергию упругой деформации стальной проволоки, растянутой на AZ = 0,01 м. Площадь поперечного сечения проволоки 5=3 мм2, начальная длина I — 2 м.
579. Медная проволока длиной / = 1,5 м и сечением 5 = 0,2 см2
при растяжении удлинилась на q = 0,01 своей начальной длины. Найти работу растяжения проволоки.
580. К проволоке, закрепленной верхним концом, подвешивается груз массой т, поддействием которого проволока удлиняется на Д/. Показать, что изменение потенциальной энергии проволоки в два раза меньше изменения потенциальной энергии груза. Как это согласуется с законом сохранения энергии?
142
581.	Какая требуется сила F, чтобы стальной стержень длиной / = 1 м с площадью поперечного сечения 5 = 1 см2 удлинить на А/ 1 мм? При какой наименьшей нагрузке Fmin стержень разорвется, если предел прочности стали о|1р = 7,85 • 10е Па?
582.	Латунный стержень длиной I = 1,5 м жестко закреплен между двумя упорами. Температура стержня t° = 0 °C? С какой силой он будет действовать на упоры, если ему сообщить количество теплоты <2 =4,19 • 105 Дж?
583.	Стальной брус при /? = 0 °C помещен вплотную между каменными неподвижными стенами. Найти напряжение материала
бруса при t'i — 20 °C.
584 . Стальная (ал ка плотно вставлена между двумя неподвижными недеформируемыми стенами при /о = 0 °C. При ?| = 10 °C
балка производит на стены давление р — 3 • 107 Па. Ка- — кое давление будет производить балка на стены при (° — = 25 °C?
585.	Стальная масштабная линейка проградуирована при ____
4= 18 °C. Какую относитель-
ную погрешность 6 допус-	рис. 186
кают, измеряя этой линейкой
при <2 = 40 °C, если не вносят поправок на тепловое расширение линейки?
586.	Стальную масштабную линейку требуется поградуировать так, чтобы миллиметровые интервалы при определенной температуре гарантировали измерение с точностью до 0,001 мм. Определить максимальную температуру, допустимую при градуировке.
587.	Колеса электровоза имеют радиус /?0 = 1 м при 0 °C. Определить разницу в количестве оборотов колеса летом при = = 25 °C и зимой при t? = —25 °C на пути пробега I = 100 км. Коэффициент линейного расширения материала колес (3 = 1,2 X X КГ6 К"1.
588.	Настенные часы имеют латунный маятник. При io = 0 °C часы спешат на Д7\=6с в сутки, а при = 30 °C отстают на Д'Г2 = 12 с в сутки. Определить температуру, при которой часы будут идти правильно.
589.	Из двух металлических тонких полосок склепана фигура в виде лестницы (рис. 186). Расстояние между полосками равно а. Полоски имеют разные коэффициенты линейного расширения fK и (Pi <€ 1; Р2 <С 1). При нагреве в результате различного удлинения полосок лестница искривляется. Каков радиус кривизны лестницы при ее нагревании на Д/°? Ширина лестницы намного меньше ее длины. Удлинением поперечных перекладин пренебречь.
590.	Стеклянный сосуд кубической формы, находящийся при температуре t°, наполнен жидкостью, масса которой т. При нагре
143
вании сосуда до t° часть жидкости вытекает, и масса ее становится равной Определить коэффициент объемного расширения жидкости а, если коэффициент линейного расширения стекла 0.
591.	В латунном цилиндрическом сосуде находится вода. При t°i = 15 °C высота столба воды = 1 м. На сколько поднимется уровень воды в сосуде, если температура воды и сосуда повысится на А/° = 5 °C?
§ 11. Свойства паров
Задача 1. Относительная влажность воздуха вечером при = 16 °C равна ф!=70 %. Сколько граммов воды сконденсируется в каждом кубическом метре воздуха при понижении температуры ночью до t°2 = 8 °C? Плотность насыщенных паров воды при 1° = — 16 °C pt= 13,6 г/м3, при = 8 °C р2 = 8,3 г/м3.
Решение. Относительная влажность
Ф1 = ^ . 1°° %, откуда	= 9,52 г/м3.
При понижении температуры до й =8 °C максимальное количество водяных паров, содержащихся в 1 м3 (при 100 % влажности), равно /2 = 8,3 г/м3, т. е.
^ = Л-/2 = 1,22 г/м3.
Задача 2. В комнате при /° = 20 °C относительная влажность <Р!= 20 %. Сколько нужно испарить воды для увеличения влажности до <р2 = 50 %, если объем комнаты V = 40 м3? Плотность насыщенных паров воды при t°= 20 °C равна р = 1,73 • 10'2 кг/м3.
Решение. При относительной влажности cpj плотность паров Pi = РФ/ и их масса во всей комнате т1— pcpil7. При относительной влажности tp2 плотность паров должна быть р2 = рср2 и масса их т2 = р<р2V. Следовательно, нужно испарить массу воды т = т2— — тх = (ср2— cpi)p V = 208 г.
Задача 3. В комнате объемом V = 50 м3 влажность воздуха Ф = 60 % при t° = 20 °C и давлении р = 108 Па. Чему равна масса т влажного воздуха в комнате? Как изменится эта масса при увеличении влажности на Аф = 10 % при неизменных температуре и давлении? Изменится ли при этом общее число молекул влажного воздуха в комнате? Давление насыщенного пара при 20 °C рн 2,3 X X 103 Па.
Решение. Масса влажного воздуха т равна сумме масс сухого воздуха ffij и водяного пара т2 : пг =	т2. Из уравнения
Менделеева — Клапейрона тх — и т2 = р2^2, где Мх ~ — 29 • 10-3 кг/моль и М2 = 18 • 10"3 кг/моль — молярные массы воздуха и водяного пара, и р2 — соответствующие парциальные
144
__	ср
давления. По условию задачи, р2 — Тоо%^н’ а согласно закону
Дальтона, рг = р — р2 = р — ра. Тогда
, V т = т1 + пг2 =
РМ1~	~ 58 кг>
Из этой формулы видно, что при увеличении влажности <р при неизменных р и Т масса т влажного воздуха в комнате уменьшается (Л4j — М2 > 0). Изменение массы при увеличении влажности на Д<р:
Дш = -дт — Мг) юо о/ ~ 50 г-
Уменьшение массы влажного воздуха в комнате при увеличении влажюсти можно объяснить с помощью закона Авогадро .При увеличении влажности воздуха общее число молекул влажного воздуха в комнате не меняется, а происходит лишь замещение части молекул воздуха на молекулы воды; увеличивается число молекул водяного пара и уменьшается число молекул сухого воздуха. Поскольку М2< < Мъ масса влажного воздуха уменьшается.
Задача 4. В объеме V содержится влажный воздух при относительной влажности ср %, температуре То и давлении pQ. Масса влажного воздуха т. Определить давление насыщенного водяного пара при этой температуре.
Решение. Запишем закон Менделеева — Клапейрона для сухого воздуха и водяного пара, находящихся в сосуде:
m	m,
р<У=~*Т и P"V = MRT-
По закону Дальтона, давление в сосуде равно сумме парциальных давлений содержащихся в нем газов: ра + рп = рй. Согласно определению относительной влажности, давление насыщенного водяного
пара /?н
Рп
<р'
• 100 и, по условию задачи, тъ + т„ = т.
Решив эту
систему, получим
100	/ ,,
Рн- <р (Л4В — Л4П) \Р^а
mRT\
V ) '
Задача 5. Человек в очках заходит с улицы, где температура воздуха t°i = 5 °C, в теплую комнату, где температура воздуха 12 = 25 °C. При какой максимальной влажности воздуха <р0 в комнате не будут запотевать очки человека? Давление насыщенных паров воды при 5 °C р± = 866 Па, а при 25 °C р2 = 3192 Па.
Решение. Возле стекол очков температура влажного воздуха становится равной /? = 5 °C. Стекла не запотевают, если относи-
сь
Тельная влажность воздуха при этой температуре не превышает 100 %:
Ф = £ . 100 % < 100 %, Pi
где pj =	---плотность насыщенного водяного пара при Тх =
= 278 К, р — абсолютная влажность воздуха в комнате, не зависящая от температуры. Поэтому максимальная абсолютная влажность воздуха, при которой стекла очков еще не запотевают, р„ =
PiM г
— Pj =	. Соответствующая относительная влажность воздуха
при Т2 = 298 К будет
Ф„ =	• ЮО % =	 100 %	30 %.
Р-2	Р-2 1 I
Задача 6. Определить отношение плотностей влажного (относительная влажность = 90 %) и сухого воздуха при давлении р0 = = 10й Па в температуре /°= 27 °C. Плотность насыщенных паров воды при этой температуре р0= 0,027 кг/м3.
Решение. Из уравнения Менделеева—Клапейрона paV = ffli п'Г	«1 МлРа ,,
= ~ RJ плотность сухого воздуха р} = ~	. При относи-
„	,	М.,Р„
тельной влажности <р плотность водяных паров р = <рр0 =	,
где р.2 — парциальное давление водяных паров. Отсюда /?2 = =	 Поскольку давление влажного воздуха ра равно сумме
парциальных давлений воздуха рг и паров р2, то р2 = р„— р2 = = ра— Плотность воздуха (без паров) при этом парциальном давлении р" =	— ~~^РРо  Плотность влажного воз-
„„V,	_____' I п»_М2р2 , М1Ро Л4,ФРО Л41Рп ,	АЛ)
духа р2 - р + р -	----м- -	+ ФР0 1 -	.
Тогда отношение плотностей
= 1 +	1 — J) «0,987.
Pi	Mipo \	М2/
ЗАДАЧИ ДЛЯ САМОСТОЯТЕЛЬНОГО РЕШЕНИЯ
592.	При /? = 22 °C относительная влажность воздуха фх = = 60 %. Появится ли роса при понижении температуры до /° = = 10 °C? Если появится, то какое количество влаги выделится из 1 м3 воздуха? Давление насыщенных паров воды при 22 °C ру = = 2,6 • 103 Па, при 10 °C р2 = 1,2 • 103 Па.
593.	Над поверхностью площадью 5=5 км2 слой воздуха толщиной h = 1000 м имеет температуру = 20 °C при относительной влажности ср =73 %. Воздух охладился до t2 = 10 °C.
146
Найти массу выпавшего дождя. Плотность насыщенного водяного пара при температурах t°i и /2 равна pt = 17,3  10~3 кг/м3 и р2 = 9,4  10~3 кг/м3 соответственно.
594.	Относительная влажность водяных паров, находящихся в объеме V = 20 л при t° = 100 °C, равна гр — 90 %. Пары изотермически сжимают, уменьшая объем в два раза. Найти массу сконденсировавшейся воды, если давление насыщенных паров при этой температуре р = 105 Па. Объемом воды пренебречь.
595.	Каким о фазой и на сколько г радусов должна измениться температура воздуха при постоянном атмосферном давлении , чтобы в нем образовался туман, если при t°— 25 °C относительная влажность воздуха ср =60 %?
596.	Влажный воздух при температуре Т и давлении р занимает объем К. Относительная влажность воздуха ср, давление насыщенных паров при этой температуре р„. Найти плотность влажного воздуха и массу содержащихся в нем паров.
597.	В салоне самолета объемом V = 600 м3 при полете поддерживаются температура = 20 °C и относительная влажность ср( = = 60 %. При посадке самолета температура в салоне возросла до = 30 °C. Сколько воды необходимо испарить, чтобы влажность осталась прежней? Давление насыщенных паров при температурах t°i и равно соответственно рг= 2352 Па и р2 =4214 Па.
598.	Температура дымовых газов, покидающих воздухоподогреватель парового котла, t°= 160 °C, а плотность водяных паров в газах р = 0,05 кг/м3. Какой должна быть минимальная температура стенки воздухоподогревателя, чтобы во избежание коррозии металла не имела места конденсация водяных паров?
599.	В закрытый сосуд объемом V = 10 л помещают несколько капель воды массой m = 0,258 г. Затем начинают нагревать сосуд, причем настолько медленно, что все время поддерживается равновесие между паром и жидкостью. При t°= 27 °C вода полностью испарилась. Определить давление насыщенных паров воды при этой температуре.
600.	В закрытой комнате объемом V — 60 м3 относительная влажность воздуха (р = 50 % при t° = 18 °C. Сколько воды необходимо испарить в этот объем, чтобы водяные пары стали насыщенными?
601.	Зимой в натопленной комнате температура воздуха = = 20 °C при относительной влажности ср£= 30 %, а на улице при температуре /° =—10 °C относительная влажность ср2 =90 %. Плотность водяных паров, насыщающих пространство при —10° С, равна р =0,0024 кг/м3. Определить количество водяных паров в 1 м3 комнатного и наружного воздуха. В каком из них больше влаги?
602.	В закрытом баке объемом V = 50 л находится m = 100 Г воды при нормальных условиях (р0= 1,01 • 10s Па; То = 273 К). Определить давление р влажного воздуха, если бак нагрели на ДУ = 100 К. Водяной пар считать идеальным газом.
147
603.	Относительная влажность воздуха при = 30 °C q)t = = 0,80. Какова будет относительная влажность <р2, если этот воздух нагреть при постоянном объеме до = 50 °C? Давление насыщенных паров воды при 30 °C равно р01 = 4,3 • 10s Па, при 50 °C р02 = 1,2  104 Па.
604.	Кондиционер пропускает через комнату ежесекундно V= = 3 м3 воздуха. Воздух забирается с улицы, где температура /? = = 40 °C и влажность (pj = 80 %, затем охлаждается в кондиционере до t° = 5 °C, а в комнате нагревается до 7° = 25 °C. Какая масса воды Ат ежесекундно конденсируется в кондиционере при таком режиме его работы? Какая влажность <р воздуха установится в помещении? Давление насыщенных паров воды при 40 °C рх= 7,4- 10sПа, при 5 °C р2= 866 Па, а при 25 °C ра= 3192 Па.
III. ЭЛЕКТРОДИНАМИКА
§ 12. Электрические заряды и электрическое поле
Задача 1. Два одинаковых шарика, обладающие зарядами qx = = 24 • 10"8 Кл и q2— 6 • 10"8 Кл, находятся на расстоянии I = = 4 см друг от друга. Их приводят в соприкосновение и затем удаляют на прежнее расстояние. Найти силу взаимодействия шариков до и после соприкосновения.
Решение. Сила взаимодействия шариков до соприкосновения
F -	0,081 Н.
1 4ле0 /2
При соприкосновении шариков между ними происходит перераспределение заряда, которое количественно определяется тем, что в момент соприкосновения шары имеют одинаковый потенциал:
<Pj = ф2, или C.q\ = C2q2,
где q{ и qi — заряды шариков после соприкосновения. Шарики одинаковы и, следовательно, характеризуются одной и той же электрической емкостью С:
Ci = С2 = С, тогда q\ = q'2 = q,	(1)
т. е. после соприкосновения шарики одинаково заряжены. В силу закона сохранения заряда
<71 + 72 = 71 + 72. откуда q = ^-±£2 = 15 . 10-8 Кл (2)
Силу взаимодействия после соприкосновения найдем, применив закон Кулона и использовав выражения (1) и (2):
л =	И-
2 4леп 4Z2	’
143
Задача 2. На шелковой нити в воздухе висит неподвижно шарик массой m = 5 г, имеющий заряд q = 10-7 Кл. Определить натяжение нити, если внизу на расстоянии I = 10 см по вертикали расположен такой же одноименный заряд. Как изменится натяжение нити, если расположенный внизу заряд будет иметь противоположный знак?
Решение. В первом случае на шарик действуют три силы: тяжести mg, натяжения нити 7\ и электростатическая сила отталки-вания F], Проектируя векторы этих сил на направленную вертикально вверх ось Ох, можем записать условие равновесия подвешенного шарика:
Гг+ Fi— mg = 0, откуда 7\ — mg —	= 4  10-2 Н.
Рис. 187
Рис. 188
Во втором случае на подвешенный шарик по-прежнему действуют три силы, причем сила тяжести не изменится, электростатическая —
сила F2 станет силой притяжения и, как следствие, изменится сила натяжения нити. Условие равновесия для подвешенного шарика
Т2— F2— mg = 0, откуда 7\ = mg-г  10~2 Н.
Задача 3. Между вертикальными пластинками, заряженными разноименно, подвешен на шелковой нити заряженный шарик массой m = 0,05 г. Найти заряд шарика и натяжение нити, если нить расположилась под углом а = 45° к вертикали. Заряд каждой пластины Q = 8,85 • 10~8 Кл, площадь 5=2 м2.
Решение. Противоположно заряженные пластины представляют собой заряженный конденсатор, причем напряженность электрического поля в пространстве между пластинами Е — -^-. Эле-ктростатическая сила, действующая на заряженный шарик, F3 = — qE =	. Отсюда q = ^-Fa. Силу F3 и силу натяжения нити
е0о	ц
Т можно определить, рассмотрев (рис. 187) условие равновесия заряженного шарика под действием трех сил mg, Т, F3:
mg + T + Ft = 0.
149
В проекциях на оси координат это уравнение запишется Tsin а — F3 =0 и Tcos а — mg = О,
откуда
Т = т~ л: 6,9 • 10-4 Н и F3 = mg tg а. cos а *	® ь
Подставив значение F3, получим q =	• mg tga = 9,8  10~8 Кл.
Задача 4. Напряженность поля в плоском воздушном конденсаторе Е, а заряд на пластинах q. Какая сила действует на каждую из пластин? Равна ли она qE?
Решение. Напряженность электрического поля в пространстве между пластинами конденсатора
Е = Et -F Е2 = 2£\, |Ei| = |Д2|,	(1)
где и Е2 — напряженности, создаваемые первой и второй пластинами конденсатора соответственно. Сила, действующая, например, на первую пластину F\, определяется только полем, создаваемым второй пластиной, т. е. вектором Е2, так как сам на себя заряд не действует. Поэтому
Fi = ^|E2|= ^qE,	(2)
где q — заряд первой пластины. Аналогично на вторую пластину, имеющую заряд —q, действует сила F2 со стороны поля, создаваемого первой пластиной (с напряженностью Е2):
F2^q\E1\=^qE,	(3)
т. е. сила, действующая на каждую из пластин, не равна qE.
Задача 5. Увеличатся ли заряды на конденсаторах, емкости которых равны С\ и С2 (рис. 188), если увеличить значения этих емкостей?
Решение. Для ответа на вопрос задачи необходимо выразить заряды на пластинах конденсатора q1 и q2 соответственно через S и Ci (t =1, 2, 3). Определим сначала электрическую емкость Слв параллельного соединения конденсаторов, емкости которых С\ и С2 и общую емкость Сов-
Г _ Г I Г ,, с — СзСлв _ <С1 + Сз)
САВ -	-т- С2 и CDB - Сз + Слв - С1 + с2 + Сз •
Заряд q3 на пластинах конденсатора емкостью С3 находим из определения электрической емкости: q3 ~Cdb$- Из закона сохранения заряда, примененного к электронейтральной системе, которую
150
образуют правая пластина конденсатора емкостью С3 и левые пластины конденсаторов, емкости которых С\ и С2, получим
?i+ ?2 = qs-	(О
Поскольку конденсаторы с емкостями Сг и С2 соединены параллельно, то на них подается одно и то же напряжение U, равное
1 = К	(2)
Решая систему линейных уравнений (1) и (2), подставляя вместо q3 найденное выше выражение, получаем
С36	С3&
qi - 1 + С2/С2 + Сз/Ci и 72 - 1 + Сх/С2 + С3/С2 
Если Cj и С2 увеличатся в одно и то же число раз, то С2!Сг и останутся без изменения, С3!СХ и С3/С2 — уменьшатся, следовательно, знаменатель уменьшится и заряды увеличатся. Если же электрическая емкость одного из конденсаторов увеличится намного больше, чем другого, то один из зарядов увеличится, а другой уменьшится, так как С3!СГ и С3/С2 уменьшатся, а С21С1 и С}!С2 изменятся по-разному (одно отношение увеличится, другое уменьшится).
Задача 6. Конденсатор емкостью С± = 2 мкФ заряжают до напряжения иг = ПО В, затем, отключив от сети, замыкают на конденсатор неизвестной емкости. Определить электрическую емкость второго конденсатора, если напряжение на нем стало U2 =44 В.
Решение. Заряд на пластинах конденсатора емкостью до замыкания q = CJU1. После замыкания он распределится между пластинами обоих конденсаторов так, чтобы выполнялся закон сохранения заряда
qi+q2=q-	(1)
Поскольку при замыкании конденсаторы оказываются соединенными параллельно, то на них устанавливается одно и то же напряжение
= = (2)
Первое из уравнений (2) и уравнение (1) образуют систему двух линейных уравнений, из которой
= С, (U.- И2}.	(3)
Подставляя (3) в (2) и разрешая полученное уравнение относительно С2, получаем
С3 = ^=^-2 С, = 3 мкф.
Задача 7. Два конденсатора емкостью Сг~ 6 • 10*11 Ф и С2^ = 25 • 10"10 Ф соединены последовательно. Определить энергию этой батареи конденсаторов, если на нее подается напряжение U — = 150 В.
151
Решение. Общая электрическая емкость батареи двух кон-С С
денсаторов, соединенных последовательно, С = -r ' 1. Тогда энергия батареи
£ _ CIP _ CiC2U2 ^73. ]Q7 дж с 2 ~ 2 (С, + С2) ~ и ДЖ'
Задача 8. На две плоскопараллельные сетки, разность потенциалов между которыми равна Ua (рис. 189), падают под разными 4
углами отрицательно заряженные частицы с энергией -^eU0. При и
каких углах падения а частицы будут «отражаться», т. е. не смогут пройти через сетки? Заряд частиц е.
Решение. Движение заряженных частиц до первой (положительно заряженной) сетки будет равномерным. Скорость его определим по известной кинетической энергии частиц £к:
= — = 3 0ТКУДа ио = V if
Движение частиц в пространстве между сетками будет равноускоренным, причем скорость v0 будет начальной скоростью этого неравномерного движения.	~ "-
Между плоскопараллельными сетками создано постоянное электростатическое поле, напряженность которого Е = Оказавшаяся между сетками заряженная частица испытывает действие кулоновской силы F = —еЕ (рис. 189), которая сообщает ей ускорение
,*. е |Е| eU0 , а = ——1 = —г = const, 1 1 т md	’
направленное в любой точке пространства между сетками от правой сетки к левой перпендикулярно к плоскости сеток. Движение частиц между сетками аналогично движению тела, брошенного под углом к горизонту в поле тяготения Земли вблизи ее поверхности. При этом роль ускорения свободного падения g играет ускорение а,
152
выбор осей координат и угла |3 =	— а, задающего направление
начальной скорости, понятны из рис. 190. Воспользуемся известным выражением для максимальной высоты подъема тела
ь Wosin2P «гпах - 2fl -
Используя формулы sin р = sin (—- а = cos a; cos2a = (1 + + cos 2сс) и полученные выше выражения для начальной скорости и ускорения, получаем
hmax = -g d(l + cos 2a).
Теперь можно сформулировать условие «отражения» частиц, которое эквивалентно условию «непрохождения» частиц через вторую (отрицательно заряженную) сетку: /imax < d. Подставляя значения /гтах и d, получим cos 2а < В силу физического смысла угла а последний изменяется в пределах аС^О; Решение тригонометрического неравенства с учетом этого ограничения приводит к окончательному результату:
Z- Iя я Г ]б ’ г[-
Задача 9. На две плоскопараллельные сетки, между которыми приложена разность потенциалов Uo, падает параллельный пучок отрицательно заряженных частиц под углом a = 60°. При каких энергиях частицы смогут пройти через сетки, если заряд частиц равен 7?
Решение. Решение аналогично решению предыдущей задачи, только условие отражения следует разрешить относительно Ек = 9 /ПОл
= — • Тогда EK>^qUa.
Задача 10. Напряженность однородного электрического поля слева от бесконечной заряженной пластины Ег, а справа — £2 (рис. 191). Определить силу f, действующую на единицу площади пластины со стороны электрического поля.
Решение. Прежде всего выясним, почему однородное электрическое поле по разные стороны от плоскости А характеризуется —' ►
разной напряженностью: ^Ег\ #= \Е^. Возможный ответ: потому, что существует не изображенная на рисунке плоскость В, параллельная плоскости А. Используя принцип суперпозиции полей, вычислим (Вл! и |Вв| — модули напряженностей полей, создаваемых плоскостями А и В соответственно:
|еа| = ± (Й + N); \Ев\ = ± (Й - |£il).
153
Зная |Ел|, определим поверхностную плотность заряда Ол плоскости А, после чего найдем искомую силу /, которая равна силе, действующей со стороны поля заряженной плоскости В на заряд, расположенный на единичной площади плоскости А:
Задача 11. Три плоские металлические пластины образуют сложный конденсатор (рис. 192, а). На пластине 1 находится заряд Q, а незаряженные пластины 2 и 3 закорочены проводником. Определить силу, действующую на пластину 2. Площадь каждой пластины равна 5.
Решение. Поскольку пластины 2 и 3 закорочены проводником, то они эквипотенциальны:
Рис. 191	Рис. 192
где <р2 и Фз — потенциалы 2-й и 3-й пластин соответственно. Тогда „ и п
Е =	= 0, т. е. электрическое поле между этими пластинами
отсутствует. Заряжены ли при этом пластины 2 и 3? Да, причем, в силу закона сохранения заряда (пластины 2 и 3 образуют замкнутую электронейтральную систему), заряды на них равны по значению q и противоположны по знаку. Между пластинами 2 и 3 произошло разделение зарядов вследствие электризации под действием электрического поля, созданного заряженной пластиной 1. Так как в результате все три пластины оказались заряженными,то каждая из них в пространстве между пластинами 2 и 3 создает свое электрическое поле. Обозначим через Elt Е,2 и Е3 векторы напряженностей электрических полей, создаваемых соответственно 1-й, 2-й и 3-й пластинами в произвольной точке А пространства между пластинами 2 и 3 (рис. 192, б). Условие Е = 0 эквивалентно следующему векторному равенству:
Ег + Е2 -t- Е3 = 0,	(1)
которое позволяет выразить q через Q. В рассматриваемом случае
154
Чтобы перейти от векторного уравнения (1) к скалярному, необходимо учесть направления напряженностей:
_!_ 2____L.9___L У. = о	(2)
2е„ S	2е0 S	2е0 S и'
Из уравнения (2) следует, что
q = j Q.	(3)
Теперь вычислим силу F, действующую на пластину 2. Обозначая через F21 и Fm силы, действующие на пластину 2 со стороны электрических полей, созданных заряженными пластинами 1 и 3 соответственно, получим F = F21 + Fi3. Знаки зарядов пластин и результат (3) позволяют заключить, что векторы F21 и FM противоположно направлены и |F2i| > Поэтому
И = |Й| - |Й| = <7 Й ~ <7 Й = '
Задача 12. Три плоские металлические пластины образуют конденсатор (рис. 193). На средней пластине заряд +<?. Крайние незаряженные пластины закорочены проводником. Определить напряженность электрического поля между пластинами, если расстояние между ними и /2 (/2 > 1г), а площадь каждой пластины 5.
Решение. Задача решается аналогично предыдущей. Необходимо учесть, что крайние пластины образуют эквипотенциальную систему и поэтому суммарная работа по перемещению заряда между ними равна нулю. Это позволяет получить соотношение между /j и /2 и напряженностями Ег и Е2 электрических полей, созданных в пространствах между левой и средней и средней и правой пластинами соответственно. Две крайние
пластины образуют электронейтральную систему, что в совокупности с законом сохранения заряда позволяет получить второе уравнение для |£j| и |£'2|. Решение полученной системы линейных уравнений имеет вид
Й = гАт А и Й = ,-4т А •
Задача 13. В пространство между пластинами плоского конденсатора, между которыми поддерживается постоянная разность потенциалов, вводится пластина с диэлектрической проницаемостью е. Во сколько раз изменится сила электростатического взаимодействия между пластинами конденсатора? Толщина пластины равна половине расстояния между пластинами конденсатора.
155
Решение. На заряд q, равномерно распределенный по (положительной) пластине /, действует сила
(О
где Ех — напряженность поля, созданного зарядом —q, равномерно распределенным по отрицательной пластине 2, в пространстве между пластинами. Ошибочно подставлять в формулу (1) вместо El напряженность Е электрического поля в конденсаторе, так как поле напряженностью Е создают заряды обеих пластин: Е = Et + + Е2, |£J = |Е2|, но заряд пластины 1 сам на себя не действует, поэтому
(2)
—> ,
Вместо напряженности Е в формулу (2) удобнее подставить ее выражение через заряд q-. I-El = что приводит к следующей формуле: е0 о
___ ч _ и рг — 2e0S 2e0S ’
Что же изменится, если в пространство между пластинами конденсатора ввести диэлектрическую пластину? Изменится емкость конденсатора и, так как разность потенциалов между пластинами поддерживается постоянной (U = const), изменятся заряды на пластинах:
С' = ~ , откуда q' = C'U.
Таким образом, после введения диэлектрика сила взаимодействия между пластинами
i/T'i —	С'2
1 I 2e0S 2e0.S	’
Заметим, что влияние диэлектрика на силу взаимодействия между пластинами заключается в изменении зарядов на пластинах конденсатора (или, что то же самое, в изменении емкости конденсатора). Сами поляризационные заряды изменяют лишь поле внутри диэлектрика EBtt (рис. 194, а), но непосредственно не изменяют силу, действующую, например, на заряд Eq пластины 1 конденсатора. Действительно, напряженность ноля, создаваемого двумя бесконечными плоскостями, заряды на которых противоположны по знаку и равны по модулю, вне пространства между ними (например, в точках пластины 1 конденсатора) равна нулю.
Далее:
156
Вычислим отношение емкостей конденсатора после введения диэлектрика и до его введения. Несколько обобщим задачу, считая толщину диэлектрика х не фиксированной (х < d), после чего число заданных в условии задачи параметров увеличится до трех: х, d — расстояние между пластинами, е — диэлектрическая проницаемость вещества диэлектрика.
Рис. 194, б, в, г иллюстрируют простую догадку, позволяющую вычислить С: после введения диэлектрической пластины получившийся конденсатор эквивалентен последовательному соединению трех плоских конденсаторов. Первый и третий из них имеют толщины
di и d2 соответственно и являются плоскими конденсаторами без диэлектрика (е = 1), поэтому их электрические емкости
z-* _ e0S _____ e0S
- d2 •
Второй из них — плоский конденсатор толщиной х, причем пространство между его пластинами полностью заполнено диэлектриком, диэлектрическая проницаемость которого е. Поэтому
3 X '
Используя формулу для емкости батареи последовательно соединен-1 1,1,1 , , , , ных конденсаторов г-, = г- + „- + и учитывая, что d1 + х d2 = = d, находим
, S qS Ed	Б	,
— ~d~ х 4- е (d — х) ~ х + е ([ _ £ 'j •	(4)
Выражая из (4) отношение емкостей С7С и подставляя в (3), получим
157
По условию, =0,5, следовательно, п —	~ 2,25.
Задача 14. Конденсатор переменной емкости имеет в основании тонкий металлический диск радиусом /? = 5 см, на который наклеен полудиск толщиной d — 1 мм из диэлектрика с диэлектрической проницаемостью е = 10. В начальном положении над незаклеенной частью основания и диэлектриком находятся проводящие тонкие полудиски ОА и OtB, соединенные проводящей осью OOj (рис. 195, а). Вычислить электрическую емкость конденсатора в начальном положении и в положении, когда полудиски ОА и OJ3 меняются местами.
Рис. 195
Решение. В начальном положении относительное расположение диска и полудисков ОА и OtB таково, что рассматриваемый конденсатор эквивалентен параллельному соединению двух плоских конденсаторов (рис. 195, б, в). Емкость первого из них, образованного половиной диска и полудиском ОХВ,
С — d ~ 2d '
Второй из рассматриваемых конденсаторов, образованный половиной диска и полудиском ОА, более сложен, поскольку пространство между его пластинами лишь наполовину заполнено диэлектриком. Для определения его емкости С2 осуществим преобразование (рис. 195, г, д), которое доказывает эквивалентность рассматриваемого конденсатора батарее последовательно соединенных конденсаторов, первый из которых совпадает с конденсатором емкостью Сп а второй — емкостью С2,— отличается от конденсатора емкостью Cj лишь тем, что пространство между его пластинами заполнено диэлектриком с диэлектрической проницаемостью е: С3 = еСх; Тогда
r _ CjC3 _ е Кол/?2
- “ Сх -|- С3 1 + е 2d '
158
Емкость конденсатора в начальном положении можно определить, как емкость батареи (рис. 195, в) конденсаторов с известными емкостями и С2, соединенных параллельно:
/ч(1) ’-'общ
— Ci + О2 =
2е + I е+1
е0л/?2
66 пФ.
Замена ОА 0LB приводит к изменению относительного поло-
жения диска и полудисков (рис. 196, а, б), что ведет к изменению ем-
кости конденсатора. На рисунке представлено преобразование, из
которого следует, что рассматриваемый конденсатор эквивалентен батарее двух параллельно соединенных конденсаторов. Первый из них — уже знакомый нам конденсатор емкости С3, а второй (емкости С4) отличается от рассмотренного выше конденсатора емкостью Ct тем, что расстояние между его пластинами вдвое больше:
Е q S	Е 0 Л R 2
= 2сГ	'
С<
В	Oi
--------1-------

о
Л
Рис. 196
Рис. 197
Емкость описанного в условии задачи конденсатора в положении О А z=t 01В теперь может быть вычислена как емкость батареи конденсаторов с известными емкостями С3 и С4, соединенных параллельно:
сх = с3 + с4 = е-^г ~365 пф-
Задача 15. Конденсатор имеет в основании тонкий металлический диск радиусом R = 5 см, на который наклеен полудиск из диэлектрика толщиной d = 1 мм. Над незаклеенной частью основания и диэлектриком находятся проводящие тонкие полудиски ОА и Ojfl, соединенные проводящей осью 00{ (рис. 197). Определить относительную диэлектрическую проницаемость диэлектрика, если емкость конденсатора оказалась равной 66 пФ.
Р е ш е н и е. Решение этой задачи аналогично решению предыдущей. Емкость описанного в условии задачи конденсатора
2f-+ 1 е+1
ennR-	‘JdC — вллА!2 п
Лг ’ 0ТКУДа 6 = 2(wR*-dC) ~ 9-
159
ЗАДАЧИ ДЛЯ САМОСТОЯТЕЛЬНОГО РЕШЕНИЯ
605.	Два одинаковых шарика массой т = 0,05 каждый подвешены в одной точке на шелковых нитях одинаковой длины. Получив одинаковые заряды, шарики разошлись так, что нити образовали между собой угол а = 90°, а расстояние между шариками г = — 30 см. Определить заряд каждого шарика.
606.	Два одноименно заряженных шарика массой т = 0,5 г каждый подвешены в вакууме на очень тонких невесомых нерастяжимых и непроводящих нитях одинаковой длины. Каждая из нитей образует с вертикалью угол а = 30°. Затем вся система погружается в диэлектрическую жидкость	'/////////
с плотностью, равной плотности материала шариков, и диэлектрической проницаемостью е =
= 2. Найти силу натяжения нитей после погружения в жидкость. Каков характер равновесия шариков?
607.	Три одинаковых маленьких шарика массой т = 0,02 г каждый подвешены в одной точке на шелковых нитях длиной I = = 30 см. Какие равные заряды следует сообщить шарикам, чтобы каждая нить составила с вертикалью угол а = 30° ?
608.	Две проводящие ссреры, радиусы которых = 1 сми г2= = 0,5 см, соединены через изолирующие прокладки пружиной с жесткостью k = 100 Н/м и длиной 10 = 9,9 см. Одной из сфер сообщают заряд </, а затем сферы соединяют тонким гибким проводом. При этом расстояние между сферами становится равным I = 10,5 см. Найти заряд q, если до его сообщения сила натяжения пружины была равна нулю.
609.	Напряженности электрического поля точечного заряда в некоторых точках А и В (рис. 198) равны Еа — 36 В/м и Ев = = 9 В/м соответственно. Найти напряженность в точке, лежащей посередине между точками А и В.
610.	Вокруг неподвижного точечного заряда Q = 10"8 Кл равномерно вращается шарик, заряженный отрицательно. Чему равно отношение заряда шарика к его массе, если радиус орбиты 7? — = 0,02 м, а угловая скорость вращения со = 3 рад/с?
160
611.	Два металлических шара с радиусами г1=0,01 м и г2 = = 0,05 м находятся друг от друга на расстоянии, намного большем их радиусов. Шары несут заряды gr= 8 • 10-8Клид2 = 4 • 10'8Кл соответственно. Их поместили в жидкий диэлектрик и измерили силу взаимодействия. Затем шары извлекли из диэлектрика, соединили проводником на короткое время и вновь измерили силу взаимодействия. Она осталась неизменной. Найти диэлектрическую проницаемость диэлектрика.
612.	На тонкой изолирующей нити длиной / = 60 см подвешен небольшой заряженный шарик массой т = 0,02 г. На расстоянии I от шарика в одной с ним горизонтальной плоскости находится металлическая сфера радиусом а = 3 м (рис. 199). Если зарядить сферу, подав на нее напряжение U = 8 • 103 В относительно Земли, нить с шариком отклоняется от вертикали на максимальный угол а = = 90°. Найти заряд q на шарике.
613.	На расстоянии R = 3 м друг от друга расположены два точечных заряда qY =— 9 • 10"8Кл и q2=—36 • 10-8Кл. Какой заряд и где нужно поместить, чтобы все три заряда оказались в равновесии?
614.	Найти расстояние между двумя точечными зарядами, если известно, что при его уменьшении на А/ = 0,5 м сила взаимодействия между зарядами увеличивается в k = 2 раза.
615.	Две бесконечные параллельные плоскости равномерно заряжены и находятся на небольшом расстоянии друг от друга. Напряженность электрического поля в некоторой точке А между плоскостями Ед = 3 • 103 В/м, а в точке В вне этого промежутка Ец = = 103 В/м. Найти поверхностные плотности зарядов плоскостей.
616.	Расстояние между зарядами q1= 10’8Кл и q2 = — 10'0Кл равно 1,1 м. Определить напряженность поля в точке на прямой, соединяющей заряды, в которой потенциал равен нулю.
617.	Между горизонтальными пластинами плоского конденсате' ра с высоты h свободно падает незаряженный металлический шарик массой т. На какую высоту поднимется он после абсолютно упругого удара о нижнюю пластину, если в момент удара на шарик переходит заряд 7? Конденсатор подключен к батарее с э. д. с., равной В. Расстояние между пластинами конденсатора d.
618.	Скорость электрона, пролетающего в электрическом поле из точки А в точку В, увеличилась от щ= 10е м/с до и2= 3 X X 106 м/с. Определить разность потенциалов между точками А и В поля. Отношение заряда электрона к его массе е/т =—1,76 X X 10Л1Кл/кг.
619.	Маленький шарик с зарядом q подвешен на тонкой шелковой нити в пространстве плоского воздушного конденсатора, пластины которого расположены горизонтально. Когда конденсатор зарядили до напряжения (/, натяжение нити увеличилось в k — — 2 раза. Определить массу шарика т. Расстояние между пластинами конденсатора d. Массой нити пренебречь.
620.	Какую напряженность поля создает протон в атоме водорода в месте нахождения электрона? Заряд протона qt = 1,6 х
6 7-253
161
X 10-18 Кл. Радиус атома водорода принять равным г— 0,5 X х 10-8 см.
621.	Имеются две проводящие сферы, одна из которых заряжена, другая нет. В каком случае заряд с заряженной сферы не будет переходить на незаряженную, если их соединить проводником?
622.	Почему заряженный проводник, покрытый пылью, теряет заряд?
623.	Две равномерно заряженные бесконечные плоскости расположены под углом ср = 90° друг к другу. Нарисуйте силовые линии электрического поля, создаваемого этими плоскостями.
624.	Пучок электронов, влетающий в плоский конденсатор параллельно его обкладкам, отклоняется на пути I — 4  10~2 м на высоту h =2 • 10~а м от первоначального направления. Какую скорость v и кинетическую энергию Т имеют электроны в момент влета в конденсатор, если напряженность электрического поля в нем Е = 2,25 • 104 В/м? Гравитационные силы не учитывать.
______________________________625. Электрон влетает со ско-ростью V = 107 м/с в плоский конденсатор, между пластинами ✓-1 -__________---------которого поддерживается посто-
рис. 200	янная разность потенциалов
U — 425 В (рис. 200). Расстояние между пластинами d = 1 см, угол а = 30°, удельный заряд электрона elm = 1,76- 1011 Кл/кг. На какое максимальное расстояние I удалится электрон от положительной пластины?
626.	Металлическая сфера диаметром О = 10 см заряжена зарядом Q = 9  10'9 Кл. Определить потенциал на сфере, для точек внутри сферы и в точке, удаленной от поверхности сферы на d = = 25 см. Нарисовать силовые линии и эквипотенциальные поверхности. Построить зависимости напряженности и потенциала от расстояния до центра сферы. Диэлектрическая проницаемость среды е = 1.
627.	Потенциал точки, находящейся на расстоянии 10 см от поверхности заряженной сферы радиусом 5 см, равен 20 В. Определить потенциал в центре сферы.
628.	Внутри равномерно заряженного шара имеется сферическая полость. Нарисуйте силовые линии электрического поля внутри полости.
629.	Два небольших металлических шарика, расположенных па некотором расстоянии друг от друга, подключены к источнику постоянного тока. Как изменится сила притяжения между шариками, если их погрузить в жидкий диэлектрик с диэлектрической проницаемостью е без изменения расстояния между ними?
630.	Оценить потенциал поверхности Земли, если известно, что Земля заряжена отрицательно, причем напряженность электрического поля вблизи земной поверхности £з= 130 В/м. Объяснить, почему электрическое поле Земли не опасно для людей, хотя оно
162
способно создать между пятками и головой напряжение около 200 В.
631.	Металлический шарик радиусом /? = 14,4 см заряжен положительно до потенциала ср = 150 В. Какое количество электронов надо перенести на шарик, чтобы он стал электрически нейтральным? Диэлектрическая проницаемость среды е =- 1.
632.	Ядро атома ?°Ne ускоряется разностью потенциалов U = 2 кВ. Какова скорость ядра? Массу нуклонов принять равной 1,67 • 10“27 кг. Дефектом массы пренебречь. Начальная скорость ядра равна нулю.
633.	Элемент атомной батареи представляет собой конденсатор, на одну из пластин которого нанесен радиоактивный препарат, ис
Рис. 201
Рис. 202
пускающий а-частицы со скоростью и0 = 2,2 • 10е м/с. Определить э. д. с. этого элемента. Отношение заряда а-частицы к ее массе k = 4,8 • К)7 Кл/кг.
634.	Определить силу, действующую на единицу поверхности (электрическое давление) сферы, радиус которой R = 20 см, равномерно заряженной зарядом Q = 8,85  10~8 Кл.
635.	Какой максимальный заряд можно сообщить шару радиусом R = 2 м из металлизированной ткани толщиной Д7? = 0,02 см? Предел прочности на разрыв материала стенки / = 5 • 107 Н/м2. Изменением объема шара пренебречь.
636.	Равномерно заряженная сфера радиусом R = 10 см разрезана на две части плоскостью, проходящей через центр сферы. Найти силу отталкивания этих частей. Заряд сферы Q = 17,7  10~10Кл.
637.	Незаряженный металлический шар радиусом г окружают концентрической сферической проводящей оболочкой, радиус которой R, а потенциал <р. Чему станет равен потенциал оболочки, если незаряженный шар заземлить (рис 201)?
638.	Три концентрические металлические сферы вставлены одна в другую (рис. 202). Заряд внешней сферы =35,4 • 10"10 Кл, ее радиус Ra = 1 м. Заряд внутренней сферы радиусом равен
17,7  16 10 Кл. Сфера, радиус которой R2= 0,5 м, заземлена. Найти заряд этой сферы.
6*
163
639.	Сосуд с маслом, диэлектрическая проницаемость которого е = 5, помещен в однородное электрическое поле, напряженность которого направлена вертикально вверх. В масле находится во взвешенном состоянии алюминиевый шарик диаметром d = 3 мм, имеющий заряд q = 10'7 Кл. Определить напряженность электрического поля, если плотность масла рм = 0,9 • 10а кг/м3.
640.	Шар, диаметр которого d — 1 см и заряд q = 10-в Кл, помещен в масло плотностью рм = 0,8 • 103 кг/м3. Плотность материала шара р = 8,6 • 10а кг/м3. В какое электрическое поле, направленное вертикально вверх, надо поместить шар, чтобы он плавал в масле?
641.	Однородный медный шар диаметром d = 1 см помещен в масло. Плотность масла р„= 800 кг/м3. Чему равен заряд шара, если в однородном электрическом поле шар оказался взвешенным в масле? Напряженность электрического поля направлена вертикально вверх и равна Е == 3,6 • 104 В/м.
642.	Восемь одинаковых маленьких капель ртути, наэлектризованных до потенциала <р = 200 В каждая, сливаются в одну. Определить потенциал <р' капли. Все капли считать шариками.
643.	Три одинаковых заряда q = 10“° Кл расположены в вершинах равностороннего треугольника со стороной г — 20 см. Какую работу надо совершить, чтобы расположить эти заряды в одну линию с расстоянием 10 см между соседними? Заряды находятся в вакууме.
644.	Три одинаковые заряженные частицы, каждая с массой т и зарядом q, поместили в вершины углов равностороннего треугольника со стороной г0. Затем частицы одновременно освободили, после чего они стали симметрично разлетаться под действием кулоновских сил отталкивания. Определить скорость каждой частицы в зависимости от расстояния г между частицами и работу, которую совершили кулоновские силы, действующие на каждую частицу, при разлете частиц на очень большое расстояние друг от друга.
645.	Между пластинами плоского конденсатора, находящимися на расстоянии d = 1 см друг от друга, приложена разность потенциалов U = 300 В. В пространство между пластинами помещаются две плоскопараллельные пластинки: из стекла толщиной d± — 0,3 см и из парафина толщиной d2 = 0,7 см. Найти напряжение на каждом слое. Диэлектрические проницаемости равны соответственно et = = 6 и е2= 2,6.
646.	Плоский конденсатор, пространство между пластинами которого заполнено керосином (е = 2), расположен вертикально, заряжен и отключен от источника напряжения. Напряженность электрического поля при этом в керосине Е = 2 • 10° В/м. Из-за дефекта в изоляции керосин начинает вытекать, а его место занимает воздух. Предельная напряженность электрического поля в воздухе, при которой наступает электрический пробой (разряд), Епр = = 3 • 106 В/м. Какая доля керосина р вытечет из конденсатора к моменту пробоя конденсатора?
164
647.	Незаряженный плоский конденсатор емкостью Сг = 1 мкФ расположен во внешнем однородном электрическом поле с напряженностью Е = 104 В/м. Силовые линии поля перпендикулярны к пластинам конденсатора. Расстояние между пластинами с!=10"3м. Конденсатор емкостью С2 = 2 мкФ, заряженный до разности потенциалов U = 10 В, подключается к конденсатору, емкость которого Cj (рис. 203). Определить заряды на каждом конденсаторе после подключения. Напряженностью внешнего электрического поля в месте нахождения конденсатора емкостью С2 пренебречь.
648.	Конденсатор емкостью С заполняется диэлектриком с диэлектрической проницаемостью е. Какой конденсатор должен быть включен последовательно с данным, чтобы их емкость осталась равной С?	________________________
649.	Три последовательно соеди- _______________________
ненных конденсатора с емкостями --------- С|ц ---------------
С2 и С3 подключены к батарее с э. д. с.  	II -----------
&. Каковы разность потенциалов и за--------------------------
ряд на каждом конденсаторе?	-------------------—----
650.	Имеются два конденсатора /	/
переменной емкости с диапазоном
каждого от 10 мкФ до 200 мкФ. Ка- _____________(kll_____I
ким будет максимальный диапазон	"
изменения емкости батареи из этих	Рис. 203
конденсаторов?
651.	Заряженный до напряжения = 210 В плоский воздушный конденсатор соединяют параллельно с таким же по размерам конденсатором, полностью заполненным стеклянной пластинкой, но незаряженным. Определить диэлектрическую проницаемость стекла, если напряжение на этой батарее конденсаторов станет равным U2 = 30 В.
652.	Электроны, вылетающие с одной из пластин заряженного плоского конденсатора, достигают другой пластины, имея скорость V. Как изменится скорость электронов, если к этому конденсатору подключен незаряженный конденсатор с увеличенной вдвое емкостью?
653.	Плоский воздушный конденсатор с емкостью С = 1 мкФ заряжен до разности потенциалов U = 120 В и отключен. Расстояние между обкладками конденсатора уменьшают вдвое. Определить работу, совершенную полем при перемещении пластин.
654.	Плоский конденсатор с емкостью С = 10 мкФ заряжен до разности потенциалов U = 1 кВ. Затем конденсатор отключают от источника и замыкают проволочкой. Какая энергия выделится в проволочке при разряде конденсатора?
655.	Плоский конденсатор получает от аккумулятора заряд q — 1,2 • 10~9 Кл. Между обкладками конденсатора находится диэлектрик с диэлектрической проницаемостью е = 6. Какой заряд пройдет через аккумулятор при'ьудалении диэлектрика из конденсатора?
165
656.	Между двумя горизонтальными пластинами, находящимися на расстоянии 1 см друг от друга, помещена маленькая капелька массой m = 2  10’12 г, обладающая зарядом q =8 • 1010 Кл. Какую разность потенциалов необходимо приложить к пластинам, чтобы капелька находилась в равновесии?
657.	Пылинка массой т, обладающая зарядом q, движется в однородном электрическом поле вертикально вверх с ускорением а. Определить напряженность электрического поля.
658.	Маленький шарик массой т = 105 г, несущий заряд q = = 10~5 Кл, помещен воднородное электрическое поле, направленное горизонтально. Шарик приходит в движение и через т = 4 с приобретает скорость v = 50 м/с. Найти напряженность Е поля.
659.	Конденсатор составлен из трех	А
плоских параллельных прямоугольных пластин (рис. 204). Расстояния между
Рис. 204
Рис. 205
пластинами 1г= 1 мм и 12 = 10 мм. Площадь пластин S = 500’см2. Определить заряд q на средней пластине, если разность потенциалов между точками А и В U — 100 В. Пространство между пластинами заполнено диэлектриком с диэлектрической проницаемостью е = 2.
660.	Три одинаковых конденсатора (рис. 205) подключены к точкам А, В и С, потенциалы которых соответственно срл = 100 В, <рв = 60 В и <рс = 50 В. Определить потенциал точки D.
661.	Электрон, получивший скорость под действием разности потенциалов U = 5  103 В, влетает в пространство между пластинами плоского конденсатора параллельно им. Какую наименьшую разность потенциалов нужно приложить к конденсатору, чтобы электрон не вылетел из него? Длина конденсатора I — 5 см, расстояние между пластинами d — 1 см.
662.	Плоский воздушный конденсатор после зарядки отключают от источника и погружают в жидкий диэлектрик. Что произойдет с энергией конденсатора?
663.	Плоский воздушный конденсатор с расстоянием между пластинами d = 3 см подключен к источнику э. д. с. В = 900 В. В конденсатор параллельно его пластинам вставлена металлическая заряженная плита толщиной = 1,5 см так, что зазоры слева и справа от плиты оказываются одинаковыми. Зиряд на плите 166
<2 =. И)-’ Кл. Определить электростатическую силу, действующую на плиту.
664.	В плоский конденсатор вставлена металлическая пластина толщиной dt с двумя диэлектрическими пластинами по бокам, толщина каждой из них d2, диэлектрическая проницаемость е (рис. 206). Форма и размеры вставки соответствуют форме и размерам пластин конденсатора. Расстояние между пластинами конденсатора несколько больше суммарной толщины вставки и равно d. Конденсатор со вставкой подключают к источнику постоянного напряжения Ua. После того как конденсатор зарядился, источник отключают. Какая разность потенциалов будет между пластинами конденсатора, если после отключения вставку вынуть?
Рис. 206
Рис. 207
665.	Плоский воздушный конденсатор емкостью С = 30 пФ с площадью пластин 5 = 500 см2 подключен к источнику с э. д. с. & = 39 кВ. Пробьется ли конденсатор, если внутрь его ввести параллельно пластинам стеклянную пластинку толщиной h —• 3 мм? Предельная напряженность электрического поля в воздухе 3 х X 10е В/м, в стекле — 107 В/м. Диэлектрическая проницаемость стекла е = 7.
666.	Плоский конденсатор, расстояние между пластинами которого d — 1 см, находится в вакууме и расположен так, что сила тяжести пер г еядикулярна к ею пластинам (рис. 207). Верхняя пластина конденсатора закреплена, а нижняя, изготовленная из фольги толщиной h = 0,1 мм, лежит на изолирующей подставке. До какого напряжения U надо зарядить конденсатор, чтобы нижняя пластина перестала давить на опору? Плотность фольги р = 2830 кг/м3. Массой соединительных проводов пренебречь.
667.	В системе (рис. 208) радиус внутренней проводящей сферы R, внешней (тоже проводящей) 3R. Расстояние от центра системы до заряда —q равно 2R. Зная q, & , R, определить заряд на внутренней сфере. Потенциал Земли принять равным нулю.
668.	В системе (рис. 209) радиус внутренней проводящей сферы R,^внешней (тоже проводящей) 2R. Расстояние от центра системы до заряда —q равно 3R. Зная q, S, R, определить заряд на внешней сфере. Потенциал Земли принять равным нулю.
669.	В плоский конденсатор, подключенный к источнику с э. д. с. <§, помещена плоская пластина, имеющая заряд q. Расстоя-
167
ния от пластины до обкладок dx и d2 (рис. 210). Площадь пластины S. Определить силу, действующую на пластину со стороны электрического поля.
Сг
Рис. 213
670.	Определить количество теплоты, выделившееся на резисторе /? (рис. 211) после замыкания цепи, если максимальная сила взаимодействия между обкладками плоского конденсатора F. Расстояние между обкладками d.
671.	Определить заряд q конденсатора емкостью С = 3 мкФ (рис. 212). Внутренним сопротивлением батарей пренебречь. В = 2 В.
672.	При каком условии переключение выключателя (рис. 213) из положения Л в положение В не приведет к изменению напряжения на конденсаторе емкостью CJ
673.	В схеме (рис. 214)	10 Ом и Т?2 =
= 20 Ом. Найти отношение напряжений на конденсаторах, емкости которых Cj и С2.
674.	На однородный стержень, оба конца которого заземлены (рис. 215), падает пучок электронов, причем на каждый сантиметр длины стержня в секунду попадает одно и то же число электронов.
168
Сопротивление стержня R. Сила тока на участке заземления CD равна /. Найти разность потенциалов между серединой стержня А и его концом В.
Рис. 214
§ 13. Постоянный электрический ток
Задача 1. Каков заряд всех электронов в куске алюминия массой 1 кг? За какое время пройдет этот заряд через поперечное сечение проводника при силе тока в 1 А?
Решение В куске алюминия массой т содержится N = Na
атомов, где A4Ai = 0,027 кг/моль — молярная масса алюминия, Na — число Авогадро.
Каждый атом алюминия содержит Z = 13 протонов в ядре и столько же электронов. Поэтому в куске алюминия массой т электронов будет
М = NZ = ^Z-
Заряд всех этих электронов
?А1 = eNi = sZNa ~	4,6- 10’ Кл.
™А1
При условии постоянства силы тока этот заряд пройдет через поперечное сечение проводника за время
о,, eZNt.ni
I =	= —А « 4 6 . 107 с.
/ /МА|
Задача 2. Ток, протекающий по проводнику, изменяется по закону / = /ое ~tt<, где /0, а — размерные постоянные, t — время. Найти количество электричества, прошедшее через поперечное сечение проводника за время Т.
Решение. По определению, сила тока равна производной ОТ заряда q(t), прошедшего через поперечное сечение проводника, по времени: / (/) =	. Поэтому заряд Q, прошедший через попе
169
речное сечение от момента = 0 до момента t2 = Т, будет равен интегралу от функции 1(f) в соответствующих пределах:
Q = f 1 (t)dt = /0 ( e-a!dt = (1 — е~аТ).
о	о
Задача 3. По проводнику течет электрический ток. Определить заряд, прошедший через поперечное сечение проводника за 10 с, если за время измерения сила тока возросла от 0 до /0 = 10 А линейно (/ (/) — kt, k — 1 А/с). Как изменится заряд, если закон изменения тока параболический (/(/) = х /2; х = 0,1 А/с2)?
Решение. Решение этой задачи аналогично предыдущей. Прежде всего необходимо определить и t2 — моменты начала и окончания процесса (пределы интегрирования):
ti = 0, 1 (12) = kt2 = /0, откуда t2 = Z-£- = 10 с,
где /0 = 10 А. Заряд, прошедший через поперечное сечение проводника от момента /х=0 до момента t2 — 10 с, будет
Q = j / (/) dt = k J tdt =	| $ —t\) = 50 Кл.
Аналогично находится заряд q, прошедший через поперечное сечение проводника за то же время, в случае, когда сила тока изменяется по параболическому закону 1 (/) = х/2:
q = j I (/) dt = x J t2dt =	|= % $ - tf) = 33 Кл.
Задача 4. Гальванометр с внутренним сопротивлением т = = 40 Ом имеет цену деления 6 = 10 мкА. Как из него сделать амперметр для измерения силы тока до I = 0,1 А или вольтметр для измерения напряжения до U = 50 В, если шкала прибора имеет N == 100 делений?
Решение. Максимальная сила тока, доступная измерению данным гальванометром, I max =	= Ю~3 А. Если сила тока в це-
пи не превышает /тах, то гальванометр включают в цепь непосредственно. Для измерения силы тока I > / тах гальванометр шунтируют, включая шунт параллельно гальванометру (рис. 216, а). Применяя закон сохранения заряда и закон Ома для участка АВ цепи, получим следующую систему уравнений:
I = /щ + /г, Uан — /ш/?ш — /рГ.
Из этих уравнений
170
Таким образом, для измерения с помощью данного гальванометра тока, сила которого не превышает /, необходимо использовать шунт с сопротивлением
D	Nfi	г NS	п, п
Rm = Г j—	~ = 0,4 Ом.
Если гальванометр подключить параллельно участку цепи АВ, то о напряжении U ан можно судить, измерив силу протекающего через гальванометр тока:
При этом, конечно, измеряемое напряжение ограничено сверху значением U тах:
UАВ	Umax = Iтая/•
Для измерения напряжений U > Umax необходимо последовательно с гальванометром подключить сопротивление R д (рис. 216, б):
и = /max (Ra + г), откуда /?д = ----г =  — г = 5 • 104 Ом.
'max
В качестве полезного упражнения предлагаем читателю определить новые цены делении гальванометра при использовании его в качестве амперметра и вольтметра в общем случае и для численных значений / и U, приведенных в условии задачи.
Задача 5. Для измерения больших токов в цепи СС (рис. 217) используется шунт Ш, параллельно которому подключается измери
171
тельный прибор с внутренним сопротивлением г = 10Ом через резисторы с сопротивлениями гг и г2. В положении А переключателя вся шкала прибора соответствует силе тока в цепи СС /х = 10 А. Каким надо взять сопротивление г2, чтобы в положении В переключателя вся шкала прибора соответствовала /2 = 100 А? Сопротивление гх = 2 Ом. Сопротивление шунта намного меньше г и гх.
Решение. Любой прибор для измерения силы тока (амперметр, гальванометр) позволяет измерять вплоть до некоторой максимальной силы тока /гпах. Для измерения токов, больших /тах, с помощью того же гальванометра используются шуиты. Сопротивление шунта выбирают намного меньшим, чем сопротивление гальванометра, и подсоединяют шунт параллельно гальванометру.
Если переключатель находится в положении А, то схема задачи оказывается эквивалентной схеме, изображенной на рис. 217, б. Из первого правила Кирхгофа следует, что
ЦА) = /</) + /<Л	(1)
Записав двумя различными способами разность потенциалов Т/12 между точками 1 и 2 схемы или применив второе правило Кирхгофа для замкнутого контура 1 — 2 — 3 — 4, получим второе уравнение
/(ЛШ = /(гЛ)(г+и),	(2)
где гш — сопротивление шунта, которое по условию задачи намного меньше остальных сопротивлений схемы:
гш <cmin {г,
Подставляя найденное из уравнения (2) /щ ’ в уравнение (1), получим
Аналогично проводится рассмотрение цепи (рис. 217, в), возникшей при положении В переключателя:
Обратимся теперь к условию задачи. Фраза «В положении А переключателя вся шкала прибора соответствует силе тока в цепи СС /х = 10 А» означает, что при силе тока в цепи СС, равной /ь через гальванометр проходит ток /тах, при котором стрелка прибора отклоняется на наибольший возможный угол. Таким образом,
№ = /max И /И) = /х.	(5)
Аналогично смысл фразы условия задачи, касающейся положения В переключателя, сводится к следующему:
/!•*' = /max И /<*> = /а.	(6)
172
Подставляя (5) и (6) соответственно в (3) и (4) и пренебрегая гш в знаменателях дробей, получим:
_ гш / ~	/
~ Г + Г1 + Гш 1 ~ r+rL 11
(7)
(8)
получим
Приравнивая теперь правые части уравнений (7) и (8),
Л = __с+_О_ 2 г + Г1 + гг
Разрешая это уравнение относительно г2, находим
гг =	^(r + rj = 108 Ом.
ч
Задача 6. Пространство между пластинами плоского конденсатора заполнено жидкостью с диэлектрической проницаемостью е и удельным сопротивлением р. Найти силу взаимодействия между пластинами конденсатора, когда через конденсатор проходит ток J. Площадь пластин конденсатора 5.
Решение. Применив закон Ома для участка цепи, найдем разность потенциалов между пластинами конденсатора
г г _
S'
откуда заряд на пластинах q = еоер/. Вычисление силы взаимодействия между пластинами производится аналогично тому, как это сделано в решении задачи 13 из § 12, что приводит к следующему результату:
,, _ еое/2р2
S ’
Задача 7. Напряжение с резистора, сопротивление которого /?2 = 10 Ом, подается на горизонтальные пластины осциллографа, а напряжение с конденсатора емкостью Со = 104 пФ — на вертикальные пластины (рис. 218). Коэффициенты усиления обоих каналов осциллографа равны. Если на вход схемы подать переменное напряжение, то на экране осциллографа будет наблюдаться отрезок прямой под углом а = 60° к оси х. Определить диэлектрическую проницаемость е диэлектрика, заполняющего плоский конденсатор, емкость которого Сх <g Со, площадь пластин 3=4 см2, расстояние между пластинами d =0,1 см. Сопротивление /?!= 6,1 кОм.
Р е ш е н и е. На экране осциллографа наблюдается отрезок прямой под углом а = 60° к оси х, это и есть траектория электронного луча. Вектор результирующей силы F коллинеарен этой же прямой —>-	—
(рис. 218, б), его можно разложить на составляющие: Fr — qEr и Fa — qEa, где q — заряд частицы пучка (в нашем случае q — — е),
173
Ег и Ея — векторы напряженности полей, создаваемых горизонтальными и вертикальными пластинами осциллографа соответственно. Из предыдущих уравнений
Ur(t), UB(t), а следовательно, и Ег, Ев изменяются с течением времени по гармоническому закону:
Ur (/) = UOr cos со/ и UB (/) = UOb cos со/,	(3)
где ш — циклическая частота переменного тока, используемого для питания осциллографа, a Uor и 4/Ов — амплитудные значения напряжений. Подстановка (2) и (3) в формулу (1) приводит к выводу
Uqb — Uо? tg а.	(4)
Итак, тот факт, что электронный луч описывает на экране осциллографа отрезок прямой, наклоненной к горизонтальной оси под углом а, означает, что имеет место определенное линейное соотношение (4) между амплитудами подаваемых на пластины осциллографа напряжений.
Напряжения Ur(t) и UB(t) подаются на пластины осциллографа с двух различных делителей напряжений (рис. 219, а), и каждое из них составляет определенную часть входного напряжения:
t/вх (С = Uо cos со/.	(5)
Какую именно часть— определяется параметрами делителей напряжения. Рассмотрим последовательно оба случая.
174
a) Ur(t) снимается с резистора сопротивлением /?2(рис. 219,а) делителя напряжения А —	R2— В. Применяя закон Ома для
участка АВ цепи, получим
UB. (t) = I (t) (Rt 4- Т?2), откуда 1 (t) =	.
Тогда
Cr(/) = /(Z)
Из (6) непосредственно следует соотношение между значениями
(6)
амплитудными
Uor = U0Tr4
(7)
Вывод (7) означает, что в каждый момент времени напряжение UY составляет такую часть входного напряжения, какую часть сопротивление /?2 резистора, с которого снимается Ur, составляет от общего сопротивления делителя Wi+ RJ-
б) Ua(t) — снимается с конденсатора Со (рис. 219, б) делителя напряжения А —Сх— Со—В. Рассмотрение проводится аналогично тому, как это сделано в предыдущем случае. Единственное раз-
личие: вместо активных сопротивлений и в новом делителе напряжения действуют реактивные (емкостные) сопротивления:
Хс* = ’ Хс° = •
Поэтому искомый результат получится из формулы (7), если в ней сделать следующие замены:
UOr^UoB,
Ao
Urtt)
A
flo-------I-----— go-----------
a	J
Рис. 219
что приводит к формуле
(/ов = с4с-^-	(8)
Lx тьо
Подставив выражения (7) и (8) для амплитуд напряжений в формулу (4), выразим Сх через Со:
г _	/?2 tg а
R, + R2 (1 - tg а) Со-Поскольку Сх =	, то
р__ dCx________dC0R2 tg ct___ __ q
80S eeS -f- R2 (1 — tg a)]
dC0R2tgct




175
Задача 8. Напряжение с резистора сопротивлением R2 (Rx/R2 = =2) подается на горизонтальные пластины осциллографа, а напряжение с конденсатора емкостью Со (С0/Сх = 3) — на вертикальные пластины (см. рис. 218, а). Коэффициенты усиления обоих каналов осциллографа равны. Определить тангенс угла а наклона прямой, возникающей на экране осциллографа, если на вход схемы подать переменное напряжение.
Решение. Решение этой задачи аналогично решению задачи 7. Прежде всего необходимо установить связь между амплитудами подаваемых на пластины осциллографа напряжений и углом а. Далее, анализ работы омического и емкостного делителей напряжения, с которых подаются напряжения на горизонтальные и вертикальные пластины соответственно, позволяет выразить С\ через
С2. Коэффициент этой прямой пропорциональности содержит tg а, поэтому можно выразить последний через безразмерные отношения R2/R2 и С21Сх-
^а = §7?ТГ = 0’75-
Задача 9. В цепи (рис. 220, а) увеличили сопротивление Rlt при этом возросла сила тока /. При каком соотношении между и это возможно? Внутренними сопротивлениями источников э. д. с. пренебречь.
Решение. Необходимо выразить силу / протекающего через резистор сопротивлением R3 тока через &lt ti2 и сопротивления Ri(i= 1, 2, 3) и исследовать полученное выражение как функцию Rt. Сначала выясним, ток какой силы создает в резисторе сопротивлением R3 источник тока с э. д. с. Поскольку внутренние сопротивления источников тока пренебрежимо малы, то рассматриваемая схема эквивалентна изображенной на рис. 220, б. Общее сопротивление этой цепи
= Rt + Ri + •
Тогда сила тока, протекающего через резистор сопротивлением Rlt по закону Ома для полной цепи, /J” =---------—> а сила тока
п I ^2^3 ''йГТХз
176
/<» определяется из системы уравнений
Л" + Д1’ = л1’-, /УЧ = /УЧ-
Отсюда
др _ _____% 1^2___
3 - R^+RiRz + R^Ri ‘
(1)
Аналогично можно вычислить силу тока Лз1, создаваемого в резисторе, сопротивление которого /?3, источником тока с э. д. с. $2. Соответствующая этому случаю эквивалентная схема приведена на рис. 220, в. Можно поступить и иначе. Эта схема отличается от схемы (рис. 220, б) лишь заменой <§2, Ri^- R%. Поэто-
Рис. 221
му и формула для /^2) получится из формулы (1), если в последней индекс «1» заменить на индекс «2» и наоборот:
/(2)	________^2^1_______
3 RiRz + R1R3 + R2R3
Сила тока /, протекающего через сопротивление Ra в рассматриваемой схеме, вследствие действия обоих источников тока, будет равна
7ГП \ лЧ) , /(2)	1R 2 + £ 2R1
+/з -+ ^, + ^3 •
Теперь для ответа на вопрос задачи достаточно выяснить свойства функции I (Rr). Вычислим производную этой функции по переменной Ri.
____________^3^3_______ I со _ /1-|- — \v 1 dRi (R1Ri + RlR3+ Я2Яз)'2Р С М’
Увеличение /?j приведет к возрастанию / (7?i) в том случае, если / Ч) — возрастающая функция Rlt а это означает, что
Задача 10. В начальный момент выключатель 5 замкнут (рис. 221, а) и в цепи проходит постоянный ток. Затем выключатель
177
размыкают. Определить количество теплоты, которое выделится в резисторе сопротивлением при следующих параметрах цепи: э. д. с. источника й0, внутреннее сопротивление г, сопротивления резисторов и /?2 равны R, емкость конденсатора С.
Решение. На рис. 221, б приведена рассматриваемая цепь и указан замкнутый контур, по которому проходит постоянный ток
Г __	^0_____
о /?, + /?, + '•
Конденсатор при этом заряжен, причем разность потенциалов между пластинами
При размыкании выключателя S ток в основной цепи прекращается (рис. 221, в), так как источник оказывается отключенным и начинается процесс разрядки конденсатора через резистор сопротивлением /?!• В силу закона сохранения энергии количество теплоты Q, выделившееся в этом резисторе, равно исходной энергии заряженного конденсатора:
п_сио _ с^о / Ъ V
Ч 2	2 [R.+R. + r) '
По условию задачи, R1 = #2 = г = R, следовательно,
Задача 11. В схеме (рис. 222) в начальный момент времени выключатель разомкнут, а конденсатор емкостью С заряжен до разности потенциалов Un. Определить, какое количество теплоты выделится в каждом сопротивлении Rlt R2 и R3 после замыкания выключателя.
Решение. В силу закона сохранения энергии сумма всех трех количеств теплоты должна быть равна начальной энергии заряженного конденсатора. Два других уравнения легко получить, рассмотрев отноше
ния QJQ2 и QJQ3 и выразив их через известные сопротивления Ri, R2 и R3. При этом следует учесть, что количество теплоты Qi пропорционально мощности JVt-, выделяющейся в t-м резисторе, т. е. пропорционально квадрату силы тока /г, протекающего через резистор сопротивлением Ri. Решив полученную систему трех линейных уравнений с тремя неизвестными Qt (t= 1, 2, 3), получим
178
R2 + R3 CU20	_________RlR2	си2
41 ~ /?1 + R2 + R3 2 ’ 42	(R, + R3) (RL 4- R2 + Ra) 2
n____________RiR3 CU*
43 (R2 + R3) (Ri + R2 + R3) 2
ЗАДАЧИ ДЛЯ САМОСТОЯТЕЛЬНОГО РЕШЕНИЯ
675, Электрическая цепь состоит из трех последовательно соединенных кусков провода одинаковой длины, сделанных из одного материала, но имеющих разные площади поперечного сечения: $1== 1 . Ю'в ма, S2 = 2  10’6 м2и S3 = 3  10"в м2. Разность потенциалов на концах цепи U = 11 В. Определить падение напряжения на каждом проводнике.
676.	Какой силы ток проходит по железному проводнику длиной I =0,5 м и диаметром d =0,6 мм, если удельное сопротивление железа р = 1,6 • 10~7 Ом • м, а напряжение на концах проводника U = 1,6 В?
677.	Сила тока на участке цепи равномерно возрастает в течение /j = 3 с от 0 до /3 = 0,5 А, а затем в течение следующих /2 = 5 с возрастает до /2= 1 А. Определить среднюю силу тока.
678.	Амперметр с внутренним сопротивлением = 2 Ом, подключенный к зажимам батареи, показывает I = 5 А. Вольтметр с внутренним сопротивлением /?в =150 Ом, подключенный к зажимам той же батареи, показывает U = 12 В. Найти ток короткого замыкания.
679.	Амперметр с внутренним сопротивлением г =0,1 Ом рассчитан на измерение сил токов до /1=2А. Каким должно быть сопротивление шунта R, подключенного к амперметру, чтобы можно было расширить пределы измерения силы тока в п = 5 раз?
680.	Найти сопротивление медной проволоки R, масса которой т, а площадь поперечного сечения S. Плотность меди р0 и ее удельное сопротивление р считать известными.
681.	Чему равно сопротивление Rx цепи между точками А и В, если она составлена из одинаковых элементов, сопротивление каждого из которых R (рис 223, а и б)?
682.	Восемь одинаковых резисторов, сопротивление каждого из которых R = 15 Ом, образуют цепь (рис. 224). Источник напряжения подключен к точкам А и О. Найти сопротивление RAo цепи.
683.	Ток идет по проводнику, форма которого показана на рис. 225. Одинакова или различна напряженность поля в местах с узким и широким сечением? Почему?
684.	Какими должны быть сопротивления г (рис. 226), чтобы входное сопротивление цепи между точками А и В равнялось R = 1,7 Ом?
685.	Найти заряд на заземленной пластине конденсатора (рис. 227).
686.	Для измерения больших токов в цепи СО (см. рис. 217, а) используется шунт Ш, параллельно которому подключается измери-
ла
Рис. 233
Рис. 232
тельный прибор через резисторы, сопротивления которых /4 = = 2 Ом и г2 = 90 Ом. В положении А переключателя вся шкала прибора соответствует силе тока /1= 10 А, в положении В = силе тока /2 = 100 А. Найти внутреннее сопротивление измерительного прибора. Сопротивление шунта намного меньше, чем гг и г2.
687.	Определить максимальную силу взаимодействия между обкладками плоского конденсатора емкостью С (см. рис. 211), если максимальный ток в цепи /0. Расстояние между обкладками d, сопротивление R.
688.	Какого диаметра был выбран железный провод длиной I = 5 м, если после замыкания источника тока с э. д. с & = 1,5 В и внутренним сопротивлением г = 0,2 Ом по нему проходит ток / = 0,6 А?'Удельное сопротивление провода р = 9,8 • КГ8 Ом • м.
689.	В схеме (рис. 228) включены два элемента с э. д. с.
и <§2 ($2 > $i) и внутренними сопротивлениями г\ и г2. Сопротивление нагрузки R. Какое напряжение покажет вольтметр?
690.	Определить внутреннее сопротивление источника тока с э. д. с. fS = 22 В, если при подключении к нему резистора сопротивлением R = 4 Ом по нему идет ток / = 0, 5 А.
691.	Два аккумулятора с э. д. с. <SX = 1,5 В и <§2 =2Ви внутренними сопротивлениями гт== 0,5 Ом и г2= 0,3 Ом соединены так, как показано на рис. 229. Определить, какой силы ток идет по резистору, сопротивление которого R = 2 Ом.
692.	Найдите сопротивление R, при котором напряжение на зажимах элемента с э. д. с. <S2 равно нулю (рис. 230). Э. _д. с. элементов = 5 В и &2 = 3 В, их внутренние сопротивления ~ = 1 Ом и г, = 2 Ом.
693.	Замкнутая цепь состоит из источника и реостата, соединенных последовательно. В цепи проходит ток /0 = 0,5 А. Если сопротивление реостата уменьшить в k = 4 раза, сила тока возрастет в I = 2 раза. Какой силы ток / будет в цепи, если сопротивление реостата уменьшить до нуля?
694.	Определить полное сопротивление бесконечной цепи (рис. 231).
695.	На рис. 232 показан участок цепи, содержащий последовательно соединенные резисторы, сопротивления которых R,= = 3,7 Ом, R2 = 5,6 Ом, и источник э. д. с. ё = 5 В (внутреннее сопротивление источника пренебрежимо мало). По этому участку течет ток / = 1 А. Определить разность потенциалов между точками А и В, В и С, А и С.
696.	Электрическая цепь состоит из двух последовательно включенных источников э. д. с. и резистора. Может ли ток через резистор возрасти, если один из источников э. д. с. замкнуть проволокой?
697.	Дана электрическая цепь (рис. 233). Считая известными величины, обозначенные на рисунке, найти заряд на пластинах каждого конденсатора.
698.	В схеме (рис. 234) выключатель сначала не замкнут. На сколько изменится заряд конденсатора, если выключатель замкнуть?
181
Известно, что /?1= 10 кОм, /?2= 15 кОм, С = 1 мкФ, ё1= 34 В, ё2 = 9 В. Внутренним сопротивлением пренебречь.
699.	Цепь состоит из элементов с э. д. с. и $2, резисторов с сопротивлениями R и гг и реостата с сопротивлением г (рис. 235). При каком значении 6% сила тока, идущего через сопротивление /?, не зависит от сопротивления реостата? Внутренним сопротивлением источников пренебречь.
700.	Для определения места повреждения двужильного кабеля используют схему, показанную на рис. 236. Определить расстояние х до места, где один провод кабеля вследствие повреждения изоляции получил соединение с Землей, если через гальванометр не
Рис. 236	Рис. 237
проходит ток, когда сопротивления плеч реостата Rab = 150 Ом и Rhc = 50 Ом. Длина кабеля L = 10 км. Сопротивлением соединительных проводов схемы пренебречь (учитывать лишь сопротивление кабеля и реостата).
701.	К батарее с э. д. с. <§] = 1 В и ё2 = 2<§j подключены резисторы с сопротивлениями rY и г2 и конденсаторы, емкости которых и С2. При этом сопротивления резисторов связаны соотношением ra=2ri, а емкости конденсаторов — соотношением С2 = = 2С, (рис. 237). Найти разность потенциалов между обкладками конденсатора емкостью
702.	Конденсаторы емкостью С\ == 5 мкФ и Са= 2С\ подключены к источнику постоянной э. д. с. ё = 300 В. Затем переключатель S (рис 238) перевели с контакта 1 на контакт 2. Найти количество теплоты, выделившейся на резисторе сопротивлением = = 200 Ом, если R2 = 300 Ом.
703.	В схеме (рис. 239) вначале выключатель не замкнут. Какой заряд Q протечет через резистор с сопротивлением R, если выключатель замкнуть? $х= 10 В, ёа= 20 В, Сг = 1 мкФ, Са= 2 мкФ.
182
704.	В схеме (рис. 240) батарея с э. Д. с. & = 12 В и внутренним сопротивлением г =3 Ом подключена к цепи, состоящей из резисторов с сопротивлениями = 20 Ом, R2 = 25 Ом, R3 = 5 Ом и конденсатора емкостью С = 10 мкФ. Найти заряд конденсатора. Сопротивлением соединительных проводов пренебречь.
705.	В схеме (рис. 241) значения С, R, известны, при разомкнутом выключателе заряд левой обкладки плоского конденсатора равен нулю. Определить начальный заряд правой пластины конденсатора, если после замыкания ключа на резисторе R выделяется такое же количестве теплоты, как и в случае, когда конденсатор вначале не заряжен.
Рис. 238	Рис. 239
706.	В схеме, изображенной на рис. 241, С, R, S заданы, при разомкнутом выключателе пластины плоского конденсатора заряжены одноименными зарядами, сумма которых равна q0. Определить начальные заряды каждой из пластин, если после замыкания выключателя на резисторе R выделяется такое же количество теплоты, как и в случае, когда конденсатор вначале не заряжен.
§ 14. Электрический ток в вакууме, металлах, электролитах и газах
Задача 1. В одну из двух электролитических ванн, соединенных последовательно, наливают раствор хлорного золота. За время работы на катоде первой ванны выделилось гщ = 2 г меди. Сколько золота выделится на катоде второй ванны? Сколько атомов золота и меди отложилось на катодах? Валентность меди «j = 2, золота п2 = 3. Молярная масса меди Мг = 64 • 10“3 кг/моль, золота 197 • 10“3 кг/моль.
183
Решение. Поскольку электролитические ванны соединены последовательно, то через них проходит ток одной и той же силы: Л = /2 = /. Кроме того, они работают в течение одного и того же времени Д/1 = Д/2 = Д/.
Массы выделяющихся на катодах металлов
1 Мг .. ,	1 м„ ,. ,
т-, = тг -- /Дг и щ, =	- 2 /Дг,
1 F щ	2 F п2 ’
6 0,33 £0
т., М«Пл
тогда — =	. Отсюда
т2 М]П2
т2 =	& 0,004 кг.
2 м2п2 1
Число молекул, содержащихся в массе т вещества с молярной массой М,
Поэтому в рассматриваемом случае
МСи = ”*1 МА » 1,88  1022 атомов и /VI ।
МДи = ^2 МА « 1,22 • 1022 атомов.
Л м2 Л
Задача 2. Газоразрядная трубка заполнена гелием. Определить разность потенциалов между элек-_ тродами, при которой начинается 2г0 4х0 бх0 дх0 Юх0 х процесс ионизации атомов элек-рис. 242	тронным ударом, если известно, что
распределение потенциала в (относительных единицах) по длине трубки в этот момент имеет вид, изображенный на рис. 242, а, длина свободного пробега электронов I = 1СГ4 м, а потенциал ионизации атомов гелия ф,= 24,5 В. Решение. Работа, которую необходимо совершить, чтобы ионизировать нейтральный атом гелия,
Ai = ecpi,
где е — елементарный заряд. Ионизация электронным ударом начнется при условии, что работа, совершенная электрическим полем, по перемещению электрона на расстояние, равное длине свободного пробега электрона, А = еЕ1 станет равной А,:
еЕ1 = бфь
Поскольку напряженность электрического поля неодинакова в различных точках пространства внутри газоразрядной трубки, то для
184
начала процесса ударной ионизации достаточно, чтобы максимальная напряженность поля в трубке £тах удовлетворяла условию
4>i р — — А-тах — i •
Чтобы выяснить, в какой части газоразрядной трубки достигается fmax, необходимо по известному распределению потенциала вдоль длины трубки установить распределение напряженности электрического поля по ее длине.
График напряженности электрического поля Е (х) также построим в относительных единицах: Ео = Характер зависимости U (х) (рис. 242, б) позволяет разделить всю длину трубки L = 10хо на три области:
область х £ [О’, 2х01;
область //: х £ (2х0; 8х0];
область /// х £ [8х0; 10хо],
в каждой из которых потенциал является линейной функцией х. Графиками этих линейных функций являются отрезки прямых различного наклона. Используя связь между напряженностью элек-
г, Лю
трического поля и разностью потенциалов Е = можно наити напряженность в каждой из областей:
1	2
и 1	1 3	2	"з и о	1
Полученный результат позволяет построить график Е (х) и определить £тах (рис. 242, в): Етак = Et= Ео. Тогда
„	1 Ue <Pi	,, 2<Pi*o
fmax = J = 7 - Откуда Uo = -j- .
Разность потенциалов между электродами трубки U — 36/О, при которой начинается процесс ионизации электронным ударом,
и = — = 14,7 кВ.
ЗАДАЧИ ДЛЯ САМОСТОЯТЕЛЬНОГО РЕШЕНИЯ
707. Скорости двух электронов равны v и лежат в одной плоскости. Расстояние между электронами а. Скорости электронов образуют с прямой, соединяющей электроны, угол а (рис. 243). На какое минимальное расстояние сойдутся электроны?
708. Электрон влетает в плоский заряженный конденсатор, двигаясь в начальный момент вдоль средней плоскости А В конденсатора со скоростью v (рис. 244). Через какое время нужно изменить направление напряженности электрического поля в конденсаторе на противоположное, не изменяя ее значения, чтобы на вылете из кон-
185
денсатора электрон пересек плоскость АВ? Длина конденсатора I. Силу тяжести не учитывать.
709. Для измерения температуры применили железную проволоку, имеющую при t°i = 10 °C сопротивление = 15 Ом. При какой температуре сопротивление равно Т?2= 18,25 Ом? Температурный коэффициент сопротивления железа а = 0,006 К"1.
710. Угольный стержень ВС, температурный коэффициент сопротивления которого а±=—0,8- 10~3 К"1, соединен последовательно с железным стержнем АВ, температурный коэффициент сопротивления которого а2 = 6 • 10~3 К-1. Сопротивление участка АС не зависит от температуры. Чему равно напряжение между точками А и В при температуре /° = 100 °C, если к точкам А и С подключить батарею с э. д. с. S = 8,5 В? Внутреннее сопротивление батареи и сопротивление подводящих проводов не
учитывать.
711. Два проводника имеют при 0 °C сопротивления Bi= 1 Ом и А?2= 3 Ом. Определить температурный коэффициент сопротивле-
ния а проводников, соединенных последовательно, если темпера-
турный коэффициент сопротивления первого проводника ах =
= 3 • К) 3 К1, а второго — а2 = —2 • 10 3 К’1.
1,А
Рис. 245
712.	Установка для электролитического получения алюминия рассчитана на напряжение на ванне = 16 В и расход электроэнергии П/1== 64 кВт • ч при получении 1 кг алюминия. Сколько алюминия было получено, если электролиз проводился при напряжении U2= 15 В и было израсходовано W2 = 60 кВт • ч? Э. д. с. поляризации не учитывать.
713.	Мощность электролитической ванны Р — 40 кВт. К. п. д. установки г] — 75 %. Найти сопротивление электролита,если известно, что за 2 ч работы выделяется т =72 г никеля. Электрохимический эквивалент никеля k = 3 • 10“7 кг/Кл.
714.	Сила тока, проходящего через раствор медного купороса, изменяется по закону, изображенному на рис. 245. В течение одной
186
секунды сила тока растет равномерно От нуля до некоторого значения, а затем в течение следующей секунды равномерно убывает до нуля. Дальнейшее изменение силы тока происходит по тому же закону. Чему равна максимальная сила тока, если за 10 мин на электроде выделилось 200 мг меди? Химический эквивалент меди А1п — = 0,03177 кг/моль.
715. При напряжении на заполненной гелием трубке в 15 кВ начинается процесс ионизации атомов гелия электронным ударом. Распределение потенциала ср между электродами в этот момент изображено на рис. 246. Определить длину свободного пробега электронов, если потенциал ионизации атома гелия 24,5 В.
§ 15. Работа и мощность электрического тока
Задача 1. При получении катодных лучей к электродам разрядной трубки прикладывают напряжение U = 3 • 103 В. Вычислить максимальную скорость электронов в катодном пучке.
Решение. Максимальную скорость будут иметь электроны, которые не испытывают столкновений в процессе ускорения электрическим полем и, следовательно, не отдают кинетической энергии. Для таких электронов, в соответствии с законом сохранения энергии, имеем
А = Ек, или eU =	, откуда t>max = ]/"^ « 3,2 • 107 м/с.
Задача 2. При равномерном движении трамвая по горизонтальному участку пути двигатели развивают силу тяги F ~ 3 кН. Сила тока, потребляемая двигателем, / = 100 А, к. п. д. г] = 80 %, напряжение в сети U = 550 В. Определить скорость движения трамвая.
Решение. Мощность, потребляемая двигателем, Рпог = IU. Полезная мощность Р, развиваемая двигателем, Р = т|Р1|ОТ = ц/Д. Эта же мощность может быть вычислена и иначе: Р — Fv. Приравнивая правые части последних уравнений, получим
IU и с / ti = i] у я 14,6 м/с.
Задача 3. Какой длины и площади поперечного сечения нужно взять нихромовую проволоку для изготовления электрического кипятильника на Р = 480 Вт, если напряжение в сети U — 120 В, а допустимая плотность тока I = 10 А/мм2. Удельное сопротивление нихрома 1О‘в Ом • м.
Решение. Из формулы мощности Р = 1U = jSU площадь поперечного сечения проволоки
S = X = 4 • 10"’ м2.
187
Для определения длины I проволоки воспользуемся формулами R = <)~ и U = 1R. Из этих формул
U = /5^ = /р/, откуда / = ~ = 12 м.
Задача 4. Два одинаковых аккумулятора соединяют последовательно и замыкают на резистор, сопротивление которого равно R. Затем их соединяют параллельно и вновь замыкают на тот же резистор. В обоих случаях на резисторе выделяется мощность Ро = = 100 Вт. Какая мощность выделится на резисторе, если на него замкнуть один из аккумуляторов?
Решение. Рассмотрим последовательно три случая, описанных в условии задачи:
а)	Последовательно соединенные аккумуляторы замкнуты на резистор сопротивлением R. В этом случае сила тока в цепи
/ - 2/° а) R+2r’
а мощность, выделяемая на резисторе,
Ра) = /?/а2) = 7?(^_у.
б)	Параллельно соединенные аккумуляторы замкнуты на тот же резистор. В этом случае
/ 2<S \2 р ( 2S Г
в)	Один аккумулятор замкнут на сопротивление R. При этом
По условию задачи, Ра) = Рб}, или R
откуда R = г. Подставляя это значение в формулы мощности, получим
р -р, - 4 -р и р - 1 е 2
'а) *б)	g р 'о И 1	r •
Выразив отношение $ZIR через Ро, получим
Р,= £ Рв = 56,25 Вт. '	16 и
Задача 5. К аккумулятору с. внутренним сопротивлением г = = 1 Ом сначала подключили проволоку сопротивлением R = 2 Ом, затем параллельно подключили вторую такую же проволоку. Во сколько раз изменится мощность, выделяющаяся в первой проволоке, после подключения второй?
188
Решение. Определим отношение мощностей, выделяющихся в проволоке в двух описанных в условии задачи случаях.
а)	К аккумулятору подключена одна проволока. Сила тока /а>* протекающего через проволоку, 1а) = R , а мощность Ра), вы-/ й \2 деляющаяся в проволоке, Ра) =	= /?-=—.— •
\К г/
б)	К аккумулятору подключены параллельно две проволоки. Сила тока, протекающего через первую проволоку, 7, составит половину /б), так как проволоки одинаковы и соединены параллельно:
7	5	. 7 __ 1 Т	S
Уб> ~ 1	,	’ 71 ~ 2 'б> ~ R + 2T--
2R + r
Мощность, выделяющаяся в первой проволоке, / я \2
Ра).
Рб) =
Таким образом, после подключения выделяющаяся в первой проволоке,
Р_.	/ о 1
второй проволоки мощность, уменьшится в п раз:
1,7 раза.
п =
Задача 6. Рассчитать длину проволоки из нихрома, необходимую для намотки спирали плитки, на которой за А/ = 10 мин можно было бы вскипятить V = 2 л воды, имеющей температуру f= 20 °C. Удельное сопротивление нихрома р = 10-аОм • м, толщина проволоки d = 1 мм, напряжение в сети U = 220 В, к. п. д. установки г] = 70 %.
Решение. По определению, q = ~. В рассматриваемом слу-и2 °
чае Qn = св(рвУ) ДТ и Qo — % ДС Подставляя эти значения Q„ и Qo в формулу для т], найдем
п_. U2^ CePeVAT”
(1)
С другой стороны,
п 1 4PZ “ р S ~ псР ‘
Приравнивая правые части уравнений (1) и (2), получим уравнение для определения I, решение которого имеет следующий вид:
. лт№А/т] ._ / =	« 45 м.
4рсврв1М7’
Задача 7. Плитка сопротивлением 300 Ом подключена к сети с напряжением 1)й= 220 В. Какое количество энергии потребляет плитка за 300 с? Как изменится количество потребляемой энергии,
(2)
189
если напряжение с момента включения начнет уменьшаться по закону U (/) = Uae~at, где а =0,01 с"1.
Решение. Энергия, потребляемая электроплиткой в первом случае,
^0
Qi = -D kt «48,4 кДж. А
Во втором случае, когда напряжение является функцией времени, энергия Q2, потребляемая электроплиткой за время от момента = 0 до момента /2= 300 с, представляется в виде интеграла от мощности
и
P(t) = RU2(t) =
в соответствующих пределах:
^2
J P(t)dt

п2
=	0 (1 — е-Щ)
2aR v
& 8,07 Дж.
Во втором случае плитка потребляет электроэнергии в п раз меньше:
п = — « 2аД/ = 6.
Задача 8. В одно из плеч моста (рис. 247) включено нелинейное сопротивление X, для которого зависимость силы тока !х от приложенного напряжения Ux дается формулой: /х = kU3x, где k = 0,25 A/В3, если сила тока измеряется в амперах, а напряжение — в вольтах. Найти мощность Рх, расходуемую в нелинейном
проводнике в условиях, когда ток через гальванометр отсутствует. Сопротивление остальных плеч моста Rt = 2 Ом, R2~ 4 Ом и R3 = = 1 Ом.
Решение. Поскольку ток через гальванометр не проходит, это означает, что разность потенциалов между точками В и С равна нулю:
/г = 0, откуда UtlC = срй — срс = 0 или <рв = фс = ср.
Отсюда легко заключить, что разности потенциалов Uab (между точками А и В) и Uac (между точками А и С) одинаковы:
VАВ = <Рв - Фл = Ф - <РЛ и UАС = срссрл = ср - фл,
190
откуда Uab = Uac = U- Применив закон Ома для участков АВ и АС рассматриваемой цепи, получим
U = /Л и U = /2/?3, откуда .
Аналогичное рассмотрение участков BD и CD цепи приводит к следующему результату:
Л_В2
/2 X ’
Приравнивая правые части последних уравнений, получим
/?2	V ^2^3
п = у , откуда X = -%-?.
л	Ki
Для нахождения мощности Рх, расходуемой в нелинейном проводнике, используем данную в условии задачи зависимость
lx = kU'x.	(1)
Искомая мощность Рх = IxUx = kU\, а само неизвестное напряжение Ux найдем из закона Ома для участка цепи:
их
Приравнивая правые части уравнений (1) и (2) и разрешая получен-_ 1
ное уравнение относительно Ux, получим Ux=(Xk) 2. Окончательно
Задача 9. В одно из плеч моста (см. рис. 247) включен нелинейный резистор X, для которого зависимость силы тока 1х от приложенного напряжения U х дается формулой lx = kU\. Сопротивления остальных плеч моста /?1=/?3 = 2 Ом и R2 = 4 Ом. При каком значении константы k мощность Рх, расходуемая в резисторе X, равна 1 Вт для сбалансированного моста (т. е. при отсутствии тока через гальванометр)? Балансировка достигается изменением силы тока в цепи источника э. д. с.
Решение. Решение этой задачи аналогично решению предыдущей. Факт равенства нулю силы тока через гальванометр позволяет выразить неизвестное сопротивление X через известные сопротивления: X = R2Ra. Нелинейная зависимость от Uх, приведенная в условии задачи, в совокупности с законом Ома для участка цепи позволяет определить Ох ' Ux ~= (kX)~l. Используя полученные результаты, можно определить мощность электрического тока. Полученная таким образом формула, выражающая Рх через
191
R2, R3 и k, является уравнением для определения коэффициента k:
*=j/^(^ = 0,125 A/В2.
ЗАДАЧИ ДЛЯ САМОСТОЯТЕЛЬНОГО РЕШЕНИЯ
716.	Какая масса воды должна пройти через турбины гидроэлектростанции, падая с высоты Н = 20 м, чтобы обеспечить нормальное горение лампочки мощностью Р =60 Вт в течение t = 10 ч? К. п. д. л принять равным 25 %.
717.	В электрическую цепь включены последовательно медная и стальная проволочки, причем длина медной проволочки в два раза больше, чем стальной, а площадь поперечного сечения стальной проволочки в 4 раза больше, чем медной. Найти отношение напряжений на этих сопротивлениях и отношение выделяющихся на них мощностей.
718.	Э. д. с. батарейки для карманного фонаря ё = 4,5 В, внутреннее сопротивление г = 3,5 Ом. Сколько таких батареек надо соединить последовательно для питания электрической лампочки мощностью Р = 60 Вт, рассчитанной на напряжение U — = 127 В?
719.	Внутреннее сопротивление источника тока г =0,1 Ом. Внешнее сопротивление цепи с этим источником тока R = 4,9 Ом. При замкнутой цепи напряжение на зажимах источника тока U — — 1 В. Определить э. д. с. источника тока, к. п. д. и силу тока короткого замыкания.
720.	Определить э. д. с. и внутреннее сопротивление аккумулятора, если при силе тока /т = 15 А он отдает во внешнюю цепь мощность Р±— 135 Вт, а при силе тока /2 = 6 А — мощность Р2 = — 64,8 Вт.
721.	Электрический самовар имеет две обмотки. При включении обмотки Rj вода (10 °C) в самоваре закипает через 15 мин, при включении R2 — через 30 мин. а) Как были включены обмотки, если вода закипела через 10 мин? б) Какова была в начальный момент температура воды в самоваре, если при включении только обмотки R2 вода также закипала через 10 мин?
722.	Две проволоки из разных металлов соединены параллельно и подключены к сети. Отношение их длин 10 : 12, а отношение площадей поперечного сечения 5 : 4. Оказалось, что за одинаковое время в них выделилось равное количество теплоты. Определить > отношение удельных сопротивлений этих металлов.
723.	Какой наибольшей мощности электропечь можно установить в конце двухпроводной линии, имеющей сопротивление г = 10 Ом, если источник тока развивает мощность не более Р = 6 кВт при напряжении U = 1000 В?
724.	Элемент замыкается один раз проволокой сопротивлением /?! = 4 Ом, другой — сопротивлением R2 = 9 Ом. В обоих случаях количество теплоты, выделяющейся в проволоках за одно и то же
192
время, оказывается одинаковым. Каково внутреннее сопротивление элемента?
725.	Какой максимальный ток протекает через нагреватель мощностью Р = 500 Вт, если эффективное напряжение в цепи t/эф = 230 В?
726.	Лампочка накаливания мощностью Р = 180 Вт используется для обогрева аквариума, содержащего V = 10~3 м3 воды. За т = 2 мин вода нагревается на АТ = 3 К. Какая часть расходуемой лампочкой энергии теряется в виде лучистой энергии?
727.	Электрическая цепь представляет собой ромб с двумя диагоналями, изготовленный из одинаковых проволочек (рис. 248). Угол а = 60°. Цепь в точках Л и С подключена к источнику с напряжением U — 100 В. Определить количество теплоты Q, выделяющееся в единицу вре-мени в каждой из проволочек: АВ, ВС, CD,	в*-—/
AD, BD и АС. Сопротивление стороны	/\ / /
ромба Р = 250 Ом.	/ \/	/
728.	Две батареи с э. д. с. В1 и <§2 со- / /Х / единены последовательно и замкнуты на Ха/ резистор. Если полярность второй батареи изменить на противоположную, то коли-чество теплоты, выделяющееся в резисторе	рис. 248
за t = 1 с, уменьшится в п =9 раз. Найти В2, если <§1=6В. Внутренним сопротивлением батареи пренебречь.
729.	Аккумулятор сэ. д. с. <§а = 12 В заряжается от источника с э. д. с. В = 18 В через лампу накаливания, потребляющую при этом мощность Р = 24 Вт. Какое время t будет заряжаться аккумулятор, если его электрический заряд Q = 40 А • ч? Внутренним сопротивлением источника и аккумулятора пренебречь.
730.	Спираль электроплитки мощностью Р = 500 Вт при включении в сеть нагревается до /° = 700 °C. Какую мощность потребляет плитка в первый момент после включения, если начальная температура спирали t0 = 20 °C, а температурный коэффициент сопротивления нихрома, из которого она изготовлена, а = 4 • 10~4 К"1 при 20 °C.
731.	От понижающей трансформаторной подстанции, действующее напряжение на выходе которой U = 220 В, необходимо протянуть до потребителя двухпроводную линию электропередачи длиной L = 1 км. Какой должна быть площадь поперечного сечения S алюминиевого провода, которым прокладывается эта линия, если потребляемая в нагрузке мощность Р = 10 кВт, а действующее падение напряжения в линии не должно превышать At/ = 20 В? Нагрузка на конце линии чисто омическая.
732.	Определить стоимость кипячения 2,5 л воды с начальной температурой 20 °C в электрической кастрюле с к. п. д. т] = 70 % при тарифе 4 коп. за 1 кВт • ч.
733.	Определить э. д. с. и внутреннее сопротивление источника тока, если при силе тока Ц = 2 А во внешней цепи выделяется
7 7-253
193
мощность Pj = 6 Вт, а при силе тока /2 = 4 А — мощность Р2 = = 8 Вт.
734.	Электрическая плитка, имеющая сопротивление7?= 24Ом, включена в сеть с напряжением U = 120 В. На плитке вскипятили т = 1,5 кг воды, взятой при начальной температуре t„ ~ 10 °C, причем 10 % воды выкипело. Определить, как долго длилось нагревание, если к. п. д. плитки г, =45 %.
735.	Сопротивления двух ламп, включенных параллельно в сеть с напряжением 120 В, относятся как 3 : 2. Определить потребляемые лампами мощности и их сопротивления в рабочем состоянии, если сила тока в первой лампе составляет 0,4 А.
736.	Выведенный из ускорителя пучок электронов со скоростью v попадает ыа мишень. Определить выделяющуюся на мишени мощность Р, если сила тока составляет I, а потенциал мишени равен нулю. Масса электрона т, заряд е. Как изменится выделяющаяся на мишени мощность Р (<р), если потенциал мишени станет равным <р?
737.	Во сколько раз изменится мощность, выделяемая в проводнике сопротивлением Р1= 6 Ом, если параллельно ему подключить второй проводник сопротивлением Р2=9Ом? Внутреннее сопротивление источника г = 4 Ом.
738.	К источнику тока подключили потребитель сопротивлением Рр Затем его отключили и подключили другой потребитель сопротивлением Р2. Полезная мощность в обоих случаях оказалась одинаковой. Чему равно внутреннее сопротивление источника тока?
739.	Обмотка мощного электромагнита питается постоянным током и потребляет мощность Р = 5 кВт. Для предотвращения перегрева обмотки магнит снабжен охлаждающим устройством, по которому течет вода, поглощающая q = 84 % выделяющегося в обмотке количества теплоты. Определить необходимый расход воды (в кубических метрах в секунду), если температура воды не должна повышаться более чем на Л71 = 25 К-
740.	Два резистора сопротивлением Р = 200 Ом каждый подключаются к источнику тока сначала последовательно, а затем — параллельно. Мощность, выделяемая на каждом резисторе, оказалась одинаковой в обоих случаях. Найти э. д. с. источника, если при последовательном соединении резисторов в цепи протекает ток 1 = = 0, 5 А.
741.	Электродвигатель подключают к аккумулятору с э. д. с. й = 12 В и внутренним сопротивлением г = 0,1 Ом. В момент включения проходит ток /1= 24 А. В установившемся режиме через аккумулятор проходит ток /2=5А. Найти полезную мощность двигателя Рп и мощность, отдаваемую батареей Ре-
742.	На нагрузке аккумулятора выделялась тепловая МОЩНОСТЬ Р1= Ю Вт. Когда к концам этой нагрузки параллельно подключили второй такой же аккумулятор, выделяемая мощность Р2 увеличилась вдвое. Определить мощность Р3, выделяющуюся на нагрузке, если параллельно подключить третий такой же аккумулятор.
194
743.	Два проводника, соединенных параллельно, имеют сопротивления 7?! = 6 Ом и Т?2 = Ю Ом. При прохождении тока в первом проводнике выделяется количество теплоты = 4 • 104 Дж. Определить, какое количество теплоты выделится за это время во втором проводнике и в обоих проводниках, соединенных последовательно, при том же напряжении.
744.	Нагреватель кипятильника состоит из двух секций сопротивлением 7? = 4 Ом каждая. Кипятильник питают от аккумуляторной батареи, э. д. с. которой & = 12 В. Вода в кипятильнике закипает за одно и то же время как при последовательном, так и при параллельном соединении элементов нагревателя. Определить силу тока, проходящего через аккумулятор при коротком замыкании.
745.	Какого сечения надо взять медный провод для устройства линии от электростанции до потребителя, расположенного на расстоянии I = 1 км, чтобы пе-	(
редать мощность Р = 8 кВт? _______s	11
Напряжение на станции U =	5 с
= 130 В, допускаемая потеря _±_	zrz
напряжения на линии k =	______U__
= 8 %. Удельное сопро- I I------------------ jy]
тивление меди р = 1,7 X	а	5
X 10-8 Ом  М.	рис 249
746.	Напряжение на ши-
нах электростанции 7/0 = 100 кВ. Расстояние до потребителя I = 500 км. Станция должна передать потребителю мощность Р = 100 кВт. Потери напряжения k не должны превышать 5 %. Вычислить силу тока в проводах, площадь сечения проводов и массу меди, необходимой для проводки. Удельное сопротивление меди р =0,17 • 10"’ Ом • м, плотность р0 = 8960 кг/м3.
747.	Электродуговая печь потребляет ток I = 200 А от сети, имеющей напряжение U = 220 В. Последовательно с печью включен ограничивающий резистор сопротивлением Р = 0,2 Ом. Определить мощность Р, потребляемую печью.
748.	Какое количество теплоты Q выделится при замыкании выключателя (рис. 249, а)? Характеристика диода приведена на рис. 249, б. <§ = 6 В, С = 1 мкФ. Какая энергия W рассеивается в диоде? Чему равна работа батареи А? Конденсатор первоначально заряжен до разности потенциалов Ua =4 В.
749.	Электродвигатель с сопротивлением обмотки Р = 2 Ом подключен к генератору с э. д. с. S = 240 В и внутренним сопротивлением г = 4 Ом. При работе электродвигателя через обмотку проходит ток 1 = 10 А. Найти к. п. д. электродвигателя. Сопротивлением подводящих проводов пренебречь.
750.	Определить напряжение на зажимах источника питания, если он обеспечивает в цепи ток I = 2 А. Цепь состоит из двух параллельно включенных лампочек мощностью Р = 30 Вт каждая. Потери мощности в проводах составляют 10 % полезной мощности.
7*	195
§ 16.	Магнитное поле. Электромагнитная индукция
Задача 1. Частица, имеющая массу т и заряд q, влетает в однородное магнитное поле, индукция которого В. Вектор скорости частицы перпендикулярен к силовым линиям магнитного поля. Доказать, что период вращения частицы не зависит от значения ее скорости.
Решение. На заряженную частицу в магнитном поле дейст-" >
вует сила Лоренца Fr, в каждый момент времени перпендикулярная к вектору скорости частицы V. Поэтому
гл = та, или qvB —	, откуда — =	= const.
Под действием силы Лоренца частица описывает окружность радиусом 7? и период ее обращения Т, равный времени, за которое частица совершит один оборот, т. е. пройдет путь длиной 2л/?, пропорционален отношению R/v и, следовательно, не зависит от v:
т 2л/? о R 2лт .
7 =----= 2л-= —= const.
v v qB
Задача 2. Сравните радиусы кривизны траектории электрона и протона, если: а) двигаясь с одинаковой скоростью v, они влетают в однородное магнитное поле с индукцией 0,1 Тл перпендикулярно к силовым линиям; б) пройдя одну и ту же разность потенциалов, они влетают в магнитное поле.
Решение. Решение этой задачи аналогично решению задачи 1, поэтому радиус кривизны траектории
= (1)
а)	Подставляя в формулу (1) вместо т и q значения физических величин для электрона и протона, получим выражения для радиусов кривизны траекторий этих частиц в магнитном поле:
т „	1836m ,,
Re = -е £ и /?р =	£ == 1836/?е.
е Ь н е d
Таким образом, радиус окружности, которую описывает протон, больше соответствующего радиуса для электрона во столько раз, во сколько раз масса протона превышает массу электрона.
б)	Начальные скорости электрона и протона различны. Рассмотрим случай, когда частица, масса которой т и заряду, с нулевой начальной скоростью ускоряется разностью потенциалов U. Из условия А = qU =	= Ек находим скорость, которую она приобре-
тает,
в=1/22.
Г т
196
Отсюда видно, что скорость v частицы определяется отношением q/m и различна для электрона и протона:
т /2е(7	/ 2eU ve
Ve~ V те и уР - У 1836те ~ /1836'
Подставив все данные для электрона и протона, получим легко сравнимые выражения для радиуса кривизны траектории электрона и протона Rp в рассматриваемом случае:
т V. 1/ 2emU	1836/тг г>	г—-—
= 7i ~ -7Г- •	— Л® = |836Л «
Таким образом, в случае б) Ш = const) радиус кривизны траектории протона также больше, но в Г 1836 « 43 раза.
Задача 3. Одновалентный ион начинает двигаться из состояния покоя и проходит ускоряющую разность потенциалов ср. Затем он попадает в однородное магнитное поле с индукцией В и описывает окружность радиуса R. Чему равна линейная скорость иона и его масса?
Решение. Решение этой задачи аналогично решению предыдущих задач. Под действием ускоряющей разности потенциалов одновалентный ион приобретает скорость v.
mv2	-i Г 2еср
еср = — , откуда v = у .
Поскольку траектория иона в постоянном магнитном поле — окружность (не спираль!), то вектор v перпендикулярен к вектору магнитной индукции В, причем между R и v существует связь
R =
т
7в
V.
Подставляя в это уравнение выражение для скорости V,получим уравнение относительно т, решение которого
еВ2/?2 m = -п— . 2<р
2<р
Скорость иона v =	.
Задача 4. Определить знак э. д', с. индукции в проводниках (рис. 250). Чему равно значение возникающей разности потенциалов между концами проводника, если его длина I = 1 м, скорость движения v = 5 м/с, индукция магнитного поля В = 0,2 Тл?
Решение. Знак э. д. с. индукции можно определить рассмотрев силу Лоренца Fjj, действующую на произвольно взятый электрон в проводнике. Направление этой силы, перпендикулярной к векторам и и В, устанавливается с помощью правила левой руки. На рис. 251 приведен результат для случая, изображенного на рис. 250, а, из которого можно заключить, что под действием магнит-
197
ного поля избыток электронов (отрицательный заряд) будет накапливаться на нижнем конце проводника. При этом верхний конец проводника окажется заряженным положительно. Совершенно аналогично проводится рассмотрение для случаев, изображенных на рис. 250, б, в, г.
XX
XX
хх
хх
XX
хх
г
а
Рис. 250
проводнике,
Э. д. с. индукции <S, возникающая в
Q = uB/sin а = vBl = I В.
Задача 5. Во вторичной обмотке трансформатора, содержащей п2 = 2000 витков, возникает э. д. с. ё2 = 600 В. Сколько витков содержит первичная обмотка, если трансформатор подключен к сети
+ + + с напряжением иг = 220 В?
---- Решение. Действующие значения э. д. с.
и <S2 и количества витков п1 и п2 связаны соотношением	, вытекающим из закона сохране-
_	ния энергии. Пренебрегая активным сопротивле-
в	пнем обмотки, можно использовать приближение
’	Тогда
П1 = п2 ~ 733.
О 2
----Q Задача 6. Проволочной квадратной рамке дли-|J ной 4 а и массой т сообщают в горизонтальном на-
правлении скорость ц0. Рамка движется в вертикаль-, ной плоскости, все время находясь в магнитном *	поле, перпендикулярном к плоскости рамки
Рис. 251	(рис. 252, а). Индукция поля изменяется по закону
В (z) = В„ — k/г, где k — постоянный коэффициент. Сопротивление рамки R. Через некоторое время рамка начинает двигаться с постоянной скоростью. Найти установившуюся скорость рамки.
Решение. Прежде всего выясним, почему движение рамки в неоднородном магнитном поле становится установившимся. Если бы не было магнитного поля, то рамка в поле тяготения двигалась бы равноускоренно и ее скорость возрастала бы неограниченно с течением времени. Дело в том, что в процессе падения рамки изме-
198
няется со временем магнитная индукция поля, пронизывающего рамку, и, следовательно, магнитный поток через рамку. Переменный магнитный поток порождает э. д. с. индукции в рамке, которая приводит к появлению переменного индукционного тока. На рамку с током в магнитном поле действует сила Ампера, и так как расположение отдельных плеч рамки относительно оси Oz различно, то результирующая сила Ампера может быть ненулевой. Покажем, что она направлена вертикально вверх и является функцией вертикальной составляющей скорости рамки v2 и остальных известных
X	X	X	X	X	X	X
X	X	X	X	X	X	X
х х хд х х С х х
X X X X X X X
ifXXXXXXX г
Рис. 252
параметров. Сейчас существенно лишь то, что увеличение индукционного тока будет происходить до тех пор, пока результирующая сила Ампера Fд не сравняется с силой тяжести mg, действующей на рамку:
Ад = mg.
Равенство это будет соответствовать наибольшему возможному значению вертикальной составляющей скорости рамки и™ах, при этом сумма сил, действующих на рамку, станет равной нулю, дальнейшее движение будет равномерным. Заметим, что перемещение по горизонтали не приводит к появлению индукционного тока и, соответственно, силы Ампера, поскольку при этом движении z — = const для каждой точки рамки и, следовательно, магнитный поток через рамку не изменяется. Таким образом, горизонтальная составляющая скорости рамки остается неизменной и равной ц0. Определив можно найти скорость установившегося движения
199
рамки vy (рис. 252, б), применив правило параллелограмма и теорему Пифагора,
Vy =	+ (vTax)2.
Теперь вычислим магнитный поток, пронизывающий квадратный контур со стороной <2 в неоднородном магнитном поле с индукцией В (г) = Во — kz. Если бы В — const, то вычисление магнитного потока через рамку не представляло бы затруднений: Ф = BS — = Ва2. Если же В изменяется от точки к точке, то для вычисления магнитного потока необходимо площадь рамки а2 умножить на некоторое среднее значение магнитной индукции Вср. В случае линейного закона изменения магнитной индукции это среднее значение будет совпадать со значением магнитной индукции в центре рамки (рис. 252, в):
Вср = В (г0) и Ф = В (z0)a2.
Эту догадку нетрудно подтвердить следующим рассуждением. Разделим квадрат, ограниченный рамкой, на большое нечетное число W равных горизонтальных прямоугольных полос шириной А/ = a/N. Магнитный поток, пронизывающий рамку, равен сумме магнитных потоков, пронизывающих каждую из этих полос. Среди полос выделяется центральная (Ц. п.), геометрический центр которой совпадает с центром квадрата. Учитывая, что А/ сколь угодно мало, можно принять магнитный поток, пронизывающий центральную полосу, равным
(АФ)ц.п = В(2о)йД/=В(2о)А5,
где AS — площадь каждой из полос. Оставшиеся (после выделения Ц. п.) полосы можно разбить на пары, равноудаленные от центра (на рис. 252, в показана одна из таких пар). Покажем, что суммарный вклад такой пары в два раза превышает вклад центральной полосы, т. е. насколько магнитный поток через верхнюю полосу больше магнитного потока через центральную полосу, настолько магнитный поток через нижнюю полосу (II. п.) будет меньше магнитного потока через центральную полосу. Действительно,
(АФ)„. „ = В(2Г, --Аг)аА/ = (АФ)Ц. п + £АгА5, (ДФ)н. п = В(г0 4- Аг)аА/ = (АФ)ц. п — £A?AS, (ЛФ)„.П + (ДФ)„.П = 2(ДФ)ц. п.
Теперь становится очевидным, что результирующий магнитный поток через рамку, равный суммарному вкладу всех полос (Ц. п. и 1/2 (N — 1) равноудаленных от центра пар), будет в N раз больше вклада центральной полосы:
Ф = Л/ (АФ)Ц. п = В (г0) У A S = В (20) а2.
200
Рассмотрим теперь падение рамки в поле тяготения. Это движение будет криволинейным. Причем движение по оси Ох будет, как уже отмечалось, равномерным со скоростью н0. Движение по оси Oz будет неравномерным, причем проекция вектора скорости рамки на ось Oz равна v, (t) =	, где г (/) — координата центра
(или любой другой точки) рамки, а — ее производная, будет изменяться от начального минимального значения ц™п = ц(0) = 0 до конечного максимального значения ц,тах — и (оо).
Выразим возникающий в рамке индукционный ток Ц (Z) через vz(t). В рамке возникает э. д. с. индукции
1 v ' dt	dt
Магнитная индукция В (г) является сложной функцией времени, поэтому вычисление ее производной по времени производится по правилу вычисления производной сложной функции:
d [В (г)] _ dB (г) dz dt ~~ dt dt
Но	= —k,	= v2(t),тогда <§i(0 = ka2u,(t). Сила индукцион-
ного тока
I	/А
Л (/) =	= R Vz
Поскольку рамка с током находится в магнитном поле, то на ее плечи действуют силы Ампера. Все эти четыре силы будут различными вследствие неоднородности магнитного поля. Так как плечи AD и ВС занимают идентичное положение в неоднородном магнитном поле (рис. 252, г), то на них со стороны этого поля действовали бы одинаковые по значению и направлению (горизонтальные) силы Ампера при условии протекания через эти плечи одинаковых токов. Но в нашем случае через плечо AD протекает ток, равный по силе току в плече ВС и противоположный ему по направлению. Следовательно, силы, действующие на плечи AD и ВС, равны по модулю и противоположны по направлению (рис. 252, <Э). Так как эти силы приложены к одной и той же рамке, то их сумма равна нулю в любой момент движения рамки. Что касается сил Ампера, действующих на плечи АВ и CD (рис. 252, е), то они будут различны по значению, противоположны по направлению и вычисляются по закону Ампера:
F1 = В (z - 1; (0 а = al; (t) (Во - kz) + | ka.4; (t), Fk = В (г +	Л (0 а -•= al; (/) (50 - kz) -1 kcFI; (t).
201
Поскольку Fa>^a. то результирующая сила, действующая со стороны магнитного поля на рамку, будет направлена вертикально вверх и равна
= F[-F[= ka4i (/) == vz (t).
В процессе движения рамки vz и F^' будет возрастать до тех пор, пока последняя не станет равной силе тяжести. Таким образом, наибольшее значение вертикальной составляющей скорости рамки определяется из уравнения
„шах _ mgR
Ui ~ k*a* •
Направление и значение установившейся скорости рамки определяем из уравнения
Задача 7. Проволочной квадратной рамке длиной 4а и массой т сообщают в горизонтальном направлении некоторую скорость. Рамка движется в вертикальной плоскости, все время находясь в магнитном поле, перпендикулярном к плоскости рамки. Индукция поля изменяется по закону В (z) = Во + kz, где k — постоянный коэффициент. Сопротивление рамки R. Через некоторое время рамка начинает двигаться с постоянной скоростью v. Найти начальную горизонтальную скорость рамки v0.
Решение. Решение этой задачи аналогично решению предыдущей. Можно показать, что изменение знака перед коэффициентом k в законе изменения индукции поля как функции координаты В (?) приводит лишь к изменению направления индукционного тока. Модуль и направление результирующей силы Ампера, действующей на рамку, остаются неизменными. Прежним будет и условие, при выполнении которого движение примет установившийся характер. Единственное отличие состоит в том, что уравнение, выражающее указанное условие, следует разрешить относительно начальной скорости н0:
Задача 8. Цепь, состоящая из двух конденсаторов емкостью Cj и С2 и катушки с индуктивностью L (рис. 253, а), первоначально разомкнута. Конденсатор С\ заряжают до напряжения U, а конденсатор С2 остается незаряженным. Определить максимальную силу тока В цепи после замыкания выключателя S. Активным сопротивлением катушки пренебречь.
Решение. Для решения задачи используем закон сохранения зарядов и закон сохранения энергии. В начальный момент времени конденсатор С2 не заряжен и, следовательно, его обкладки электро-202
нейтральны, а конденсатор заряжен и, таким образом, на его обкладках содержатся заряды противоположных знаков: q = CYU. После замыкания выключателя начинается разрядка конденсатора Cj через катушку индуктивности L. При этом, очевидно, насколько уменьшится заряд верхней пластины конденсатора С1( настолько же увеличится заряд нижней пластины конденсатора С2 (рис. 253, б). Обозначим первый из зарядов в момент времени t через qr (/), а второй — через q2 (/). Из определения электроемкости следует, что qx(t) — CxUx (/') и 72(Z) = C2f/2(/), где 1Ц (t) и U2 (/) —напряжения на конденсаторах соответственно в момент времени /. Таким образом, закон сохранения заряда qx(t) -f- </2(/) — q позволяет установить связь между напряжениями иг и U2 в любой момент времени:
C^i) + C2U2(t) = C,U.	(1)
Рис. 253
Применим теперь закон сохранения энергии к рассматриваемой замкнутой системе. В начальный момент времени лишь конденсатор Сг с и2 обладал энергией. Поэтому начальная энергия системы £„ = -Лр-В качестве конечного состояния системы будем рассматривать ее состояние в произвольный момент времени t, в который оба конденсатора заряжены до напряжений и U2(t) и через катушку индуктивности протекает ток / (t). Энергия системы будет складываться, таким образом, из энергии электростатических полей конденсаторов (?! и С2 и энергии магнитного поля катушки индуктивности L:
CrUiq) c2ul(t) L14i} =----2----'---— + “Г•
Поскольку активные сопротивления всех трех элементов и соединительных проводов пренебрежимо малы, то систему можно считать замкнутой и, следовательно, выполняется закон сохранения энергии
§£/?(/) +	+	=	= const.	(2)
Заметим, что напряжения на конденсаторах Сг и С2 имеют противоположную полярность, поэтому, если в одно и то же уравнение должны входить оба напряжения и Ux (t) считается положительным, то напряжение на втором конденсаторе должно входить в это же
203
уравнение со знаком минус. Запишем двумя различными способами разность потенциалов между точками / и 2 (рис. 253, в), или, совершив обход контура (против часовой стрелки), запишем в соответствии со вторым правилом Кирхгофа уравнение, связывающее падение напряжения (в нашем случае и —t/2(Z)) и действующие в контуре э. д. с. (в нашем случае э. д. с. самоиндукции катушки
. В любом случае придем к следующему результату:
=	(3)
где — производная силы тока как функции времени.
a	S	h
Рис. 254
Полученная система уравнений (1), (2) и (3) позволяет ответить на вопрос задачи. В соответствии с необходимым условием экстремума максимальная сила тока будет соответствовать моменту вре-.	dl
мени t = t0, при котором производная обращается в нуль:
Лпах = / (t0), откуда = 0.	(4)
Q
Подставив (4) в (3), получим (/0) = U2 (t0) = с с U. Максимальную силу тока в цепи /тах можно найти из уравнения (2), если рассмотреть его в момент времени t — tQ. Подставив в это уравнение вместо /(/0), t/,(f0) и U2(t0) их значения, получим уравнение относительно /тах, решение которого дает
/ = 1/_21£=_______и
Задача 9. Две одинаковых катушки подключены через выключатели Sj и S2 к источнику с э. д. с. ё и внутренним сопротивлением г (рис. 254, а). В начальный момент времени оба выключателя разомкнуты. Затем замыкают выключатель Sj. Определить силу тока, протекающего через выключатель Sj в момент замыкания S2, если известно, что установившийся ток через выключатель Sj после замыкания S2 в два раза больше установившегося тока через выключатель S2. Активными сопротивлениями катушек пренебречь.
204
Решение. Когда замыкают выключатель (рис. 254, б), сила тока возрастает от нуля до значения /0, которое будет достигнуто в момент времени t = t0. В момент времени t = t0 замыкают выключатель S2, после чего силы тока и /2(/) продолжают возрастать до некоторых предельных значений, формально соответствующих бесконечно большому времени (т. е. предельному переходу / —<- оо). Эти предельные (установившиеся) значения сил тока обозначим через
/Г = lim Л (0 и /Г = Ит/г(().
Рассмотрим момент времени t t0 К -* °°)> когда токи через катушки станут установившимися. В этом случае цепь станет эквивалентной изображенной на рис. 254, в. Причем сила тока через источник (на участке /1 — & —В) легко определяется по закону Ома для полной цепи, так как для установившегося (постоянного) тока катушки индуктивности представляют собой нулевые сопротивления: Л == |. В то же время из закона сохранения заряда следует, что
Д” Учтя условие задачи, имеем /“ = 31™. Тогда можно записать первое уравнение для неизвестных сил установившихся токов:
/Г + /2” = /” = з/г = у.
Для получения второго уравнения, связывающего 7 Г и /?, применим второе правило Кирхгофа к схеме при t > t0:
г 4/i (0 । I {I) л ~L~dT + L~dT~ и’
где и — производные функций Д (/) и /, (/) по времени.
В рассматриваемом контуре действуют только э. д. с. самоиндукции катушек, а в условии задачи указано, что подключены две одинаковые катушки самоиндукции. Тогда
4 [Л (0-^(01 . л dt
и, используя свойства производной, получаем
/ДО— /2 (t) = С = const.	(1)
Постоянная С задана в условии задачи — это сила тока /0 при
Л (Q	^2 (Q “ = Л'
Переходя в уравнении (1) к пределу при 7->оо, получим искомую связь между установившимися значениями сил токов: /Г — /2” = /п,
205
а это означает, что превышение силы тока над силой тока 12 на 10 с течением времени сохраняется. В результате получаем
Задача 10. Две катушки с индуктивностями L2 и L2 подключены через выключатели и S2 к конденсатору с емкостью С (рис. 255, а). В начальный момент времени оба выключателя разомкнуты, а конденсатор заряжен до напряжения U. Сначала замыкают выключатель S, и, когда напряжение на конденсаторе становится равным нулю, замыкают выключатель S2. Определить мак-
J
Рис. 255
симальную и минимальную силы тока, протекающего через катушку после замыкания S2. Активным сопротивлением катушек пренебречь.
Решение. Замыкание выключателя S, при t = 0 приведет к тому, что сила тока /](/) станет возрастать от нуля до некоторого значения /,(/0). Момент tQ определяется тем условием, что конденсатор успеет полностью разрядиться. Начальная энергия электро-CU1 2
статического поля конденсатора £э = — полностью превратится в момент времени t = tQ в энергию магнитного поля катушки само-LJi (С)
индукции, Еы = -—’ > т- е- = Еы, откуда
(1)
Таким образом, в момент замыкания энергия конденсатора равна нулю, а через первую катушку протекает ток В момент времени t = tfx сила тока
Ш) = 0,	(2)
206
поэтому из закона сохранения энергии следует, что как бы далее не распределялись токи между катушками и как бы не изменялось напряжение на конденсаторе, энергия магнитного поля в первой катушке может вследствие перераспределения энергии лишь уменьшаться, и, следовательно, наибольшее значение силы тока будет /™ах = л(и = Viy-
Теперь определим наименьшее значение /?|п.
Рассмотрим произвольный момент времени t > /0. Применим второе правило Кирхгофа к внешнему контуру 1 — 3 — 4 — 6 — 1 схемы (рис. 255, б), содержащему лишь катушки:
Это уравнение можно преобразовать, используя известные свойства производной:
+ L2/2(01 = 0, откуда + Т2/2(0 = ФОбщ= const. (3)
Постоянная в правой части уравнения ФОбЩ представляет собой общий (суммарный) магнитный поток обеих катушек. Уравнение (3) выражает закон сохранения магнитного потока. Значение ФОбщ вычисляется из начальных условий (1), (2) при t = t0:
фобщ = L1/1 (t0) + L2/2 (t0) = УсЦи.	(4)
Записывая двумя различными способами разность потенциалов между точками 3 и 5 схемы или применяя правило Кирхгофа к контуру 2 — 3— 4 — 5 — 2, получим
=	(5)
где U (0 — напряжение на конденсаторе.
Уравнение, связывающее I^f), /2(/) и Z7(/), получается из закона сохранения энергии:
LjiCt) Ь2/2(0 . CU2(t) „ CU*	1К.
~j— 4----= — •	(6)
Вид уравнения (3) позволяет сделать вывод: наименьшее значение /j(Z) соответствует наибольшему значению /2 (/), и наоборот. Иными словами, экстремальные значения функций 1$) и 12(/) достигаются одновременно при t = 4Kc. Таких моментов времени t3KC, по-видимому, бесконечно много, поскольку процесс периодического изменения сил токов в нашей идеализированной схеме, не содержащей активных сопротивлений, будет продолжаться бесконечно долго. В силу необходимого условия экстремума
^а(^экс) _ «
dt ~ 1
207
и из уравнения (5) следует, что
U (/ЭК(:) = 0.
Поэтому в точках t = /экс уравнение (6) значительно упрощается (исчезает третье слагаемое в левой части), и, как следствие, в этих точках уравнения (4) и (6) образуют систему двух уравнений с двумя неизвестными:
I1 /1(Скс) + ^72(^ке)==^,	(7)
T-i^i (^экс) L212 (£Экс) = угСЬги.	(8)
Решением этой системы будут две упорядоченные пары значений: //mln. /тлах\ ztmax, »т1п\
VI , 12	' И (/j	, 12	).
Предлагаем самостоятельно решить системы (7), (8), например, методом подстановки и приведем лишь результат:
(Уг,и-0)4ц^УСс,и-с^.иУ <9)
Решение (9) позволяет идентифицировать /™п и 12ах, что приводит к окончательному ответу на вопрос задачи:
На рис. 255, в, г приведены зависимости /j(/) для различных соотношений между и L2. Интересующийся читатель может попытаться расширить исследование и определить /х (/) и /2 (/) как функции времени.
Задача 11. В схеме (рис. 256) L, С, ё известны, конденсатор вначале не заряжен, выключатель на некоторое время замыкают, а затем размыкают. Определить силу тока через катушку в момент размыкания, если максимальное напряжение на конденсаторе 2 &. Сопротивлением катушки пренебречь.
Решение. При замыкании выключателя напряжение на конденсаторе сразу установится <§, а сила тока через катушку будет возрастать прямо пропорционально времени. Пусть в момент размыкания ключа сила тока, проходящего через катушку, была 1а. Тогда в колебательном контуре к моменту размыкания будет запасена начальная энергия
с L/o , С&г £н=^- + — •
После размыкания цепи в колебательном контуре начнутся незату-хающие электромагнитные колебания с периодом Т =2nVLCs в процессе которых энергия будет периодически превращаться то в энергию магнитного поля катушки, то в энергию электростатического поля конденсатора. Второму случаю будет соответствовать 208
максимальное напряжение на конденсаторе Uca* — 2S и конечная энергия колебательного контура
С((77?ах)2
Ек = -	- = 2CS2.
В силу ^закона сохранения энергии Ен = Ек, откуда сила тока
Задача 12. В схеме (рис. 257) сразу после выключения магнитного поля, в котором находилась катушка индуктивности, через резистор сопротивлением Rr проходит ток 1. Пренебрегая омическим сопротивлением катушки с индуктивностью L, определить, какое количество теплоты выделится на резисторах, сопротивления которых Rt и /?2-
Решение. Обозначим через Q, (Z =1, 2) количество теплоты, которое выделяется на резисторе сопротивлением Ri- Первое уравнение получим, выразив отношение QJQ2 через сопротивления и R2. При этом следует учесть, что количество теплоты Qi пропорционально мощности Pi, выделяющейся на резисторе сопротивлением Ri, и, следовательно, обратно пропорционально сопротивлению Rt. Существенно, что в течение всего процесса напряжение на резисторах с сопротивлениями Rt и R2 одинаково вследствие их параллельного включения в схеме. Второе уравнение для неизвестных величин Qi выражает закон сохранения энергии: сумма (Q1+ <2г) должна быть равна начальной энергии магнитного поля катушки, которая пропорциональна квадрату начальной (t — 0) силы тока /к(0) через катушку. Решив систему двух линейных уравнений с двумя неизвестными, находим
п _R1 + RiLP п _Ri(R, + R2)LP
41-----n----2“ И (^2 — ---—2-----g- .
^2 Z	R2 Z
Задача 13.Из идеального проводника изготовлен угол, по сторонам которого скользит стержень так, что остается все время перпендикулярным к биссектрисе (рис. 258). Площадь поперечного сечения стержня 5 = 1 мм2, удельное сопротивление р = 5 X X 10~7 Ом • м. Перпендикулярно к плоскости угла направлено однородное магнитное поле с индукцией В = 1 Тл. Определить силу
209
тока в цепи Ap42A3, если проводник движется с постоянной скоростью и=5 м/с.
Решение. Задача решается на основании закона электромагнитной индукции. В процессе движения стержня Л2Л3 изменяется площадь контура, ограниченного двумя отрезками идеального проводника (сопротивления плеч AtA2 и равны нулю) и стержнем (его сопротивление пропорционально возрастающей с течением времени длине отрезка Л2Л3). Поэтому в контуре АгА 2А3 возникает переменная э. д. с. индукции
сопротивление контура 7? (/) также является функцией времени:
fl(0 = 2sPtgf vt.
На основании этих формул и закона Ома для полной цепи вычисляем силу протекающего в контуре индукционного тока
BSv Р
10 А.
АВ _ О _
Д?' Р показания к точкам
Задача 14. Круглое кольцо составлено из трех проводников одинаковой длины и сечения, но с разными удельными сопротивлениями: pi=p; р2= 2р; р3 = Зр (рис. 259). Центральная область круга радиусом г0 = 1 см пронизывается перпендикулярно к плоскости кольца переменным магнитным полем с постоянной скоростью роста = 10 Тл/с. Определить вольтметра, подключенного К и А/, где KN = NL.
Решение. Используя законы электромагнитной индукции, закон Ома для полной цепи и правило сло-последовательном их соединении, полу-
жения сопротивлений при
чим выражение для силы индукционного тока
ЗВлгоР
I (Р1 + Рз + Рз) ’
где S — площадь поперечного сечения кольца, I — общая длина проволоки, из которой изготовлено кольцо. Можно приближенно положить / равным 2лг0 при условии, что толщина проволоки намного меньше г0. Тогда
2 (Pi + Ра + Рз)
210
Сопротивление участка Л/М находим, исходя из того, что точка Л/ — середина дуги KL\
О О
а искомое напряжение на участке K.N (из закона Ома для участка цепи)
Ukn ~ IRkn
2 (Pi + Р2 + Ра)
Подставив из условия в эту формулу значения pt- (i = 1, 2, 3), имеем
VKN =	= 0,26 мВ.
Задача 15. Заряженные частицы ускоряются в циклотроне в однородном магнитном поле с частотой ускоряющего напряжения v = 8 МГц. Пучок ускоренных частиц со средней силой тока 1 = = 1 мА выводится с орбиты радиусом R = 1 м. Определить индукцию магнитного поля В, если известно, что температура воды, охлаждающей «ловушку», в которой тормозятся частицы, повышается на АТ = 6 К при расходе воды Q = 1 л/с.
Решение. Кинетическая энергия АЕК частиц, ускоренных циклотроном и попавших в «ловушку» в течение 1 с, будет полно стью затрачена на увеличение At/ внутренней энергии охлаждающей воды, которая в течение 1 с пройдет через «ловушку»:
АЕК = At/.
Изменение внутренней энергии А Г/ = свАтАТ. Поскольку Am = рвО то At/ = сврв(2АТ.
Изменение кинетической энергии частиц
. „ mv2 .
А£\< —	АЛ/,
где первый сомножитель —средняя кинетическая энергия заряженной частицы, попавшей в «ловушку», а ДМ — среднее число частиц, попадающих в «ловушку» за 1 с. Так как сила тока / численно равна заряду, попавшему в «ловушку» за 1 с, то
ДМ = — . q
Движение заряженных частиц в дуантах циклотрона происходит - >
в постоянном магнитном поле, индукция которого В перпендикулярна к скорости частиц V. Поэтому на заряженные частицы действует сила Лоренца, которая выступает в роли центростремительной силы: Т'ц = Ел, откуда v = ~ BR. Подставляя значения ДМ и V, получим
Д£к = -^ /В2/?2.
211
Для определения отношения заряда частицы к ее массе вспомним условие «синхронизма» в циклотроне: Та— Т, где То — ^, аТ— период вращения частицы в магнитном поле — вычисляется по известной скорости движения частицы по круговой траектории: гр_________________________ 2л/? _m 2л
и q В
Отсюда ==	. Тогда Д£к = itvBIR2. После подстановки полу-
ченных выражений для AU и AFK в уравнение теплового баланса получим уравнение относительно искомой величины В:
В =	« 1 Тл.
ЗАДАЧИ ДЛЯ САМОСТОЯТЕЛЬНОГО РЕШЕНИЯ
751.	Электрон, прошедший разность потенциалов U = 3500 В, влетел в однородное магнитное поле с индукцией В = 0,1 Тл и начал двигаться по окружности. Вычислить радиус окружности.
752.	Заряженная частица с энергией Е = 1,6 • 10"16 Дж движется в однородном магнитном поле по окружности радиусом R = = 1  10~3 м. Какова сила, действующая на частицу со стороны поля?
753.	Неизвестная заряженная частица с малой начальной скоростью ускоряется сначала в однородном электрическом поле, а затем попадает в область, где имеется однородное магнитное поле. Скорость частицы в магнитном поле перпендикулярна к его силовым линиям. Определить, какая наблюдается частица, если известно, что радиус кривизны ее траектории в магнитном поле в 1^2 раз больше, чем у протона, прошедшего ту же систему полей. Заряд неизвестной частицы в два раза больше заряда протона.
754.	Как располагается свободно подвешенный виток с током в однородном горизонтальном магнитном поле?
755.	Как направлено магнитное поле протонов, равномерно удаляющихся по прямой от нас?
756.	Могут ли замыкаться силовые линии электрического поля?
757.	Прямой постоянный магнит падает сквозь металлическое кольцо. Будет ли магнит падать с ускорением свободного падения? Сопротивлением воздуха пренебречь.
758.	В однородном магнитном поле с индукцией В = 0,02 Тл расположен плоский проволочный контур с сопротивлением R = — 3 Ом. Вначале плоскость контура была перпендикулярна к силовым линиям поля. Затем контур поворачивают и при этом подключенный к контуру гальванометр регистрирует протекание через него заряда Q = 10-6Кл.На какой угол был совершен поворот? Площадь контура S — 10 см2. Сопротивлением гальванометра пренебречь.
212
759.	Почему при включении в электрическую цепь двух внешне одинаковых катушек, намотанных одинаковым проводом так, что их сопротивления равны, одна обнаруживает магнитное действие, притягивая железный предмет, а другая нет?
760.	Определить индуктивность проводника, в котором равномерное изменение силы тока на Л/ = 2 А в течение А/ = = 0,25 с приводит к возникновению э. д. с. самоиндукции = = 20 мВ.
761.	Как изменится индуктивность длинного соленоида, если растянуть его вдвое, а число витков оставить неизменным?
762.	Внутри катушки, находящейся в свободном пространстве, помещен тяжелый электрически заряженный шарик. Катушка подключается к источнику постоянного тока и в ней устанавливается постоянное магнитное поле. Изменится ли при этом состояние покоя шарика?
763.	Автомобиль движется прямолинейно и горизонтально со скоростью v = 120 км/ч. Определить разность потенциалов, возникающую в поперечном направлении на оси автомашины, если ее длина I — 180 см, а вертикальная составляющая индукции магнитного поля Земли В = 5 • 10~5 Тл.
764.	Показать, что правило Ленца для закона электромагнитной индукции Фарадея является следствием невозможности создания вечного двигателя, действие которого основано на использовании электромагнитных явлений.
765.	Электрон влетает в совпадающие по направлению однородные электрическое и магнитное поля. Напряженность электрического поля Е = 100 В/м, индукция магнитного поля В — 10-4 Тл.
- ►- — ► Скорость электрона v = 10е м/с перпендикулярна к векторам Е и В. Найти ускорение движения электрона.
766.	Электрон влетает в область пространства с однородным электростатическим полем с напряженностью Е — 6 • 104 В/м перпендикулярно к линиям напряженности. Определить значение и направление индукции магнитного поля, которое надо создать в этой области для того, чтобы электрон пролетел ее, не испытывая отклонения. Энергия электрона W = 1,6 • 10~16 Дж.
767.	Электрон движется в однородном магнитном поле с индукцией В. В начальный момент времени электрон находился в точке О и его скорость и была перпендикулярна к вектору магнитной индукции. Найти расстояние I электрона от точки О в момент времени t. Массу электрона т и его заряд q считать известными.
768.	На виток радиусом R = 18 см в зазоре между полюсами электромагнита действует максимальный момент сил М = = 0,65 Н • м. Какова индукция В магнитного поля в зазоре, если сила тока / = 4 А. Магнитное поле Земли не учитывать.
769.	По П-образной рамке, помещенной в однородное магнитное поле, перпендикулярное к плоскости рамки, движется без трения с постоянной скоростью v = 2 м/с, перемычка, сопротивление которой R = 0,2 кОм (рис. 260). К перемычке приложена сила
213
F = 4 Н. Найти силу тока 1 в перемычке. Сопротивлением рамки пренебречь. Силу тяжести не учитывать.
770.	Из одинаковых проводников изготовлены два контура — квадратный и круглый. Оба они помещены в одной плоскости в однородном, изменяющемся во времени магнитном поле. В круглом контуре индуцируется постоянный ток /t — 0,4 А. Найти силу тока /а в квадратном контуре.
771.	По бесконечному проводу, изогнутому под прямым углом, проходит ток / = 103 А. Найти магнитную индукцию В в точке, лежащей на продолжении одного из участков провода на расстоянии а = 1 см от вершины изогнутого провода.
772.	Два электрона влетают в однородные магнитные поля с индукциями Вг =0,1 Тл и 52= 0,2 Тл перпендикулярно к ли
Рис. 260
Рис. 262
ниям индукции. Сравнить периоды их обращения по окружностям, описываемым в магнитных полях.
773.	Поток жидкого металла течет по керамической трубе. Для измерения скорости потока трубу помещают в однородное магнитное поле с индукцией В =0,1 Тл, перпендикулярной к оси трубы. В ней закрепляют два электрода, образующие плоский конденсатор, причем силовые линии магнитного поля лежат в плоскости электродов. Измеренная разность потенциалов Дер оказалась равной 0,4 мВ при расстоянии между электродами а = 2 см. Найти скорость потока v.
774.	Проволочное кольцо радиусом R = 0,1 м помещено в неоднородное магнитное поле, индукция которого одинакова по модулю в точках пересечения силовых линий поля с кольцом и составляет угол а = 60° с его плоскостью. При пропускании по кольцу тока / = 12 А для обеспечения равновесия пришлось приложить силу F =0,6 Н. Найти индукцию В неоднородного магнитного поля в точках кольца.
775.	По проволочному кольцу радиусом R = 0,1 м проходит ток / = 2 А. Кольцо находится в однородном магнитном поле с индукцией В = 1 Тл, силовые линии которого нормальны к плоскости кольца. Какую силу F надо приложить, чтобы растянуть кольцо по диаметру на у = 10 % при постоянной силе тока в цепи?
214
.776. Электрическая цепь имеет форму окружности радиусом R = 0,5 м (рис. 261). Четыре гальванических элемента с э. д. с. <§! = 2 В, ё2 = 4 В, <§3 = 3 В, <§4 = 5 В соединены последовательно друг с другом и имеют внутренние сопротивления ,i\ = 1 Ом, г2 = 2 Ом, r3 = 1 Ом, г4 = 0,5 Ом соответственно. Однородное магнитное поле перпендикулярно к плоскости цепи, направлено за чертеж и возрастает со временем по закону В = kt, где k = 4 Тл/с. Чему равна сила тока в цепи? Сопротивлением соединительных проводов пренебречь.
777.	Из провода длиной I = 2 м, обладающего сопротивлением R = 4 Ом, спаян квадрат. В стороны квадрата включены источники с э. д. с. =10Виё2=8В (рис. 262). Цепь помещена в однородное магнитное поле, перпендикулярно к плоскости квадрата, направленное за чертеж и возрастающее во времени по закону В = = kt, где k = 16 Тл/с. Найти силу тока I в цепи. Внутренним сопротивлением источников пренебречь.
778.	На отстоящие друг от друга на а = 10 см горизонтальные рельсы положен медный стержень, который может катиться вдоль них. Рельсы подключены к батарее с э. д. с. = 3 В и внутренним сопротивлением г = 5 Ом, перпендикулярно к плоскости рельсов создано магнитное поле. Определить индукцию В поля, при которой стержень будет двигаться с наибольшей установившейся скоростью, и эту скорость итах. Сопротивлением рельсов и стержня пренебречь, силу сопротивления движению стержня считать равной F =0,06 Н (F— механическая сила).
779.	Два неподвижных параллельных горизонтальных рельса, находящихся на расстоянии I =0,1 м друг от друга, помещены в однородное магнитное поле, вектор индукции которого В = = 0,1 Тл направлен вертикально. На рельсах перпендикулярно к ним лежит металлический стержень массой т = 0,05 кг. Определить ускорение а этого стержня, если по нему идет ток I = 20 А, а коэффициент трения между стержнем и рельсами ц = 0,2.
780.	Рамка, имеющая форму равностороннего треугольника, помещена в однородное магнитное поле с индукцией В =0,1 Тл. Перпендикуляр к плоскости рамки составляет с направлением магнитного поля угол а = 30°. Определить длину а стороны рамки, если при равномерном уменьшении магнитного поля до нуля за время т = 0,01 с в рамке индуцируется э. д. с. ё = 10“3 В.
784.	Металлическое кольцо массой т и сопротивлением R начинает падать под действием тяжести с высоты И в магнитном поле с индукцией, изменяющейся с высотой х по закону В (х) = = В0(1 — --) (рис. 263). Определить установившуюся скорость падения кольца, если его радиус г, а плоскость кольца остается перпендикулярной к линиям магнитной индукции.
782.	Пучок протонов влетает в область однородного магнитного поля с индукцией В = 0,1 Тл. Направление поля перпендикулярно к скорости пучка. В этом поле протоны движутся по дуге окружности радиусом г—0,2 ми попадают на заземленную мишень.
215
Найти тепловую мощность, выделяющуюся в мишени. Ток в пучке / = 0,1 мА. Удельный заряд протона elm. = 108 Кл/кг.
783.	Электрон, имеющий скорость v, влетает в однородное магнитное поле с индукцией В под углом а к ее направлению. Определить радиус и шаг винтовой линии, по которой будет двигаться электрон.
784.	Медный провод сечением S, согнутый в виде трех сторон квадрата (рис. 264), может вращаться относительно горизонтальной оси. Провод находится в однородном магнитном поле, направлен
ном вертикально. Когда по проводу проходит ток I, провод отклоняется на угол а. Определить индукцию магнитного поля. Плотность меди р.
785.	Конденсатор емкостью С, заряженный до напряжения (/, подключен через выключатель к двум параллельно соединенным катушкам с индуктивностями Lj и L2. При замыкании выключателя конденсатор через некоторое время полностью перезаряжается (изменяется знак напряжения на конденсаторе). Какие заряды Qj и Q2 протекут через катушки за это время? Омическим сопротивлением катушек пренебречь.
786.	Контуры I и // (рис. 265) находятся в переменных магнитных полях. Поток вектора магнитной индукции в первом контуре изменяется по закону Ф1== A±t, во втором контуре— по закону Ф2= А2Л На остальных участках цепи магнитного поля нет. Найти токи в контурах. Сопротивления считать заданными.
216
787.	Контур АОВ образован дугой АВ радиуса R = 10 см из голого проводника и двумя проводниками ОА и ОВ с сопротивлениями /?!= 1 Ом, #3=2 0м (рис. 266). В центре О шарнирно закреплена перемычка ОС, другой конец которой может двигаться вдоль дуги, сохраняя с ней контакт. Сопротивление перемычки #3= 3 Ом. Контур находится в магнитном поле с индукцией В = = 2 • 10"4 Тл, направленной перпендикулярно к его плоскости. Найти силу тока / в перемычке при ее вращении с постоянной угловой скоростью и=11 рад/с Сопротивлением дуги пренебречь
788.	Электрон влетает в однородное магнитное поле с индукцией В. В некоторой точке А он имел скорость v, которая состав-
ляла с направлением поля угол а. При каких значениях индукции магнитного поля электрон окажется в точке С? Точки А и С нахо
Рис. 267
дятся на прямой, параллельной вектору магнитной индукции поля, расстояние между точками L.
789.	Для подзарядки аккумулятора с э. д. с. <§ = 12 В от мощного источника напряжения U = 5 В собрана схема, состоящая из катушки с индуктивностью L = 1 Гн, диода D и прерывателя 5 (рис. 267), который периодически замыкается и размыкается на одинаковые промежутки времени Tj = т3 = 0,01 с. Определить средний ток зарядки аккумулятора.
790.	В одном из проектов получения электроэнергии предлагалось использовать морские течения и магнитное поле Земли. Проект заключается в том, что в море погружают две горизонтальные металлические пластины, расположенные одна над другой на расстоянии L = 100 м, площадь каждой пластины S = 1 км2. Морская вода, удельное сопротивление которой р = 0,25 Ом  м, протекает между пластинами с запада на восток со скоростью v = 1 м/с. Магнитное поле Земли в данном месте однородно, направлено с юга на север, индукция поля В = 10~4 Тл. Определить максимальную электрическую мощность Ртах, которая может выделиться на нагрузке, подсоединенной к пластинам.
791.	Перемычка KL (рис. 268) неподвижно закреплена, а перемычка MN скользит с угловой скоростью со = 50 рад/с вокруг центра 0 по двум кольцам с радиусами 10 см и г2 = 20 см, находящимся в постоянном однородном магнитном поле, индукция которого В = 0,01 Тл перпендикулярна к плоскости колец. Какой ток течет через перемычку /<Т в тот момент, когда псЬемычка MN проходит положение Кольца сделаны из проволоки, пло
217
щадь поперечного сечения которой S = 3,4 мм2, а удельное сопротивление р = 1,7 • IO’8 Ом • м. Сопротивлением перемычек пренебречь.
792.	Тонкое медное кольцо радиусом г = 2 см помещают в однородное магнитное поле, направленное нормально к плоскости кольца и изменяющееся по закону В = —а/, где а = 1 Тл/с. За какое время кольцо нагреется на ДТ =0,1 К? Теплопередачей пренебречь. Удельное сопротивление меди р =0,17 • 10-7 Ом • м, плотность D = 8960 кг/м3.
793.	По участку АВ цепи (рис. 269) проходит ток, сила которого изменяется по закону / = at. Здесь а = 0,01 А/с — постоянный коэффициент. Омическое сопротивление цепи 7? = 0,01 Ом, индуктивность L =0,01 Гн, емкость С =0,1 мкФ. Найти заряд на конденсаторе в момент времени т = 1 с.
Рис. 269	Рис. 270	Рис. 271
794.	Цепь (см. рис. 253, а) первоначально разомкнута. Конденсатор заряжают до напряжения U, а конденсатор С2 остается незаряженным. Определить максимальное напряжение на конденсаторе С2 после замыкания выключателя. Активным сопротивлением катушки пренебречь.
795.	Две катушки с индуктивностями Lx и L2 подключены через выключатели Sx и S2 к источнику с постоянной э. д. с. & и внутренним сопротивлением г (см. рис. 255, а). В начальный момент времени оба выключателя разомкнуты. После того, как выключатель Sj замкнули и сила тока через катушку Lr достигла некоторого значения /0, замыкают выключатель S2. Определить установившиеся значения сил токов через катушки Lx и L, после замыкания выключателя S2. Активными сопротивлениями катушек пренебречь.
796.	Две одинаковые катушки подключены через выключатели Sx и к конденсатору (рис. 270). В начальный момент времени оба выключателя разомкнуты, а конденсатор заряжен до напряжения U. Сначала замыкают выключатель 5Х, и когда напряжение на конденсаторе становится равным нулю, замыкают выключатель S2. Определить максимальное напряжение на конденсаторе после замыкания выключателя S2. Активным сопротивлением катушек пренебречь.
797.	Последовательно с катушкой индуктивностью L и конденсатором емкостью С через выключатель подключили батарею с э. д. с. ё0 (рис. 271). В начальный момент времени выключатель разомкнут, конденсатор не заряжен. Определить максимальную
218
силу тока в цепи после замыкания выключателя. Омическими сопротивлениями в цепи пренебречь.
798.	Последовательно с катушкой индуктивности L и конденсатором С через выключатель подключили батарею (см. рис. 271). В начальный момент времени выключатель разомкнут, а конденсатор не заряжен. Определить э. д. с. источника, если известно, что после замыкания выключателя максимальная сила тока в цепи будет /max*
799.	Катушка из гц витков, площадь каждого из которых S, расположена в однородном магнитном поле с индукцией В, направленной перпендикулярно к виткам катушки (рис. 272). Вне поля расположена вторая катушка. Обе катушки соединены проводниками. Пренебрегая омическим сопротивлением катушек и проводников,
Рис. 272
определить силу тока, возникающего в катушках после выключения поля. Индуктивности катушек и L2.
800.	Катушка из nt витков, площадь каждого из которых S, расположена в однородном магнитном поле, индукция которого перпендикулярна к виткам катушки (см. рис. 272). Вне поля расположена вторая катушка. Обе катушки соединены проводниками. Пренебрегая омическим сопротивлением катушек и проводников, определить индукцию магнитного поля, если после выключения поля через катушки проходит ток /. Индуктивности катушек Ц и Ь2.
801.	В схеме (рис. 273) выключатель замыкают на время т, а затем размыкают. Сила тока в катушке индуктивности в момент размыкания /0. Через сколько времени после размыкания ток в катушке достигнет максимального значения 2/0? Построить график //.(/) (0 < t < оо). D — идеальный диод.
802.	Для подзарядки автомобильного аккумулятора с э. д. с. <§ = 12 В от сети с напряжением U = 5 В собрана схема (см рис. 267), содержащая катушку индуктивности с L =0,1 Гн, идеальный диод D и прерыватель S, который периодически замыкается и размыкается на одинаковое время Tj = т2= 0,1 с. За сколько времени можно таким образом осуществить подзарядку аккумулятора на 20 А • ч? Сопротивлением всех узлов схемы и диода в прямом направлении пренебречь.
219
803.	В схеме (см. рис. 256) L, С, & известны, конденсатор вначале не заряжен. Выключатель на некоторое время замыкают, а затем размыкают. Определить силу тока /0 через катушку индуктивности в момент размыкания, если максимальная сила тока, протекающего через нее, оказалась 2/0. Сопротивлением катушки пренебречь.
804.	В цепи (рис. 274) в начальный момент времени выключатель размыкают. Определить количество теплоты, которое выделится в резисторе сопротивлением Rr при следующих параметрах цепи: э. д. с. источника 6% ; г = Rt= R2 = R; индуктивность катушки L.
805.	Из идеального проводника изготовлен угол, по сторонам которого скользит стержень так, что остается все время перпендикулярным к биссектрисе угла. Площадь поперечного сечения стержня S = 1 мм2, удельное сопротивление р =2,1 • 10"7 Ом • м. Если проводник двигать со скоростью о =2 м/с, то сила тока в цепи / =5 А. Определить индукцию однородного магнитного поля В, направленного перпендикулярно к плоскости угла.
806.	Круглое кольцо составлено из двух проводников одинаковой длины и сечения, но с разными удельными сопротивлениями: р12=р; р21 = 2р. Внутри кольца проходит концентрическая цилиндрическая трубка радиусом г0 = = 2 см, в которой создается перпендикулярное к плоскости кольца переменное магнитное поле,
ДВ .
скорость возрастания которого постоянна и равна = k = = 10 Тл/с. К точкам 1 и 2 подключен конденсатор емкостью С = = 300 мФ (рис. 275). Определить заряды на каждой обкладке конденсатора.
807.	Заряженные частицы ускоряются в циклотроне в однородном магнитном поле с индукцией В = 1 Тл и частотой ускоряющего напряжения v =7,5 МГц. Пучок ускоренных частиц со средней силой тока / = 1 мА выводится с орбиты, радиус которой R = 1 м. На сколько градусов будет повышаться температура воды, охлаждающей «ловушку», в которой тормозятся частицы, если расход воды <? = 1 л/с?
IV. КОЛЕБАНИЯ И ВОЛНЫ
§ 17. Механические колебания и волны
Задача 1. Материальная точка совершает гармонические колебания, описываемые уравнением х =0,05 sin (4- Опреде
220
лить амплитуду А, круговую частоту со,период Т, частоту v и начальную фазу колебаний ср0. Определить также фазу колебаний <р, и координату колеблющейся точки xt в момент времени t == = 0,5 с. Совпадает ли начало наблюдения с началом движения, если колебания точки вызваны толчком из положения равновесия?
Решение. Запишем уравнение гармонических колебаний в общем виде х = yJsin (со/ + <р0) и сопоставим его с заданным в условии. Из уравнения следует, что А ~ 0,05 м; со = ~ с-1; ср0= __	2 тс
—	. Период колебаний определим из соотношения Т = — , или
Т =	= 4 с. Частота колебаний v = у —	с-1 = 0,25 Гц.
При / = 0,5 с ф/ = -5- • 0,5 + ~	, xt = 0,05 sin cpz = 0,05 х
Z	4 Z
X sin ~ = 0,05 m. Значение xt совпало с амплитудой.
По условию, колебания точки вызваны толчком из положения равновесия. Ее координата х = 0. Если функция х (/) имеет вид х = A sin ср, то при х — 0 и sin ср = 0, значит, соответствующее значение фазы
ср = со/ 4- ср0 = 0.
Если наблюдение за движением началось в этот момент, то в последнем равенстве следует положить / = 0, тогда начальная фаза ф0 = 0. В нашем случае ф0=	, следовательно, наблюдение за
движением началось позднее.
Задача 2. За какую часть периода тело, совершающее гармонические колебания, проходит: а) весь путь от среднего положения до крайнего; б) первую половину этого пути; в) вторую его половину?
Решение. Уравнение гармонических колебаний в общем виде записывается х = A sin (со/ 4- ср0).Для удобства положим начальную фазу ср0 равной нулю. Это всегда можно сделать, выбрав начало отсчета времени в момент, когда колеблющееся тело проходит через положение равновесия (х = 0). Тогда уравнение колебаний примет вид х = A sin со/. Следует иметь в виду, что в зависимости от того, в каком направлении тело проходит положение равновесия, его начальная фаза будет 0 или л. Но это не существенно в данной задаче, так как от этого зависит только направление (знак) последующего смещения от положения равновесия.
Время, затрачиваемое на прохождение всего пути от среднего положения до крайнего, найдем, полагая в формуле х = /Isin со/ смещение х равным амплитуде х — А (или х =—А при ф =л). Тогда А =. Л sin со/, откуда sin со/ = 1 и со/ = ~ . Это соответствует времени первого прохождения через крайнее положение (х — А). Отсюда
,__ л т
1 ~2й~ V
221
Этот же результат можно получить качественно из симметрического характера движения колеблющегося тела по отношению к положению равновесия.
Время прохождения первой половины этого пути найдем, полагая х = ^А:
4- А = A sin at, отсюда sin at, = i-; at, = ~ и t, = ~	.
Вторую половину этого пути тело пройдет за время
Задача 3. За какую часть периода точка, совершающая гармоническое колебание, пройдет путь, равный: а)половине амплитуды, если в начальный момент она находилась в положении равновесия;
о Г . I _	—.___________2 Д._________
»	.1-., ;^г|	*	“Д+/I *
Рис. 276	Рис. 277
б) одной трети амплитуды, если в начальный момент она находилась в крайнем положении?
Решение, а) Путь /1= -^А, пройденный точкой в гармоническом колебании при движении от положения равновесия к крайнему положению, равен смещению х, определяемому уравнением х = A sin (at + <р0), которое можно записать так: х '= = A sin + <р0.)
Чтобы найти начальную фазу <р0, воспользуемся начальными условиями задачи: х = 0 при t = 0. Подставив эти значения х и t в предыдущее уравнение, получим <р0 = 0, следовательно, х = . . 2nt
= A sin-уг.
J	I
Подставив в последнее уравнение значение х — А, найдем искомое время: t = Т.
б) Точка движется из крайнего положения, поэтому начальные условия будут такие: х — А при t = 0. Подставив эти значения х и t в уравнение гармонического колебания, получим ф0=-2-л. Следовательно, х = A sin \ 2^t +	= A cos
Чтобы не допустить ошибки, необходимо учитывать, что исходное уравнение х = A sin (at -f- ср0) выражает смещение х точки при гармоническом колебании, отсчитанное от положения равновесия (рис. 276), но не путь, пройденный точкой; лишь в частном случае движения точки из положения равновесия к крайнему положению эти величины численно равны (этим мы воспользовались
222
в случае а). Если точка, двигаясь из крайнего положения, прошла путь 12 = ^А, то, как видно из рис. 276, ее смещение равно х =
2
= А —12 — у А. Подставив это значение х в формулу х = А х 2л/	2л/	2	„	.
Xcos-jr-, получим cos-jr = -g. Отсюда, пользуясь таблицей косинусов, найдем искомое время в долях периода:
2л, 4ЯО. ,48° Т
Г( = 48' 1 = 360° 7 = 7?5 
Задача 4. Записать уравнение гармонического колебательного движения с амплитудой в 5 см, если в одну минуту совершается 150 колебаний и начальная фаза колебаний равна 45°.
Решение. Запишем уравнение гармонического колебания в общем виде: х = A sin (at + <р0). Чтобы записать уравнение конкретного колебательного движения, нужно знать значения всех постоянных, входящих в уравнение.
По условию задачи, А = 0,05 м; <р0 = 45° = рад. Циклическая частота равна числу колебаний, совершаемых в течение 2л. Число колебаний за одну секунду получим, разделив число колебаний Л/, совершаемых за At, на промежуток времени At. Значит, со = 2л-^- = 5 лс-1. Тогда уравнение колебательного движения запишется
х = 0,05 sin (5л/ + -7-1.
Задача 5. Определить начальное положение х0, период Т, начальную фазу <р0 и начальную скорость у0 колебаний, заданных л ... . 39,2/4-5,2 уравнением х = 0,05 sin —g-1----.
Решение. Уравнение гармонических колебаний в общем виде записывается х = A sin (со/ + сра). Перепишем данное в за-даче уравнение в виде х = 0,05 sin (—g- / 4- -g-1 . Сравнивая эти
5 2	39 2
уравнения, найдем, что ср0 =-g-рад = 1,04 рад; со — —g- рад/с ж
х 7,84 рад/с; А =0,05 м. Период Т =	& 0,8 с. Смещение х„
найдем, полагая / = 0 в уравнении колебаний: x0 = Xsincp0. Для заданных в задаче колебаний х(1 = 0,05 sin 1,04 л/ 0,043 м.
Для определения начального значения скорости необходимо найти производную от смещения по времени х (/). В общем виде
dx
v(t) =	— AacQs(at 4- <P0) и 7 ~ y(0) = 4cocoscp0« 0,198 м/с.
Положительное значение скорости означает, что в момент времени / = 0, когда смещение было равным х0 = + 0,043 м, скорость тела и0 направлена в ту же сторону, что и смещение (рис. 277).
223
Задача 6. Когда груз неподвижно висел на вертикальной пружине, ее удлинение было А/ = 5 см. Затем груз оттянули и отпустили, вследствие чего он начал колебаться. Каков период этих колебаний?
Решение. Если бы груз совершал колебания только под действием упругой силы пружины ЕуПр = — kx, их период можно было бы определить по формуле Т = 2л В данном случае на тело еще действует сила тяжести mg. Чтобы выяснить ее влияние на колебания груза, рассмотрим силы, действующие на тело, в двух положениях:
1) груз неподвижно висит на пружине. Равнодействующая сил, приложенных к телу, Г1 = 0. Приняв направление вниз за положительное, запишем FL = mg — k&l.
2) Груз смещен из положения равновесия на Будем считать величиной алгебраической. Пружина в этом случае растянулась на А/ + Д/Р Равнодействующая сил, приложенных к грузу, равна F2=mg— k (А/ + A/J. Раскрывая скобки и учитывая (1), получим	Отсюда видно, что равнодействующая
сил ГуПр и mg пропорциональна растяжению пружины и противоположна ему по направлению, если только это растяжение отсчитывать от положения равновесия висящего на пружине груза. Следовательно, и при наличии силы тяжести груз будет совершать гармонические колебания. Период этих колебаний Т = 2л |/= = 2л ]/» 0,44 с.
Задача 7. К пружине весов подвешена чашка с гирями. Период вертикальных колебаний чашки 7\. После того как на чашку положили добавочные гири, период стал Т2. На сколько удлинилась пружина от прибавления добавочного груза, если первоначальное удлинение было Z?
Решение. Запишем закон Гука для пружины для случаев, когда на чашке нет добавочного груза и когда он есть: mg = kl и (m + A/n)g = k (/ + А/), где Am — масса добавочного груза, а А/ — удлинение пружины от прибавления добавочного груза. т.	„ IЦ- Д/ m 4- Am . , /т 4- Ат	, i ,
Из этих уравнении —— и А/ = (—-----------------1 : I.
„	т 4- Ат
Отношение ——— можно наити по известным периодам колебаний чашки без добавочного груза и вместе с ним: 7\ = 2л
и Г, = 2л I/ —4—. Отсюда - = 1^- . Тогда
д/ = [(£’-ф.
Задача 8. На чашку массой тъ подвешенную на пружине с коэффициентом жесткости k, падает с высоты h груз массой т2
224
и остается на чашке. Под действием удара груза о дно, который можно считать абсолютно неупругим, чашка начинает колебаться. Определить амплитуду ее колебаний. Сопротивление воздуха и изменение внутренней энергии не учитывать.
Решение. В результате неупругого удара груз с чашкой получит скорость v, которую можно вычислить на основании закона сохранения импульса для системы груз — чашка: m2v1 = (/«1+ 4- m2)y, где vx — скорость, которую приобрел груз, падая с высоты h на дно чашки. Так как 2gh, то v = —— ]^2gh.
Запишем закон сохранения энергии в применении к колебательному процессу системы, состоящей из чашки с грузом и пружины, учтя, что под действием силы тяжести чашки в первоначальный момент пружина была растянута на А/ (/гД/ = m^g);
-2-----2~ = -2 (mi + m2) V2 + (Wj + m2) (х — A/)g,
где х — смещение чашки с грузом.
Подставим в уравнение колебательного процесса выражение для V.
= 1 2?/1 + ('"1 +
Учтя, что А/ =	, решим уравнение относительно х:
. ,/<g2 , ,	.
л ~ k s ± у k2 (тг + т2) k •
Первый член в полученном выражении характеризует положение равновесия, около которого происходят колебания груза с чашкой, второй член — амплитуда колебаний.
Задача 9. К потолку лифта подвешен математический маятник. В неподвижном или движущемся с постоянной скоростью лифте период колебаний маятника равен Та. Чему равен период колебаний этого маятника, если лифт: а) поднимается вертикально вверх с ускорением а = 0,5g; б) опускается вертикально вниз с ускорением а = 0,5 g; в) опускается вертикально вниз с ускорением а = = 1,5g?
Решение. Рассмотрим сначала положение равновесия маятника в системе отсчета, связанной с Землей (рис. 278). В этом положении равнодействующая R силы тяжести mg и силы натяжения нити N должна обеспечивать маятнику ускорение, равное ускорению лифта а. Запишем динамическое уравнение движения маятника в неподвижном или движущемся с постоянной скоростью лифте N + mg = 0 и в лифте, движущемся с ускорением a, mg —
8 7-253	22S
= та. Проектируя оба равенства на ось Оу, получаем для четырех случаев движения лифта:
mg — N = 0; mg —	= —mar\ mg — N2 = ma2 и
mg — M3 = ma}.
Отсюда N = mg\ Nt = m (g -|- aj; = m (g — a2) и N3 ~ ~ m (g—a3). Таким образом, при ускоренном подъеме маятника натяжение нити возрастает, а при ускоренном спуске — уменьшается.
Период колебаний маятника в движущемся с ускорением лифте будет таким же, как для маятника той же длины, колеблющегося под действием эффективной силы N1=in(g + al), N2 — т (g—
Рис. 278
Рис. 279
— а2) или М3 = т (g — а3) с ускорением g + alt g — а2 или
§ аз-
а)	При подъеме период колебаний маятника
7\ = 2л j/ g + ai = 2л ]/ уд; =	]/ д-.
б)	При опускании лифта период колебаний маятника
Т2 = 2л ]/ g_02 = 2л = Т’о 1^2.
в)	Если лифт опускается с ускорением аа > g, колебания маятника происходят не около нижнего положения равновесия, а около верхнего. Период колебаний в этом случае
Та = 2л V -——: = 2л 1/ nV = Л/2-d Г |g —а3| Г 0,5g	°
Задача 10. Маятник длиной I = 1,2 м подвешен к потолку вагона, движущегося горизонтально по прямой с постоянным ускорением а = 2,2 м/с2. Определить положение равновесия и период колебаний маятника.
Решение. Свяжем систему отсчета с Землей (рис. 279). При движении вагона с ускорением маятник отклонится от вертикали на угол а и относительно вагона будет неподвижным. В этом положе-
226
нии равнодействующая 7? силы тяжести mg и силы натяжения нити F должна обеспечивать маятнику ускорение, равное ускорению ва-—>-	—h-
гона а, поэтому R — та. Тогда равновесный угол а между нитью и вертикалью определяется условием
= Д = | , откуда а = arctg | да 13°.
Период колебаний маятника в вагоне будет такой же, как для маятника той же длины, но колеблющегося под действием эффективной силы F = V\mg)2 + R2 — т V g2 + а2 с ускорением g± = F г-----------
= — — |/ g2 -j- д2:
rn > ь <
Т = 2л У - = I /да 2,1 с.
У th \/ Va2+ll2
Задача 11. Как изменится период колебаний математического маятника, поднятого с поверхности Земли на высоту //?
Решение. Период колебаний маятника на поверхности Земли
2л 1/ — , на высоте Н над Землей 7\ = 2л --, где g и r S	’ gi
gt— ускорения свободного падения на поверхности Земли и на высоте Н. Если пренебречь суточным вращением Земли, силу тяжести маятника в обоих случаях можно приравнять к силе земного при-mM	тМ	g (R-\-H)2
тяжения: mg = у и mgt =	. Отсюда £ = '—!__с.
n	Т. 1/g Т\	R + Н	,
Поскольку ?	= у g	’ то Т =	—Т?- >	1>	т. е. с увеличением вы-
соты Н период колебаний маятника возрастает.
Задача 12. Как изменится период колебаний маятника при перенесении его с Земли на Марс, если масса Марса в 9,3 раза меньше массы Земли, а радиус Марса в 1,9 раза меньше радиуса Земли?
Решение. Периоды колебаний математического маятника на
Марсе и Земле равны соответственно 7\ — 2л и 7\ = 2л х
Г /	„	Л?м
X I/ — . Согласно закону всемирного тяготения, £.м У ~ и
У g3	_____ R^
ёз==У~?-- Таким образом, I- =	=•= l/" да 1,6.
s R;	н т* V Ум V MMR‘3
Задача 13. Материальная точка совершает колебания согласно закону х0= 0,03sin20n/. Определить фазу и смещение точки 1, расположенной на расстоянии у = 5 м от материальной точки, через время t =0,1 с от начала колебаний при скорости распространения волн v = 200 м/с.
Решение. Запишем уравнение волны х= 0,03sin20л —-у*.
Фаза колебаний точки с координатой у <р = 20л	Для
8*
227
/	5 \
точки 1 при t = 0,1 с <Pj = 20л 10,1 —^1 = 1,5л. Искомое смещение этой точки Xj = 0,03 sin 1,5л = —0,03 м.
Задача 14. Волна с частотой v =5 Гц распространяется в пространстве со скоростью 0=3 м/с. Найти разность фаз волны в двух точках пространства, отстоящих друг от друга на расстояние I = 0,2 м и расположенных на прямой, совпадающей с направлением распространения волны.
Решение. Длина волны Х =	=0,6 м. Так как на расстоя-
нии длины волны .1 разность фаз равна 2л, то на расстоянии / ,	.	2л/	2
разность фаз Лер — — = л.
Задача 15. Скорость распространения звуковых колебаний в воде (упругой среде) v = 1450 м/с. На каком расстоянии друг от друга находятся две ближайшие точки, расположенные на одном луче и колеблющиеся в противоположных фазах (разность фаз равна л)? Частота колебаний v = 725 Гц.
Решение. Задачу можно решить двумя способами.
1.	Запишем уравнение звуковой волны х = A sin со [t —	. Ки-
нематические уравнения колебания частиц, расположенных от источника волн на расстояниях /, и /2 (/г >/2); Xj = /lsin<o^— и x2 = j4sinco^—У . Отсюда разность фаз колебаний частиц
= <Р2 — Ф1 — У — “ (? — 7 I = ш = 2яv 1'1-^  По условию, 2nv v = л. Отсюда — /2 =	= 1 м.
2.	Две ближайшие точки, колеблющиеся в противоположных фазах, находятся на расстоянии 1г —1.2= ~ X. Так как X =	, то
11 —= 1 м.
ЗАДАЧИ ДЛЯ САМОСТОЯТЕЛЬНОГО РЕШЕНИЯ
808.	Материальная точка совершает колебания согласно закону у = 0,02 sin (со/ 4- -"-j . Определить фазу колебаний и координату точки в моменты времени ( = 0; t = Т и t — ^Т.
809.	Материальная точка совершает гармонические колебания с периодом Т = 2 с. Движение начинается из положения х0 = 0,54,
где А — амплитуда колебания, а через после начала движения точка находится в положении х = 0,5 м. Записать уравнение движения точки.
228
810.	Записать уравнение гармонического колебательного движения с амплитудой в 0,1 м, периодом 4 с и начальной фазой, равной нулю.
811.	Материальная точка совершает гармонические колебания с амплитудой А = 0,6 м, начав движение из положения равновесия. Записать уравнение движения точки, если через после начала движения ее смещение х = 0,3 м. Какова частота колебаний точки?
812.	Амплитуда гармонических колебаний А = 0,05 м, период Т = 4 с и начальная фаза ф0= Записать уравнение этого колебания. Определить смещение колеблющейся точки от положения равновесия в моменты времени t — 0 и t = 1,5 с.
813.	Материальная точка совершает гармонические колебания с частотой v = 5 Гц. Амплитуда колебания А — 0,5 м. Движение начинается из положения = 0,3 м. Записать уравнение движения точки.
814.	Через сколько времени от начала движения точка, совершающая гармоническое колебание, будет иметь смещение от положения равновесия, равное половине амплитуды? Период колебаний Т = 24 с, начальная фаза tp0 = 0.
815.	Через сколько времени от начала движения точка, совершающая колебательное движение согласно закону х = 7 sin ~ nt. проходит путь от положения равновесия до максимального смещения?
816.	Уравнение колебаний точки имеет вид х = 0,08cos л (t + + 0,2). Определить амплитуду, период и начальную фазу колебаний.
817.	Материальная точка совершает гармонические колебания, уравнение которых имеет вид х = 0,05 sin (”-/ + ?). На сколько увеличится фаза этих колебаний за время At = 2 с?
818.	Какой путь при гармоническом колебательном движении пройдет тело за время t = ^Т? Амплитуда колебания А = — 0,08 м. В начальный момент времени смещение тела было максимальным.
819.	Точка совершает гармонические колебания. В некоторый момент времени t смещение точки хг = 0,05 м. При увеличении фазы вдвое смещение точки стало х2 = 0,08 м. Найти амплитуду А колебаний.
820.	Материальная точка массой tn =0,1 кг совершает гармонические колебания, уравнение которых имеет вид х = 0,04 cos nt. Определить модули скорости точки v, ее ускорения а и возвращающей
4
силы F в момент времени t = Зс. Найти максимальные значения этих величин.
821.	Записать уравнение гармонического колебательного движения, если максимальное ускорение точки йтах= 0,493 м/с2, период
229
колебаний Т = 2 с и смещение точки от положения равновесия в начальный момент времени х0 = 0,025 м.
822.	Уравнение колебания материальной точки массой пг ~ = 1,6 • 10~2 кг имеет вид х — 0,Isin 4- Найти значение максимальной силы, действующей на точку.
823.	Амплитуда колебаний груза массой tn = 1,5 кг, скрепленного с горизонтальной пружиной, жесткость которой k == 1200 Н/м, равна А = 0,1 м. Определить кинетическую и потенциальную энергии системы при фазе ср = 50°, ее полную механическую энергию в этот момент и период колебаний.
824.	Амплитуда гармонических колебаний материальной точки А =0,02 м, полная энергия колебаний Е = 3 • 10-7Дж. При каком смещении от положения равновесия на колеблющуюся точку действует сила F = 2,25 • 10~5 Н?
825.	Материальная точка массой m — 0,01 кг совершает гармонические колебания, уравнение которых имеет вид х — 2 • 10~4Х X sin 8л/. Найти возвращающую силу F в момент t =0,1 с, а также полную энергию Е точки.
826.	Медный шарик на пружине совершает колебания с периодом Ti— 1 с. Каков будет период Т2 колебаний алюминиевого шарика такого же радиуса, подвешенного к той же пружине?
827.	Два математических маятника совершают за одинаковое время соответственно п±= 10 и п2 = 6 колебаний. Разница в длинах маятников А/ = 16 см. Определить длины и /2 маятников.
828.	Период колебаний математического маятника при температуре t°x равен 7\. Каким будет период колебаний, если температура поднимется до /“? Коэффициент линейного расширения материала, из которого изготовлена нить маятника, равен 0.
829.	Во сколько раз изменится период колебаний математического маятника при перенесении его с Земли на Луну? Масса Луны в 81 раз меньше массы Земли, а радиус Земли в 3,7 раза больше радиуса Луны.
830.	Какой глубины скважина должна быть пробита вдоль радиуса Земли, чтобы период колебаний математического маятника па дне её был такой же, как и на Луне? Радиус Луны в 3,7 раза меньше радиуса Земли (/?:> = 6400 км), а плотности вещества Земли и Лупы равны соответственно р, = 5,52- 103 кг/м3 и рл =3,34Х X 103 кг/м3.
831.	На сколько секунд в сутки будут отставать маятниковые часы, поднятые зондом на высоту h = 400 км, если на Земле они шли правильно (То = 1 с)? Принять радиус Земли равным R = = 6400 км.
832.	К пружине подвешено тело массой m = 2 кг. Если к нему присоединить тело массой тх= 300 г, то пружина растянется еще на A/j = 2 см. Каков будет период колебаний, если трехсотграммовый довесок снять и предоставить телу массой 2 кг колебаться?
230
833.	Два грузика подвешены на нитях одинаковой длины I. Первый грузик поднимают по вертикали до точки подвеса, второй (при натянутой нити) — отклоняют на малый угол а от вертикали и одновременно отпускают. Какой из них быстрее достигает положения равновесия?
834.	Сферическая цистерна радиусом 7? находится на платформе, движущейся по горизонтальному прямому пути с ускорением а (рис. 280). С каким периодом будет колебаться находящееся внутри цистерны тело, если его отклонить на угол а <7° от положения равновесия и отпустить? Внутренняя поверхность цистерны гладкая.
Рис. 280
Рис. 282
835.	Кабина, к потолку которой подвешен математический маятник длиной I = 1 м, опускается вниз с ускорением а =2,4 м/с2. Определить период колебаний маятника.
836.	Математический маятник длиной Z совершает колебания вблизи вертикальной стенки (рис. 281). Под точкой подвеса маятника на расстоянии от нее в стену забит гвоздь. Найти период колебаний маятника.
837.	Набухшее бревно, сечение которого постоянно по всей длине, погрузилось вертикально в воду так, что над водой находится лишь малая (по сравнению с длиной) его часть. Период вертикальных колебаний бревна Т = 5 с. Определить длину бревна.
838.	На чашку, подвешенную на пружине жесткостью k (рис. 282), с высоты h падает груз массой m и остается на чашке (удар абсолютно неупругий). Определить амплитуду хтях колебаний чашки (ее массой пренебречь).
839.	Ракета поднимается с ускорением а — 3g. Сколько полных колебаний совершит помещенный в ракете маятник длиной I = 1 м за время, в течение которого ракета поднимается на высоту Н = = 1480 м? Зависимостью ускорения свободного падения от высоты пренебречь.
231
840.	Маятник представляет собой стальной шарик, подвешенный на нити. Масса маятника т = 5 г, период колебаний Т — 1 с. Какова длина нити? Когда под шариком поместили магнит, период колебаний уменьшился до 7\ = 0,8 с. Определить силу притяжения шарика к магниту.
841.	Шарик, имеющий массу m = 10 г и заряд q = 2  10“4Кл, подвешен на нерастяжимой нити длиной I = 25 см в поле плоского горизонтального конденсатора. Разность потенциалов между пластинами конденсатора U = 1206, расстояние между ними d = = 30 см. Чему равен период колебаний шарика на нити?
842.	По грунтовой дороге прошел трактор, оставив следы в виде ряда углублений, находящихся на расстоянии I = 0,3 м друг от друга. По этой дороге покатили детскую коляску, имеющую две одинаковые рессоры, каждая из которых прогибается на Д/г = 2 см под действием силы тяжести F = 10 Н. С какой скоростью катили коляску, если от толчков на углублениях она, попав в резонанс, начала сильно раскачиваться? Масса коляски m = 10 кг.
843.	При какой скорости поезда рессоры вагонов будут особенно сильно колебаться под действием толчков колес о стыки рельсов? Длина рельса I, нагрузка на рессору F, рессора прогибается на h при нагрузке /.
844.	Капли воды падают через одинаковые интервалы времени с некоторой высоты на пластину, закрепленную на пружине. Частота собственных колебаний пластины ю0. Известно, что амплитуда колебаний пластины при этом оказывается максимальной. Найти расстояние между отрывающейся и ближайшей к ней падающей каплей.
845.	Маленький металлический шарик, имеющий массу tn = = 10 г и заряженный положительным зарядом q = 10“4 Кл, висит на нити длиной I = 1 м в электрическом поле с напряженностью Е = 40 В/м. Силовые линии поля направлены вертикально. Шарик отклонили от положения равновесия и отпустили. Определить период колебаний шарика и зависимость частоты колебаний от напряженности поля.
846.	Нить, на которой подвешено тело массой т, привязана к концу доски (рис. 283). Доска совершает гармонические колебания в вертикальной плоскости с амплитудой А. С какой максимальной частотой может колебаться доска, чтобы нить не порвалась, если прочность ее на разрыв Мо?
847.	Подставка, на которой лежит тело массой т, совершает гармонические колебания в горизонтальной плоскости. Чему равна при амплитуде А максимальная частота колебаний подставки, при которой тело еще не скользит по ней, если коэффициент трения ц?
848.	На два вращающихся одинакового радиуса ролика, расстояние между которыми 21, положили симметрично доску МЗССОЙ ГП (рис. 284). С какой силой действует она на каждый ролик при смещении от положения равновесия на Дх, если коэффициент трения между роликами и доской ц? Записать кинематическое уравнение колебаний доски.
232
849.	Скорость распространения волны в среде v = 200 м/с. Вычислить период колебаний, если ближайшее расстояние между точками, колеблющимися в противоположных фазах, I = 20 см.
850.	Волны распространяются вдоль резинового шнура со скоростью v = 3 м/с при частоте v = 2 Гц. Какова разность фаз между точками, отстоящими друг от друга на I = 75 см?
851.	Определить разность фаз колебаний двух точек звуковой волны, распространяющейся в воздухе, расстояние между которыми I — 0,25 м, если частота колебаний источника v = 680 Гц. Скорость звука в воздухе v = 340 м/с.
853 Источник .незатухающих гармонических колебаний движется по закону х = 5 sin3140n/. Определить смещение от поло-
Рис. 283
жения равновесия, скорость и ускорение точки, находящейся на расстоянии I = 340 м от источника, через t = 1 с после начала колебания, если скорость распространения волн v = 340 м/с.
Рис. 284
853. На поверхности озера возбудили волну, которая добежала до крутого берега за t = 1 мин. Расстояние между соседними гребнями волн I = 1,5 м, а время между ударами волн о берег /0= 1 с-На каком расстоянии от берега возбуждена волна?
854. Уравнение колебаний источника волн у — 0,04 sin 600л/. Колебания распространяются в упругой среде. Записать кинематическое уравнение волны, определить период колебаний Т и отклонение от положения равновесия точки, находящейся на расстоянии х = 75 см от источника через t — 0,01 с от начала колебаний при скорости распространения волны v = 300 м/с.
§ 18. Электромагнитные волны
Задача 1. Колебательный контур, состоящий из воздушного конденсатора с площадью пластины 5 = 100 см2 и катушки с индуктивностью L = 10'6 Гн, резонирует на волну длиной X = 10 м. Определить расстояние между пластинами конденсатора.
Решение. Расстояние между пластинами конденсатора можно найти из формулы емкости плоского конденсатора С =
=	, откуда d = -2^- .
233
Емкость найдем из формулы Томсона, определяющей период колебаний в электрическом контуре: Т = 2л У LC, откуда С —
Та _	,	е0в54л2£
~ 4л2£ ’ Тогда d —	.
Неизвестный в условии задачи период колебаний Т можно определить, зная длину волны X, на которую резонирует контур. Длина волны X = сТ, откуда Т = —. Окончательно
d = 4Ji2e^c2S£ з 14 . 10-3 м<
Л2
Задача 2. Колебательный контур состоит из конденсатора емкостью С =5 • 10"8 Ф и катушки индуктивностью L =0,2 Гн. Определить максимальную силу тока /0 в контуре, если максимальная разность потенциалов на обкладках конденсатора U = 90 В. Омическим сопротивлением /? контура пренебречь.
Решение. Задачу можно решить двумя способами: а) исследуя уравнение свободных электромагнитных колебаний; б) применив закон сохранения энергии.
а)	Если пренебречь омическим сопротивлением R контура, то в нем возникнут незатухающие колебания
q = <70sin (aot + ср0),
где <в0 = |/ \/LC — резонансная круговая частота колебаний. Сила тока есть производная от заряда по времени. Поэтому, дифференцируя обе части уравнения для q по времени, получим для силы тока в контуре уравнение
/ = cos (©„/ + <р0).
Величина /0 = является амплитудным, т. е. максимальным значением силы тока в контуре. Подставив значение <о0 и учитывая соотношение = CU0, получим /0 = aaqa = ]/ \fLCCUa = Uo X X У Сс = 0,45Л.
б)	В процессе незатухающих электромагнитных колебаний полная электромагнитная энергия контура, равная сумме энергий электрического поля конденсатора -^-CU2 и магнитного поля катушки ^Ll2, остается постоянной. При этом в те моменты, когда конденсатор максимально заряжен (// = Uo), сила тока равна нулю. Следовательно, полная энергия контура
г = 4 CUZQ.
Когда конденсатор разряжен (U = 0), сила тока достигает максимального значения/0. Тогда полная энергия контура
W = 4 ul
234
Приравняв правые части последних формул, найдем
/0 = £/<,]/£- = 0,454.
Задача 3. Колебательный контур, состоящий из катушки индуктивности и конденсатора с воздушным диэлектриком, имеет резонансную частоту v = 41,405 • 103 Гц. После того как контур поместили под вакуумный колпак и откачали воздух, резонансная частота стала vQ= 41,418- 103 Гц. Определить диэлектрическую проницаемость воздуха.
Решение. Емкость конденсатора пропорциональна диэлектрической проницаемости диэлектрика, заполняющего пространство между пластинами, т. е. С — еС0- Поэтому резонансные частоты v и v0 контура до и после откачки воздуха определяются формулами
1	1	1	/VO\2
2л VLC 2л у e.LC„ 0	2л У LCa	)
Отношение частот очень близко к единице, и при его вычислении приходится учитывать много значащих цифр. Удобнее вычислять не само значение е, а разность между е и единицей:
р — 1 — 2? _ 1 — V(i ~ v2 — v) (v" + v) 1 — v2 1	v2	v2
Частоты v0 и v очень близки, поэтому можно заменить v0 + v® т 2v. В результате получаем е — 1 =	— л; 0,00063. Сле-
довательно, диэлектрическая проницаемость воздуха е = 1,00063.
Задача 4. В катушке колебательного контура находится сердечник из материала с магнитной проницаемостью ц. Как изменится резонансная частота контура, если сердечник вынуть из катушки?
Решение. Обозначим через L индуктивность катушки с сердечником, а через Lo — индуктивность катушки без сердечника. Пусть резонансные частоты контура с сердечником и без него соот-, 1 1 ветственно равны v и v0. Из формул v = —=- и v0 = ——== 2лу LC	2лу Lfi
после подстановки L = ц.Е0 получим v0 = vj/p.. Если проницаемость ц > 1 (сердечник изготовлен из ферромагнитного или парамагнитного материала), то v0 > v, т. е. удаление сердечника ведет к повышению частоты собственных колебаний контура. Когда р. < 1 (сердечник изготовлен из диамагнетика), v0 < v, т. е. удаление сердечника ведет к уменьшению частоты собственных колебаний контура.
ЗАДАЧИ ДЛЯ САМОСТОЯТЕЛЬНОГО РЕШЕНИЯ
855.	На какой длине волны работал передатчик автоматической станции «Луна-9», если частота излучаемых им волн была v = = 183,538  10е Гц?
235
856.	Колебательный контур, состоящий из катушки индуктивности и воздушного конденсатора, настроен на длину волны = = 300 м. При этом расстояние между пластинами конденсатора dl — = 4,8 мм. Каким должно быть это расстояние, чтобы контур был настроен на длину волны Х2= 240 м?
857.	Какой емкости конденсатор нужно соединить с катушкой индуктивностью L = 5 • 1О’Й Гн, чтобы получившийся колебательный контур имел частоту собственных колебаний v = 6 X X 104 Гц?
858.	Радиоприемник настроен на радиостанцию, работающую на длине волны )^= 25 м. Во сколько раз нужно изменить емкость приемного колебательного контура радиоприемника,} чтобы настроиться на длину волны Х2 = 31 м?
859.	Колебательный контур состоит из катушки индуктивностью L =2  10-3 Гн и конденсатора емкостью С =8 • 10~8 Ф. Значение емкости может отклоняться от указанного на 2%. Вычислить, в каких пределах может изменяться длина волны, на которую резонирует контур.
860.	При изменении силы тока в катушке индуктивности на А/ = = 1 А за А/ = 0,6 с в ней возникает э. д. с. 8 = 0,2  10~3 В. Какой длины X будет радиоволна, излучаемая генератором, колебательный контур которого состоит из этой катушки и конденсатора емкостью С = 141 пФ?
861.	Емкость переменного конденсатора изменяется от = = 56 пФ до С2 = 667 пФ. Какой комплект катушек индуктивности нужно иметь, чтобы колебательный контур можно было настраивать на радиостанции в диапазоне длин волн от Х1=40 м до Х2 = = 2600 м?
862.	Радиоприемник можно настраивать на прием радиоволн от 25 до 2000 м. Что необходимо делать при переходе к приему более длинных волн — уменьшать или увеличивать емкость конденсатора колебательного контура?
863.	Контур радиоприемника судовой радиостанции настроен на частоту v = 9 МГц. Как нужно изменить емкость переменного конденсатора колебательного контура приемника, чтобы он был настроен па длину волны А = 50 м?
864.	В колебательном контуре происходят свободные колебания. Зная, что максимальный заряд конденсатора q = 10~6 Кл, а максимальная сила тока Z = 10 А, найти длину волны, излучаемой контуром.
865.	Колебательный контур состоит из дросселя индуктивностью L = 0,2 Гн и конденсатора емкостью С = 10~5 Ф. Конденсатор зарядили до напряжения U = 2 В, и он начал разряжаться. Какой будет сила тока в момент, когда энергия контура окажется распределенной поровну между электрическим и магнитным полями?
866.	Колебательный контур содержит конденсатор емкостью С = 8 пФ и катушку индуктивностью L = 0,5  10-3 Гн. Каково максимальное напряжение Um™ на обкладках конденсатора, если максимальная сила тока /щах = 40 мА?
236
867.	Имеется два колебательных контура с одинаковыми катушками и конденсаторами. В одном из контуров в катушку вставили железный сердечник, увеличивший ее индуктивность в п = 4 раза. Найти отношение резонансных частот контуров и их энергий, если
максимальные заряды на конденсаторах равны.
868. В колебательный контур с индуктивностью L, емкостью С и активным сопротивлением R включили последовательно источник синусоидальной э. д. с. с амплитудой <§тах. Затем, изменяя частоту э. д. с., добились того, что амплитуда силы тока стала максимальной. Найти значение /т.|Х.
з™ д
Рис. 285
869.	Электроемкость колебательного контура С = 10 мкФ. Построить график зависимости частоты колебаний силы тока от индуктивности L данного контура.
870.	Длина воздушной линии электропередачи I = = 300 км. Частота напряжения v =50 Гц. На какую долю периода отличаются фазы напряжения в начале и в конце этой линии?
871.	После того как конденсатору колебательного контура сообщен заряд q = 10'а Кл,
в контуре происходят затухающие электрические кое количество теплоты Q выделится в контуре к тому времени, когда колебания полностью затухнут? Емкость конденсатора С = 0,01 мкФ.
872.	Колебательный контур, состоящий из катушки индуктив
Рис. 286
колебания. Ка-
ности и конденсатора, через выключатель подключен к источнику постоянной э. д. с. с внутренним сопротивлением г (рис. 285). Первоначально выключатель замкнут. После установления стационарного режима выключатель размыкают, и в контуре возникают колебания с периодом Т. При этом амплитуда напряжения на конденсаторе в п раз больше э. д. с. батареи. Найти индуктивность катушки и емкость конденсатора. Омическим сопротивлением катушки пренебречь.
873.	Антенна телевизора (точка С на рис. 286) наряду с волной, идущей непосредственно от передающей станции (точка А), принимает волну, отраженную от железной крыши здания (точка В). Вследствие этого изображение двоится. На сколько сантиметров сдвинуты изображения друг относительно друга, если антенна, станция и крыша здания расположены на расстояниях, указанных на рис. 286? Ширина экрана телевизора I = 50 см. Учесть, что изображение в кадре телевизора разлагается на 625 строк и в секунду передается 25 кадров. Временем обратного хода лучей пренебречь,
237
V. ОПТИКА
§ 19, Фотометрия
Задача 1, Биспиральные электролампы накаливания с криптоновым наполнением мощностью 40, 100, 1000 Вт имеют световую отдачу соответственно 11,5; 13,5; 18,6 лм/Вт. Определить полный (номинальный) световой поток, излучаемый каждой электролампой, и силу света.
Решение. Световая отдача электролампы равна отношению полного светового потока Ф к номинальной мощности лампы:
1]	=Ф//У, откуда Ф = r\N.
Подставляя численные значения ц и М, находим:
ф] = 460 лм; Ф,= 1350 лм: Фа= 18 600 лм.
Для определения силы света воспользуемся соотношением между полным световым потоком Ф и силой света /: Ф = 4л/, откуда / = ф/4л. Тогда 36,6 кд; 1107,5 кд; /3 = 1480,9 кд.
Задача 2. Сила света точечного источника / — 200 кд. Определить освещенность поверхности, перпендикулярной к направлению лучей, которая находится на расстоянии R = 5 м от источника.
Решение. Можно применить первый закон освещенности, так как лучи падают перпендикулярно к освещаемой поверхности (i = 0°). Тогда
Е = -j, = 8 лк. i\ 
Задача 3. На столбе высотой h — 8 м подвешена электролампа, сила света которой / = 300 кд. Определить освещенность поверхности Земли вблизи основания столба и на расстоянии I — 4 м от его основания (рис. 287).
Решение. Освещенность вблизи основания столба
£i = ^==/^~4'7 лк-
В точку В лучи падают под углом /, который равен углу BSA (как внутренние накрест лежащие при параллельных прямых ЗЛ и BD).
Из д ASB определяем SB= R2 и cos i:
R2 = V'h2 4-	8,9 m; cos i — x 0,9.
<J D
Подставляя значения R2 и cos i в формулу закона освещенности, получаем
Е„ = cos / « 3,4 лк.
Задача 4. Свет от электролампы силой света / — 400 кд падает на тетрадь ученика под углом i = 60° (рис. 288) и создает осве-238
щенность Е = 150 лк. Определить, на каком расстоянии от тетради расположена лампа и на какой высоте относительно тетради она подвешена.
Решение. Освещенность в центре тетради
£ = ^2 cos i, откуда R = I/ g да 1,2 м.
Из A SAB определяем высоту, на которой подвешена лампа: h = R cos ZBSA; поскольку ZBSA = I (как внутренние накрест лежащие при параллельных прямых и BD), то
h — R cos i = 0,6 м.
Рис. 287	Рис. 288	Рис. 289
Задача 5. Электрическая лампа накаливания мощностью N = 40 Вт излучает световой поток Фп = 415лм. Она подвешена на высоте h, = 1,5 м над центром круглого стола диаметром D = = 2 м. Определить световую отдачу лампы, силу света источника, освещенность в центре и на краю стола (рис. 289).
Решение. Световая отдача лампы
Ф
ц = уда 10,4 лм/Вт.
Сила света
/ = -а « 33 кд. 4Л
Освещенность в центре стола
/ Ф„ £1== / =г-^да 14,7 лк, 1 h1 4n/i2
так как угол падения лучей в центре стола i = 0 и cos i — 1. Освещенность на краю стола
£2 = -2 cosi = 4^cost, где г = |/ h2 +	. cos i
Ф„й
Поэтому Е2 = — ----—зу2 да 12,7 лк.
4л (Л2+ т)
239
Задача 6. Какую среднюю освещенность создает электрическая лампа накаливания мощностью N = 100 Вт, световая отдача которой г] =13,5 лм/Вт, если на поверхность S = 6 м2 падает 50 % светового потока, излучаемого лампой?
Решение. По определению, освещенность Е = В данном случае АФ составляет 50 % всего светового потока Фо, излучаемого лампой, т. е. АФ = 0,5Фо. Весь световой поток Фп = т)АП Тогда средняя освещенность, создаваемая лампой на площади S,
£ =	= 112,5 лк.
Задача 7. Какой мощности нужно взять электролампу для создания нормальной освещенности (100 лк) обрабатываемой на станке детали, находящейся на расстоянии £ = 50 см от лампы? Лучи от лампы падают нормально к поверхности детали. Световая отдача лампы г] = 13,5 лм/Вт.
Решение. Сила света лампы / = £/?2. Световой поток, излучаемый лампой, Ф = 4л/ = 4л£/?2. Поскольку световая отдача
т]	= Ф/ZV, то N = ФА] = 4л£/?2/ц = 23 Вт.
Задача 8. На каком расстоянии следует поместить электролампу мощностью N = 200 Вт, световая отдача которой г) = = 13,5 лм/Вт, чтобы освещенность рабочего места ювелирного мастера была £ = 360 лк?
Решение. Расстояние от лампы до рабочего места /? = = VНЕ. Силу света лампы определим из формулы полного светового потока Ф = 4л/, откуда / = Ф/4л. Полный световой поток Ф = т] N. Следовательно,
/? = ]/Ф/4л£ = ]£т]У/4л£ « 0,6 м.
Задача 9. На каком расстоянии друг от друга следует подвешивать люминесцентные лампы мощностью по N = 80 Вт и светоотдачей т) = 65,3 лм/Вт, чтобы освещенность на поверхности рабочего места в точке, лежащей посредине между двумя лампами, была не меньше £ = 100 лк? Высота рабочего места hY — 0,7 м, расстояние от поверхности стола до лампы h2 = 3,5 м, лампы подвешиваются на расстоянии /i3=0,3 м от потолка.
Решение. Освещенность в точке М (рис. 290) равна сумме освещенностей, создаваемых обеими лампами, и должна быть не меньше нормы £, т. е. £t+ £2 > £. Вследствие симметрии £1 = = Е2. Следовательно, 2£] = £, откуда £j = Е/2. Но £j = ^cosZ. Учитывая, что г =	, получим £1 = Z,cos3j.
Расстояние между лампами I = 2/i2tg i. Косинус угла падения i cosl = |/ —=|/
240
Поскольку световой поток Ф = 4л/, то 1 = Ф/4л. Световой поток ф — r\N. Подставив значение Ф в формулу для 1, а значение 1 — в формулу для cos i, получим
cos 1 = 1/ —г/«0,9; i = 25°48'.
У T)/V
Расстояние, на котором следует подвешивать лампы,
I = 2/i2 tgi л; 2,4 м.
Задача 10. Прожектор дает пучок света в виде узкого конуса с телесным углом 0,03 ср. Какую освещенность дает прожектор на расстоянии R = 2 км, если в качестве источника света используют
Рис. 290	Рис. 291
трубчатую ксеноновую лампу «Сириус» мощностью N — 100 кВт, светоотдача которой ц = 50 лм/Вт? Поглощением света в воздухе пренебречь.
Решение. Освещенность, создаваемая прожектором, Е = = -^cos i. Будем определять освещенность на поверхности радиусом R. В этом случае S = О£2. Световой поток Ф = т]2У. Тогда £ =	« 83 лк.
Задача 11. Точечный источник 5, сила света которого / = = 50 кд, находится над поверхностью стола на высоте R — 1 м (рис. 291). Найти освещенность в точке М, в которую лучи от S падают нормально. Как изменится освещенность в точке М, если сбоку от S на расстоянии R поместить плоское зеркало Z, отражающее свет в М? Считать, что коэффициент отражения равен 1.
Решение. Освещенность в точке М. при отсутствии зеркала
= 50 лк. После установки зеркала Z в точку М. будут попадать также лучи, отраженные от зеркала. Освещенность в точке М создается как бы двумя источниками света: действительным S и мнимым S'.
241
При коэффициенте отражения, равном 1, мнимый источник S' обладает такой же силой света, как и источник S. Следовательно, дополнительная освещенность в точке М будет
£д = г4-созг,
где г = ОМ + OS', а I = 45° (по построению).
Из £±SOS' видно, что OS = OS' (как стороны равнобедренного треугольника); но так как по построению SO = SM, то OS' = SM. Из Л OSM находим: ОМ = VSM2SO'1 = R lz2. Следовательно, г -= К2/? + OS' = V~2R + R = R (V2 -j- 1). Возведем в квадрат левую и правую части этого выражения:
г2 = R2 (3 + 2 ]/ 2) « 5.82R2.
Полная освещенность в точке М равна сумме освещенностей и £д:
„ г , С / , / cos 45°	/ /. , cos 45° I
С —	— ^-2 + 5 82у?2 — R2 ( 1 +	5(82 j —
= 1,12 ^2= 1,12^.
ЗАДАЧИ ДЛЯ САМОСТОЯТЕЛЬНОГО РЕШЕНИЯ
874.	Прожектор дает пучок света в виде узкого конуса с телесным углом О = 0,03 ср. Какую освещенность дает прожектор на расстоянии R = 2 км, если световой поток прожектора Ф = = 6 • 10е лм? Поглощением света в воздухе пренебречь.
875.	Параллельный пучок лучей, несущий однородный световой поток плотностью 200 лм/м2, падает на плоскую поверхность, внешняя нормаль к которой образует с направлением лучей угол 120°. Какова освещенность этой поверхности?
876.	Какой части освещенности Солнцем на экваторе равна освещенность горизонтальной поверхности в Киеве в полдень в день равноденствия? Географическая широта Киева 50°.
877.	Максимальная освещенность горизонтальной поверхности в летний полдень Е = 106 лк. Сколько энергии падает на 5 = 1 га поверхности земли в течение t = 1 ч? Механический эквивалент света q = 1,6 • 10-3 Вт/лм.
878.	Ширина кадрового окна киноаппарата dr = 0,012 м, ширина экрана с!а=2,4м. Освещенность экрана должна быть не менее Е = 4 лк. Определить минимальную освещенность кадрового окна киноаппарата.
879.	Вычислить дневной расход электроэнергии в теплице, освещаемой лампами с удельной мощностью 1 Вт/кд, если продолжительность светового дня для растений t = 12 ч, а освещенность Е = — 250 лк. Над каждым квадратным метром расположено п = 2 лампы; площадь теплицы S = 10 м2. Считать, что освещенность создается за счет т] = 40 % полного светового потока ламп.
242
880.	Лампа накаливания и лампа дневного света, потребляющие по Ni = 40 Вт. дают на расстоянии R = 2 м при нормальном падении лучей освещенности соответственно Е1--= 7 лк и £2 = 56 лк. Какова удельная мощность каждой из этих ламп? Определить полный световой поток, излучаемый лампами.
881.	Норма освещенности рабочего места для точных работ £’ = 100 лк. Какой силы света лампочки надо подвесить над рабочим местом на высоте не менее 1г = 2 м?
882.	В настольной лампе используется 40-ваттная лампа. Она излучает Ф = 355 лм. Высота лампы над столом h — 0,3 м. На каком расстоянии от тетради ученика должна находиться лампа, чтобы освещенность тетради составляла Е ~ 150 лк?
883.	Лампа мощностью W = 500 Вт дает на расстоянии R = — 5 м при нормальном падении лучей освещенность Е — 28 лк. Определить световую отдачу лампы.
884.	Норма освещенности для выращивания рассады огурца в теплицах Е = 400 лк. Для освещения используются лампы дневного света мощностью N = 65 Вт, световая отдача которых г; = 50 лм/Вт. Определить, на какой высоте над рассадой следует подвешивать эти лампы. Считать, что свет падает нормально к поверхности почвы.
885.	Расстояние от источника, сила света которого £ = 20 кд, до фотометра R}-= 0,2 м, а до исследуемого источника R2~~ 0,6 м. Чему равна сила света исследуемого источника?
886.	На расстоянии 1,2 м друг от друга находятся две лампы, силы света которых R = 20 кд и 40 кд. Определить, где следует поместить между ними экран, чтобы он имел одинаковую освещенность с обеих сторон.
887.	Две лампы силой света R = 20 кд и /2 = 30 кд находятся на расстоянии R— 2 м друг от друга. На каком расстоянии от первой лампы (по линии, проходящей через центры ламп) надо поместить лист бумаги, чтобы его освещенность со стороны первой лампы была в п — 1,5 раза больше, чем со стороны второй?
888.	Лампа в 1 = 500 кд подвешена над серединой круглого стола на высоте /г = 1 м. Определить освещенность края стола, если его диаметр D =1 м.
889.	На столбе высотой h = 5 м висит лампа в 1 = 200 кд. На каком расстоянии от основания столба освещенность на земле равна Е = 1 лк?
890.	На двух мачтах на высоте h = 8 м подвешены лампы по / = 1000 кд. Определить освещенность поверхности Земли под каждой лампой, если расстояние между ними I — 30 м.
891.	Для выращивания рассады ранней капусты выбрали площадку квадратной формы со стороной d = 1,3 м. Лампа силой света / =400 кд подвешена над центром площадки на высоте £ = 2,2 м. Определить максимальную и минимальную освещенности площадки.
892.	На высоте h — 3 м над центром круглой арены диаметром d = 10 м висит лампа в I = 250 кд. Принимая лампу за точечный
243
источник, определить, сколько процентов площади арены имеют освещенность не менее Е = 6 лк.
893.	Над серединой чертежной доски, образующей с горизонтальной плоскостью угол а = 30°, на высоте h, = 2 висит лампа, сила света которой / = 200 кд. Определить освещенность поверхности доски. Соответствует ли эта освещенность принятой Е„ = 50 лк? Лампу считать точечным источником света.
894.	Классная комната площадью 3 == 60 м2 освещается лампами дневного света. 40 % светового потока, излучаемого лампами, распределяется на уровне высоты ученических парт. Светоотдача 30-ваттных ламп ц = 63 лм/Вт. Нормальная освещенность ученических рабочих мест Е = 150 л. Сколько ламп необходимо разместить в классной комнате?
895.	Над горизонтальной поверхностью помещены на высоте h = 2 м и на расстоянии I = 1 м друг от друга два источника света, дающие световые потоки по Ф = 300 лм каждый. Определить освещенность поверхности в точках под источниками и на середине расстояния между ними.
896.	Над небольшой сценой на высоте h = 5 м размещены два светильника, излучающие световые потоки Ф,= 9420 лк и Ф2 = = 12 560 лм. Расстояние между светильниками I = 8,66 м. Чему равна освещенность сцены под светильниками на середине расстояния между ними (считая по проекциям)?
897.	Лампа на / = 200 кд находится на расстоянии d = 1 м от рабочего места чертежника. Освещенность поверхности чертежа Е — 100 лк. Под каким углом падают лучи света на чертеж и на какой высоте над рабочим местом подвешена лампа?
898.	Над вогнутой полусферой находится точечный источник света на высоте, равной радиусу полусферы. Определить освещенность в точке полусферы, наиболее удаленной от источника, и в точках на краю полусферы. Сила света источника I = 100 кд, радиус полусферы /? = 2 м.
899.	Над партой, наклоненной к горизонту под углом а = 20°, на высоте h = 2 м висит лампа, излучающая полный' световой поток Ф = 2715 лм. Какова освещенность поверхности парты, создаваемая этой лампой?
900.	При фотографировании объекта, освещенного 100-ваттной лампой на расстоянии d = 1 м, требуется экспозиция /1=8с. Какова должна быть экспозиция при освещении объекта двумя 100-ваттными лампами, находящимися на расстояниях t\ — 3 м и г2 = 4 м от объекта, если общее количество энергии, попавшей на светочувствительную пластинку, должно быть одинаковым?
901.	Для печатания фотоснимка при лампе силой света в 1г = = 40 кд на расстоянии Rr = 1 м требуется продолжительность печатания tl — 2 с. Какова должна быть продолжительность печатания при лампе /2= 30 кд на расстоянии R2 — 1,5 м? Предполагается, что общее количество энергии, полученной фотоснимком, должно быть в обоих случаях одинаковым. --------------
244
902.	Лучи от лампы падают нормально на селеновый фотоэлемент. Площадь фоточувствительного слоя фотоэлемента S = = 2,5 • 10~3 м2. Гальванометр, подключенный к фотоэлементу, показывает силу тока i = 18 мкА. Чувствительность фотоэлемента k = 360 мкА/лм. Расстояние до лампочки R = 2 м. Определить силу света лампочки.
903.	На расстоянии /? =0,7 м от фотоэлемента помещена лампа силой света /г = 240 кд. Определить полный световой поток лампочки и силу тока, которую покажет гальванометр, подключенный к фотоэлементу, если его рабочая поверхность 5 = 10~3 м2, а чувствительность k = 280 мкА/лм.
904.	Автомобильная лампочка, которую можно принять за точечный источник силой света в / = 20 кд, находится на расстоянии 7? = 2 м от собирающей линзы диаметром d{--= 0,08 м. За линзой расположен экран, на котором лучи света, прошедшие через линзу, дают светлый кружок диаметром d2 = 2 см. Определить освещенность кружка. Поглощением света в линзе пренебречь.
905.	Точечный источник света помещен над центром круглого стола. Сила света источника / = 50 кд, радиус стола г — 0,5 м, высота источника над столом h = 1 м. Определить: а) зависимость освещенности Е стола от расстояния г до его центра; б) освещенность в центре и на краю стола.
906.	На вертикально висящую картину площадью 5 = 0,48 м2 падает свет под углом а = 25° к нормали от источника силой света 1 — 100 кд с расстояния /? = 1 м. Какой световой поток падает на картину, если на противоположной стене подвешено большое зеркало на расстоянии d — 2 м от лампы?
§ 20. Прямолинейное распространение света.
Отражение света
Задача!. Вертикально поставленная веха высотой hr= 1,5 м при солнечном освещении отбрасывает тень длиной /х= 2 м. В то же время заводская труба отбрасывает тень длиной 12 = 50 м. Какова высота заводской трубы?
Р е ш е н и е. Из подобия треугольников АВС и А 'В'С (рис. 292) следует: ~ — у2 , откуда высота заводской трубы
б Ч
/г2 = h1 = 37,5 м.
Задача 2. Определить длину тени, отбрасываемой Луной во время новолуния, когда расстояние между центрами Луны и Солнца приблизительно равно 1,5 • 108 км. Отношение диаметров Луны и Солнца около 1 : 400.
Решение. Луна отбрасывает тень в виде конуса (рис. 293, а). Длину тени (высоту конуса О'В) можно определить из подобия треугольников ОАВ и О'А'В (рис. 293, б):	— откуда
245
О'В =	• OB, где О А и O'A' — соответственно радиусы Солнца
и Луны. Из рис. 293, б видно, что ОВ = 00' + О'В, где 00' — расстояние между центрами Солнца и Луны. С учетом сказанного О' А'
длина тени, отбрасываемой Луной, О'В = -„-т- (00' + О'В), или С/ Pl
Рис. 292
откуда
°'b=woo7('-
О'А'
от,~3.8.№
КМ.
Задача 3. Лампа помещена в матовый стеклянный шар радиусом 0,2 м и подвешена на высоте 5 м
над полом. Под лампой на высоте 1 м держат мяч радиусом 0,1 м. Определить размеры тени и полутени, образуемых мячом.
Решение. После включения лампы на полу образуется тень мяча, окруженная широкой полутенью (рис. 294, а). Вследствие прямолинейного распространения света в область тени не попадают
Рис. 293
лучи от светящейся поверхности источника (поверхности стеклянного матового шара), а в область полутени попадают лучи только от некоторых точек светящейся поверхности.
Из рис. 294,6 видно, что радиус тени А'К' ~ NK = (АК— — AN), где АК — радиус стеклянного матового шара. Из подобия треугольников АМС и ANA': =-4^, откуда AN = ft IV LVL	L И10
где A'N = 5 м, МС = 5 м— 1 м = 4 м — расстояние между мячом 246
и источником света, AM — АК — МК — 0,1 м. Тогда AN = 0,125 м; А'К' = 0,075 м.
Радиус полутени А"К' = А'А" — А'К'- Из подобия треугольников А А 'А" и ACD:	, откуда А'А" — CD — 0,25 м,
так как CD = 0,2 м — диаметр мяча. Следовательно, радиус полутени А"К' = А'А" — А'К' = 0,175 м.
Задача 4. Человек ростом h = 1,85 м, стоящий на берегу озера, видит в небе по направлению, составляющему угол i = 50°с
Рис. 296
Рис. 294
горизонтом, Луну. На каком расстоянии от себя увидит человек отражение Луны в воде озера?
Решение. Луна находится на значительном расстоянии от наблюдателя, поэтому можно считать, что от Луны на поверхность воды озера падает параллельный пучок света и, отразившись, попадает в глаз человека (точка О, рис. 295). Из треугольника О АВ искомое расстояние АВ = О A tg а, где О А — расстояние от поверхности Земли до глаза человека, которое приблизительно равно росту человека. Угол отражения I' равен углу падения: i = 90° — 50° = = 40°. Тогда АВ « 1,55 м.
Задача 5. Солнечный луч, проходящий через отверстие в ставне окна, образует с поверхностью стола угол а = 48°. Как надо поставить плоское зеркало, чтобы изменить направление луча на горизонтальное?
Решение. Если плоское зеркало лежит на горизонтальной плоскости стола (рис. 296), то угол падения луча i — 9(?° — 48° =
247
== 42°. Угол отражения Г для этого случая также равен 42°. Угол между падающим и отраженным лучами i + г" = 21. Если плоское зеркало повернуть на такой угол а, чтобы отраженный луч S" имел горизонтальное направление, то Z SOS" — 180° — 48° = 132°. Но Z5OS" = 2Z1. Отсюда следует, что j1=ZSOS''/2, или z\ = 66°.
Угол поворота NON' перпендикуляра к зеркалу, а следовательно, и угол поворота зеркала а составляет
ANON' = а = ц —1 = 24°.
Задача 6. На плоское зеркало падает луч света. Зеркало поворачивают на 5° вокруг оси, находящейся в плоскости зеркала, перпендикулярно к лучу. На какой угол а при этом повернется
отраженный луч? На какое расстояние х переместится световой зайчик на экране, расположенном перпендикулярно к отраженному лучу на расстоянии I = 10 м от точки падения лучей на зеркало?
Решение. Пусть угол падения луча i (рис. 297). Луч света отразится от зеркала под углом I' = i и на экране будет виден световой зайчик. Угол между падающим и отраженным лучами р = i I = 2/.
Повернем зеркало Z вокруг оси, лежащей в плоскости зеркала и проходящей через точку О, на угол а (расположение Zj). При этом перпендикуляр п к зеркалу Z повернется на угол а и займет положение n±. Вследствие этого угол падения луча на зеркало уменьшится на а, т. е. = z— а. Согласно закону отражения света, угол между отраженным лучом 3 и перпендикулярем пг (угол отражения) z,' = z\ = z— а. Угол между лучами 1 и 3 будет у =	(( = 2i1 = 2(1 — а). Отраженный луч при переходе из по-
ложения 2 в положение 3 повернется на угол 6 = р— у—21— •—2(z—а) — 2 а. Следовательно, при повороте плоского зеркала на угол а отраженный луч поворачивается на угол S = 2а. По условию задачи, а = 5°, следовательно, 6 = 10°.
248
Смещение х светового зайчика при повороте зеркала на угол а равно MN. Из прямоугольного треугольника OMN следует, что MN = ОМ • tg 6 = 1,8 м.
Задача 7. Доказать, что источник S и два его изображения, полученные с помощью зеркал, расположенных под углом а друг к другу, лежат на окружности. Указать положение центра этой окружности.
Решение. Построим мнимое изображение источника S, которое можно получить с помощью плоского зеркала Zr Для этого из S опускаем перпендикуляр на плоскость, в которой лежит зеркало Zj (рис. 298). Он пересекает плоскость в точке О'. На про
Рис. 299
Рис. 300
должении перпендикуляра SO' откладываем отрезок O'S' = SO'. Точка S' является искомым мнимым изображением источника S. Аналогично находим мнимое изображение S", получаемое с помощью зеркала Z2.
Рассмотрим треугольники SO'O и S’O'O. Поскольку у них по два равных катета (SO' = S'O' по построению, а катет 00' — общий), то они равны. Следовательно, OS — OS'. Аналогично доказывается равенство треугольников SO"O и S"O"O, откуда SO = = S"O. Таким образом, если радиусом OS провести окружность, то источник S и его мнимые изображения S' и S", полученные с помощью зеркал Zx и Z2 соответственно, будут находиться на окружности, центр которой О лежит на прямой пересечения плоскостей, в которых находятся зеркала Z2 и Z2.
Задача 8. Перед плоским зеркалом поставлена лампа. На сколько изменится расстояние между лампой и ее мнимым изображением, если зеркало отодвинуть от лампы на AZ = 0,2 м?
Решение. Расстояние от лампы до ее мнимого изображения, полученного с помощью плоского зеркала, равно 21 (рис. 299, а). Если зеркало Z отодвинуть от лампы на А/ = 0,2 м, то расстояние между зеркалом Z и лампой S будет I AZ, а между S и S' —
249
2 (/ + AZ) (рис. 299, б). Следовательно, если зеркало отодвинуть от лампы на A Z, то расстояние между лампой и ее мнимым изображением увеличится на 2(1 + A/) — 2Z = 2AZ — 0,4 м.
Задача 9. Перед зеркалом воткнуты булавки А и В. Как должен расположить глаз наблюдатель, чтобы мнимые изображения булавок накладывались друг на друга?
Решение. Мнимые изображения А' и В’ (рис. 300) для наблюдателя накладываются тогда, когда его глаз размещен на прямой, проходящей через точки А' и В' этих изображений, т. е., например, в положении М.
ЗАДАЧИ ДЛЯ САМОСТОЯТЕЛЬНОГО РЕШЕНИЯ
907.	Тень от летящего вдоль реки самолета упала на мост и покрыла его на 2/3 его длины. Какова длина моста, если размах крыльев самолета d = 12,4 м?
908.	Как следует расположить точечный источник света,
плоский предмет и экран, чтобы
Рис. 301
контур тени на экране был подобен контуру предмета?
909.	Какова длина полной тени, отбрасываемой земным шаром, освещаемым Солнцем? Радиус Земли R = = 6400 км, радиус Солнца равен НО земным радиуоам. Расстояние между центрами Земли и Солнца равно примерно 23 900 земным радиусам.
910.	Изображение предмета АВ можно получить с помощью камеры с маленьким отверстием. Глубина камеры 0,2 м, расстояние до предмета 0,4 м, диаметр отверстия 1 мм. Можно ли при таких условиях различать на изображении детали предмета, размеры которых 2 мм?
911.	Падающий и отра
женный лучи образуют угол а — 120°. Чему равен угол падения луча?
912. При выполнении маркировочных работ и для контроля правильности установки конструктивных элементов каркаса в проектное положение используют лазер (рис. 301). На каких законах оптики основан этот метод? Какой должен быть угол падения лучей на плоские зеркала?
913. Плоское зеркало вращается с постоянной частотой v = = 2,5 с-1. С какой скоростью будет перемещаться «зайчик» по сфе-
250
рическому экрану радиусом R = 5 м, если зеркало находится в центре кривизны экрана?
914.	Человек, стоящий на берегу озера, видит в гладкой поверхности воды изображение Солнца. Как будет перемещаться изображение при удалении человека от озера? Солнечные лучи считать параллельными.
915.	На какой высоте h находится аэростат А, если с башни высотой //он виден под углом а над горизонтом, а его изображение В в озере видно под углом |3 под горизонтом (рис. 302)?
Рис. 304
Рис. 306
Рис. 305
916.	Построить изображение треугольника АВС в плоском зеркале MN.
917.	Узкий пучок света У падает на двугранный угол а = 60 °, образованный одинаковыми плоскими зеркалами ОМ и ON, закрепленными на оси О (рис. 303). После отражения от зеркал свет фокусируется линзой и понадает в неподвижный приемник Л. Зеркала вращают с постоянной угловой скоростью. Какая часть световой энергии пучка за время, намного большее периода вращения, достигает приемника, если пучок проходит на расстоянии от оси, равном половине длины отрезка ОМ?
918.	Построить изображение светящейся точки в системе двух взаимно перпендикулярных зеркал.
919.	В каком направлении нужно пустить луч света из точки М (рис. 304), находящейся внутри зеркального ящика, чтобы он попал в точку /V, отразившись по одному разу от всех четырех сте
251
нок? Точки М и N лежат в одной плоскости, перпендикулярной к стенкам ящика (в плоскости рисунка).
920.	Определить графически, при каких положениях глаза наблюдатель может видеть в зеркале конечных размеров изображение отрезка прямой (рис. 305).
921.	Человек и плоское зеркало расположены так, как указано на рис. 306. Сможет ли он увидеть свое изображение в зеркале? Где нужно расположить глаз наблюдателя, чтобы увидеть изображение этого человека?
922.	Какой наименьшей высоты должно быть плоское зеркало, укрепленное вертикально на стене, чтобы человек мог видеть свое изображение в нем во весь рост, не изменяя положения головы? На каком расстоянии от пола должен быть нижний край зеркала?
923.	На горизонтальном столе лежит шар. Под каким углом к плоскости стола нужно установить плоское зеркало, чтобы при движении шара к зеркалу наблюдаемое изображение шара двигалось по вертикали?
924.	Два плоских зеркала расположены под углом друг к другу и перед ними помещен источник света. Где следует расположить глаз наблюдателя, чтобы одновременно видеть оба изображения источника в плоских зеркалах?
§ 21. Преломление света
Задача 1. Свет падает из вакуума на поверхность стекла, абсолютный показатель преломления которого п = 1,63. Определить угол преломления, если известно, что угол между падающим и отраженным лучами 90°.
Решение. Абсолютный показатель преломления среды п = =	, где с и Cj — скорости света в вакууме и стекле, i —
угол падения луча, г — угол преломления. Из этой формулы sin г =	. Угол падения I равен углу отражения i' (рис. 307).
По условию задачи, i + i' — 21 — 90°. Следовательно, угол падения £ = 90'72 = 45°. Тогда sin г = s‘^|-0,44. По таблице синусов находим г = 26°.
Задача 2. Луч света падает из воздуха на поверхность стекла под углом 40°. Как изменится угол преломления, если угол падения увеличится на 20°?
„	sin t n
Решение. Согласно закону преломления света, п = ^=-=11о sin ц sin г, sin (0 + 20°)	__„ . „ ,
условию задачи, п = =— =	= —=—-------. Зная п, о и г2,
311L Г1	Ы11Г2	0111/2
определим г1 и г2:
sin г, =	« 0,402; г, = 23°42';
sin г, = ^^0,541; г2 = 32°45'.
2 п	>	’	2
252
Следовательно, угол преломления изменится на
Аг = Г2_ Г1== 32°45' — 23°42' = 9°3'.
Задача 3. На горизонтальную поверхность среды падает луч света из воды под углом 0 = 20° к поверхности, преломляется и идет под углом у = 46° к поверхности. Определить абсолютный показатель преломления среды.
Решение. Показатель преломления среды относительно воды
п„, —	== -1- . Абсолютные показатели преломления для воды и
21 sin г с2
r,	Sin I	с	с	п„
Подставив	значения	с, и с,, получим	п.21	= -—=	_	;	—	=-Д
122	21	Sill Л	/1,	п,	п,
п, sin i
откуда п., =	.
-	2 sin г
Из рис. 308 видно, что i = 90° — 20° = 70°; г = 90° — 46° = = 44°. Таким образом, абсолютный показатель преломления среды
Задача 4. Высота сваи строящегося железнодорожного моста 10 м. Глубина водоема 6 м. Какова длина тени сваи на дне водоема, если солнечные лучи света падают под углом а = 40° к горизонту?
Решение. Длина тени DB состоит из тени ОС надводной части сваи АС и тени BN' подводной части сваи CD (рис. 309). Из прямоугольного Л АСО следует, что ОС = AC tg Z О АС = = AC tg г; I — угол падения, равный 90° — а.
Длину подводной части тени BN' находим из Л ON'В: BN' = = OATtg A BON' = CDtg г, где г — угол преломления солнечного луча на поверхности воды в точке О.
Таким образом, длина тени сваи:
BD = ЛС tgr-F- CD tg г.
253
• ,	„	,	si П i	 st n I
Угол г найдем из формулы n =	, откуда sinr = — =
= 0,576, a r= 35° 10'. Тогда
BD = 4tg50° + 6tg35°10'«9 m.
Задача 5. В дно озера и на берегу вбиты вертикальные сваи одинаковой высоты h = 1,5 м. Определить: а) зависимость между длинами их теней, образуемых солнечными лучами; б) длину тени на дне озера. Лучи Солнца падают под углом а = 30° к горизонтальной поверхности. Глубина озера больше, чем длина сваи.
Реше н ие.а) Длину тени находим из прямоугольного дАСВ: (рис. 310): ВС = /ICtg р — /ICtg i. Обозначим ВС — lj, АС = /г^ тогда = hjg i.
Рис. 310
Рис. 309
Тень В'С от сваи на дне озера образует солнечный луч ОА'В', который преломился под углом г, а упал на поверхность воды под углом I. Длину этой тени определим из А А'С'В’: В’С = A'C'tgy = = A'C'tg г. Обозначим В'С = 12; А’С = 1г2, тогда 1.2 = /i2tg г.
Отношение длин теней от сваи на берегу и на дне озера
Д = /1[ *8 ‘
12 A tg г '
По условию задачи,	с учетом этого получим соотношение	. Следовательно, отношение длин теней па берегу
озера и на дне от сваи одинаковой высоты равно отношению тангенса угла падения к тангенсу угла преломления.
б) Длину тени на дне озера определяем по формуле l2 — htgr. Для нахождения численного значения угла г воспользуемся фор-u	sin i . sin i
мулои для показателя преломления: п =	, sin г =	=
=	—— = 0,6511; по таблицам г=40°37'. Подставив значе-
ния h и г в выражение для /а, получим
/2= 1,5 tg 40°37' = 1,3 м.
254
Задача 6. Доказать, что если угол между отраженным и преломленным лучами составляет 90°, то показатель преломления сре-
ды равен тангенсу угла падения.
Решение. Из рис. 311 видно, что угол преломления г = — 180° — 90° — i' = 90° — Г = 90° — i. Показатель преломления fin i sin i sin i .
среды n = -— = .	—- =-----. = tg i.
r	sinr sin (90—i) cos i s
Задача 7. При определении на глаз в вертикальном направле
нии кажется, что пузырек воздуха находится в стеклянном изделии на глубине h' = 2 см. Определить действительную глубину нахож-
дения воздушного пузырька, если показатель преломления стекла п = 1,8.
Решен не. Наблюдателю кажется, что изображение точки 5 (рис. 312) находится па пересечении продолжения входящих в глаз лучей, т. е. в точке S', на глубине h'. Из рисунка видно, что /i = 00j/tgi; h' = OOj/tg г, откуда Il _ h' tg i ’
Рис. 311	Рис. 312
Вследствие того что углы i и г малы, их тангенсы можно за-,	....	~	h sin г
менить синусами: tgr=sinr, tg i — sin t.	1огда & = =— = n.
Отсюда h — nh' = 3,6 см. Следовательно, воздушный пузырек находится на глубине 3,6 см.
Задача 8. Найти предельный угол падения при переходе луча света из стекла в воду, если абсолютные показатели преломления стекла и воды соответственно п1= 1,52 и п2 = 1,33.
Решение. Чтобы определить предельный угол в формуле
= ^212 = нужно принять г = 90°, или sin г - = 1 и i = Znp; 21 Sin Г flL ' J	г
тогда sin inp = п- , где п2 и —абсолютные показатели преломлено
ния воды и стекла соответственно. Подставив значения п2 и пь найдем sin г|1р = 0,875. По значению sin inp находим /пр = 61°.
Задача 9. При переходе света из воздуха в некоторое вещество предельный угол полного отражения света оказался равным 24°37'. Определить, какое это вещество.
255
Решение. Формула для определения показателя преломления sin I 1	„
имеет вид	. 1 юскольку для предельного угла полного
отражения угол преломления г = 90°, то предыдущее равенство принимает вид sin гпр = , откуда н = l/sininp = 2,4. Такой показатель преломления имеет алмаз.
Задача 10. Луч света (рис. 313) падает на стеклянную плоскопараллельную пластинку под углом 60° к нормали. Определить, под каким углом он выйдет из пластинки и на сколько сместится выходящий луч, если толщина пластинки АС = 10 см.
Решение. Запишем формулу закона преломления света соответственно для точек А и В:
sin i   п2 ' sin 1'   Щ sin rt	n2 ’ sin r2 n2 ‘
Перемножим эти два уравнения и, приняв во внимание, что — i', получим sin i — sin г2, а так как i и r2 — острые углы, то i = r2. Таким образом, из этого следует, что лучи 5Л и BS' параллельны. Однако луч BS' смещен по отношению к падающему на пластинку лучу 34 на BD. Определим это смещение из AADB : BD — = А В sin (i — rt). Определим значение угла t\ по формуле показателя преломления:
•	Sint
sin г, =---= 0,552,
1 п
откуда rt=31°20'. Отрезок АВ находим из прямоугольного треугольника АСВ: АВ = ЛС/cos гР Искомое смещение луча
BD= sin (i — г Л « 3,7 см.
COS Г] v 17
Задача 11. Луч света падает под углом 45° на плоскопараллельную стеклянную пластинку. Начертить ход лучей: отраженных, преломленных и выходящих из пластинки. Найти направление выходящих лучей и смещение лучей внутри пластинки. Толщина пластинки 10 см.
256
Решение. Отраженный луч OS' (рис. 314) составляет с перпендикуляром N\Nri угол tj = ilt где = 45° — угол падения света.
Преломленный луч ОА идет под углом к перпендикуляру Определим этот угол гг по формуле у- = п:
sin Г! =	= 0,4504; гг = 26°46'.
Луч выходит из пластинки под углом г2 к перпендикуляру М2М2. Он параллелен падающему лучу SO и смещен внутри пластинки на расстояние DA. Следовательно, r2= iv
Смещение луча
DA — —— sin (й — г А = 3,5 см. cos rx 1	17
Отраженный луч АВ пойдет внутри пластинки под углом i’.,= = i2 = /-j к перпендикуляру N2N'2-
Выходящий луч BS" будет параллельным отраженному лучу OS'. Действительно, поскольку i3 = rlt то из формул показателя преломления света при переходе света из стекла в воздух и из воздуха в стекло следует:
si nr,	1	sin t,
-—4 = — ;	—1 = ti.
sin t3	n	sin r,
Перемножив два последних равенства, получим:	= 1, отку-
да sin й = sin i3. Поскольку углы й и i3— острые, то из последнего равенства следует, что i1=i3. Поскольку й = й> то и ia = = й-
Задача 12. Преломляющий угол призмы ф = 45°. Монохроматический луч света падает на боковую грань призмы под углом i = 30°. Найти угол отклонения луча, если показатель преломления материала призмы п =1,6.
Решение. Пусть на левую грань призмы падает луч <3/1 (рис. 315). Здесь он преломляется и идет в направлении АВ. В точке В луч второй раз преломляется и идет в направлении BS’. Продолжим лучи S4 и BS' до их пересечения в точке С. Угол ср, образованный пересечением продолжения луча, падающего на призму, с продолжением луча, вышедшего из призмы (угол отклонения призмы), нужно определить.
Из А АВС следует, что гр = Z.CAB A- ZCBA, но из рис. 315 видно, что ,/САВ — i — г и /_СВА — гх — ilt поэтому ср = i— — r -I- rt — й = i + П — (П + г). Из AABD следует, что й -J-- гх= = /_Bl)N', a ZBDN' = ф. Поэтому й + Н — Ф- С учетом сказанного выражение для угла отклонения луча будет иметь вид ср = = i + — ф.
Чтобы определить гъ применим последовательно закон преломления света к первой и второй граням: sin г =	= 0,3125 и
9 7-253
257
соответственно r=18°13';	откуда sin rt = п sin it.
Из ДЛВ£) видно, что х\ = -ф — г. Следовательно, sinr1 = nsinx X (ф — г) = 0,720; г1 = 46°3'.
Подставляя найденные значения в формулу для угла отклонения луча, получим: <р = 30° + 46°3' — 45° = ЗГ°3'.
Задача /3. Луч света SD падает на правильную трехгранную призму перпендикулярно к грани АС (рис. 316). Определить дальнейший ход луча в призме и за гранью ВС. Показатель преломления стекла призмы п = 1,8.
Решение. Луч SD падает перпендикулярно на грань АС, поэтому он проходит эту грань, не преломляясь. Из прямоуголь
ного AADE: /_AED =90° — z_DAE =30°. Угол падения i = = Z_N'ED = /_N'EA - Z_A£ZJ = 60°.
Таким образом, на грань АВ луч DE падает под углом i = 60°. Поскольку луч DE падает на границу раздела среды, оптически более плотной (стекло), и среды, оптически менее плотной (воздух), то здесь могут быть три случая: 1) угол падения света i меньше, чем предельный угол полного отражения света /пр. При этом в точке Е луч разделится на два луча — отраженный и преломленный; 2) i = = 1пр; тогда в точке Е луч DE разделится на два луча — отраженный и преломленный, который пойдет в направлении ЕВ (угол преломления равен 90°); 3) i > inp; на грани АВ будет полное отражение света.
Чтобы определить дальнейший ход луча от точки Е, надо определить угол inp для стекла, из которого изготовлена призма:
sin 1Пр = — = 0,5556; inp = 33°45'.
Поскольку i > inp, то в точке Е произойдет полное отражение света. Угол отражения i' = i — 60°.
Из АВРЕ видно, что ZBFE=\8O°—/.РВЕ—/ВЕР = = 120°—^BEF(^.FBE— преломляющий угол равноугольной призмы). /BEF = 180° —(/AED + /DEF)= 180° — (^AED + + 2i) = 30°. С учетом вышеприведенных вычислений /В££ = 90°.
558
Следовательно, луч EF падает перпендикулярно на грань ВС, поэтому не будет преломляться и пойдет в направлении FS'.
Задача 14. В сосуде с водой находится полая равноугольная трехгранная призма, склеенная из тонких стеклянных пластинок. Внутри призмы — воздух (рис. 317). Определить дальнейший ход луча SO, параллельного основанию призмы АС, и найти угол отклонения луча. Толщиной стеклянных пластинок, из которых изготовлена призма, пренебречь.
Р е ш е н и е. На границе раздела вода — воздух (грань призмы АВ) луч света SO будет преломляться. Поскольку вторая среда оптически менее плотная, чем первая, то преломленный луч OD будет отклоняться от перпендикуляра AW). Угол преломления гх определим по формуле показателя преломления света: sin = и sin ix. Из построения видно, что угол г\ = 30°; показатель преломления воды п = 1,33. Тогда sin rr = = 0,665; а = 41°4Г.
Луч OD на границе раздела воздух — вода преломится, но поскольку свет переходит из оптически менее плотной среды в оптически более плотную, преломленный луч DS' приблизится к перпенди-
куляру А\У'. Угол преломления г2 определим по формуле no-sin i2 казателя преломления света: sin г2 =	. В данном выражении
неизвестен угол i2 —угол падения на грань ВС. Из рис. 317 видно, что i2 = 90° — Z0DB. Из Д OBD: Z0DB = 180° — (ZBOD + + ZOBD); zBOD = 9Q° — г, = 48°19'; ДОВО = 60°. С учетом этого z_0DB = 7Г41', а искомый угол г2 — 18°19'.
Подставив значения i2 и п в выражение для sinr2, получим: sin г2 = 0,2361; г2 = 13°40'. Угол отклонения луча <р находим по формуле <р =	+ г2 — ф =—12°20'. Знак минус указывает на
то, что выходящий из призмы луч DS' отклоняется не к основанию призмы, а к вершине В.
ЗАДАЧИ ДЛЯ САМОСТОЯТЕЛЬНОГО РЕШЕНИЯ
925.	На поверхность раствора сахара с показателем преломления п = 1,38 падает луч света. Чему равен угол падения света, если угол между отраженным и преломленным лучами составляет 90°?
926.	Определить, на какой угол отклоняется луч света от первоначального направления при переходе из стекла (п — 1,6) в воздух и из воды в воздух. Угол падения света 30°.
9*
259
927.	На прозрачную среду из воздуха падает луч света под углом i = 40° к поверхности. При переходе в среду луч отклоняется от первоначального направления на а = 24°49'. Определить показатель преломления среды.
928.	Луч света переходит из алмаза в воздух и образует угол преломления 85°. Определить угол падения света.
929.	Определить скорость распространения света в среде, ёсли при угле падения света из воздуха в среду, равном i = 45°, угол преломления оказался г = 23°8'.
930.	Луч света переходит из скипидара в воздух. Предельный угол полного отражения 42°23'. Определить скорость распростране
ния света в скипидаре.
931. Луч света переходит из воздуха в стекло, показатель пре-
ломления которого п =1,6. При каком угле падения угол преломления будет в 2 раза меньше угла падения? Выполнимо ли такое условие при переходе света из воды в стекло?
932.	На дне кубического сосуда с непрозрачными стенками лежит монета на расстоянии d = 5 см от одной из стенок (рис. 318). Наблюдатель разместил глаз так, что в него могут попадать лучи, отраженные от стенки сосуда CD. Какой объем воды необходимо налить в сосуд, чтобы наблюдатель мог увидеть монету? Ребро сосуда а = 30 см.
933.	Находясь под водой, водолаз видит Солнце на высоте 45° над горизонтом. Определить действительную высоту Солнца над горизонтом.
934.	Наблюдатель смотрит на предмет, лежащий на дне водоема. Ему кажется, что предмет находится на глубине h = 1 м на расстоянии I = 5 м от глаза по прямой линии. Глаза наблюдателя находятся на одной и той же высоте И = 1,5 м над поверхностью воды. На какой действительной глубине h0 лежит предмет?
935.	Водолаз рассматривает из воды светящийся предмет, находящийся над его головой на расстоянии ft = 3 м над поверхностью воды. Определить видимое расстояние предмета над поверхностью воды.
936.	В воде распространяются два параллельных луча света и S2B (рис. 319). Луч выходит в воздух непосредственно, а" луч S2B проходит сквозь горизонтальную плоскопараллельную пластинку, а) Будут ли лучи б'хЛ и S2B параллельны по выходе
260
В воздух? б) Выйдет ли в воздух луч S2B, если луч падает под углом 40° к поверхности воды?
937.	Луч света падает на плоскопараллельную стеклянную пластинку толщиной h = 5 см под углом t = 35°. Определить смещение луча внутри пластинки.
938.	Луч света, падающий под углом i = 70° на лист бумаги, образует на ней светлое пятно. На сколько сместится это пятно, если на бумагу положить стеклянную пластинку толщиной h — = 6 см?
939.	На плоскопараллельную пластинку толщиной h = 10 см падает луч света под углом i = 40°. Проходя через пластинку, он смещается на I = 3 см. Найти показатель преломления вещества пластинки.
940.	Параллельный пучок света падает из воздуха на стеклянную плоскопараллельную пластинку под углом i = 50 °. Ширина пучка в воздухе d. = 5 см. Определить ширину пучка в стеклянной пластинке.
941.	В сосуд со стеклянным дном налиты вода и масло. На поверхность жидкости под углом i = 60° падает свет. Под каким углом будет выходить свет через дно сосуда?
942.	Сечение стеклянной призмы плоскостью, перпендикулярной к основанию, имеет форму равнобедренного треугольника. Одна из равных граней призмы посеребрена. На другую (непосеребрен-ную) грань падает перпендикулярно луч света, и после двух отражений выходит через основание призмы перпендикулярно к нему. Определить угол между равными гранями призмы.
943.	У призмы с преломляющим углом ф = 30° одна боковая грань посеребрена. Луч света, падающий па другую грань под углом а = 45°, после преломления и отражения от посеребренной грани вернулся назад в прежнем направлении. Найти показатель преломления материала, из которого изготовлена призма.
944.	У прямоугольной равнобедренной стеклянной призмы посеребрены обе грани-катеты. Доказать, что лучи, падающие под любым углом на грань-гипотенузу, будут выходить из призмы параллельно первоначальному направлению.
945.	Для получения с помощью проекционного аппарата проекций опытов в прямом, а не перевернутом виде употребляют «оборотную призму», преломляющий угол которой ф — 90°. Начертить ход в призме и за ней двух лучей, падающих на призму параллельно ее основанию. Изменится ли расстояние между лучами? Одинаковы ли пути первого и второго лучей внутри призмы?
946.	Монохроматический луч света падает перпендикулярно на боковую грань призмы с преломляющим углом ф = 30° и выходит из призмы под углом а = 64°10'. Определить показатель преломления материала, из которого изготовлена призма.
947.	В спектрографе луч света падает под углом i — 45° на кварцевую призму, преломляющий угол которой ф = 60°. Показатель преломления кварца п = 1,55. Под каким углом выходит луч из призмы?
261
948.	В воде находится полая равноугольная призма, изготовленная из тонких стеклянных пластинок. Определить угол наибольшего отклонения.
949.	Каков преломляющий угол у стеклянной призмы, если угол наименьшего отклонения равен преломляющему углу призмы?
950.	Призма с преломляющим углом ср = 50° дает угол наименьшего отклонения а == 35°. Какой будет угол наименьшего отклонения, если эту призму погрузить в воду?
§ 22. Линзы
Задача 1. Построить изображение светящейся точки Sb собирающей (выпуклой) линзе.
Решение. Для построения изображения точки S (рис. 320)
выбираем лучи построения: а) луч S/1, параллельный главной
оптической оси линзы 00 преломленный луч пойдет в направле-
нии AF2 (через фокус); б) луч SO, проходящий через оптический центр линзы О (в направлении побочной оптической оси), или в) луч SFlt проходящий через фокус линзы, после преломления в линзе он пойдет параллельно главной оптической оси. Точка пересечения S' этих преломленных лучей является изображением светящейся точки 3.
Задача 2. Построить изображение светящейся точки S, которая лежит на главной оптической оси линзы.
Решен и е. Проводим фокальную плоскость MN (рис. 321). Из точки
S проводим какой-либо луч к линзе в точку А. Параллельно лучу проводим побочную оптическую ось ВО, которая пересечет фокальную плоскость М N в точке С. Луч SA после преломления в линзе также должен пройти через точку С. В качестве второго луча построения возьмем луч, направленный вдоль главной оптической оси. Точка пересечения S' лучей AS' и SS' является изобра-
жением точки S.
Задача 3. Построить изображение предмета, получаемое с помощью выпуклой линзы, если предмет находится от линзы на расстоянии: а) а > 2/; б) / < а < 2f; в) а < f.
262
Решение, а) Из точки А (рис. 322, а) проводим два луча построения: АС, параллельный главной оптической оси 00 г линзы, и АО— через оптический центр О линзы. Точка пересечения А' лучей CF и СМ является изображением точки А. Аналогично находим изображение В’ точки В. Все остальные изображения точек предметам.8 находятся между А' и В'. Изображение А'В' является действительным, перевернутым, уменьшенным и находится от линзы
Рис. 322
на расстоянии Ь, которое меньше двойного фокусного расстояния линзы и больше фокусного расстояния, т. е. f < b < 2f.
б) Из рис. 322, б видно, что изображение А'В' находится за линзой на расстоянии b > 2/; оно действительное, перевернутое и увеличенное.
в) В качестве лучей построения выберем луч АС || 00х (рис. 322,а) и луч АО. Лучи CFN и АОМ не пересекаются. Для получения изображения предмета необходимо расходящиеся пучки света, выходящие из линзы, преобразовать в сходящиеся. Если расположить глаз, как показано на рис. 322, в, то расходящиеся лучи соберутся на сет-
263
чатке глаза и образуют изображение А” точки Л. Мнимое изображение точки (предмета) находится там, где пересекаются продолжения лучей, входящих в глаз, в данном случае — в точке А'. Аналогично можно построить мнимое изображение В‘ точки В. Соединив точки А' и В', получим мнимое изображение А'В' плоского предмета АВ. Оно лежит по ту же сторону от линзы, что и предмет, мнимое, увеличенное и прямое.
Задача 4. На рисунке 323, а изображена светящаяся точка S и ее изображение S', образуемое выпуклой линзой, главная оптическая ось которой ММ}. Найти положение линзы и ее фокусов.
Решение. Поскольку светящаяся точка S и ее изображение S' находятся по одну сторону от оптической оси линзы, то, очевидно, изображение S' является мнн-* мым. Точка S и ее изображение S'
должны находиться на луче, проходящем через оптический центр О линзы. Следовательно, для нахождения положения линзы достаточно провести прямую через точки S и S' (рис. 323, б). Точка пересечения О этой прямой с главной оптической осью ММ] является оптическим центром линзы. Положение фокусов линзы определим так. Из точки S проводим луч S.4 || /ИЛД. Преломленный линзой луч пройдет через фокус F2. Поскольку S' — мнимое изображение точки S, то оно должно находиться на продол
жении преломленного луча, проходящего через фокус F2. Следовательно, прямая, проходящая через точки S' и А, пересечет главную оптическую ось /WAfj в точке, которая является фокусом F2 линзы. Другой фокус F] линзы — это точка на главной оптической оси, симметричная F2 относительно
точки 0 — оптического центра линзы.
Задача 5. Задана главная оптическая ось ММ] линзы, светящаяся точка S и ее изображение S' (рис. 324, а). Найти построением оптический центр линзы и положение ее главных фокусов.
264
Решение. Точка S и ее изображение S' лежат по разные стороны от главной оптической оси Из этого можно сделать вывод, что точка S' является действительным изображением точки А. Светящаяся точка и ее изображение всегда лежат на прямой, проходящей через оптический центр линзы. Таким образом, точка пересечения О прямой, проходящей через точки S и S', с главной оптической осью ММг является оптическим центром линзы (рис. 324, б). Для нахождения главных фокусов линзы проводим через точку 0 плоскость, перпендикулярную к оптической оси ММг из точки 5 проводим луч SA || MMt Преломленный линзой луч AS' пройдет через главный фокус F}— точку пересечения луча AS' с главной оптической осью ММг. Вторым главным фокусом линзы
будет точка F2, лежащая на главной оптической оси и симметричная Fj относительно точки О.
Задача 6. Построить изображение предмета, которое можно получить с помощью двояковогнутой линзы, если предмет находится от линзы на расстоянии: а) а > 2/; б) /< а < 2f‘, в) предмет расположен между фокусом линзы и линзой.
Решение, а) Выберем два луча построения, например, луч АС, параллельный главной оптической оси A4A4j, и луч AD, продолжение которого проходит через мнимый фокус F2 (рис. 325, а). Луч АС после преломления пойдет так, что его продолжение будет проходить через мнимый фокус Flt а луч AD выйдет из линзы параллельно главной оптической оси. Преломленные лучи СЕ и DK не пересекаются. Если на пути лучей СЕ и DK расположить глаз, то наблюдатель увидит изображение точки А в точке А', где пересекаются продолжения лучей ЕС и KD. Аналогично строится изображение точки В. Таким образом, изображение предмета будет мнимым, уменьшенным, прямым и расположено между фокусом Е1; лежащим по ту же сторону от линзы, что и предмет, и линзой.
б) Для построения изображения точки А выберем два луча построения: луч АС || и луч, проходящий через оптический центр 0 линзы. Луч АС, преломившись в линзе, пойдет так, что его продолжение пройдет через фокус Fv Луч АО преломляться не будет (рис. 325, б). Точка пересечения А' продолжений выходящих из
265
линзы лучей СЕ и AOD является местом нахождения мнимого изображения точки А. Аналогично строим изображение точки В. Следовательно, и в этом случае изображение А'В' является мнимым, прямым, уменьшенным и находится между фокусом Fr и линзой.
в) Изображение А'В' предмета АВ строим аналогично случаю б). Изображение А'В' (рис. 325, а) является мнимым, прямым, уменьшенным и находится между предметом и линзой.
Задача 7. Оптическая сила собирающей линзы 8дптр. Предмет установлен на расстоянии а = 0,5 м от линзы. На каком расстоянии b от линзы будет находиться изображение и какое линейное увеличение дает линза?
1 1 1 п 1
Т- = ------. Поскольку т —
b f a	J f
= D — оптическая сила линзы,
Решение. Из формулы линзы
куда b = =---г =0,17 м. Ли-
J Da — 1 u	,6
неиное увеличение k = — = «0,34.
Задача 8. На каком расстоянии от предмета нужно поместить экран, чтобы двояковыпуклая линза с радиусами кривизны поверхностей
R =0,2 м и показателем преломления п = 1,6 давала изображение предмета, увеличенное в 2 раза?
Р е ш е н и е. Из условия задачи следует, что предмет расположен на расстоянии, большем, чем фокусное, и меньшем, чем двойное фокусное расстояние. Для построения изображения точки А (рис. 326) проводим два луча построения: луч АС, параллельный главной оптической оси OOL, и луч АО, проходящий через оптиче-
ский центр О линзы. Лучи, прошедшие через линзу, встретятся в точке А'. Все лучи, выходящие из точки А, пройдя сквозь линзу, соберутся в точке А'.
Найдем соотношение между расстояниями между линзой и предметом а и между линзой и изображением Ь. Из подобия ДА 05 и ДА'ОВ' следует: = 4^- = 7 . По условию задачи, 4 = 2, или \Jt\ /ixj Ct	Cl
. С Л	1	1	,	1	3 ,
b = 2а. Формула линзы 7 = = + - , откуда а = f.
Фокусное расстояние f линзы определим по формуле у=(п— 1)Х
X (к + щ У Учитывая, что R^ = R2, находим: 4 — (п —- 1) , от-^2/	I
п
куда / = п~.--. Подставив это значение f в формулу для а, получим:
2 (П
3 г 3R л пе а~ 2 f~ 4(n —1) “ 0,25 М‘
266
Следовательно, расстояние между предметом и экраном будет а + Ь= а + ‘2а=За= 0,75 м.
Задача 9. На каком расстоянии от экрана нужно установить .проекционный аппарат, чтобы изображение диапозитива точно уложилось на экране размером 1,7 X 1,7 м? Размеры диапозитива 8,5 X 8,5 см. Фокусное расстояние объектива 30 см.
Решение. Расстояние от проекционного аппарата до экрана определяем по формуле линзы j = — +	. Определим а из фор-
..	,	h' b	bh	г,
мулы линейного	увеличения k =	j	= —	, откуда а = -f--, .	Подстав-
ляя значение а в формулу линзы, получим
1	Л*. 1	.	/ (h 4- fl') r п
т = bh + т  0ТКУда b = Т = 6-3 м-
Задача 10. При фотографировании с расстояния 100 м высота дерева на негативе равна 12 мм. Определить действительную высоту дерева, если фокусное расстояние объектива фотоаппарата 50 мм.
Решение. Действительную высоту дерева h = АВ (рис. 327) можно определить из подобия AABF и ADCF. Замечая, что CD = = h', можно записать: & = ^- = — , откуда h =	. Поскольку
s0 = а — / — расстояние от дерева до фокуса объектива, то
Л =(а ~ h’- = 24 м.
Задача 11. Под каким горизонтальным углом зрения виден экран в обыкновенном, широкоэкранном и панорамном кинотеатрах, если ширина I экранов в них соответственно 6; 14 и 30 м? Считать, что зритель находится на расстоянии L = 10 м от середины экрана и одинаково удален от его краев.
267
Решение. Из рис. 328 видно, что tg . Следовательно,
a)	tg = 0,30. По таблице тангенсов находим = 16°42' и ах = = 33°24'.
б)	tg“2? = 0,70; % = 35°; а2 = 70°.
в)	tg|3 = 1,50; у3 = 56°24'; а3 = 112°48'.
ар а2 и а.—горизонтальные углы зрения, под которыми виден экран в обыкновенном, широкоэкранном и панорамном кино-
театрах.
Задача 12. Найти увеличение, которое можно получить с помощью лупы, еслифе окусное расстояние f = 2 см; а) для нормального глаза с расстоянием наилучшего видения L =
= 25 см; б) для близорукого глаза с расстоянием наилучшего видения L = 15 см.
Р е ш е н и е. Из рис. 329 видно, что если предмет АВ находится
от глаза на расстоянии ЛВ
а = —; если же этот
наилучшего видения L, то угол зрения предмет находится вблизи фокуса лупы
и мы рассматриваем его через лупу, то он будет виден под углом А'В'
зрения = —у.
По формуле линейного увеличения линзы k = -^g- = — . Под-ставляя в это соотношение вместо А'В' и АВ их значения, получим: k = arblaL; но поскольку мнимое изображение находится на расстоянии наилучшего видения, то b = L, следовательно, k« « cq/a « L/a. Учитывая, что предмет находится возле фокуса лупы, можно принять a^f. Таким образом, увеличение лупы при-268
ближенно определяется по формуле k = ~ . Поэтому увеличение для нормального глаза, которое можно получить с помощью лупы, kj= 12,5 раза; для близорукого глаза — k2 = 7,5 раза.
Задача 13. Чему должны быть равны радиусы кривизны поверхностей, ограничивающих лупу (А\= /?а = 7?), чтобы при ее помощи можно было получить увеличение/г = 10? Показатель преломления стекла, из которого изготовлена лупа, п =1,6.
Решение. Из формулы линзы -j- = (п—1)	= (п—
— 1) определяем	—1). Значение f определяем по фор-
муле увеличения лупы k — LIf, откуда / = L/k. Подставляя значение f в выражение для R, получим
91
R = j (п— Г) = 3 см.
Задача 14. Зрительная труба с фокусным расстоянием объектива /об = 50 см установлена на бесконечность. После того как окуляр передвинули на некоторое расстояние, стали ясно видны предметы, удаленные от объектива на b = 50 м. На какое расстояние переместили окуляр при наводке?
Решение. В первом случае фокусы объектива и окуляра совпадали (рис. 330), а изображения предметов, находящихся на расстоянии 50 м, находились за фокусом объектива на расстоянии Ай = Ь — / об. Для того чтобы изображения были ясно видны, глаз должен быть аккомодирован на бесконечность. Отсюда следует, что изображение должно находиться в фокальной плоскости окуляра. Поэтому окуляр надо переместить на расстояние \b = b — — /об. Значение b определим из формулы линзы: b ==	=
= 0,505 м; А6 = 0,005 м = 5 мм. Следовательно, при наводке окуляр переместили на 5 мм.
Задача 15. Микроскоп состоит из объектива с фокусным расстоянием /об = 2 мм и окуляра с фокусным расстоянием /ок = ₽ 40 мм. Расстояние между фокусами объектива и окуляра 18 см. Определить увеличение, даваемое микроскопом.
269
Решение. Объектив микроскопа устанавливается так, чтобы рассматриваемый объект располагался на расстоянии, немного большем фокусного расстояния объектива (рис. 331). При этом объектив дает действительное, обратное и увеличенное изображение А'В' объекта АВ, которое находится по другую сторону объектива на расстоянии b от него, между оптическим центром и фокусом окуляра. Это действительное изображение рассматривают через окуляр, как через лупу. Линейное увеличение объектива йоб — — » Д , а /об так как а м fo6 — переднему фокусному расстоянию объектива. Следовательно, линейные размеры изображения, образуемого объективом, А'В' = АВ • Ь- .
/об
Рис. 331
Видимая картина изображения А'В', рассматриваемого через окуляр, увеличится в /гок = А//Ок раз. Общее увеличение, которое можно получить с помощью микроскопа, будет k = k06 • k0K = =	. Расстояние b можно приближенно считать равным длине
/об/ок
d тубуса микроскопа (так как /об < d и /ок d); тогда формула для определения увеличения микроскопа примет вид
k =	« 562,5 раза.
/об/ ок
ЗАДАЧИ ДЛЯ САМОСТОЯТЕЛЬНОГО РЕШЕНИЯ
951.	Построить ход луча за линзой (рис. 332). Показатель преломления среды по обе стороны линзы одинаков.
952.	В фокальной плоскости собирающей линзы расположено плоское зеркало (рис. 333). Построить ход луча в пространстве между линзой и зеркалом, а затем по выходе из линзы влево.
953.	Определить показатель преломления материала линзы, если радиусы кривизны ее поверхностей /?1 — 40 см и R2 — 50 см, а оптическая сила D = 3 дптр.
954.	Определить фокусное расстояние плосковыпуклой линзы, изготовленной из стекла с показателем преломления п = 1,6, если радиус кривизны ее поверхности R — 20 см.
270
955.	Двояковыпуклая линза из стекла («!= 1,57) имеет в воздухе оптическую силу £\=5дптр. Определить оптическую силу и фокусное расстояние линзы после погружения в жидкость с показателем преломления п2 = 1,67.
956.	Фокусное расстояние собирающей линзы в воздухе f = = 0,1 м. Чему равно оно в воде?
957.	Фокусное расстояни*’ линзы, находящейся в воздухе, Л=5 см. Фокусное расстояние той же линзы, погруженной в некоторую жидкость, /2 = 35 см. Определить показатель преломления жидкости, если показатель преломления стекла нс = 1,8.
958.	Предмет и его изображение находятся от собирающей линзы соответственно на расстояниях а = 0,4 м и Ь = 1,2 м. Опре-
делить оптическую силу линзы.
Рис. 332
Рис. 333
959.	Между предметом и экраном установлена двояковыпуклая линза с радиусами кривизны сферических поверхностей А\== 30 см и /?2= 20 см. С помощью данной линзы получено изображение с линейным увеличением k ~ 2. Определить расстояние от предмета до экрана. Показатель преломления материала линзы п = 1,6.
960.	Расстояние от лампочки до экрана L = 1 м. Линза, помещенная между ними, образует резкое изображение лампочки при двух положениях, расстояние между которыми I = 0,6 м. Определить фокусное расстояние линзы.
961.	Расстояние от предмета до экрана L = 1 м. На каком расстоянии от предмета следует поместить собирающую линзу с фокусным расстоянием / = 12,5 см, чтобы получить на экране резкое изображение предмета?
962.	Предмет находится на расстоянии а =0,1 м от двояковыпуклой линзы, оптическая сила которой D = 20 дптр. Определить, на каком расстоянии от линзы будет изображение предмета и чему равно линейное увеличение изображения.
963.	Линза, расположенная между лампочкой и экраном, дает на экране резко? увеличенное изображение лампочки. Когда линзу придвинули к экрану на Да = 30 см, образовалось резкое уменьшенное изображение. Расстояние от лампочки до экрана d = = 60 см. Определить фокусное расстояние линзы.
964.	Предмет находится на расстоянии L = 1 м от экрана. Собирающая линза, помещенная между предметом и экраном, дает четкое изображение. Не меняя расстояния между предметом и экраном, перемещают линзу и снова находят четкое изображение пред
271
мета. Какова оптическая сила линзы, если расстояние между обоими положениями линзы I — 20 см?
965.	В экране сделано круглое отверстие, через которое проходит сходящийся пучок. Вершина пучка 5 лежит на расстоянии I = 15 см от экрана. Как изменится расстояние точки встречи лучей до экрана, если в отверстие вставить собирающую линзу с фокусным расстоянием / = 30 см? Построить ход лучей после установки линзы.
966.	Двояковыпуклая линза из стекла (гц = 1,57) имеет в воздухе фокусное расстояние / = 30 см. Определить расстояние от линзы до изображения, если расстояние от линзы до предмета а — 15 см. Где будет находиться изображение, если линзу и предмет опустить в воду (п2 = 1,33)?
967.	Фокусное расстояние рассеивающей линзы / =0,15 м. Расстояние между предметом и линзой а = 0,10 м. Определить расстояние между предметом и изображением.
968.	Расстояние от предмета до линзы а =0,1 м. Найти расстояние от изображения предмета до линзы, если линза: а) собирающая; б) рассеивающая. Фокусное расстояние линзы f = 0,2 м.
969.	Плосковыпуклая линза с фокусным расстоянием = 30 см и плосковогнутая с фокусным расстоянием /2 = 10 см сложены вплотную. На расстоянии а = 15 см от этой системы линз поместили предмет. На каком расстоянии от системы линз получится изображение предмета?
970.	Плосковыпуклая стеклянная (п = 1,56) линза, посеребренная с выпуклой стороны, образует вместе с вогнутым зеркалом и плосковыпуклой линзой оптическую систему. Определить фокусное расстояние такой оптической системы, если радиус кривизны 7? = 50 см.
971.	Определить положение изображения, образуемого оптической системой, состоящей из вогнутого зеркала с фокусным расстоянием = 10 см и собирающей линзы с фокусным расстоянием /2 = 20 см. Расстояние от зеркала до линзы d — 30 см, а от линзы до предмета аг = 40 см.
972.	Фокусное расстояние объектива проекционного аппарата f =0,15 м. Объектив находится на расстоянии Ь = 6 м от экрана. Определить линейное увеличение изображения на экране.
973.	При раскрое листовой стали используется проекционный аппарат. В качестве диапозитива берут фотоснимок чертежа детали, который проектируют на лист стали. На каком расстоянии от экрана находится объектив проекционного аппарата, если снимок чертежа изготовлен в масштабе 1 : 40, а диапозитив находится от объектива на расстоянии а = 10 см?
974.	Фотоаппаратом, фокусное расстояние объектива которого f = 12 см, фотографируют предмет. Найти расстояние от предмета до объектива, если размер изображения на пленке /ц = 10 мм. Высота предмета 1г = 2 м.
975.	Требуется сфотографировать чертеж размером 500 X X 750 мм. Фокусное расстояние объектива / =0,12 м. На каком рас
272
стоянии от объектива нужно поместить чертеж, чтобы его изображение на фотопластинке имело размеры 100 X 150 мм?
976.	Фотоаппаратом с фокусным расстоянием f = 12,5 см фотографируют рисунок размером 40 X 40 см таким образом, что изображение на пленке имеет размеры 8x8 см. Какое должно быть при этом расстояние между: а) рисунком и объективом фотоаппарата; б) между фотопленкой и объективом?
977.	Фотоаппаратом с фокусным расстоянием объектива / = — 22 см необходимо сфотографировать дерево высотой Н = 12 м так, чтобы высота его на пленке была h — 12 см. Каково расстояние между: а) фотоаппаратом и деревом; б) между фотопленкой и объективом фотоаппарата?
978.	При наибольшем удалении объектива от пленки фотоаппарат дает резкие снимки предметов, расположенных на а — 1,5 м от объектива. С какого наименьшего расстояния можно будет получать резкие снимки, если на объектив насадить линзу с оптической силой D = +3 дптр?
979.	Фотокамерой «Зоркий» с размером кадра 24 X 36 мм и фокусным расстоянием / = 50 мм переснимают текст книги с размером страницы 12 X 18 см. Определить: а) линейное уменьшение переснимаемого объекта; б) на каком расстоянии от поверхности пленки следует расположить переснимаемую страницу; в) какой оптической силы следует использовать насадочную линзу. Площадь кадра используется максимально, и объектив фотоаппарата сфокусирован на бесконечно удаленный предмет.
980.	Двояковыпуклая линза изготовлена из стекла с показателем- преломления п = 1,8. Радиусы кривизны поверхностей линзы равны R = 10 см. Какое линейное увеличение дает такая линза, если ею пользоваться как лупой?
981.	Расстояние между фокусами объектива и окуляра внутри тубуса микроскопа Z = 16 см. Фокусное расстояние объектива /об = 4 мм. С каким фокусным расстоянием следует взять окуляр, чтобы получить увеличение в k = 500 раз?
982.	Фокусное расстояние объектива микроскопа /Об = 5 мм, окуляра /ок = 2 см. Расстояние от объектива до окуляра I = 16 см. Какое линейное увеличение дает микроскоп для нормального глаза?
983.	При рассмотрении небесных светил телескоп дает увеличение в k = 150 раз. Фокусное расстояние окуляра телескопа /ок = = 20 мм. Какое угловое увеличение дает один объектив телескопа, если убрать окуляр?
984.	В телескопе, имеющем объектив с фокусным расстоянием /Об1 = 2 м, некоторый окуляр дает увеличение в 50 раз. Какое увеличение даст этот окуляр при объективе с фокусным расстоянием /Об2 = 5 м?
985.	В телескоп рассматривают удаленные предметы на линии горизонта. В каком направлении и на сколько нужно переместить окуляр, чтобы отчетливо видеть предметы, находящиеся на расстоянии L =100 м, если фокусное расстояние объектива /Об = 1 м?
273
986.	Объектив телескопа состоит из двух сложенных вплотную линз: собирающей (fx = 1 м) и рассеивающей (/'2— 3 м). Фокусное расстояние окуляра f0K = 25 мм. Какое линейное увеличение дает телескоп?
§ 23. Световые волны
Задача 1. Разность хода интерферирующих волн от двух когерентных источников света равна 0,4 к. Определить разность фаз этих волн.
Р е ш е н и е. Расстояние от когерентных источников света и З3 до точки М, где встречаются волны, излучаемые этими источниками, соответственно равны и г2(рис. 334). Тогда уравнения волн (рассматриваем только электрическую составляющую электро-1агнитного поля), приходящих в точку М ОТ ИСТОЧНИКОВ Sj и S2, имеют вид
Ej = E’otsin (со/ — kr-^, Е2 — = E02sin(o»Z—kr2), , 2л где « = у — волновое число.
Разность фаз между этими волнами в точке М
Д<р12 = at — kr2 — (со/ — kr2) = k(r2 — гг) = — (г2 — rj =
2л .
= гДг.
Подставив в последнее равенство значение Дг = 0,4%, получим дф1а =	. 0,4% = о,8л рад.
Задача 2. Разность хода интерферирующих лучей белого света (0,76 мкм > % ^ 0,40 мкм) Д — 2 мкм. Найти все длины волн видимого излучения, которые будут: а) максимально усилены; б) максимально ослаблены.
Р е ш е н и е. а) Максимально будут усилены те волны, для которых в разности хода интерферирующих лучей укладывается целое число длин волн, т. е. условие максимума при интерференции имеет вид Д = &% (где k = 0; 1; 2; ...), откуда % = у-.
Поскольку в условии задачи указано, что имеет место разность хода лучей, то k Ф 0. Подставляя в последнее равенство значения k = 1; 2; 3; ..., определим длины волн %, которые попадут в интервал 0,76 МКМ > % > 0,40 мкм. Найденные длины волн и будут максимально усилены:
2 %а = — мкм «0,67 мкм;
%4 = 0,5 мкм; %л = 0,4 мкм.
274
б) Максимально ослаблены будут те волны, для которых в разности хода интерферирующих лучей укладывается нечетное число полуволн, т. е. условием минимума интерференции является соотношение
Л = (2k + 1) (где k = 0; 1; 2; ...), откуда
i 2Д
Л - 2k + 1 '
Подставляя значения k в выражение для длины волны, находим, что в интервал длин волн видимого излучения попадает волна % — 0,555 мкм (/г = 3). Следовательно, эта волна будет максимально ослаблена.
Задача 3. Два параллельных луча падают перпендикулярно на грань стеклянной призмы с преломляющим углом ср = 30° и после преломления выходят из нее (рис. 335). Определить оптическую разность хода лучей, вышедших из призмы.
Решение. Произведение геометрической длины пути света на показатель преломления среды называют оптическим ходом луча. В рассматриваемом случае лучи 1 и 2 входят в первую грань в одной фазе. Поэтому оптическая разность хода лучей возникает вследствие разной геометрической длины пути в призме и оптической разности хода лучей после преломления в призме, т. е. Л = (12 — l^n — /3, где (/2 — /])и и 13 — оптическая разность хода лучей в призме и после преломления в призме соответственно.
Из рис, 335 видно, что 1.2 — 1г = dtg ср и /3 = c/ntg ср. С учетом этих замечаний Л = dn (tg ср — tg ср) —. 0.
Таким образом, оптическая разность хода лучей, вышедших из призмы, равна нулю.
Задача 4. Вода освещена зеленым светом, для которого длина волны в воздухе = 0,555 мкм. Какой будет длина волны в воде? Какой цвет увидит человек, открывший глаза под водой?
Реше и и е. Отношение скорости распространения света в одной среде к скорости распространения света в другой среде равно показателю преломления света второй среды относительно первой, т. е. п21 = —. Поскольку свет переходит из воздуха в воду, то
275
п = c/v, где п — абсолютный показатель преломления воды, поскольку он мало отличается от показателя преломления относительно воздуха. Скорость распространения света в вакууме с — Xov, скорость распространения света в воде (среде) v = Xv. Подставляя в формулу показателя преломления выражения для с и v, получим
п =	, откуда X = —°« 0,417 мкм.
Zv X * J	п
Таким образом, длина волны света в воде будет в п раз меньше, чем в воздухе. Однако человек, открывший глаза под водой, будет видеть зеленый свет, поскольку воспринимаемый глазом свет зависит не от длины волны, а от частоты.
Задача 5. Экран освещается двумя точечными когерентными источниками, колеблющимися в одной фазе и находящимися на расстоянии I = 0,5 мм друг от друга. Источники дают монохроматическое излучение с длиной волны X = 0,5 мкм. Расстояние от плоскости источников света до экрана L = 1,5 м (рис. 336). Определить расстояние первого и второго интерференционных максимумов от центрального максимума; расстояние между двумя соседними максимумами.
Решение. Поскольку источники света колеблются в одной фазе, то разность фаз между волнами, приходящими в любую точку экрана, определяется только разностью расстояний ту и г2, проходимых волнами от источников и S2 до рассматриваемой точки экрана Л-1:
л 2л ,	, 2л .
Аф =	(f2 — rj = Аг.
Определим разность хода Аг, с которой волны приходят в точку М экрана, лежащую на расстоянии х от его середины (Ц. Из подобия AOOjM и /\S2NS1 следует:	= л-п • Приняв во внима-
О 2 /V	Uj м
ние, что I и х намного меньше L, последнее уравнение можно переписать в виде
l L	А 1
ъ = 7 ’ °™уда Аг = ьх-
Зная выражение для разности хода волн, можно записать выражение для определения условия получения интерференционных максимумов:
Аф = v Аг — т г х~ ±2ял, Л	г*. L
откуда х = ±Л-у , где k = 0; 1; 2; ... .
Центральный интерференционный максимум будет при k = 0, т. е. он проходит через точку Ot.
276
Для определения расстояния первого и второго интерференционных максимумов от центрального в выражение для х необходимо подставить значения k, равные соответственно 1 и 2 , т. е.
~ ±	= ±1,5-. 10"3 ми х2 = ±	— ±3 • 10-3 м.
Расстояние между двумя соседними максимумами находим из условия Ьх = |х/г+1 — хи | = | ±(^ + 1)	| = Т =1,5х
X 10~3 м.
Таким образом, расстояние между двумя иЕгтерференционными максимумами не зависит от порядка максимума и интерференционная картина имеет вид светлых и темных полос равной ширины.
Задача 6. Два когерентных источника белого света и 32 освещают экран, плоскость которого параллельна отрезку (рис. 337). Какова ширина первого максимума, если расстояние между источниками света I = 3 мм, а их расстояние до экрана L = 5 м? Как расположены относительно центрального максимума максимумы красного (Xx= 0,76 мкм) и фиолетового (Х2 = 0,40 мкм) Света?
Решение. К точкам экрана Л4Х и Л12 от когерентных источников белого света приходят волны всех излучаемых длин. Однако интерференционный максимум будет наблюдаться только для тех длин волн, для которых разность фаз кратна четному числу л (или разность хода волн кратна X). Вследствие этого на экране образуются разноцветные интерференционные максимумы (для видимого излучения — от красного до фиолетового). Условие максимумов имеет вид х = + kKL/l-
Определим положение на экране первого максимума красного X. L
света:	±-у-= ± 1,3 • 10-3 м и фиолетового света: х2 =
= ±7^ = ± 0,7 • IO"3 м.
Поскольку длины волн видимого излучения?, находятся в интервале X > Х2, то все другие максимумы видимого излучения будут расположены между максимумами красного и филетового света. Поэтому ширина первого максимума (спектра первого порядка) равна расстоянию между первыми красным и фиолетовым максимумами, т. е.
Дх = |xj — х2| = 0,6  10’3 м.
277
Чем больше длина' волны X, тем дальше от центрального (нулевого) максимума будет находиться максимум света, соответствующий данной длине волны. Таким образом, максимум красного света расположен дальше, а максимум фиолетового света — ближе от центрального максимума.
Задача 7. Над плоским зеркалом Z (рис. 338) размещен монохроматический источник света S (X = 0,43 мкм). На экран АВ, перпендикулярный к плоскости зеркала, в точку /И падают два луча: непосредственно луч SM, параллельный плоскости зеркала, и луч S0M, отраженный от зеркала в точке О. Что будет наблюдаться в точке Л/ — усиление или ослабление света? Источник находится на расстоянии 1 мм от зеркала иЗмот экрана, а точка О — па расстоянии 1,5 м от экрана.
Решение. При определении оптической разности хода интерферирующих лучей необходимо учитывать, что при отражении света от оптически более плотной среды фаза колебаний изменяется на л. Разность хода Л равна геометрической разности хода лучей S0M и SM и дополнительной разности хода Х/2, обусловленной изменением фазы колебаний при отражении луча SO от поверхности зеркала в точке О, т. е.
Д = SO + ОМ — SM + .
Учитывая, что SO — ОМ (как гипотенузы равных прямоугольных треугольников SCO и MD0), преобразуем выражение для Д к виду
Д = 2SO — SM + \ .
Для нахождения Д построим точку S', симметричную точке 3 относительно плоскости зеркала, т. е. найдем положение мнимого изображения источника света 5. Из рис. 338 видно, что ^SOC = /_ = S'OC, Z_SOC = Z_M0D. Следовательно. Z_M0D = ZJS'OC. Поскольку эти углы имеют общую сторону CD, то они вертикальны, а значит, точки Д', О и М лежат на одной прямой. Так как ОД = ОМ и SO = OS', то ОМ --= OS'. Поэтому S'М = 2OS. Отсюда видно, что для расчета интерференционной картины можно рассматривать точку 3 и ее мнимое изображение 3' как когерентные источники света. Такой прием значительно упрощает расчет интерференционной картины, поэтому он широко используется при решении задач.
С учетом сказанного геометрическую разность хода лучей можно определить по формуле Д' = х.
Разность хода интерферирующих лучей, с учетом потери полуволны при отражении света от зеркала в точке О, будет
А А ' ।	i I . К Г2 LX
Д = Д + 2 = 1 2 "г 2 “ 2L 
Учитывая, что I = 2х = 2 • 10-3 м, Д = 0,88 • 10 8 м.
278
Вычислим, сколько полуволн помещается в этой разности хода лучей:
А	2Д	.
,п X/2	X	4‘
Поскольку разность хода равна целому числу полуволн, то в точке М будет интерференционный максимум.
Задача 8. Угол между двумя плоскими зеркалами (бизеркало Френеля) а = 12' (рис. 339). На них падает свет от щели, параллельной линии пересечения зеркал, находящейся на расстоянии г = 15 см от нее. Длина световых волн Л = 0,6 мкм. Отраженный от зеркал свет образует интерференционную картину на экране, расположенном на расстоянии Lo — 3 м от линии пересечения зеркал.
Определить расстояние между соседними интерференционными полосами на экране.
Решение. С помощью бизеркала Френеля можно получить два мнимых изображения светящейся щели. Они расположены параллельно щели (источнику света) и линии пересечения зеркал. Проекциями светящейся щели и ее мнимых изображений являются соответственно точки 5, S', и S", лежащие в плоскости рисунка. Любая пара мнимых изображений, полученных с помощью бизеркала Френеля, одной и той же точки светящейся линии может рассматриваться как пара точечных когерентных источников света, так как оба изображения воспроизводят колебания одного и того же действительного точечного источника. Различные точки светящейся линии не когерентны между собой, поэтому их мнимые изображения представляют собой некогерентные источники.
Для расчета интерференционной картины можно воспользоваться только двумя мнимыми изображениями 3' и S" точки S. Точки S, S' и S" лежат на окружности радиуса OS = г. Из построения видно, что OS — OS' = OS". Центральный угол ZS'OS" = t= 2 Z.S'SS", т. e. ср = 2(T где [3 — угол между хордами SS' и SS". Угол |3 = а, как углы со взаимно перпендикулярными сторо-
279
памп. Следовательно, <р = 2а. Поскольку угол а мал, то дугу S'S" = = гср == 2га можно заменить отрезком / — расстоянием между когерентными источниками света, т. е. I = 2га. Кроме того, расстояние от точки OL (середина отрезка /) до экрана равно L = Lo + г.
Тогда расстояние между соседними интерференционными полосами на экране
Ax = ^-L^^(Lo+/j=l,8.10-3 м.
Задача 9. В опыте с бизеркалом Френеля расстояние между мнимыми источниками монохроматического света 1 мм. На расстоянии 5 м от источников на экране наблюдается интерференционная картина, в которой расстояние между соседними светлыми полосами 3 мм. Определить длину волны света, излучаемого источником.
Р е ш е н и е. В предыдущей задаче было показано, что расстояние между соседними светлыми полосами интерференционной картины, полученной с помощью зеркал Френеля, определяется соотношением Ах = у L. Отсюда А = 1-~ = 0,6 • 10~° м— 0,6 мкм.
Задача 10. Пучок параллельных лучей монохроматического света (А =0,7 мкм) падает на мыльную пленку (п = 1,33) под углом i = 45°. При какой наименьшей толщине пленки отраженные лучи будут: а) максимально ослаблены вследствие интерференции; б) максимально усилены?
Решение. Из пучка параллельных лучей, падающих на пленку, выберем один луч S4 (рис. 340). Этот луч частично отразится от пленки (луч ЛА'), а частично преломится (луч АВ). На второй грани пленки Л/Лф луч АВ частично преломится и выйдет из пленки (луч BD), а частично отразится (луч ВС). На грани ММг луч ВС частично преломится (луч CF), а частично отразится. Нас будут интересовать лучи Л А' и CF. Если их свести с помощью линзы или глаза в одну точку, то они будут интерферировать. Найдем оптическую разность хода лучей ЛА' и CF: А = (АВ + ВС)п — АЕ +
Из рис. 340 видно, что АВ = ВС = d/cosr, АЕ = 4Csini = ,,,	. . o/Sinr . о, sin2r
= 2dtgrsin i = 2d — nsinr = 2dn--.
°	cosr	cc sr
Учитывая выражения для AB и АЕ, перепишем выражение для оптической разности хода лучей в виде
А = 2d —----2dn	=- 2dnc(\sr + л? = 2d.n У1 — sin2r + =
cosr	cosr 2	2	2
= 2d У rF — H2sin2r +	= 2d У rF — sin2t ф-	.
а)	Отраженные от мыльной пленки лучи будут максимально ослаблены при такой толщине пленки, которая удовлетворяет условию
А = 2d/n2-sin8i + \ = (2k + 1) .
280
Отсюда d — —,	. Выбрав значение k = 1, получим d =
2 у п2 — sin2i
= 0,3 мкм.
б)	Отраженные от мыльной пленки лучи будут максимально усилены при интерференции, если толщина пленки d удовлетворяет условию
Д = ‘IdV п1 — sin2i + у = kX.
Отсюда d =	. Приняв k= 1, получим d = 0,15 мкм.
4 п2 — sin2i
Задача 11. На поверхность стеклянного объектива (пг = 1,6) нанесена тонкая пленка, показатель преломления которой п2 = = 1,26 (просветляющая пленка). При какой наименьшей толщине
этой пленки произойдет максимальное ослабление фиолетового света (X = 0,38 мкм), падающего на пленку перпендикулярно?
Решение. Падающий луч ДЛ (рис. 341) частично отражается на границе воздух — пленка (луч ЛД'), частично проходит в пленку и отражается на границе пленка — стекло и выходит из пленки (луч BS”). Отраженные лучи когерентны. Они интерферируют между собой и при определенной толщине пленки d могут практически полностью «погашаться», т. е. максимально ослабить друг друга.
Наименьшую толщину пленки d, при которой произойдет данное явление, определяем из условия для интерференционных минимумов Л = (2/г + l)-^- при k = 0. Следовательно, оптическая разность хода лучей ЛД' и BS" равна Д = М2.
Поскольку отражение света в обоих случаях происходит на границе с оптически более плотной средой («воздуха < п2, П2 <П]), ТО происходит потеря полуволны как при отражении в точке А, так и в точке В. Поэтому разность хода лучей ЛД' и BS" равна Д = = 2dn2. Приняв во внимание, что Д должно быть равно Х/2, находим
4dn2 —	, откуда d =	= 0,075 мкм.
Задача 12. Мыльная пленка, расположенная вертикально, образует клин вследствие стекания жидкости. Наблюдая интерфе
281
ренционные полосы в отраженном свете гелий-неонового лазера (X = 0,63 мкм), находим, что расстояние между пятью полосами равно I = 1,5 см. Определить преломляющий угол клина. Свет падает на поверхность пленки перпендикулярно. Считать, что показатель преломления пленки п = 1,33.
Решение. Падающий на пленку свет отражается от левой и правой граней клина. Отраженные лучи Г и 1", 2' и 2" и т. д. (рис. 342) когерентны, поэтому наблюдается интерференционная картина. Поскольку преломляющий угол <р очень мал, то отраженные лучи Г и 1", 2' и 2", ..., 5' и 5" будут практически параллельны. В тех местах мыльной пленки, где ее толщина d отвечает условию интерференционного максимума А = 2d |/ п2— sin2/ 4-	~ k\, бу-
дут наблюдаться светлые полосы. Приняв во внимание, что угол падения лучей i = 0, получим выражение для условия максимума
2dn + = /гХ; 2dn = 2-^1 X.
Первая светлая полоса (первый интерференционный максимум) будет наблюдаться на пленке при k = 1, а пятый — при k = 5. Со-ответственно находим: dt = и а5 = ^. 11о условию задачи, расстояние между пятью интерференционными полосами 1= 1,5 см. Тогда из рис. 342 sincp = у d' . Поскольку <р очень мало, то можно принять, что
d.— d. 9Х— X 2Х со 1А_= sincp ~ ф =	_1; ф =	« 6,3 - 10 рад.
Задача 13. Дифракционная решетка имеет М = 2000 штрихов на I = ] см. На ее поверхность нормально падает пучок света от лазера, активным элементом в котором является рубин (X = = 0,6943 мкм). Определить: а) направление максимума в спектре первого порядка; б) сколько всего максимумов дает данная дифракционная решетка.
Решение, а) Направление максимума ср в спектре первого порядка определяем из условия: (а + ft)sin ср = /гХ, где а + b — период решетки, ср—угол отклонения лучей, соответствующих дифракционному максимуму k = 0, 1, 2, ... Для спектра первого порядка k= 1; поэтому sincp =	. Приняв во внимание, что
а -р b =	, находим: sin ср = ^ = 0,1389; ср = 8°.
б) Для вычисления общего числа дифракционных максимумов, которые можно получить с помощью данной дифракционной решетки, определим femax из условия предельного угла <ртах = 90°:
(а + 6) sin<pmax _ _/_____
«тах —-------
282
Поскольку k может быть только целым числом, то femax=7. Общее число максимумов равно 2femax + 1 =15, так как имеется центральный максимум, от которого вправо и влево наблюдается еще по 7 максимумов.
Задача 14. На плоскую дифракционную решетку, имеющую N = 2000 штрихов на I = 1 см, нормально к ее плоскости падает пучок света от ртутной дуги. Определить: а) период дифракционной решетки; б) под каким минимальным углом к направлению первоначального пучка совпадают максимумы двух линий спектра ртути:
0,6128 мкм и Х.2= 0,4047 мкм.
Решение, а) Период дифракционной решетки
а + & = ~ = 5 • 10- м = 5 мкм.
б) Совпадение максимумов, соответствующих двум длинам волн, возможно только в том случае, если дифракционные спектры различных порядков перекрывают друг друга. При этом углы отклонения лучей решеткой для длин волн и Х2 должны совпадать. Это условие находим из условий для дифракционных максимумов двух длин воли:
sin q> = k±= k„ , или /jjAj = /?2Х2; -- = ~ , где /гх и k., — порядки тех спектров, для которых максимумы, соответствующие длинам волн Х2 и совпадают.
X	3
Найдем численные значения и k2: ~ ж 1,5 — g-. Следовательно, kr = 2 и k., = 3.
По значению kx определим направление (угол ф), при котором максимумы двух спектральных линий Х2 и Хх совпадают:
sinq> = -^ = 0,2451; <р=14° 11'.
Задача 15. На плоскую дифракционную решетку нормально падает пучок света от разрядной гелиевой трубки. Под углом 20° видна красная линия спектра (Xj = 0,6680 мкм). Чему равен период дифракционной решетки, если под тем же углом видна и синяя линия (к, = 0,4470 мкм) в спектре более высокого порядка? Наибольший порядок спектра, который можно наблюдать при помощи данной решетки, равен 5.
Решение. Период дифракционной решетки определяем из условия максимума:
(а + b) sin ф = ±kk a + b = ^~.
Значения k и sin ф находим из условия совпадения дифракционных максимумов для линий и Х2. Поскольку наибольший порядок спектра, который можно получить с помощью данной решетки, равен 5, то ни klt ни k2, не могут быть больше 5.
283
Условие совпадения максимумов для двух линий спектра гелий Zj и Л2 определяется соотношением
sm<p=^=4^; k^ = k^ г; = ^=2-Следовательно, k2 = 3 и kt = 2.
Период дифракционной решетки а + b =	= 3,9 мкм.
Задача 16. На стеклянную призму, преломляющий угол которой ф = 60°, падает луч белого света под углом i = 60°. Определить угол 0 между крайними лучами спектра после выхода луча из призмы. Показатели преломления стекла призмы для крайних лучей видимого спектра н,< = = 1,62 и «ф= 1,67.
Решение. Белый луч света SD на грани призмы АВ разложится на монохроматические составляющие (рис. 343). Наибольшее преломление претерпевает фиолетовый луч DE (угол преломления 4), наименьшее — красный луч DM (угол преломления г'). На грани АС эти лучи вторично преломляются и выходят в воздух соответственно (луч MN). Угол между этими край-
под углами 4 (луч EF) и i{
ними монохроматическими лучами и является искомым углом 0.
Как видно из A QQ'Р, 0 4- у2 — у1, где и у2 — соответственно углы отклонения призмой фиолетового и красного лучей. Они равны Yj — i 4- I, —ф и у2 = i 4- i\ — ф. Приняв это во внимание, найдем 0 = Yi — у2 = - ц— t'.
Для определения угла 4 воспользуемся законом преломления света на гранях А В и ВС:
sin i
---= п;
Sill
Из /\DEE’ следует, что t\ 4- r2 со взаимно перпендикулярными сторонами). Следовательно, 44-4- г2= Ф-
Добавим к этим трем уравнениям еще четвертое уравнение: у1= i 4- 4— ф; получим систему четырех уравнений с четырьмя неизвестными sin 4, sin r2, sin 4, yp
sin i = n sin 4; sin ir = n sin r2;
G + r2 = Ф; Yi = t + 4 — Ф-
Решая эту систему, получаем уравнение 2sin24 + V 2sinir + 2 ~ — n2 =0, из которого
—1 * V4п2 — 1 sin 4 —-----.
1	2/2
sin ra   i
sin i,	n
= фР Кроме того, <Pi= ip (как углы
284
Учитывая, что физический смысл имеет решение этого уравнения со знаком плюс перед радикалом, находим
sin z\ = 0,7755; z\ =50°5Г;
sin z'= 0,7382; = 47°34'.
Таким образом, угол между крайними лучами спектра после выхода света из призмы
0 =50°5Г — 47°34' = 3°17'.
ЗАДАЧИ ДЛЯ САМОСТОЯТЕЛЬНОГО РЕШЕНИЯ
987.	Гелий-неоновый лазер излучает монохроматические волны ‘k = 10,6 мкм. Определить частоту этих электромагнитных колебаний. Какова длина волны и частота этого излучения в воде?
988.	Лазер, активным элементом которого является стекло с примесью неодима, излучает монохроматический свет с частотой V = 2,83 • 1014 с-1. Сколько длин волн этого света уложится на пути I = 1 мм: а) в вакууме; б) в воде; в) в стекле (п = 1,6)?
989.	Какой путь пройдет фронт волны монохроматического света в стекле (п — 1,6) за то же время, за которое он проходит I = 1 м в воде?
990.	Какая длина волны подразумевается в выражении для разности фаз Дер интерферирующих световых волн, оптическая разность хода которых Д (Дф = 2лА/Х): длина волны в среде, в которой распространяются волны, или длина волны в вакууме?
991.	Разность хода двух когерентных лучей 2,5 мкм. Определить длины волн видимого излучения (0,400 мкм < X 760 мкм), которые дадут интерференционные максимумы.
992.	В воздухе распространяются два параллельных монохроматических когерентных луча (А = 0,63 мкм). На пути одного из них поставили стеклянную плоскопараллельную кювету с раствором сахара так, что луч падает на ее стенку нормально. Найти: а) оптическую разность хода лучей; б) длину волны света в растворе сахара; в) изменение разности фаз колебаний. Толщина стенок кюветы d = 1 мм, ее длина I = 7 см. Показатели преломления воздуха, стекла, из которого изготовлена кювета, и раствора сахара соответственно щ = 1,00026, и2 = 1,57, па = 1,397.
993.	На пути луча света поставлена стеклянная пластинка (п = 1,5) толщиной d = 1 мм так, что угол падения луча равен z = 30°. На сколько изменится оптическая длина пути луча?
994.	Расстояние между двумя щелями в опыте Юнга 1= 0,5 мм. Расстояние от щелей до экрана L = 3 м. Определить расстояние между светлыми интерференционными полосами на экране, если опыт проводится с зеленым светом (X = 0,555 мкм).
995.	Расстояние между двумя когерентными источниками света I = 0,9 мм. Источники излучают монохроматический свет с длиной волны X = 0,64 мкм. Расстояние от источников до экрана L = 3,5 м. Определить число светлых полос на 1 см длины.
285
996.	Расстояние между двумя когерентными источниками света (X = 0,6943 мкм) I = 0,2 мм. Расстояние от источников до экрана L = 2,5 м. Определить расстояние между светлыми полосами в средней части интерференционной картины.
997.	Два когерентных источника света Si и 52 (X =0,5 мкм) находятся на расстоянии I =2 мм друг от друга. На расстоянии L = 2 м от линии SiS2 находится экран (рис. 344). Точка Л4 на экране является основанием перпендикуляра, опущенного из Sj на экран. На пути луча 52Л4 перпендикулярно к нему помещается плоскопараллельная пластинка толщиной d= 10,5 мкм. Она вносит дополнительную оптическую разность хода лучей А = 5,25 мкм. Определить: а) что будет наблюдаться в точке экрана М (ослабление или усиление света) в отсутствие стеклянной плас-тинки на пути луча S2/W; б) показатель преломления
I	стекла, из которого изготовлена пластинка.
/	998. В опыте с зеркалами Френеля расстояние
/	между мнимыми источниками монохроматического
г-L света (X = 0,6 мкм) I = 1 мм. На расстоянии L = =5 м от источников поместили экран. Определить / расстояние между светлыми полосами в средней час-/ ти интерференционной картины.
/	999. Определить расстояние между мнимыми источ-
/	никами света в опыте с зеркалами Френеля, если рас-
ЧМ_______ стояние между темными полосами на экране d =3мм,
Рис 344 а Расстояние от мнимых источников до экрана L — = 2 м. Длина световой волны точечного источника X = 0,6 мкм.
1000.	На мыльную пленку одинаковой толщины (показатель преломления п — 1,33) падает белый свет под углом i = 30°. При какой наименьшей толщине пленки отраженный от нее свет будет красным (X = 0,63 мкм)?
1001.	На прозрачную пленку толщиной d = 0,367 мкм падает под углом i параллельный пучок белого света. Показатель преломления пленки п = 1,40. В какой цвет будет окрашена пленка в случае, если угол i равен: а) 30°; б) 60°?
1002.	От лазера на рубине падает параллельный монохроматический пучок лучей (X = 0,6943 мкм) на мыльную пленку (и = = 1,33) под углом i = 30°. При какой наименьшей толщине пленки отраженный от нее свет будет максимально ослаблен?
1003.	Поверхность стеклянной линзы покрыта тонкой пленкой. Показатели преломления пленки и стекла, из которого изготовлена линза, равны соответственно пг = 1,34 и п2 = 1,8. На линзу падает белый свет. Определить наименьшую толщину пленки, обеспечивающую максимальное ослабление отраженного света, соответствующего середине интервала длин голи видимого излучения (X ~ 0,57 мкм). Считать, что свет на поверхность линзы надает нормально.
1004.	На стеклянный клин падает нормально к его поверхности пучок монохроматических лучей (X = 0,6 мкм). Расстояние между
286	"
светлыми интерференционными полосами d = 2 мм. Определить угол между поверхностями клина.
1005.	Между краями двух отшлифованных стеклянных пластинок положена полоска бумаги (рис. 345) Два противоположных края пластинок вплотную прижаты друг к другу. Определить число N интерференционных полос, наблюдаемых па единице длины пластинок, если отраженный свет (А = 0,63 мкм) рассматривают, как показано на рисунке. Толщина бумаги h =0,1 мм; длина пластинок / = 10 см; угол падения лучей I =45°.
1006.	На плоскую дифракционную решетку падает нормально пучок монохроматического света (X = 0,59 мкм). Под какими углами к первоначальному направлению лучей будут видны дифракционные максимумы первого и второго порядков, если решетка имеет 500 штрихов на 1 см?
1007.	На плоскую дифракционную решетку падает нормально свет натрия (А = 0,590 мкм). Сколько штрихов на 1 мм длины содержит решетка, если угол между двумя спект-рами первого порядка ср = 13°34'?	'
1008.	На дифракционную решетку нормаль-
но падает свет, длина волны которого А =	_
= 0,589 мкм. При этом для спектра третье-	м
го порядка получается угол отклонения = Рис 345 = 10°11'. Какова длина волны света, для
которого угол отклонения в спектре второго порядка срх = = 6°16'?
1009.	На дифракционную решетку, период которой d — 6 мкм, нормально падает монохроматический свет. Угол между спектрами первого и второго порядков 4°36'. Определить длину световой волны.
1010.	При освещении дифракционной решетки белым светом (0,40 мкм < А с 0,76 мкм) в спектре третьего порядка под углом ср наблюдается спектральная линия, соответствующая длине волны А =0,72 мкм. Будут ли видны под этим углом еще какие-нибудь спектральные линии?
1011.	Падающий на плоскую дифракционную решетку свет состою из двух спектральных линий с длинами волн Ах = 0,490 мкм (голубой свет) и А2 = 0,600 мкм (оранжевый свет). Первый дифракционный максимум для линии с длиной волны Aj располагается под углом ср = 10°. Найти угловое расстояние Дер между линиями в спектре второго порядка.
1012.	Свет от газоразрядной трубки, заполненной водородом, нормально падает на дифракционную решетку. Наименьший угол к первоначальному направлению световых лучей, при котором совмещаются спектральные линии, соответствующие длинам волн Ах = = 0,656 мкм и А3== 0,410 мкм, составляет ср = 41°. Определить период дифракционной решетки.
1013.	На дифракционную решетку с периодом d = 2 • 10"5 м нормально падает пучок белого света. Определить разность углов отклонения начала и конца спектра первого порядка.
287
1014.	На плоскую дифракционную решетку, имеющую 430 штрихов на 1 мм, нормально падает пучок желтого света (X = 0,589 мкм). Определить угол отклонения трубы спектроскопа, при котором наблюдается последний дифракционный максимум. Каков порядок этого максимума?
1015.	На плоскую дифракционную решетку нормально к ее поверхности падает монохроматический свет. Решетка имеет 500 штрихов на 1 мм длины. Какого наибольшего порядка дифракционный максимум дает эта решетка в случае: а) красного (Х]= 0,6943 мкм) и б) зеленого (Х2= 0,56 мкм) света?
1016.	На плоскую дифракционную решетку, период которой d =4 мкм, падает нормально монохроматический свет. За решеткой расположена собирающая линза с фокусным расстоянием f—40 см, которая дает изображение дифракционной картины на экране. Определить длину волны падающего света, если первый максимум находится на расстоянии I = 5 см от центрального.
§ 24. Основы теории относительности
Задача 1. Длина стержня, неподвижного в системе отсчета К, 2 м. Какую продольную скорость нужно сообщить ему относительно системы К, чтобы релятивистское сокращение длины составило 0,2 м?
Решение. Связь между длиной покоящегося и движущегося стержня выражается соотношением I — 101/ 1 — и2/с2. По условию задачи, Д/= /0 — I, или Д/ — 10 — l0V 1 — г»2/с2. Отсюда и = =	1 — (1 — уГ « 1,31 • 108 м/с.
\	*0/
Задача 2. Мезоны космического излучения достигают поверхности Земли с разнообразными скоростями. Найти релятивистское сокращение размеров мезона, имеющего скорость, равную 95 % скорости света.
Решение. Размеры мезона связаны соотношением d = d0 X Xp^l — о2/с2. С учетом этого соотношения hd=d0 — do]^ 1 — ц2/с2 = = d0 (1 — V1 — у2/с2), откуда — = I — J/Ч — v^jc2 = 0,688, или Uq 68,8 %.
Задача 3. Ионизированный атом, вылетев из ускорителя со скоростью 0,5 с, излучил фотон в направлении своего движения. Определить скорость фотона относительно ускорителя.
Решение. Свяжем с ускорителем систему координат К, а вылетевший из ускорителя ионизированный атом примем за тело отсчета подвижной системы координат К' (рис. 346). Скорость фотона относительно системы К' и' = с. Скорость фотона относительно ускорителя
и' 4- V
U ~ 1 + u'vjc2 ~ С‘
288
Следовательно, скорость фотона относительно ускорителя равна скорости фотона относительно атома, излучившего этот фютон, т. е. и = и' == с.
Задача 4. Имеются две инерциальные системы отсчета Л и К', относительная скорость которых неизвестна. Параллельный оси Ох' стержень, движущийся относительно системы X' со скоростью и*- = 0,100 с, имеет в этой системе длину Г = 1,10 м. В системе /< длина стержня I = 1,00 м. Определить скорость стержня их в си-
стеме К и относительную скорость систем и.
Решение. Поскольку длина стержня в системе отсчета К'
больше, чем его длина в системе /С, то естественно принять систему К' за неподвижную, а систему К — за подвижную. Обозначим длину покоящегося стержня в системе К' через Го. Тогда I = /0' У1—и2/с2,
откуда их^сУ\
Определим Г„ из соотношения /' = = Г0У1 — и2-/с2 и, подставив в формулу для их, получим
^Г(1-«>)~У;
их = с/1 - (///')2(1 ~	^0,43 с.
Относительную скорость систем v най-
Рис. 346
дем из соотношения
“l + ч	z	, ихих’\ 1	„ о.
Ых1 = , ,	, ,; ц = («х'~«,) 1---------—А ~ —0,34 с.
1 + UxV/c* ’	V	\	С2 /	’
Знак минус указывает на то, что система Л движется относительно >
системы Л' со скоростью v, направление которой противоположно направлению оси О'х'.
Задача 5. Электрон прошел ускоряющую разность потенциалов и приобрел кинетическую энергию 1,02 МэВ. Определить: а) во сколько раз его масса больше массы покоя; б) скорость электрона.
Р е ш е н и е. а) Отношение массы движущегося тела т к массе покоя т0 можно найти из релятивистской формулы кинетической энергии тела WK = тс2 — т0с2:
т
т0
т№с2 + Wk { , Wk
тис2	~ т0с2
= 3 раза.
б) Для определения скорости электрона воспользуемся зависимостью массы тела от скорости его движения т =	 •
V1 — о2/с2
Отсюда — = r -1_______
то У 1—иа/с2 ’
Решив последнее уравнение относительно и, получим
v = с У1 —	« 2,83 • 108 м/с.
Va Ю 7-263
289
Задача 6. Электрон прошел ускоряющую разность потенциалов U = 500 кВ. Определить импульс электрона.
Решение. Импульс электрона можно определить из соотношения между импульсом и энергией. Полная энергия тела W = тс2 = т°с — • отсюда Ю7! — v2/c2=m0c2. Возведем в квад-/1—t)2/C2
рат левую и правую части последнего равенства:
W2—IT2 = /п>; W2 — т2с* = tnoc\
Учитывая, что m2v2 = р2 (где р — импульс тела), получим
U72 — р2с2 mj/?4, откуда р = W2 — тй(А.
Найдем выражение для W через ускоряющую разность потенциалов U'.
eU = тс2 — тос2 = W — Wo, откуда W = eU -ф VP'q.
Подставим это значение в формулу импульса:
о	]/ (d/ + Го)2 - Г;, = j V(eU)2 + 2е(Ж0 = 4,7 • 10“22 кг-м/с.
ЗАДАЧИ ДЛЯ САМОСТОЯТЕЛЬНОГО РЕШЕНИЯ
1017.	Какую продольную скорость v должно иметь движущееся тело, чтобы его длина была в два раза меньше длины, которую оно имеет в состоянии покоя?
1018.	С какой скоростью v должна лететь частица относительно системы отсчета К для того, чтобы собственное время Ат0 частицы было в 10 раз меньше промежутка Ат, отсчитанного по часам, находящимся в покоящейся системе отсчета Л?
1019.	Из двух ускорителей частицы вылетают навстречу друг другу со скоростью 0,9 с. Определить относительную скорость сближения частиц.
1020.	Электрон движется со скоростью v = 0,9 с. Определить импульс р и кинетическую энергию электрона.
1021.	При какой скорости кинетическая энергия любой частицы вещества равна ее энергии покоя?
1022.	Определить импульс электрона, обладающего кинетической энергией WK = 5 МэВ.
1023.	В электронно-лучевой трубке телевизора электрон ускоряется разностью потенциалов А<р = 80 кВ. Чему равно отношение его массы после ускорения к массе покоя т/т0? Определить относительную погрешность, которая получится, если для вычисления скорости электрона вместо релятивистской формулы кинетической энергии воспользоваться классической.
1024.	Электрон двигался с начальной скоростью у =0,1 с. Над ним была совершена работа А — 8,24 • 10~14 Дж. Найти
290
изменение скорости, импульса и кинетической энергии электрона.
1025.	Солнце излучает ежеминутно энергию 6,5 • 1021 кВт • ч.
На сколько уменьшается масса Солнца за 1 сут?
§ 25. Световые кванты
Задача 1. Лазер излучает в импульсе длительностью t — = 1,5 мкс энергию W = 250 Дж в виде почти параллельного пучка с площадью сечения £ = 1 см2. Длина волны, излучаемой лазером, X = 1,06 мкм. Определить плотность фотонов в пучке.
Решение. Зная энергию фотона, можно найти общее число излучаемых фотонов: па = W/hv или, учитывая, что с = hv, получим n0 = hW/hc.
Считая, что мощность лазера постоянна в течение времени излучения и что поперечное сечение пучка также постоянно, находим выражение для плотности потока фотонов
Задача 2. Определить массу фотона: а) инфракрасного (% = = 300 мкм); б) красного (0,76 мкм); в) фиолетового (0,4 мкм); г) рентгеновского (2 • 10-4 мкм) и д) гамма-излучения (5 • 10“’мкм). Сравнить ее с массой электрона.
Решение. Из соотношения между энергией и массой £ = = hv = /пс2 следует, что т =	, или, учитывая, что с = Xv, можно
ch
Хс2
записать т —
Подставляя численные значения, получим
а)	тк = 7,4 • 10-39 кг; б) тк = 2,9 • Ю-30 кг;
в) гпф — 5,5 • 10-36 кг;	г) тр = 1,1 • 10-32 кг;
д) тг = 4,42 • 10-30 кг.
Масса электрона те —9,11 • 10-31 кг, а масса гамма-фотона т, = 4,42  10~зв кг, т. е. в 4,85 раза больше массы электрона.
Задача 3. Лазер излучает свет с длиной волны X = 0,6943 мкм. За один импульс он излучает энергию 2 Дж. Определить: а) энергию фотона (в электрон-вольтах); б) массу фотона; в) импульс фотона; г) количество фотонов, излучаемых в одном импульсе.
he
Решение, а) Энергия фотона Е — hv у да 2,9 • 10-19 Дж = = 1,8 эВ.
б)	Массу фотона определим из соотношения между энергией и массой Е = /НфС2, откуда /Пф = Е/с2, = 3,2 • 10-34 кг.
в)	Подставив в равенство Е = тс2 вместо т его значение т — тп	п т«с'2 г>
= — ° — , получим Е = —г . Возведя это равенство в квад-
У \—v2/c2 J	/1 —w2/c2
рат, после преобразований найдем
ЩцС4 = £2 — £2 ~2 , откуда т“с4 = £2 — р2е2.
С
Ча Ю!
291
Учитывая, что фотон имеет нулевую массу покоя, подставим в последнее равенство значение т0 = 0; получим
Е hv	hv	hv	h
p = — = —	, ИЛИ	P =	— = .— =	Y .
' с c	r	C AV	Л
Следовательно, в рассматриваемом случае импульс фотона будет р = 9,6 • 10-28 кг • м • с-1.
г)	Количество излучаемых лазером фотонов за один импульс N = W/E = 7 • 101и фотонов/импульс.
Задача 4. С какой скоростью должен лететь электрон, чтобы: а) его кинетическая энергия была равна энергии фотона зеленого света (X = 0,555 мкм); б) его импульс был равен импульсу фотона с длиной волны X = 0,555 мкм?
Решение, а) Кинетическая энергия электрона U7K = -^-. he
Энергия фотона Е = у . Поскольку U7K = Е, то лги- he	-1 Е2hc , „	,	,
-о- = v • откуда v = I/ —г « 1,6  10е м/с.
б) Импульс электрона р = mv, а импульс фотона = у;
й	h
так как по условию задачи они должны быть равны, то mv = т ,
А
откуда v =	« 1,3 • IO8 м/с.
Задача 5. Глаз человека наиболее чувствителен к зеленому свету (X = 0,555 мкм), для которого порог чувствительности глаза соответствует п0 — 80 фотонам, падающим на сетчатку за 1 с. Какой мощности света соответствует этот порог?
Решение. Мощность света N = W/t. Энергия фотонов W = = паЕ, где Е — энергия кванта света. Так как E = hv—~, то W =	. Подставив значение W в формулу для мощности, получим: N =	« 2,9 • IO’17 Вт.
Задача 6. При какой температуре атом гелия будет иметь кинетическую энергию, достаточную для того, чтобы ударом возбудить атом другого химического элемента, излучающего фотоны с длиной волны 0,63 мкм? Какова средняя квадратичная скорость атома гелия при этой температуре?
Решение. Средняя кинетическая энергия атомов одноатомного газа
W к —	~ kT.
Для того чтобы атом гелия мог ударом возбудить атом другого химического элемента, кинетическая энергия атома гелия должна быть не меньше энергии фотона, излучаемого атомом другого эле-
292
__	2	he
мента, т. е. WK > Е, или kT > hv = -г . Из последнего равенства находим
I-5 • 104 К.
3 ЛЙ
Средняя квадратичная скорость атома гелия ус. к =Д/Да =
= V^~ 9700 м/с'
Задача 7. У некоторого металла фотоэффект начинается при частоте падающего света 8,1 • 1014 Гц. Определить работу выхода электрона из металла в электрон-вольтах.
Решение. По второму закону фотоэффекта, Е = А + WK. В условии задачи указана частота света, при которой начинается фотоэффект, т. е. красная граница фотоэффекта. При этой частоте кинетическая энергия фотоэлектронов равна нулю. Поэтому уравнение Эйнштейна для красной границы фотоэффекта будет иметь вид hv = есро- Следовательно, работа выхода электрона из металла в этом случае равна энергии фотона (кванта):
А = hv = 5,4 • 10'1!1 Дж « 3,4 эВ.
Задача 8. На поверхность вольфрама падают ультрафиолетовые лучи, длина волны которых 0,04 мкм. Найти: а) красную границу фотоэффекта для вольфрама; б) максимальную кинетическую энергию фотоэлектронов; в) максимальную скорость фотоэлектронов.
Р е ш е н и е. а) Красную границу фотоэффекта находим из соот-ношени я hvK = А, откуда vK = A/h. Из таблицы находим значение А для вольфрама: А =4,5 эВ. Следовательно, граничная частота
vK = 1,1 • Ю1-1 Гц.
Соответственно длина волны света, отвечающая красной границе фотоэффекта, будет = — = 2,7 • 10-7 м.
VK
б) Максимальную кинетическую энергию фотоэлектронов находим из уравнения Эйнштейна для фотоэффекта:
Е = А 4- ИД, откуда 1ГК = Е — Д; Г,< = ~ — А = 4,28 • 10"1а Дж =
= 26,75 эВ.
в) Кинетическая энергия фотоэлектрона может быть найдена по классической формуле 1ГК = m<r/2 или по релятивистской формуле WK = (т — т0) с2, в зависимости от того, какая скорость сообщается фотоэлектрону. Вычислим энергию покоя электрона и сравним ее с энергией фотона:
№0 = 8 2 • 10 14 Дж; ^=1,6 ДО4.
293
Следовательно, E<^Wa, поэтому для определения скорости фотоэлектрона воспользуемся классической формулой кинетической энер-
w, mv2	-|/~2117к	п
гии: U/K = — , откуда v = у	Подставим сюда выражение
для найденное из уравнения Эйнштейна; получим
v = 1/2(£~Л) » 3,1  10е м/с.
Задача 9. Фотон (длина волны X = 0,002 нм) вырывает из поверхности платины электрон. Определить: а) скорость фотоэлектрона; б) его импульс; в) задерживающее напряжение.
Решение, а) Сравним энергию фотона с энергией покоя электрона. Если последняя будет значительно больше энергии фотона, то скорость фотоэлектрона будем находить по классической формуле кинетической энергии частицы; если же энергия покоя электрона будет сравнима или меньше энергии частицы, то по релятивистской.
Энергия фотона Е = —я: 10“13 Дж = 0,625 • 10е эВ = Л
= 0,625 МэВ. Энергия покоя электрона = /nftc2 = 8,2 • 10"14Дж = 0,512 . 10е эВ = 0,512 МэВ. Как видим, энергия фотона больше, чем энергия покоя электрона, т. е. эти энергии сравнимы по значению. Поэтому для определения скорости фотоэлектрона воспользуемся релятивистской формулой кинетической энергии частицы
Й7К = тс2 - т0с2 =	- т0с2 =	— №0.
°	/1-И2/са	/1— v2/v2	°
Решив последнее уравнение относительно и, получим
V	^о+1^к
Значение WK найдем из уравнения Эйнштейна для фотоэффекта Е — А + WK. Так как работа выхода электрона из платины А = = 5,3 эВ Е = 0,625 МэВ, то в уравнении Эйнштейна ею можно пренебречь. Следовательно, можно принять, что Е = WK. С учетом сказанного определяем численное значение скорости фотоэлектрона
п = 2,7 • 108 м/с.
Скорость фотоэлектрона определим по классической формуле кинетической энергии WK =	, откуда v = ~\/~	«4,7- 108 м/с.
Как видим, при применении классической формулы для скорость фотоэлектрона получилась большей, чем скорость света в вакууме с = 3 • 108 м/i, что не имеет физического смысла. Это и подтверждает правомерность применения релятивистской формулы кинетической энергии.
294
б) Импульс фотоэлектрона p = mv, где т = ——т<>	Тогда
V 1 — и2/с2
р «5,6 • IO'22 кг • м/с.
в) Задерживающее напряжение можно определить по закону сохранения энергии №к = eU3, откуда
Г из =	= 0,63  10° в.
Задача 10. Уединенный цинковый шарик облучают монохроматическим светом длиной волны X = 4 нм. До какого потенциала зарядится шарик? Работа выхода электрона из цинка А = 4,0 эВ.
Решение. Под действием света происходит вырывание электронов из металла, вследствие чего цинковый шарик заряжается положительно и его электрическое поле тормозит вылет последующих электронов. Очевидно, наступит такой момент, когда разность потенциалов между шариком и бесконечно удаленной точкой будет равна задерживающему напряжению, т. е. [/3 = <ртах — <р«, где фтах — потенциал, до которого зарядится цинковый шарик при фотоэффекте, = 0 — потенциал поля в бесконечности. Следовательно, U3 = фтах. Задерживающее напряжение определяется максимальной кинетической энергией фотоэлектронов:
W eU3 = WK; ^3 = фтах = —к.
Максимальную кинетическую энергию фотоэлектрона определим из уравнения Эйнштейна: hv = А + WK; UZK = hv—А. Отсюда
ЗАДАЧИ ДЛЯ САМОСТОЯТЕЛЬНОГО РЕШЕНИЯ
1026.	Определить энергию, массу и импульс фотона рентгеновского излучения с длиной волны X =0,1 нм.
1027.	Сколько фотонов падает в 1 с на 1 см2 поверхности, если она облучается с мощностью N = 10~3 Вт/см2 у-излучением с длиной волны X = 10-11 м?
1028.	Мощность, рассеиваемая в виде излучения во всех направлениях лампочкой карманного фонарика, N = 1 Вт; средняя длина волны излучения X = 1 мкм. Сколько фотонов проходит за 1 с через площадку 1 см2, расположенную на расстоянии I = 10 км от лампочки перпендикулярно к лучам?
1029.	У некоторого металла фотоэффект начинается при частоте падающего света v = 1,14 • 1C1S Гц. Найти работу выхода электрона.
1030.	На поверхность литиевой пластинки падают фиолетовые лучи (X = 0,4 мкм). Работа выхода электронов из лития А = = 2,4 эВ. Определить красную границу фотоэффекта. Будет ли происходить фотоэффект?
295
1031.	Работа выхода электрона из никеля А = 5 эВ. Будет ли происходить фотоэффект, если никель освещается лучами длиной волны X = 0,3 мкм?
1032.	Какая часть энергии фотона расходуется на работу вырывания электрона, если красная граница фотоэффекта Хо = 0,4 мкм и кинетическая энергия электрона W = 2 эВ?
1033.	Сколько процентов энергии фотона расходуется на работу вырывания электрона из металла, если красная граница фотоэффекта X = 0,555 мкм и кинетическая энергия фотоэлектрона WK = 4 эВ?
1034.	Красная граница фотоэффекта для некоторого металла Хо = 0,275 мкм. Найти: а) работу выхода электрона из этого металла; б) максимальную кинетическую энергию фотоэлектронов, если на металл падает свет длиной волны X =0,18 мкм; в) максимальную скорость фотоэлектронов.
1035.	На фотоэлемент с оксидно-цезиевым катодом падают инфракрасные лучи (7 = 1 мкм). Определить максимальную: а) кинетическую энергию фотоэлектронов; б) скорость фотоэлектронов, если красная граница фотоэффекта = 1,2 мкм.
1036.	На металл падает рентгеновское излучение с длиной волны X = 1,5 нм. Определить скорость электронов, вылетающих из металла. Работой выхода электрона из металла пренебречь.
1037.	Какова максимальная скорость фотоэлектронов, вылетающих из цинка, при освещении его лучами с длиной волны X = = 0,2 мкм, если работа выхода А = 4 эВ?
1038.	Под действием света из платины вырываются электроны, скорость которых v = 5 • 105 м/с. Определить частоту падающего света.
1039.	Красная граница фотоэффекта цезия v0 =4,59 • IO?4 Гц. Определить скорость фотоэлектронов при облучении цезия фиолетовыми лучами длиной волны X = 0,4 мкм.
1040.	На металлическую пластинку падает монохроматический свет длиной волны X = 0,42 мкм. Фототок прекращается при задерживающем напряжении U = 0,95 В. Определить работу выхода А электронов с поверхности платины.
1041.	Фотоэлемент освещается светом длиной волны 7 = = 0,38 мкм. Электроны, вырываемые с поверхности катода, полностью задерживаются разностью потенциалов Дер = 1,7 В. Определить работу выхода для металла фотокатода и краевую границу фотоэффекта.
1042.	При фотоэффекте с поверхности серебра задерживающий потенциал оказался равным ф = 1,2 В. Вычислить частоту падающего света.
1043.	Красная граница фотоэффекта для рубидия Ло = 0,81 мкм. Какое задерживающее напряжение надо приложить к фотоэлементу, чтобы задержать электроны, вырываемые из рубидия ультрафиолетовыми лучами длиной волны к =0,1 мкм?
1044.	Рентгеновская трубка работает под напряжением U — — 60 кВ. Определить: а) максимальную энергию фотона рентгеновского излучения; б) минимальную длину волны рентгеновского излучения; в) импульс излучаемого фотона.
296
VI. АТОМНАЯ И ЯДЕРНАЯ ФИЗИКА
§ 26. Физика атома и атомного ядра
Задача 1. Согласно теории Бора, радиус первой орбиты электрона в атоме водорода равен = 0,53 • 10“10 м. Определить линейную и угловую скорости движения электрона по этой орбите, а также радиус, линейную скорость и частоту вращения электрона в атоме водорода для второй боровской орбиты. Существует ли какая-либо связь между частотой обращения электрона вокруг ядра атома водорода и частотой его излучения?
Р е ш е н и е. В теории Бора к атому водорода применимы классические законы. Так как кулоновская сила притяжения электрона в атоме водорода к его ядру обеспечивает движение электрона по орбите с центростремительным ускорением, то по второму закону динамики можно записать
г2 mvn 4лейг“ = 77 ’
откуда для первой орбиты (n = 1) линейная скорость электрона а; 2,2 • 106 м/с. Угловая скорость в1 = -1л4,1х
У 4neomr1	Г1
Радиусы воровских орбит связаны соотношением гл = «2г1, где п — номер орбиты. Отсюда r2 = 22rt = 2,12 • 10-10 м. Линейная скорость движения электрона по второй орбите v2 = е. « 1,1 X
У 4neomr2
X Ю6 м/с. Частота обращения электрона на n-й орбите определяется формулой vn =	. На второй боровской орбите частота обра-
щения электрона v2 « 7,5 • 1013 с-1.
Частота излучения атома по теории Бора определяется изменением его энергии при переходе из одного стационарного состояния в другое, поэтому никакой связи между частотой излучения атома и частотой обращения электрона вокруг ядра не существует.
Задача 2. На неподвижный невозбужденный атом водорода налетает другой невозбужденный атом водорода. Какова должна быть минимальная кинетическая энергия налетающего атома, чтобы в результате столкновения мог излучиться фотон? Энергия ионизации атома водорода Е — 13,6 эВ. Частоты излучения атома
водорода определяются формулой v = R	где R — по-
стоянная, пит — целые числа.
Решение. При соударениях двух тел максимальные потери кинетической энергии их движения происходят при абсолютно неупругом ударе, когда тела после столкновения двигаются с одинаковыми скоростями. Эта часть кинетической энергии может превращаться в другие виды энергии, в частности, пойти на возбуждение
11 7-263
297
одного из сталкивающихся атомов. В дальнейшем, при переходе атома из возбужденного состояния в основное, может излучиться фотон. Минимальная энергия возбуждения Emin атома, находящегося в основном состоянии, достигается при переходе электрона с первого энергетического уровня на второй (п = 1, т =2). Напротив, энергия ионизации атома, необходимая для отрыва электрона от него, соответствует переходу с первого уровня на бесконечно удаленный (п = 1, т = оо). Поэтому из условия задачи следует, что константа R в формуле для частот излучения равна энергии ионизации Ек, деленной на постоянную Планка h, а минимальная энергия возбуждения атома
Из законов сохранения энергии и импульса можно получить мини-1 ° мальную кинетическую энергию mv“0 налетающего атома водорода:
У mv0 = 2 -у + £min и mv0 = 2mv, откуда mv0 = 2£min =
= 4 Еи = 20,4 эВ. 4
Задача 3. Протон, летящий горизонтально со скоростью о0 = = 4,6 • 104 м/с, сталкивается с неподвижным свободным атомом т-г	1
гелия. После удара протон отскакивает назад со скоростью а атом переходит в возбужденное состояние. Вычислить длину волны света, который излучает атом гелия, возвращаясь в первоначальное состояние.
Решение. Запишем для системы протон—атом гелия законы 1	1	2
сохранения импульса и энергии: mv0 = — mv0 + Mv и mv0 = = -i- тог„ 4- g1 Mf2 4- E, где m = 1 a. e. м. — 1,67 • 10-27 кг — масса протона, M = 4 а. е. м. — масса атома гелия, Е— энергия возбуждения атома. Отсюда найдем Е — mv20 (1 —	.
Возвращаясь в первоначальное состояние, атом излучает квант света, энергия которого равна энергии возбуждения: hv = Е, откуда длина волны излучаемого света
Задача 4. Определить энергию связи ядра атома урана g25U.
Решение. Энергия связи ядра атома определяется из соотношения А£ = АМс2, где АЛ4 — дефект массы ядра, с — скорость света в вакууме. Дефект массы ядра определяется по формуле AM = Zmp + Nma — Мя, где Мя — масса ядра.
298
В таблицах масс изотопов приводятся значения масс нейтральных атомов, а не массы ядер. Поэтому последнюю формулу целесообразно преобразовать так, чтобы вместо массы данного ядра Мя в нее входила масса соответствующего нейтрального атома М3-Так как Мя = Ма— Zni&, то ДЛ4 = Ztnv + Nmn — (Д4а— Zrne), или ДЛ4 = Z(mp 4- me) + Nmn — Ma. Ho mp + me = tru . Следовательно,
ДУМ =	+ Nm„ — Мя.
Подставив в последнюю формулу числовые значения масс в а. е. м., получим Ш = 92  1,00783 + 143 • 1,00899 — 235,04394 = = 1,96199 а. е. м. « 3,26 • 10~2’ кг. Тогда энергия связи ядра 2mU Е = 1,96199 -931,1 МэВ « 1826,8 МэВ « 29,34 • КГ11 Дж.
Задача 5. Ядро атома урана поглощает медленный (тепловой) нейтрон и распадается в соответствии с реакцией 2g2U + *п —> -> ^Хе + ggSr + 2*п. Какая энергия выделяется при этой реакции?
Решение. Чтобы определить выделившуюся при ядерной реакции энергию, необходимо найти сумму масс частиц до реакции и их сумму после реакции. Если первая сумма превышает вторую, то реакция идет с выделением энергии (экзотермическая реакция), если же сумма масс частиц после реакции превышает сумму масс частиц до реакции, то реакция идет с поглощением энергии (эндотермическая реакция).
Сумма масс частиц до реакции:
Ми = 235,04394 а. е. м.
Л4П = 1,00899 а. е. м.
S Мг = 236,05293 а. е. м.
Найдем сумму масс частиц после реакции:
МХе = 138,91844 а. е. м.
MSr = 94,92446 а. е. м.
2Л4п = 2,01798 а. е. м.
£ Л42 = 235,86088 а. е. м.
Энергия, выделяющаяся при ядерной реакции, определяется соотношением
кЕ = (SMj — ZM2) 931,1 МэВ « 178,8 МэВ.;
Учитывая, что 1 МэВ = 1,6 • 10~13 Дж, ответ можно представить в единицах СИ: ДЕ = 178,8 • 1,6 • 10"13 Дж ж 2,86 • Ю'11 Дж.
Задача 6. Ядерная реакция ,-iN + 2Не J7O + р может идти, если налетающие на неподвижные ядра азота а-чаетицы имеют энергию, превышающую пороговую энергию Еп = 1,45МэВ. На сколько энергия а-частиц должна быть больше пороговой, чтобы кинетическая энергия образующихся в реакции протонов была равна нулю?
11*
299
Решение. Пороговая энергия £п — это минимальная энергия налетающей частицы, при которой происходит ядерная реакция. Она включает в себя энергию Е, поглощаемую при реакции, и минимальную кинетическую энергию Ек продуктов реакции (Ек не может быть равной нулю, исходя из закона сохранения импульса). Эта кинетическая энергия минимальна, если ядро кислорода и протон движутся как единое целое, т. е. с одинаковыми скоростями. Согласно закону сохранения энергии, в первом случае
Еп = Е +
таЕп . то + «р ’
где та, т0 и тр — массы а-частицы, ядра кислорода и протона соответственно. Во втором случае энергия а-частицы будет равна таЕа
Еа — Е + —т . Тогда увеличение энергии а-частицы
тг,тр
Д£а = Еа — Еп = Еп
----j---гт-------г ~ 25 кэВ. (m0 + тр) (тй — та)
Задача 7. Какую минимальную энергию необходимо затратить 12~	.
для разделения ядра -^С на три равные части?
12
Решение. При делении ядра -^С на три равные части обра-зуется три ядра -% Не. Для этого нужно затратить ЕЕ = 931,1 Ет = = 931,1 (тс — Зтне) ~ 7,3 МэВ х 11,7 • 10-13 Дж.
Задача 8. Определить энергию, освобождающуюся в водородной бомбе при синтезе 1 кг гелия.
Решение. Уравнение ядерной реакции синтеза гелия из ядер дейтерия и трития 4D + ,Т *Не Jn. При синтезе одного атома гелия освобождается энергия ЕЕ = 931,1Ет ~ 931,1 (mD-f-mT — -mHe-m„)«2,8- 10“12 Дж.
Число атомов, содержащихся в 1 кг гелия, можно найти, зная число Авогадро МА = 6,02 • 1023 и число молей этого количества вещества, т. е. М = -^-Мд. Тогда суммарная энергия, освобождающаяся при синтезе 1 кг гелия, будет
Е = NEE = -^-N.EE « 4,2 • 104 Дж. /И А
Задача 9. Написать недостающие обозначения в следующих
1 \ г ^L_T	। 1	1^8 г т . 1	198 *	.
ядерных реакциях: 1) 4Ве + 2Не -> х + оп; 2) 80Hg + оп -* ,вАи+х; 3) “F + }Н -> “О + х; 4) “Мп + х “Fe + Jn; 5) х + }Н -> ??Na+ + ,Не; 6) x + fiv-^W + Jn-
Решение. По закону сохранения электрического заряда, сумма нижних индексов после реакции должна равняться их сумме
зоо
до реакции. Сумма массовых чисел, т. е. верхних индексов, после реакции также должна равняться их сумме до реакции. Поэтому сначала нужно записать недостающие индексы: 1) 4 Be + 2Не-► 11х + +	2) ^Hg + Jn -> ™Au + Jx; 3) “F + *H -> “O + ‘x; 4) *86Mn +
+ ix + Jn; 5) “x + JH ->^Na + ‘He; 6) \842x + hv + Jn.
Теперь, зная зарядовые числа, вписываем наименования элементов: 1)	2) ^Hg + o^^Au + jH; 3) \9F+
+ JH -> “О + 24Не;	4) “Мп +	-+	+ Jn; 5) J25Mg + JH
-> ^Na + ‘Не; 6)	+ hv ->	+ Jn.
Задача 10. Мощность атомных установок подводной лодки «Наутилус» (США) Р = 14,7 МВт. Ядерным топливом служит обогащенный уран (25 % 236U). Определить запас топлива, необходимого для месячного плавания лодки, если при делении одного ядра урана выделяется q = 200 МэВ = 3,2 • 10'11 Дж энергии
Решение. Определим сначала количество энергии, выделяющейся при делении всех ядер, содержащихся в 1 кг 235U. В 1 кг
урана содержится атомов N = МА. При делении всех этих ядер выделяется энергия Е = Nq =	Тогда на один месяц пути
надо брать 235(7
Pt 24 • 3600 • ЗОРМ .пс т3 = F =---------—---------« 495 г.
с
mNAq
Но 235U в горючем составляет лишь 0,25 часть всей массы топлива, поэтому запас топлива на месяц пути должен быть в 4 раза больше, т. е. приблизительно 1,98 кг.
ЗАДАЧИ ДЛЯ САМОСТОЯТЕЛЬНОГО РЕШЕНИЯ
1045.	При переходе электрона в атоме водорода с третьей стационарной орбиты на вторую излучаются фотоны, соответствующие длине волны X = 6,52 • 10-7 м, дающие красную линию водородного спектра. Какую энергию теряет атом водорода при излучении этого фотона?
1046.	Электрон в невозбужденном атоме водорода получил энергию Е = 12 эВ. На какой энергетический уровень он перешел? Сколько линий можно будет увидеть в спектре излучения при переходе электрона на более низкие энергетические уровни? Энергия основного состояния атома водорода Ег =—13,5 эВ.
1047.	Атом водорода при переходе из одного стационарного состояния в другое испускает последовательно два кванта с длинами волн Х1 = 40 510 • 10'10 м и Х2 = 972,5 • 10'10 м. Определить изменение энергии атома водорода.
1048.	Электрон, обладающий вдали от покоящегося протона скоростью v = 1,875 • 10е м/с, захватывается последним, в резуль
301
тате чего образуется возбужденный атом водорода. Определить длину волны фотона, который испускается при переходе атома в нормальное состояние.
1049.	При лобовом столкновении атома водорода, двигавшегося со скоростью и0 = 7 • Ю4 м/с, с покоившимся атомом водорода был испущен световой квант с длиной волны X = 0,122 мкм. Пренебрегая импульсом фотона, определить скорости атомов после столкновения.
1050.	Определить энергию связи, приходящуюся на один нуклон в ядре атома ^N, если масса последнего т = 22,99714 а. е. м.
1051.	При бомбардировке алюминия 13AI а-частицами образуется фосфор i°P. Записать эту реакцию и подсчитать выделившуюся энергию. Массы изотопов 23А1 и равны соответственно 26,99010 и 29,97867 а. е. м., а массы а-частицы и нейтрона — 4,00260 и 1,00894 а. е. м.
1052.	Какую минимальную энергию должен иметь у-квант для вырывания нейтрона из ядра ,’2С?
1053.	Найти энергию связи ядра, которое имеет одинаковое число протонов и нейтронов и радиус которого в полтора раза меньше радиуса ядра 13А1.
1054.	При облучении бериллия а-частицами образуются неизвестный элемент и нейтрон. Записать реакцию и определить неизвестный элемент.
1055.	При термоядерной реакции слияния дейтерия 2Н и трития образуются нейтрон, неизвестная частица и выделяется Ео — 17,6 МэВ энергии. Определить неизвестную частицу и полную энергию, которая выделится, если прореагирует tn = 1 г дейтерия.
1056.	Ядро урана 2дг^7, захватив один нейтрон, распалось на два осколка и выбросило два нейтрона. Одним из осколков является ядро бария ^Ва. Записать реакцию и определить, какой еще химический элемент образуется при этом распаде.
1057.	При взаимодействии ядер алюминия 13AI с х-частицами образуются ядра изотопа магния ^2Mg и ^-частицы. При взаимодействии же ^-частиц с ядрами алюминия ”А1 образуются ядра изотопа магния ^Mg и z-частицы. Какие широко известные частицы х, у и z участвуют в этих ядерных реакциях?
1058.	При бомбардировке изотопа азота нейтронами получается изотоп углерода 13С, который оказывается 0-радиоактивным. Написать уравнения обеих реакций.
1059.	В результате взаимодействия ядер дейтерия и трития образуются ядро гелия и нейтрон: 2Н + 3Н = 4Не + п. При этом выделяется значительная энергия. Какую часть ее уносит с собой нейтрон? Кинетическими энергиями дейтерия и трития до реакции можно пренебречь по сравнению с выделившейся энергией.
302
1060.	Записать недостающие обозначения в следующих ядерных реакциях: 1) j,Al + Jn -> х + ‘Не; 2) ‘‘С + ‘Не-»-1^) + х\ 3)gLi+ + х -> ‘Не + ‘Не; 4) х + ‘Н -> "К + ‘Не.
1061.	Какое количество урана расходуется в сутки на атомной электростанции мощностью Р = 5000 кВт? К. п. д. т] принять равным 17 %. Считать, что при каждом акте распада выделяется энергия Ео = 200 МэВ.
1062.	Определить месячный расход топлива ядерным реактором РБМК-1500 тепловой мощностью Р =4800 МВт, считая, что при делении одного ядра g’5U выделяется ДА = 200 МэВ энергии. Топливом служит обогащенный уран с содержанием в 1 т природного урана 20 кг , причем вследствие захвата нейтронов делению подвергается г] = 85 % всех ядер.
1063.	Два ядерных реактора атомного ледокола «Сибирь» развивают мощность Р = 55 000 кВт. Определить суточный расход 235U при работе ледокола на полную мощность, считая, что при делении 026U выделяется ДА = 3,2 • 10-11 Дж за один акт деления. К. п. д. двигателей т] принять равным 35 %.
ОТВЕТЫ И РЕШЕНИЯ
I. МЕХАНИКА
$ 1. Кинематика
1. | г | = | г2[ — | Г] | = 20 м; s = sx Ц- з2 = 220 м.
2. г = V+ (2R)2 = 50 км; счетчик показывал 40372,8 км.
4.	i = v s\ + s.2 = 400 м.
Pj — Р2
5.	Каждый корабль встретит в открытом море 23 корабля (рис. 347).
6	/ =___L___= 20 с х- ——— = 200 м. Во втором случае время до встре-
01 + О2 ’ О] + о2
чи уменьшится в 2 раза, а место встречи не изменится. Графическое решение задачи приведено на рис. 348.
304
7.	t = %V1 „ = 3 мин 45 с.
8.	vt = 12 км/ч; v2— 15 км/ч.
9.	t = -Ak_ = 75 с. ‘1 ------ »2
10.	f = ^/-+4Z1Za ~r (рис. 349).
11.	n = J2-=^ = 2. ‘1	*2
12.	Формулой можно пользоваться в случае, если зависимость скорости движения от времени — линейная функция. Из графика (рис. 350) следует vc =
Рис. 350
As 2 (з *1} ”н + ук „	„
= — --------------------------------Линейная
At t2 — ti	2
дающая таким свойством.
13.
функция — единственная, обла-
v2 = 54 км/ч; v2 = 36 км/ч.
3lV А , v^T+wr6K/c-
v = .- 3uit>2Ug^- ~ 38 к
Направление катера относительно воды должно быть перпендикулярным к берегу (рис. 351). Модуль этой скорости vr = V оа + и3. Время переправы /=1 =60 с. и
14.
15.
16.
17.	/=-!+-	.
“ \	У и3 — V3)
18.	а=11°29'; v = rijcosa— oacos 45° » 21,5 м/с.
19.	a) t0 = - ; б) = t0--1—г»; в) t2 = ...	.
'° f	° i _ /и. у1 /1 — (и/и)2
20.	Курс лодки должен составить угол в 39° с нормалью к берегам.
21.	Время полета в обоих случаях одинаково.
22.	х = 2ojl = 10 км.
23.	t =	, не зависит; I, = — YI3 + d3 + /.
305
24.	u2 = Vс, +1>2 = 100 км/ч; a = arcsin E? « 36°54',
25.	6 = arcsin cos a); t =-------------*S‘.n -—57 •
r	' x ’ t'o cos (a — p)
26.	/=^ + T* 2 = 0,8 4. v? + v22
27.	sa=Si — = 750 m. _________________________________________
28.	smIn = (BO - 40) )/ L±^? „ 850 m.
29.	a2l =	— 2u1z.'2cos (a2 — aj) ~ 3,2 м/с.
30. s = t — км; графики при-
Рис. 353
Рис. 352
2/
31.	t = — = 3 мин 20 с.
«1
32.	t = ^2—Ей . Координата места встречи х =?V1 ^2*—Ей. Расстояние между телами изменяется по закону Дх = (□! — t'2) t + -g- at2.
„„	.	,	. , 1	, о, — v2zt У(н2 — о>)2 — 2аха
33.	хх = vrt и х2 = х0 + и2/ + — at2; t = —---------------2—r v ag--------—----------a <
(t>a — oj2
2x0	'
35. a=— =0,1 м/с2.
”1
36. Sj = 4- at? = 30 m; As = 35х = 90 м; и = тфт = 12 м/с,
2	1	u *1 ~Г *2
3’-« = 2^^ = О.2«»/А
38. a=s7 « 2,81 м/с2; t = — « 26,6 с; v. = 4- = 37,5 м/с.
21	v	с t
39. Из графиков (рис. 353): 1) t = 18 с; х = 83 м; 2) vK = 8 м/с; 3) Si = 75 м.
306
40.	t = 40 с; x = 80 м; а = —0,1 м/с2. Графики приведены на рис. 354.
41.	и (/) = ^ =—30/4+30; а(/)=^£ = —120/3; х (т) = 24 м; V (т) = 0 и а (т) =—120 м/с2.
42.	См. рис. 355, а и б.
43.	См. рис. 356, а, б, в; v = — v0.
«,0.= /<ЦР»-5м/с.
45.	/3 — /.j + V1% (/2 — ^1)'
46.	Дхтах = 5 м в конце 8-й секунды.
47.	См. рис. 357 (а —график пути, б — ускорения).
48.	См. рис. 358 (а — график пути, б, в — ускорения).
307
49.	t»0 =	233 м/с.
V?.	1 I v V
s = 5g+28 V-g°) =39-2M-
3,7 м/с.
50.
51.
52.
/2(Wx-A)
/x—2 — 7 m; /2—3 — 3 m; Z^_4 — 3 m; /4-5	1 m.
Рис. 357
„ f ^о + 2£лл-”<> 1Л
DO. t —------------- « 1(J C.
8л
54. /1 = |/4--]/(a2)2+£/2=s3,5 c.
65. Спустя /1=1/ —- упадет половина осколков, а спустя /2=--------------------
упадут на землю все осколки.
56.	т = — — = 3 с; h = — ‘ф « 85 м. g 2	2g 8
57.	ft = Ь ai2t + а/х/, — 1 g/| = 101,6 м.
58.	va = gt « 15 м/с; ftmax = j gt2 « 11,2 м.
308
59.	Д/ « 0,59 с.
60.	ft = 6gt2«212 км.
61.	Два раза на высотах ht «17,1 м
и Л, « 27,9 м; при т =	——
2	F	2guj
« 10,41 с.
62.	ay = ajitga. 63. а = axsin а.
64.	а) / =	с; б) s = о0 «
« 26,7 м/с.
65.	t = 1/ —-— « 1,7 с. г acosa
66.	— tg2a. 67. v = gt 1^2 « 70,6 м/с.
^2
68.	и2 = 5,6 м/с.
69.	s = 1/2g cos60°t « 2,5 м. г stn260
70.	v = У (o0sina — gt)2 + (u0cosa)2 « 5 м/с.
VooT?	t>nSin2a
71.	a = arcsin « 20°30', 1 = --------------«
v0	g
I
72.	a = arctgy0 у
731 X° V 2(Й-Л) < "° < Х» V 2 (h - уг)
107 м.
75.	О третью.
76.	В обоих случаях первый камень полетит дальше.
ПОп
77.	S =-4 = 314 м2.
ГПаХ gi
gS1cosa	/gs2cosa
78‘ V	V 2tga	____
79. v = Vva + 2gH « 743 м/с; a = arctg « 57е.
80.
81.
82. I
v i ^Vgh и ------; tg a > -p^-
cos a	у 2v
Vp sin2a
x = 2.t0 — —-----; t)0 
i- / г
(i>osinao)
o0sina0 g
X 1 + 1/" 1 + 7—2gft	« 1,5 c.
V (voSinao)2J
83. u0TH = 2t.i(Jsina1 « 2,8 м/с.
84. и = /и2 + 4n2n2r2 . График зависимости v (г) показан на рис. 359; v% = = |<u2 4- 4л2л2/?2 « 2,2 м/с.
309
85.	® =	= 2 рад/с‘	861 п = 9^77 = 4,9 с-1- /<	2Л//
87.	Uj = 314 м/с; винтовая линия с шагом винта 1,34 м.
88.	9,.
89.	ш = ~ = 2,5 рад/с.	92- а = 4“2Я = 0.2 м/с2.
90.	R	v г =	  -- .	93. со = п. /4л2 + 1	3
§ 2. Динамика
94.	F = W-[(-2s/Z2) + Pgl ~ 4,5 Н.
cosa + psina
F	т
95.	а =----------= 2 м/с2; Т = F------------— = 0,8 Н. Во втором случае Т =
пг, + m2	mj + т2
д/	F
96.	а = k — = 0,5 м/с2.	97. х = я7 = 0,05 м.
т	2k
98.	F > pg (ffij + т2) = 59,3 Н.
99.	а = ^osa-p(^-fsina) ~ т
100. F12= | F = 34,5 Н; FM = 1 F = 23 Н; F3i = ~ F = 11,5 Н.
101.	9	9 F	у2 — Vl р = —	Д-Д = 0,01. mg	2gl
102. 103.	amax = MS = 0.8 g. T — -.1 (m0.+	. T _ £1"'2 + Fj (mt + ma)
104. 105. 106.	1	m, + m0 + m2 ’	2	mL + m0 + m2 s =	I2 « 2,8  IO'2/2. 2 (Л4 — m) См. рис. 360. Па	F ~ 10-* м/с2' 2)	0 5 м/с2-
	U a-“+M ~ 1U м/с , -1 ab	m --u,o м/с, aT M
= 0,25 м/с2. 107.	t = 1/ —— «2 с. ' а — pg 108.	F = nm(a+^); Ах, = 1т<а + ^. kln (—		1 bin a 109.	р = 	 , Д,			) = °-2- mg — kl„ (1—cosa! T ПО. Да, если спускать с ускорением, большим, чем a = g— — = 1,8 м/с2.	
111. 112.	л	aV Am = p —;— « 7,8 кг. g+ a /=~l/;mi+OT2?^0,21 c. Г (ma — m,}g
310
113.
114.
115.
116.
117.
т =	^131 н. t . 1 /27/ (mi + w2) t j
mi + m2 ~	'	(m2 — "h) g ~ ’
1 / (mg — 2FC\ h
v= I/ . —— «0,8 м/с-
'	2 (M + m) '
T = 2 mim2	= 2^aa,
«1 + «8	mj + ffia
2g (sj + s2)
1 = 1/ -^-rr = 0,28 c.
V g + F b tn
c.
F
118. a = 2gsina-= 2,5 м/с2.
Рис. 360	Рис. 361
125.
Г U2 — D1	1
F = ml ——---------f- g (sina + p.cosa)
« 6,4 • 103 H.
126. F = 2250 H.
127 n = 2^2g	• a =	. T = ^m^g _
1 4mr + m2 ’ 2 4mx + m2 ' 4т1аг2
«.о m2 —fflisina „ n. , „ „	„
128. a = —------*-----g — 3,92 м/с2; T =---------V-g (1 + sin a) « 17,6 H.
«1 + ^2	m1 + m26'' 1	1
Г • cos a (liim, -4- u,,m,l 1
129. a = g sina-----------I ^2 n 1
I	«1 + m2 J
130 т = OTlffl2g ^2 ~ C0SK = 2 3 H ml + m.i	’
311
131.	a) v = va— = 4,9 м/с; б) s = vat---------^-г.	= 0.
М + т ‘	" т + Л4 2
132.	а = g Зт2т3 — М («а + 4ffi3) _ (mt + т3) т2 — 2тгт2 (1 + р).
1 s mim2 4-т2т3 + 4mjm3 ’ 2 g - 1 - - 1 •..................... ’
а _ т2т3 — [(2 + р) та — 4т3] т1 .
3 ® mj/nj + т2т3 + 4т1т3
133.	N = mgcos2a; р > tga.
юл tga —р	tga + p
134.	g т—;—г2- < а < g	, или
Б 1 + ptga ь 1 — ptga
135 А — g (sina — pcosa) — т (2pcosa 4~ p2sina — sina) . ° M — msina (2pcosa-j-p2s>na— sina)
a = YA2sin2a + (g — Asina)2 p2.
136 A = e (mi + ^mi + S‘n a т1ГП2 (sln a ~ COS a* 	=
® /И (mx + ff*a) + 2mima + (mi s'n “— m2cosa)2 ’ aix ~
тгт2 + m2m3 + 4z?z1m3 m1OT2W3 (3 р)
тТт2 + rn3m3 + Ьт^тг
8 м/с2 < а < 12 м/с2.
т2 т,+ т2
X[g+A(cosa — sina)]; a11(=Asina; a2x=Acosa; a2!/= mt . .	,	.	8
-----г— (cos a — sin a). 772j -j-27 кг < fflj < 403 кг. a = —p-- g xs 5,27 м/с2. I + ps
/=<^^2Г-=0.45 c. 2 (2m! + m2) g .. F (m! +m2}t. 2т2т3 — 4 4/rai ~	_ m2 — 4mi
fll lOmj +' ma ’ °2 ~ 16m! + ma ^тй-хА? . .
В=-Г=1М/С-£sin a + a rt . /ла = tny 5-^—г—!— «2,5 кг. 2 ^sinp —a p (	u2\
F = m g cos a — n- • \	A /
145. F «= mg | cos a —	) as 1,14 • 106 H [a ~	*= 30°) .
\_____gRJ	\ 2nR /
146. n = Г 7й) ’ “ 1 -1 с’1. 2л/
137.
138.
139.
140.
141.
142.
143.
144.
147. F, = m
2800 H; F2 = m
4200 H.
gR — 140 м/с.
149.	F = 3mg « 30 H.
150.	T = 3mg cos a = 25,4 H.
151.	Ti = m IWl-g) « 8,58 H; T2 = m (4nWl + g) = 9,17 H.
152.	T = mg (3 — 2 cos a) « 20 H.
148. v
153. FH = mg ^3— 2 cos -y j «
154. N = Wlg (3^1 — ma) = 2
 103 H.
mx + m2
= o- m3g = 2 H.
312
4 (mA + mA) Z2sin2-y
155.	T = m2g 1 4----------—r,------2------
, Г2g (1 — cosa) (m.,/2—mJ,)
156.	t»2 = ‘2]/	-------~i ~ >-------если в положении устойчивого
у	+ m2/j
равновесия масса т1 находится выше оси вращения и и2 = /2 X
ч, . / 2g (• + cos a) (mA — mA)
X I / --------------------J---, если масса mt находится ниже оси вра-
у	А т2/2
щения.
157.	<р = ± arccos I (cos <р0 + У 3 + с о s2 <р0) | = ± a rccos 1
L □	J	®
158.	k = 4n2m (2«2 — /if) « 1,8 • 102 Н/м.
160.	^(H-2^2^
gR
161. f =	« 1,3 CM.
4л2л2
162.
a
y±R2 _ a2
0,05.
p C
0,24 m.
1	/	, di ,	1 Йп\
1 — cos ^arctg + arctg j
163.
164.
165.
„ 1Л & n c , m o iZ/cosa . _
v = I/ ---—;« 2,6 м/с; T =8 2л I/ -------« 1,7 c.
r cos a sin a	V g
ш “ 1/ t & « 3,4 рад/с-r I cos a r
166. T	2=Г11/-А_££^. r g — a cos cq
167. <o	l0 cos a + ^-
168. a	y2 = arctg^~28°.
169. h	= ^«8,4 см.
170. R	т1п = ^«147 M‘. a = arctg^«73o20'.
171. v,	nax = VpgR = 20 м/с = 72 км/ч.
172. a	y2 = arctg-«30“.
173. a	R (M + m) n < arctg--7J—- =	. Im	4
174. p	<v>+^.
175. 1)	1 Груз массой 2m ,будет опускаться вниз. 2) Равновесие не нарушится.
313
176. Возможны два случая: 1) <р — 0 и 2) ср = arccos —. Во втором случае _ a^R
равновесие будет устойчивым, если со < 1/
т. + Г2>/5,. 1.2 с.
178.	а) Скорости одинаковые, б) Скорость в конце пути 1 больше.
$ 3. Закон всемирного тяготения
179.	Т =	« 1,2 • Ю-з с.
Г VP
180.	Л « 0,41 /?3 « 2624 км.
А3
181-	£ = йо j =3.9 м/с2-
182.	Н = R (/£ — I) « 6,4 • 10е м.
183.	g = ~ npyR ss 0,015 м/с2.
О
184.	h = R3 J « 3,8 • 10е м.
(О_ R /2л\2	. . ,,
185.	g = — I. «1,1 м/с2.
т] \ t /
186.	Возможны два решения задачи: 1) при <ос > <о3 будет
2) при <ос < со3 будет
187.	R = 1Z= 2,5 • 10’ м. г 4л2
188.	k =---^—„=1,24.
YP — Злп2
189.	Т = 2л ]/— « 7150 с « 2 ч.
I».	(g)‘-3.8 '<>•».
191.	Л= 1Z1600 км-Г 4ла °
192.	v «=	1 /~ — ~ 39 км/с.
' у ГЮ
193.	Если бы не было сопротивления
ДИЯ.
194.	Уменьшить примерно в 2,4 раза.
195.	/ =	10.5 ч.
1/ g 2п
' п3гэ Т3
196.	М = — « 2 • 103“ кг, V
воздуха, то при выходе из жерла ору-
314
197.	gx = 7- 103g.
198.	/?ю = /?3 ]/	« 786 • 10» км.
199.
200.
201.
р — 3,03 • 103 кг/м3.
== 0,85.
\ 3/2
/ 3 \2
202.	F = I -j ) mg = 66 Н.
203.	Н = R3 (/2 — 1) « 2560 км.
204.	На 12 км/с.
205.	Д£ — mg (h1 — й2) = 3 • 10s Дж.
206.	При = 6 км/с корабль улетит, при упадет на Землю.
207.	v = ]/" £л/?л » 2,1 км/с.
v2 = 5 км/с и va = 4 км/с —
208. Н = R^
209. v = gR ~ 7,9 км/с.
210. В момент достижения центра Земли ракета имеет вторую космическую скорость, а вылетает из шахты с первой космической скоростью.
211. a) v = Иц = 11 км/с; б) v = Vj = 8 км/с.
212. С той же скоростью, 213. ц„ = /3^.
214. Звезды движутся mJ которых Г, =----=--
	mi ~г тг
х 1/	/3	
Г y(mi + m2)
; r2 = ml + mi 2
215.
по
и
с которой была запущена в туннель.
окружностям вокруг центра масс системы, радиусы
г2 = —-у----. Период обращения Т = 2л X
wii + т2
mJ . т2 + т2 ’
» _т ?(mi4-m2)	2
216. r 4r
217. <0 = /Т(/И1 + Л12)/?3; ах = ?^; a2 = y^.
2.9.
§ 4.	Статика
220.	arnln = arctgbz±lH-2 = 45°.
4	Z"2
221.	F = ~^mg tga.
222.	См. рис. 362.
315
223. a > arcctg
224.
225.
J /1 _ M 1
p2 \ m + M 2	/
sin 2q .
1 + 2 sin2 a ~	'
u (m + M) I si n a —% Л4 / cos a m cos a
226. T = 150 H; Fc 513 H; FB = 173 H.
227. a > arctg — = arctg 10 = 84°. 1*
228. M = 1 m = 25 кг; R, = i mg = 490 H; Ra = V3Mg = 419 H.
„„„ ,,	sin P	sin a ,	2 sin P
1 ~~ ms sin (a + P) ’	2 — sin (a + P) ’ g ~~ sin a sin (a + P)
Рис. 362
— ctga.
230.	T — mg x .COzsa—— .	Равновесие возможно, если
s 2 sin (a — P)
a > p.
oqi p [MgL + mg(2L — ZJJcosa,
231.	F = --------------------------.
232.	Л/j = 1 Mg + mg « 22 • IO® H; n2 =
2	L	§
= jMg + mg-~	17,3 • 103 H-
m2 + m2 — mi	m2 + mi — m2
233.	a = arccos----5; P = arccos---------------5.
2mma	2ттл
2M- H1
235. TAB = mig^-^ 122 H; ла sin a
m2 = mx
236. a = arcsin
mR
(m + M)(l + R)
/sin P cos a \ sin a
« 135 кг.
237. R = mg sin 30° = 9,8 H.
238. tg a = 2 tg P.
239.	m2 = ~ m, tg a.
д2
240.	a= arctg « 14°.
241.	F = Rctg a + i mg= 11,6 H. A
М21Г = ®^а)й12 4 H. m + M pmg
243. F =---;.
cos a + p sin a
«*• Ып-тггг-
-0A '
246.	F >---_ . a = arccos .
r + p /R2 — r2	R
3’6
247.
248.
249.
250.
251.
252.
^т+Л4_Л11 м + k2 т 2т] м ________mg______• д? ________________
1 cosa + ctg Р sin а '	2 sin Р ctg а + cos Р '
F = mg tg aj = 0,98 H. a = arctg ~ «31°.
a
% C ~r=-----------— 1,5 m.
У 2 (sin a + M cosa) , 1 «i+ 2"m X = I -:----;-
«i + m-i + m
= 0,27 м.
mg
/ 1 , , „ \ \ m2 / + /?а^ — mi 253. Правее центра стержня на х =--------------;----:---
н	«1 + тг + т3
254. х = / =----, = 0,01 м.
/?2 — г2 , а mg tg-y ---------------- = 102 Н/м.
=4,5 см.
255. k = -
I
х
256.
257.
258.
Сила упругости во всех стержнях равна 10 Н.
F = 25 mg ctg а.
tg Р = 3 tg а.
$ S. Закон сохранения импульса
259.	f = 2m^na = 15 н Д/
260.	F = 2pt)2Ssina= 86,4 Н.
261.	F =	= 80 Н.
262.	&р = т (vi + р2) = 0,18 кг • м/с; F = mg +	= 18,2 Н.
263.	F =	.
2в4. а)	= 58,8 кг/с; 6)	= 176,4 кг/с.
265.	Др = т + v2B = 10 кг • м/с.
„„	, т.п. — m,v ,
266.	v, = ——-----— = 1 м/с.
mi
267.	т2 =	= 120 кг.
S
268.	Д/= (т\~т11-м.
fill -f- /Mg ~р Л1
//g
269.	v =-------------г- .
/8.(1+^)
317
п-. mtVi cos a _ n, , 271. v =	----= °.07 м/с.
272.	v = t/j 4- Vi —-г—г? = 1,36 м/с. 1	2 m-j- M	'
273.	v = 10,4 м/с.
274.	При одновременном прыжке в одну
= 2 м/с; в раз-

ные стороны v = 0; при последовательных прыжках в одну сторону v = и х ,, / т , т \ п „ ,	т2и „ пс ,
X —пп + ля , г, = 2,2 м/с; в разные стороны v =	—--ту- = 0,25 м/с.
М-\-2т)	г	м (2т+М)
/Moi — mv,, ,
275.	v = —’ = о.З м/с; направление скорости не изменится.
и (mi — т2) | J_ (М + fflj + m2)J и
276. s = о
277. с. =---!—VM2v2 4- т2и2 + 2mMvu sin a.
1 т + М
, mNv, ,	M,v,,	,	, AiiVj
278. о, = nr,  4- t,—,  ; после обратного прыжка v, = -г-.—:-п 4
2 mN 4- Л12 ~ М2 4- mN	н н * 1 * * * У Ма 4- mN
, mNv2
mN 4-Mt '
279.	v, = —= 1,5 м/с. 1 fflj + m2
280.	о = 5 км/ч.
281.	ц =	+ м/с.
cos а
282. v, =  -^и1- = 0,9 м/с.
ЯЦ 4“ тч
„„„	Nil ..	ml ,,
283. s, =----;—г. = 44 м; s2 =----:—= И м.
1 т 4- М	2 т+ М
Og sin 2a
284. о2 = Юо0 cos a; х = 5,5------— .
285. 0с = (vi —	= 250 м/с.
286.
287.
288.
289.
то — пцгц _ о» =-------— = — 300 м/с.
2 т2
o0sin2a mo0Msina х = —------------------•
g	Mg
Вторая порция должна сгорать непосредственно за первой.
s3 = ^/~ sf 4- S2 = 5 км.
290. | Др | = 2mov sin a « 2,5 . 10~2з кг • м/с; F = /VAp = 2Nm0v sina = 0,25 Н. о2 sin 2a 1П
291. х —---------;-----21 *= 10 м.
g
у Оу 4-
292. и —-------------- 1,7 м/с; угол между скоростями и и ц равен а =
1 — п
= arctg —2 « 37°.
Vi _____________
1 Г a 2 J 2 2
У 4- mavt 293. v = ---------;--------- 10 м/с.
«! + m2
318
. fHlVl ------- /И, (2f„ + f 1) I
294.	| Ut | = — — Г„,2	= 1 м/с.
ml + m2 I
295.	v — j/ (f0 — fi)a у2 — 2fi (f0 — fi) cos 2ф ~ 720 м/с.
296.	P = 90° —a = 70°. 3
297.	M = g- m.
298.	o2 = 4fa + о 2— 4ffj cos a, P = arcsin /---------- U1 s‘n a ------------
\	4f2 + f2 — 4ffi cos a
$ 6. Работа и анергия. Закон сохранения анергии
299.	А == mh ^g + J = 15 510 Дж. ,22.	,22
/0,-01	\	/У2 — V1
800. А = mg[ —т------|- ps j те1,48• 10® Дж; N =mg —------1- ps
«1,3- 10е Вт.
301.	s =	; при р > tg а санки останутся на месте.
302.	о= р/"2 ^gft------те 11,5 м/с.
303.	А = mh № + gj = 1,35 • 10® Дж.
307. Fc
= 3 • 10» Н;
804. Е = ~ (oj + g2/2) = 302,6 Дж.
2 fn
305. h = -г =61,25 м.
4g
306. F = ,- ^?гт + (т + М) g = 125 720 Н.
(т + М) h	5
2ч t =	= 8 • 10-® с.
/2gft
308.	xmaj[ = М + V(МУ + 2йД/ = 20 см.
309.	= 980 кВт.
Т]*
310.	N =	100 % те 20,4 кВт.
Т]/
311.	//=^ = 5 м.
PgQ
т (о2 + 2gh — о?)
312.	Fc = —i—2-^----------= 3 • 10®
2/ .2	2
/У2 V1
313.	А = tn 2- — ~2 + £л
^fyfnax
«О
314.	Wmax
315.	h
Н.
• 10’
Дж.
4s2/n .nn D = —» 400 кВт
Г]/3
N = ^Ь-р =	= 200 кВт.
ср	|)fS
. „ ,-----------Г те 15 М.
2g (1 + р Ctg а)
319
316.	fmax = mg + v V km « 3,2 • 10s H.
>17. " tn
_	— 1^217.
2//gsin|-
=	« 6250 H.
2/ (m + M)
tn2 (и ul2
319.	Нет, так как пройденный ящиком путь / =—2p,gAl2	= 0,125 м мень“
ше расстояния d = 0,5 м до стены.
2
VQ
318. F,
320.	Пуля
пробьет «= =--------------„ = 2,8
v0 - О,64о02
досок, т. е. застрянет в третьей
доске.
321.
2 ад
322.
fl —- __________
~ 2g (т± + m2) '
т.щ (m, -4- m„)
A =	\	----- ъ 390 Дж.
323.
324.
%tn2
ТП 2и2 s =	« 0,23 м.
2pgAl2
А =	----ч = 48 Дж.
2 (mj + m2)
325.
А =
к* 347 Дж.
326.
327.
50 м.
328.
tl2 расстояние s •
329.
330.
331.
m -4- M -./~z—i—:— v =----!--- у 2gl sin a.
m cos a и
2mg
Состав и дрезина проедут одинаковое
V,V, п va = —=—— 2 cos a.
Vi + V.2
„ pgVH
Мощность должна быть ™mln > —7— Т]Г
A = ml [a + g (sin a + Ц cos a)] = 910,5 кДж.
N = mg (sin a + p cos a) 0 « 2mgv -y « 11,8 кВт.
, где г] — к. и. д. двигателя.
332.
h
333. Лт1п = mg (h + р /Р— Л2) = 5 • 103 дж;	« 0,8.
334 /_———= 10 m. Это расстояние не зависит от коэффициента трения М—т
_____	m.,v„ — mjV,
335.	т„ = ——------— = IO2 кг.
0 Vi — v2_________________
336.	о= У 2gH (1	~ 14 м/с.
ЧЧ7 V	{{H-hypS2
337.	N _	+ 2s^s .
338.	В 8 раз большую.
339.	k = 2mg д,^А/' = 30 Н/м.
ДР
320
340.
_ mg 1 Г m2ga 2mgh max k T у k2 t- k •
341 * (m + M)g , -l/(m + M)2g2 34L *max —--k----Г V ---fe2-
1 /2Е (k, +fe2)
т- ’-V- мГ 
2m2g/i k (tn + M) *
F2m22x0
345. E„ =	- .yg = 0,1 Дж.
п 2 (mj + m2)2F„
4m?/
340.	h = ----:---rs =12,5 cm.
(«! + m2)2
2mjm2g/ sin
347.	Q =---------:----- ~ 6,9 Дж.
Mi + m2
348.	^ = 4^-= 0,36 = 36 %.
E \v vl /
349.	— = 1 + 2 cos a.
m2
 m.
350.	amav = arcsin — . max	m2
351.	Возможны два случая: а) налетающий шар массой т продолжает двигаться в прежнем направлении. Тогда а = -1 р +	• Этот случай возмо-
жен, если М « т с ЗЛ1; при этом < а < 1. б) Шар после удара отлетает назад. Тогда а = -J- (1 + 9	) . Этот случай возможен при 0 < m <	Л4;
4 \ /И /	о
1
при этом — < а < 1.
_ , / gfi (2М2+ 2тМ + т2) + т2^
OoZ« и — I /	________ .	.		—.1
у	т + М
353.	Е = mgl (1 — cos а) » 0,3 Дж; v = K2g/ (1 — cos а) « 2,4 м/с.
354.	u0 = V5g/?.
5
355.	йт1п = -2- R = Ю м.
5
356.	Н = -^ R от основания. о
357.	А = mg (н — -|- У? ) = 2 • 10~2 Дж.
358.	v = ]/и0 р —	« 2 м/с-
359.	CD = [п„ - 2gR (1 - cos а)]	« 6,9 м.
321
о2 — vr
360.	Н = h Я-------— = ft + I slns a ~ 22 см (рис. 363).
361.	ft = Н Гд .
362. Л = H t.g“ , - = 0,8 m. tg« + H
363. / = 2 У (Я — ft) ft (n + 1) = 1,6 m.
364. 7\ = mg и T2 = mg. 10	IO
„	т
; из условия пробивания пулей шара следует, что < < 1, поэтому 0,5 < k < 0,75, 366. Р = -2 +-*2 = 0,6.
ftj — ft2
367. А= (т— р V) g (Н— 0,5 frv) + + (т — 0,5рV) g fy~V + mg(h — — 0,5 jZv) « 147 Дж. При выходе куба из воды значение силы Архимеда
368.
365. k =
§ 7. Механика жидкостей и газов
369.
= 10 cm.
изменяется от pgV до 0.
Лт1п = PSSA2 = 200 Дж-Ьх = Xj (У 1 + n2 — 1) =
Р — Ра
370.	АЛ = Л -5---2 = 0,17 см.
Рр-Рб
371.	р = 3P1Pag/t « 7 . IO2 Па.
Pi + 2р2
372.	ft = 49,5 м.
373.	Fa = 4а2	+ pg « 6 . 10’ Н
ное атмосферное давление).
374.	F = ПРе/>г (3i? ~ А tg а) «29,2 Н.
3 cos а ft2
375.	F = pcpS = pgd j = 1,25 • 10» H.
(здесь Ро = 1,01 • 106 Па — нормаль-
376. Aft =	/1 —	= 4 см.
Рм* Рв)
4 Я8(Р1 —4-Ра)
377. V = Д- п-i65,7 см2.
3	Р,
379.	т = -5- [Р1^ - (Р1 - р2)] « 90 г.
2/ЗлгЗр — т, , _с
380.	т2 =	;----- = 1,75 кг.
2	1 ~ Р1/Ра
322
381.	Кубик утонет, так как его масса (8,9 кг) больше массы вытесненной воды (8 кг).
/ЛРВ \
383.	F'= mg ( —2 — 1 = 39,2 Н.
" \4РП )
384.	h = d (— — I 'i = 1,8 см.
\Рв /
385.	у =	__ -1 « 15 - 10-» м».
п Рв Рм .	, Рм
«1 — («I — т2) -
386.	тг = р.----------------5 и 35 г и т = 210,6 г.
с ° Рс “ Рв nd2Ah р,р, п _ 387. т ==—j------^-2- « 2,5 кг.
4 Pi — р2 м
388,	Л — —— = 5 см; т — p„ld (h, — Л2) — М = 6,8 • Ю3 кг = 6,8 т. и рв1а	в	'
389,	р — ~ р0. Тело совершает незатухающие (но не гармонические!) колебания о
между начальным положением и точкой максимального погружения в жидкость.
390.	Т =	<Р* - Pi) ~ °-107 Н-
О
391.	ДЕп = ш§Я (-1—1 ) = —0,49 Дж.
'Рз	nd2
392.	/' = [pgA4-p0g(//-A) + p]^-«l,93- 10» Н.
393.	Льдина не удержит человека.
394.	т = рв5й = 3,24 • 10» кг = 3240 т.
395.	т =	~ 63 г.
О
396.	т = РлРв/|- « 3,5 кг.
Рв“ Рл
397.	V = V —= 1545 м»; Уп = 1350 м«. л Рв“ Рл
398.	V, = Р'! ~ Рз V, = 1,7 м3.
Рз — Pi nd2l
399.	М = (рв - р) ~ 74 кг.
400.	рА = р + (рв/7 — рмй) g « 1,22  10» Па.
401.	р = р0 4- pg (Н — й) « 2 • 10» Па.
402.	А = p0S (И - й) + p0S = 1,5  10» Дж.
NIS2 ,с
403.	п = п —тгЯ- = 45.
1 mgSih
404.	Изменение показания динамометра связано с действием на гирю выталкивающей силы, которая равна весу воды в объеме погруженной части гири: AF
&F = pgSAh, откуда S =	6,25 см2.
405.	V >	« 73 м».
(Р — Pi) g*2 — 2Р1й
323
у 4л/?3 (2р — р )
406.	-^- = 1 +----------------
х	ЗМ
407.	р = -	= 2,5 • IO3 кг/м3.
rrti — т2
408.	р = рв = 0,75 • Ю3 кг/м3.
II	. МОЛЕКУЛЯРНАЯ ФИЗИКА. ТЕПЛОВЫЕ ЯВЛЕНИЯ
§	8. Основы молекулярно-кинетической теории. Законы идеального газа
409.	N = Na ~ 3,3 • 10».
410. v =	~ 11,1 моль;
М
N = vNA ~ 6,68  104
pV
411. v = яз 0,4 моля. а/
412. М=	1,45 • 104
А
413.	Д'= Д',^^2,7  1025; та = 1,66 • 10-2’Л1г « 3,3  Ю"3’ кг. A	«	Г
414.	Д' = Д'А Ра\(^а ~ Г1).	415, .V = pV^ = 2,4 • 10«.
1\1 1/ 2
416.	т, =	~ 0,295 кг; т2 ~ 0,96 кг; т3 як 0,017 кг; Д\ •= Л'А^- ~
я» 0,55  1026; N2 2,1 • 1028; N3 яа 0,26 • 104
417.	ра = ftpg Z1 + /а = 9,97 • 10* Па. И *2
418.
h -1 - £ + /(й +1 - h^+ 4ph/pg
&h =----------—----------~ 42,5 см.
419.
л0 + /-Кл02 + /г
2
яа 252 мм.
421. ДТ = — Т Pg/ =s —11 К. 422. I = hp + P—.
Р + pg//2	Р
423. h= <oPa/Pg~^ft° + / — — — ho = 40 мм.
Ip — l	Pg
4M-“-Ki®('-r)“25w/c'
425. T = — 2pliTl.— x 320 K.
(p + Pg-y) li
Св. Д(д«-ц[1-0.M
324
«7. .= ]/|(/+Л + Л.), + И-1(	l + h + ^] Pg 1	~ 12 см.
428. Т. = Т1^- = 633,6 К, или = 360,6	°C.	
Pi		
429.	= 1 _ Ъ = о 04 = 40 %.		
Р2	Т2		
440 Т	_ ОПП V		
43°’ (I -а)(I + Р)- I		
431. р2 < Р1.	433. Т =	:	= 300 к.	
432 о — PlV1 + PaVa	434 Т — — 432‘ р~ v1 + v2 	Г	= 350 К.	
435. р =	= 180 кг/м3; 7\ =-^ Т = I\J	р	342 К, или	ti = 69 °C,
436. рг = Р1^-^ 19 - 10» Па.		
437. 1о =ъ1~ °-06 м; 1н =	~ °.94 и« 17	п« 17	м.	
438. р = j? Pf* ~ 6,8 - 10» Па. Г1 + У2		
439 Л Н	125 м		
Л- fe3	l?3pg -].“5м.		
440. h = М27? Т Г1) « 8,4 м.		
РвЯЛ		
441. г2 = гг 1/	—г	1,4 см. 2	Г ра + pgh Л		
PoVMn — mRT		
442‘ р=	7мв	= 3-2' 105 Па-		
443. F = Ро (V1 ~ Va) 5 = 400 Н.		
v2		
444. v=	« 1,52 м/с.		
MpiS		
A	MV 1 О,	П„ \ 445. Am =	-£г	1 — 3 г.		
R \ Тх Т2 ]		
446 р _		
Ро Т„ + Т		
447. р =
448. См.
449. См.
Т (рУг Vi + V2 Тх рис. 364. рис. 365, а, б.
' Т I ' 1 2 /
450. Пропорционально корню квадратному из температуры:
И
где а — тангенс угла наклона прямой АВ к оси объема.
451.	Это изобарический процесс; объем изменялся пропорционально температуре.
452.	См. рис. 366.
Т
453.	V . = /? — , VB = V..
А Ра в л
325
454. VC>VA>VB, РА>Рс>Рв-
459. См. рис. 370.
460. р2 = 2pi = 8 • 10* Па.
461	= т/? 41 _	= 360 кг.
4bl Mgh g
Рис. 367	Рис. 369
326
463.	Г, = -	PgV1 		= 254,8 К, где Vo = 22,4 л. fPlLi-E? о k	vj к pVMH	44н
464.	т™- Rt' «44Н;~°’О76КГ 76" rljVJ
465.	ра = Гя(-^-^)~3,6.10‘Па.
466.	т (44 п — 244н ) RT Р^=	МОМ,У	-2.1-Ю‘Па.
467.	Г2 = -(pV/J?)	« 310 к. ‘	тг!М
468.	pRT рМ2 . .	4	pRT — рМ, 4 2.1Q2& м-з "N ЙГ (442- 442) "3,5	’ Н *Г(442 - 44,)~4’2 ‘°
469.	Р =^Г (-Тг + -%г} ” *>33  10’ Па. V \ 44х 1 М.2)
470.	P<s  triyT2/MiT 1 + /и2/Л4а _ 12  р, — т21Мг + m2IMt	7 ~
471.	► V = ^-j- = 0,1 м3. 0,1 р
472.	V Т ла = пг ~ =г « 2,3 моля. '1 •* 2
473.	Vo АТ1 х~ S 2Т0 + АГ ~ 1 СМ‘
474.	D _ D	~ 1 1 - 10® Па p-p»r(v1r2 + V2r1j~1, 4
475.	з ’
476.	Pi = Ро V- -у?- « 0,5 кг/м3. Ро 1
477.	Р, у п= е .- = 333. Количество заполненных трубок лимитируется гелием 5/6роп)
478.	р =	+	+	= 3 . 10з Па. чт •'гт м
479.	_! _±_ тп8\ м АК т~ \Ро+ S / R АГ '
480. F = ,.P°hS.^ = 60 H. Vo — hS
481.
482.
483.
484.
485.
= pmToPiPa--------	0 48 кг/мз.
PaT (miP2 + ^aPi)
. 2pS (n — 1)	„
a = g stn a H----= 25 м/с3.
s 1 mn (2 — n)
fti = 2,4-
AV 44,-442
V ~Мг + М,‘
V1= V —^ = 0,42 л; Va= V-Vt = 0,58 л.
«Ой p + T’a) Т’з _ 16 „по 48в> B 2T\Tt ~ 15 ра3’
327
487.
488.
489.
тг = Т1 « 563 К, или /° = 290 °C. Pin
а = —- = 3 • Ю-a м.
ml + tn2
Лпах = «^ ДГ^- = 26 Н-
490.
nR
491.
= 150 К; /а = -123 °C.
492.
493.
494.
495.
т\ = 7\
Ек=^рУ = 1500 Дж.
Т mRT	-г
р = р“77+л17 ~'-9- 10 Па-
/4йа,г = 4-
Л =	« 1.04 м.
М ,, , 2£ Mg + —
496.
497.
498.
499.
500.
501.
У] _ ты2 г У 2 Ро$
Р2 = (Pi — Р£Л1) ^+Т—У + PS^2 = 9." • >04 Па.
Ра = РА + лУ\~Р'л « Ь01 • Ю6 Па-
а	АЛ —Л2Р1/Р£
__	4
П~ lg (1 + WV) ‘
Vc	1
V =-----!----~ 0,37.
Уц Iх Po/P— 1
VTr.
„ = 9^ « 7600.
1,28.
502.
§ 9. Внутренняя энергия и механическая работа
Ci/nj (0° — О + m2X + C1m2 (0° — /„) mn = -------------------—----—-------------« 115 г.
— 0°)
503.
504.
505.
506.
CimiCjfflJ/j —/j)
Q =-.------------------ « 5850 Дж.
^lml + c2m2
0,0005 кг льда, ‘0,0045 кг воды и твердый свинец при температуре 0 °C.
с2т, (/2 — 6°) С
= _2_2Д2--------------«2,17 - Юз Дж/(кг • К).
tn, (0° - /°)
__	+ С2т2 (^2--- ^1) — (С + С)Ш1) (t'S — ^1) ~ Q 364 кг
tn <=
607.
_ 1(с1от1 + с2та) h + (С2^2 — тз1 14 ор
с,т, + c2 (m2 + m3)
328
608. 0» = (^ + ^'° + ^з _ 2<) ос Cj/Wj Н" ^2^2 + ^8^8
сгтг(/г — t°3) —	— li)
------------------------------« 1,6 • 10я Дж/с.
609.
510.
’ =------------i-------
о /8(^ + Й-^з + О —	О , ,О л ,О	ОХ,Х kJ.
'1 + 'з-2/а
,,, AQ cimi ((з — z°) + c*mi (*з — £)	, „
511. — --------------------------=1 Дж/с.
512.	=	^0025 кг
513.
514.
515.
сх = 564 Дж/(кг • К) в первом опыте и сх = €03 Д ж/(кг • К) во /1 *о \	Pto
т = Р I —------г = 300 г и /п = — = 700 г.
в \са л /	Л л
<2—(Cj/nj + cama) (/“ — /°) k = ----------7------------- «й 0,01.
Лт2
втором-
516.	= у. [X + с (t° — /„)) ~ 4,4 кВт.
517.
т [св (/а - 0) + Л] тх =-------------з---~ 740 г.
сл (0- /х)
сва°-^)
518. Испарится --------- « 1,8 % перегретой воды, независимо от
чества долитой воды, поскольку температура последней не изменяется. со (^к — ^о)
коли*
519.
6,5 кг.
520.
521.
522.
т = ---о---
*+^пл-0
Т=-2-^Г ~ 273 К1 или *° = ° °с-
Хт т, =-5-5- = I час 46 мин.
cpV (/i — /а) та cpWa т =---—-------— »= 600 г.
Л
523.
< =	Д/° « 1340 с.
N
524.	Т = Ти —	« 333 К, или Г = 60 °C.
к ГЦ
525.	v —	« 9,7 м/с.
x\mq
526.	m^»UC(100°-/°) + rU61 f
П?
N<i
527.	m = — « 0,08 кг.
T]?O
52R m рУМСГг — Т^щ ~е2кг
528’ mx=----RT^cbT--------62 КГ>
Nl
529.	m = — «1,3- 10» кг.
12 7-253
329
530.	L = — = 2,24- 10е Дж/кг. T|
cm(i°-t°)
531.	mx =---Ц« 1,6 r.
532.	_ 2ст2 А/0 _ ntigh
533.	/ 2 (с (/“ — /’) +А] v - |/	л	« 420 м/с-
534.	АГ= ^(1-.е)/Ю0°-=214 К. 2с
535.	х~	2Л	~°’35-
536.	Д/° = -£ [*1 - ”2 ~ тг(У1 “ "г)’] 216 к-
537.	/ 2h (Vo — 2с А/°) 1 = 1/ ——		 « 286 м. 2	2
538.	_ т^ mv! _ {тл + mV1)2 _ У ~ 2	1	2	2 (т„ + т) 4U Д
539.	Г	/	si VI (уо cos a)’+ uosina	« AT = i	Ц;	Ojcosa/J 8 I0-3 K. 2c
54 0. %1п = V 2 [сл (/°л - Г) + A + cB (t°K - t°) + г] « 2,5 - 10» м/с.
541.	Q =	=15Дж.
2 (m, + m2)
542.	tnx = mx r-	« 45 r.
x 4(1—ctg a)
543.	A = т.Ро.£Т « 8,5 • 104 Дж.
AT
544.	AT = —Ц- = 5,76 K.
545.	A = ~RAT.
546.	A =	~ 3,46 Дж.
547.	A = (p„S + mg) = 30 Дж.
548.	At/ = Q = ДГ « 155 Дж.
549.	AU = Q — nR (T2 — T2) = 207 Дж.
550.	AU = Q - (p0 + V у = 33 Дж.
551.	Q = Л + 1) ~ 3,5 • 10’ Дж.
V 0
552.	1 « 3 — объем газа увеличился приблизительно в 3 раза.
^0 А'О
330
<2 = А.
45 Дж.
106 Дж; б) MJ = 3 (р2Г2- pjVj) = 0;
/п PoVo.	. (Т, Т-Л	.
553.	А = 57 (Pi - Ps) Hr----И • где р„, То и Vo - давление, тем-
М 1 о	\ Рг Pi /
пература и объем 1 моля газа в нормальных условиях. Q = А.
S64,	т, - т + ~ - 2S4 К; V,--------------т1>Т‘	» 0.024 «».
•» л-ТГ *(НЙ)<v-
556. А = -^(7'2-71)(^-1].
657. Л=^-/?Ь(^-И2).
-Rfi д_РгУо (Тз. ^1^ /у _IZ 1 ~
559.	<2 = cvm (Т3 - Г4) +	/?7Х (1
водорода при постоянном объеме.
560.	а) А =	(V2 - VJ = 4,5 .
в) Q = А + MJ = А.
561.	А = R (7, - Т2 + Т3 - Т4).
562.	<2 = Cptn (Л — тч + Т3 — Л) -|~ cvm (72 — Т3) = срт(7! — TJ — -^-R X X (Т2 — Т3), где ср и cv — удельные теплоемкости гелия при постоянном давлении и объеме.
563.	На участке 1—2 совершаемая газом работа отрицательна (А<0), и от газа отводится некоторое количество теплоты окружающей среде (<2<0). Аналогично на участке 2—3 А > 0 и Q > 0; на участке 3—1 А — 0 и Q < 0; в процессе 1—2—3—1 А>0 и <2 > 0.
564.	U2 — U1 — cvm (Т4 — Tj), где Ut—внутренняя энергия в состоянии D, a Ut—внутренняя энергия в состоянии 4.
565.	А = Р1 (V2 -	+	(V» - vj) + ??	- t£.
566.	А =(р2 — pj (1/2 — Vj).
567.	А = (р2 — pi) (V2 — Vj).
568-	/.тах=^(1-^=М5м.
569.	Г2<Л^-л.
570‘ с = pv\f~2H-T^ = 4,2 ’103 Дж/(кг ’ К)> 0,90	1 о
".-'-т^-т + т’-0.55-
S72.
573. N =	« 54 кВт.
71 з
12*	331
— I , где Су — удельная теплоемкость
$ 10. Упруги* и тепловые свойстве твердых тел и жидкостей
674. а= 1Л  Ю’ Па; ДЛ= 1,4 • 10~“ м. ltd3	Е
Fh
575.	AZ =	" -g- « 2,02 см.
576.	х «а 0,3 м.
577	г 4s (Л + Z.J ЕгЕг lOOfEJj + Е2/2)'
578.	Еп = AZ2 = 7.3 Дж.
579.	А= 1 £SZp2 = 120 Дж.
580.	Уменьшение потенциальной энергии груза mg Ы. Увеличение по-Д / 2
тенциальной энергии проволоки ДЕ2 = —%— . Поскольку Л Д/= mg, то A£2=
Д/ 1 АЕ, „	Д/
= mg £- = -% Д£\. Часть энергии mg идет на увеличение внутренней энергии проволоки.
581.	F =	= 21,6 • 10а Н; Fmln = onpS = 78,5 . 103 Н.
582.	F= ^s= 1,6- 10s Н.
cpl
583.	о = £0 (Т2 — Л) ss 4,4  10’ Па.
Z,
584.	ра = рх —3- = 7,5 • 10’ Па. ti
585.	6 = 1000 AZ° = 0,026 %.
586.	t° = 91 °C.
587.	An = n д I-!—г-------3) ss 9,6 оборота.
2л/?0^+р/°	1+р,°/
5Ю.,  [,_(22±й+>^№№>.у]=з.с.
Р L \	1 + АТг/Д7\	/.
589,	R= (02 —0J '
590.	а = 30 + m о W1 - •
m (t° — t°)
591.	Д/i = ftj	“ hi A/° (“ - 2₽> = 5’5 • 10'4 M-
1 -f- 2p ill
§ 11.	Свойства паров
„	.	. <Pi At V / p! p2 \	_
592.	Роса появится, выделится Д/n = ,л- n  —-=-------v~l = о г.
iuu% к \ J 1	* а /
593.	m=(-^7-p2 Sft=s 1,6 - 10’ кг. \ 1 Uu 7Q	/
р„VM [ <р 1 \
594.	дт = ._	ss4,7 г.
332
695. Температура должна уменьшиться на А/0 = 8,3 °C.
596. р = рв + рп = J\lB^p — — рнj 4-Л1п — pHj , тп =	— рн.
б97-Д/п=тооЧ _ 4?)=4,6 кг-
598.	= 45 °C.
599.	р -	= 3573 Па.
V
600.	Am = рн (1 — ф) « 0,46 кг.
601.	Сухой комнатный воздух содержит больше водяных паров
= 0,0052 кг/м3 I , чем сырой наружный воздух

602.	р = Ро -X- + р = 1,4  106 Па.
* о
603.	фа = ф1£ф~30 %.
P02l 1
fi04 Am =	_ P*MV	TlPi
RT\ • 100 % RT2 R \ 7, • 100 % = P-^? • 100 % ~ 30 %.
P$* 2
Ф1Рн1
Pl 100 %
<P =
--^-О,! кг;
III. ЭЛЕКТРОДИНАМИКА
§ 12.	Электрические заряды и электрическое поле
605.	q = у 4neor2mg tg = 7,1 • 10~8 Кл.
606.	Т = S*n а ~ 3,5 • 10 4 Н; равновесие безразличное.
607.	q = 21 sin а V2neomg tg а cos а га 2,2 • 10-8 Кл.
608.	<?=(! +	) (/ + и + г,) 1/ 4л80й (/ — /0) — « 2,1 • 10"» Кл.
\ Г1 /	'	Г2

609. Ес —
16
В/м.
610. А = 4лео<Д2^3	8 . 10-6 Кл/кг
т Q
вп. е=^_1^Ш2=1,6.
(?1 + <7г)2	г1Г.,
612.	q = -- : «= 5,4 • 10-’ Кл.
aU (’	/5 )
613.	х = —у'	R = 1 м от заряда q0 =	| q21 = 4  10"8 Кл
I 42 1 — I 41 1	К
614.	Г=	га 1,7 М.
Vk — 1
ззз
815. Ol = e0 (ЁА + EB)=3,54- IO'8 Кл/м2; o2 = 80 (Еа-Ел)=-1,77-10~8 Кл/м».
616. Е = lgll2- + It?alj- = 990 В/м, 4neori 4ne0r2
точки до зарядов q± и q2 соответственно.
6|7. х — ..—т^-
mgd — q£
где и — расстояния от искомой
618.	^fl = -g-(^-vi) = -22,7 В.
qu
619.	m = —г---.. т 
(k — 1) gd
620.	E = ~-----^-«5,75- IO” в/м.
4 л e0 r1
621.	Если незаряженная сфера находится внутри заряженной.
622.	Пылинки, осевшие на поверхность заряженного проводника, заряжаются. При этом напряженность поля в не-
посредственной близости от заряженной нылинки достигает большого значения (как известно, напряженность поля пропорциональна г-2). Под действием этого поля ионы воздуха, заряд которых противоположен знаку заряда проводника, интенсивно притягиваются к проводнику и разряжают его.
623.	Задачу можно решить на основе принципа суперпозиции полей. Получение вектора напряженности результирующего поля по известным напряженностям полей, создаваемых заряженными плоскостями, проиллюстрировано на рис. 371 и 372 для всех четырех областей 1, II, III и IV, на которые де-
лят пространство пересекающиеся заряженные плоскости. Искомая картина силовых линий электрического поля, создаваемого этими заряженными плоскостями, приведена на рис. 373. Легко видеть, что поля различны и однородны в каждой из областей I—IV.
624.	о=1/ 4g-«4- 10’ м/с.
У 2пт
625.
dv'2 sin2 а
2 (е/т) U
« 0,17 см.
334
626. Потенциал и напряженность электрического поля, ной сферой, 0
создаваемого заряжен-
D 2 ’
ф(И =
2ne0D Q  4ле0г
О, 0<
£(') =
4пе0г2
D
2 < . _О ' 2 ’ D
Графики ср (г) и Е (г) представлены на рис. 374, а, D\
-g-1 равен потенциалу
б. Потенциал в любой
любой точки на сфере
точке внутри сферы ср (г)
тДтг= 1620 В, 2ле0£)
а потенциал в точке, удаленной
сферы на расстояние d = 0,25 м, составляет ср d
627.	ср = 60 В.
628.	См. рис. 375.
629.	Увеличится в в раз.
630.	ф = Е3г3 « 8,32 • 108 В. Поле Земли не опасно для человека, поскольку тело человека является проводником. Под действием Земли электрические заряды распределяются по телу так, что результирующее поле внутри тела становится равным нулю.
631.	Уе = 4лед/?ср _ ] g . |q10 элекТр0Н0В_
от поверхности
__ _____Ф_______ 970 о - 2лв0 (b+2d)
632.
4,4 • 10» м/с.
633.
634.
635.
636.
637.
638.
2 Vn = dH5 105 в-
С= 355^- ''75  1<Г’
Q = 8л/? /в„//?Д/? ~ 2,1  IO”2 Кл.
О2
F =	~ 3,5  10-’ Н.
32ле0/?3
R — r Фг=Т<Р-
Q2 = _Q! _ q3 -35,4  10-’» Кл. “а
Q
Ф
v
335
639.	£ = ^(Ра-Рм)~ 1.2 - 10- В/м.
640.	£ = ^(р_рм)~4. Ю4 в/м.
641.	(р-р )^й=	_а Кд oh
642.	<р' = 4<р = 800 В.
643.	А =	~ 9 • IO-3 Дж. 2ле0г
644.	у = 1/2 «!_(!_	а = -^-. г	\ г	r0 J	4леого
645.	ut = (	) и ~ 47 В; и2 =	,е^2 . и ~ 253 В. \ ead1 +	)	2	+ 6jd2
646.	в /. E \	2 P *->( U"3’ c,c,	c.c., [ c, \
647.	^Ci + cV Ed)-°’ ^-сАсДи + с[ЕЬ2-^^
648.	cx = -?—c, X E— 1
649. и _	CjC^	_	C2CaS - d2 -?3- C1 Ci + CiCs + CaCi . L'l Ctc2 + CLC3 + C2C3 • САСЯ^	,,	CiC^
2 CjCa + C.Cj + CaC» ’	3 CjC2 + CiC3 + C2C3
650.	5 мкФ < Собщ < 400 мкФ.	654. W' = CU2.
651.	e = U1~U2 = 6.	655. Д<7 =	<7=1O-8 Кл. Ui	E
652.	Уменьшится в р^З раз.	656. U =	
653.	Л = - CU2.	657. Е = т	• 2	<7
658.	£ = f/U)’-8’-’7.5 В*
659.	q = e(leSt7	9,7 • 10"6 Кл. /|/2
660.	Фо=4011 л+tpc)=70 в>
661- <4,in
2dU
I
662.	Уменьшится на Д1Р = 1Р0 11 — -
663.	F =-r^-r = 6  10» H. d — di
664’	= d - 2d2 - d,. - 2d2/e Un'
665.	Пробьется в воздушной части.
666.	U = d |/ ^=8  IO2 в.
336
1	4
687. Q =	<? + 4ле0/?$.	668. Q = 8ле0/?$—=- ?•
О	м
fififi Р - I	I Я* Г
I dx + d21 28qS di + d2
1	2
670. Q = dF. 671. 4~-C$ = 4 10~° Кл. 672. При <$2 = 0.
™-uAb^'R-
§ 13.	Постоянный электрический ток
6751 t/1“S1Sa+ 5^ +S.253 6 B’ 2
l, _ S'S*U = о В "’"ЗД+ЗД + ад
676.
S^U
S'jS2 SjS3 + SaS3
= 3 B;
677.
678.
I*.
^2" 5,6 A.
4p/
71 (h	= 0,56 A.
2 (It +
U(Rb-Ra)
3~(u-ira)Rb 2916 a-
679.	R = —^—r = 0,025 Ом. n — I
680.	R = P-~-
681.	a) Rx = 1,5/?; 6) Rx = R.
7
682.	/?Л0 = ГЁ/? = 7 Ом.
683.	В узком сечении напряженность больше, причем во столько раз, во сколько площадь узкого сечения меньше, чем площадь широкого.
684.	г = —= 1 Ом.
/3
4
685.	Q = — g- CU при условии, что все конденсаторы были не заряжены до подачи напряжения U.
688. d= |/ - 2Р</тл- °.52 мм-т л (й — !г)
689. u = ^ + R)+^i.
Г1 + т2 Ч-- R
690. r= j-— R = 40 Ом.
691 1 —	4-	, R (ri + г2) + rir2 '	« 0,83 А.
692. R	« 2,3 Ом.
Со
337
893. 7 = J-Ц.—- 1,5 А, ft — I
894. Rx = 1 [R, + R3 + /(R, + R3)* + 4R2 (R. + R3)l-
895. UAB =lRt = 3,7 B, UBC = -S 4- !R2 = 0,6 B, UAC=l(R1+R^~ — Й = 4,3 B.
696. Может.
RjRuCS
697. 91 -	- g3 - 94 - 2[{Ri +??2) r + RiRz] •
698.
699.
700.
Д? = (R^R)	=25-8 • 10*‘ Кл-
& R's'
-TWF^-S“'
701.
702.
703.
704.
705.
~b <2/^1 g _ 8 p ^ = 0,04 Дж.
U1 (Cj/C2+1) (r2/n + l)
0 =	R]
V1 C1 + C2R1 + R2
Q^C1S1-|-C232 = 5- IO'5 Кл.
_____________RjR^S_____________ _ __s „
q ' (R1 + Rs + Rs) + Я1 (Я2 + Ra)
q = 4CS.
706. 91 = ± [(9„ + 2C6) - J/90 + 4C»S2];
92=4-2cs)++4c2fi2i-
§ 14. Электрический ток в вакууме, металлах, электролитах и газах
___________е2
707' Гт1п ~ 4ле0ту2 cos2 а
2 — У2 I
708.	Д1 = ±—t----------
2 v
R 2 (1 -h ct/i) — R1
709.	f = —	« 48,3 °C.
aRi
710.	иАВ =	ё = 1,6 В.
АВ I — а2/а!
711.	а =	=-7,5 • 10~^ К"1.
Ri + Ri
712.	т2 -	= 1 кг-
”3- % °270"'
7,4.	/тах = -Д^ =2 А.
715.	I ~ 7  10"6 м.
333
§15. Работа и мощность электрического тока
716.	/тк « \ 0 / gli
717.	У» RM	Рм^м^м	. ос Рс~	~ ’ и2
718.	&U-Pr-i5‘
719.	S = (R±l\u = \,02 В; п = ММ • 100 о/о = 980/0; \ К )	\к -Г Г1
'к.з	=	и « 10.2 А. \ Rr )
720.	' = Т/11^Р1ТТ = °'2 Ом=	гт\7~Т)^ 12 в‘
721.	1) Параллельно; 2) 70 °C.
722.	Р1 __ ^$1 __ о . р2 /2S2	2
723.	Ртах =(1-^)Р = 5.64 «Вт.
724.	Г = /ЯЛ = 6 Ом.
725.	^тах = /2^-3>1 А-
726.	6=1-	и 0,4. Рх
727.	U2	U2 Qbd—®> Qab Que Qco Qad	10 Дж, Qac — sjn a —
= 23,1 Дж.
728.	= (К?— 1 ) 3, = 3 В или &, = И Si = 12 В.
2 \/п+ 1/ 1	\И«- У
C(S-Sa) р
729.
= 10 ч.
730. Ра = Р [ 1 + a (t° - $| = 636 Вт.
м2=1’4 см2*
732.	1,33 коп.
733.	& = Рг~г\ =4 в-, г =	- о,5 Ом.
734.	56 мин.
735. /?! = 300 Ом, R2 = 200 Ом; Р, = 48 Вт, Р2 = 72 Вт. _ mv2! _ . . mv2! .
™-Р=~2Г’ Р(<Р)= — -,<р-
737 Р»	lR1R2 + ' № + P*)i2
’ Р2	+
738.	г =
739.	9 = —« 4 • IO’6 м3/с. срДТ
743. Qs= J1 Qi = 2,4 • 104 Дж;
= 1,6.
740.	S = 31R = 300 В.
741.	Рп = 47,5 Вт, Рб = 60 Вт.
742.	Ра = 27 Вт.
Q3= ^y,-Qi = 1.5- Ю< Дж.
Al Н“
339
744. 1 = A = 3 A. A
745. S =--------------= 22- IO'4 m2
fc(l— k)U* '	•
p 746. /, = ----------
1	k)
m = 2p0/S = 33 256 кг. 747. P= 1U- UR = 36 кВт.
748. Q = 1 C (<S2 - L/„) = Ю-s дж;
A = C& (S—Uo) = 1,2  IO’5 Дж.
1R =0,9.
= 1,05 А; 5=-----------------------г = 3,6 • 10"1 м’;
k (1 - k) Uo
117 = С<70 (& - Uo) = 0,8 • IO-» Дж;
749' 11	‘ S- //?
750. U = 33 В.
§ 16. Магнитное поле. Электромагнитная индукция
751. R = -4 1/ — « 2  IO’2 м.
В V е
1Ю. F = =^	3,2 • 10~12 Н.
А
753.	Ядро атома 2Не.
754.	Так, чтобы вектор магнитной индукции был перпендикулярен к плоскости витка.
755.	Силовые линии магнитного поля протонов представляют собой окружности с центром на оси пучка, направленные по часовой стрелке, как в случае магнитного поля ли-нейного тока.
756.	Силовые линии напряженности вихревого электрического поля могут замыкаться сами на себя. В частности, силовые линии вихревого электрического поля, возникающего при изменении магнитного поля, представляют собой замкнутые линии, которые охватывают лилии переменного магнитного поля. 757. Нет.
Г.	<?Я]	2я
758.	а = arccos 1---= —т— 
[	DJ J	О
759.	Катушка, не проявляющая магнитного действия, имеет, по-видимому бпфиллярную намотку, выполненную сложенным вдвое проводом.
760.	L = -2— = 2,5 • 10-‘ Тл.
А/
761.	Уменьшится в 2 раза.
762.	Заряженный шарик будет вращаться поокружности внутри соленоида.
763.	U == Blv = 3 мВ.
764.	Рассмотрим некоторый электрический контур, замкнутый на внешнюю нагрузку. Допустим, что в законе электромагнитной индукции Фарадея вместо знака «—» стоит знак «-(-», т. е. правило Ленца имеет противоположный смысл. Тогда в результате, например, случайного увеличения силы тока в контуре возникает э. д. с. самоиндукции, которая в таком случае приводит к еще большему росту силы тока и т. д. Таким образом, практически из ничего получается источник неограниченной электроэнергии, выделяющейся на внешней нагрузке. Невозможность создания вечного двигателя приводит к выводу, что в законе Фарадея должен быть именно знак «—» в соответствии с правилом Ленца.
765.	а =	КВ2 + п2В2 ~ 2,5  1012 м/с2.
766.	В= \f — Ея>3,2- 10*2 Тл, причем В ± Е.
340
... . 2mv . (qPf\ 767. I = —% sin ~ •
qB \2m)
M
768. В =	= 1,6 Тл.
771. B = ^- = IO’2 Тл. 4ла
769. I =	= 0,2 A.
770. /,=	= 0,314 A.
*	4
774.
b = __2— = o,i6 2л/?/ cos a
772.
773.
Тл.
Ll = Ё2 = 2.
rt B, v --	= 0,2 м/с.
775.	F = nv/BR = 0,063 H.
776.	/ =	+	—^3 — ^4 +Д,
rl + r2 + r3 + ri
777.	1 = 1	= 0,5 A.
778.	В = U = 2 Тл, vfflax = ^ = 7,5 м/с.
779.	„Л^=2н/с.
m
780.	a = 1/ •„ ,2^T	® 1,63 • IO’2 m.
F В sin a cos 60
781.
782.
„/ H Г
v = mgR \1^bJ
B2Ir2e p =	= 2 Вт.
m
„ ma sin a . 2nmvcosa
783.	R =----5 ; h =-------5---
eB	eB
7M.
788.	4.
787.
Вш/?2(/?1 + /?2)
2	+ R2R3 + Л1^?з]
788. В
2rtmv cos a ------t---n.
eL
U2t?
789’ 7cp 2L(®-t/)(T! + T2) »	В-
4p
791 J = «>В (r3 —£i) 5 = g, 18 A ftp
8,9 мА.
792.	&l =	= 60 c = 1 мин
a2r2
793.	q = aC (Rx + L) = 2  IO"11 Кл.
7e. iztnax _ ^£i£_
794.	U2 ~Cl+Ci-
341
L2 g
4“ ^2 Г
795. /“
V2 796. (/max = V-^U-
797. /max= |/^o-
M'O . J ea *-'1	®
Li + Lt' -Lr + L^r
799. 7 = ^;.
M "T *-2
800. B = i(b+b). rtjS
801. Д/ =	T.
\ О /
798. g0 = /max.
График зависимости силы тока от времени приведен на рис. 376.
803. /0 = 1/	806. q = Ckr-Q = 3,14- 10"1 ’ Кл.
г oL	<3
804. Q = ^°.	807.
IV. КОЛЕБАНИЯ И ВОЛНЫ
$ 17. Механически* колебания и волны
808.	ф0= j л; <pi = п; <р2 = ± л; уп = 0,02 sin j « 1,4 см; ух
X sin Д л =а 1,93 см; (/2 = 0,02 sin л = 0,02 м. 12	2
809.	х — sin ^л/ 4- j .
810.	х = 0,1 sin 0,5 л/.
811.	* = 0,6 sin 0,5 nt; v = 0,25 Гц.
342
0,02 X
812.	х = 0,05 sin (” I + f *i 0,0352 m; x2 = 0.
813.	x = 0,5 sin (10 nt + arcsin 0,6).
814.	По условию, A = A sin t, откуда ( = 2 c.
W.I.JC
816.	A = 0,08 m; 7 = 2 c; <p0 = 0,2 л = 36°.
817.	Д<р = <p2 — <pj = л.
5
818.	x = A sin -г- л « 0,04 м.
о
820, v = (оД ] sin ср | « 0,11 м/с; a = to21 x | = 0,2 м/с2; F = inaA I x | « 0,02 H; ^max =	0’13 M/c> amax =	~	М/С2, 7lnax = nl0max “ ^,04 H.
822.	fitiax = mo}2A 2,4 • i0-« H.
823.	=	Дж; £n =	« 3,6 Дж; £ = £K + £n =
= 1 kA2 = 6,1 Дж; T = 2л 1/^ « 0,22 c.
2	V k
FA2
824.	x =	= 1,5 . 10"2 m.
825. F « 0,747 H; E= 125 Дж.
826. T2 = Tj « 0,55 c.
n£ AZ
827. Z. =-J------, = 9 cm; Z2 =
nf — «2
828. Ta = Vl-f- 3
nf M "o o' "i-«2
= $ CM.
830. ft = R3 1
5357 km.
831. At = 86400	« 5400 c = 1,5 ч.
A
832.	T = 2л 1/	= 0,73 с.
Г tr^g
833.	Время движения грузиков t± = и
ZA -ZlEjk 1
h 4	________
834.	T = 2n 1/ R  .
1 Vai + gi
835.	7 = 2л ]/-— == 2,3 c.
r g — a
t. e.
T =
343
838.	Т =	+
837. I -	« 6.25 м.
4ftJ
язо r _ V (m£)2 + ^mghk
edtt- Xmax -------------1---------
839.	(l + f) = l°-
eT2	(Tl	\
840.	/ = ^-5- « 0,25 m; F = mg -4 _ ] = 3 . |0-« H.
4n4	\Т,	/
7\ = 2л j/"g lq(J/rm °’74 с’ если вектоР напряженности поля направ-вниз; Т3 = 2л 1/ ----1-гг—
3 У g — qU/rm направлен вверх.
/ * / ~2F~
842. и = — j/ —— ~ 0,48 м/с.
2л У mAh
841.
лен
®=* 2,34 с, если вектор напряженности поля
843.	t, = 1 1/^ .
844.	1 = ?^. “о
845.	7' = 2лЪ'^—• Знак плюс берем в случае, У mg zt qE 2л г I ml	н J ’
—>
если вектор Е направлен в ту же сторону, что и сила тяжести, а знак минус— когда эти векторы направлены в противоположные стороны, причем в послед-
нем случае задача имеет смысл, если Е <	•
<4
846.	v= ‘
Т 2л г mA
847.	v= ' Т/Й.
2л 2л У А
,, I + Ах	I — Ах	, . 1 Г tig.
848.	/V, =	— mg; N2 = 2/  mg; х = A sin у t.
21
849.	Т = -- = 2 • IO"’ с. v
lv
850.	А<р = 2л — = 3,14 рад. . 2n/v
851.	— <ра =	—— = zfcn.
852.	х, = 0; о=1,57 • 104 м/с; а = 0.
853.	s = ^- = 45 м. / г \	|
854.	у = 0,04 sin 600 л 11-I ; Т = с; (/, = 0,04 м.
J	i v 300 J'
244
9 18. Электромагнитные волны
855.	X = - = 1,6 м. v
/1 \2
856.	d2 = dj J = 7,5 мм.
857.	С = -г-^т = 1.4 • Ю-e Ф.
858.	~2 = (М « 1,54. С, \k1J
859. ДХ = (2,382 ± 0,024) • 108 м.
860.
861. Две катушки, индуктивности
Л2
которых L, =	« 8,1 • 10-" Гн и Л2=
тjL С j
4п2с2С2
== 2,9
Ю-з Гн.
862. Увеличивать собственный период колебаний контура (X = сТ = 2лс ЕС), т. е. увеличивать емкость конденсатора колебательного контура.
863. Увеличить в --j = 2,25 раз. 864. X = 2лс -у- ~ 189 м.
865. I = U jA? « IO’2 А.
868. f/max = /тах « 316 В.
867. -1 = V~n v2
2; энергии
868- /тах
max
869.	См. рис. 377.
870.	= ‘V = 5  IO"2.
1 с
871.	Q = ^, = 5 • Ю-з Дж.
872.
Ггп. с Т 2п ’ 2лгп ’
контуров одинаковы.
873. А/=~ 7,8 см, где t = с — время, за которое луч прочер-I С
чивает одну строку.
V. ОПТИКА
§ 19. Фотометрия
874. Е =	= 50 лк.
875. Освещенность поверхности параллельными лучами не зависит от расстояния ее от источника света. Она зависит только от интенсивности светового потока и угла падения лучей на поверхность. Е = 100 лк.
876. k	0,64.	878. Еи = 1,6  10s лк.
877. W = EStq = 1600 кВт • ч. 879. W = 4,3 • Ю8 кДж.
345
880.	«1 я	а 1,42 Вт/кд: и2 ==	г 0,18 Вт/кд; Фх = 352 лм; Ф2 = 2813 лм.
881.	/ =	Eh'1 = 400 кд.	884. h = 0,8 м.
882.	d =	0,8 м.	/ п \ 2 885. /а = Ц	= 180 кд. \'Х 1/
883.	Т) «	17,6 лм/Вт.	
886.	/?! =	= 1,24 м и R2 = 1	— /?! = 0,76 м.
887.	</ =	0,8 м.	892. 64 %.
888.	Е =		—« 356 лк.	893. £=s43,5 лк. Не соответствует,	
		(й2+ О2/4)0/х	
889.		8,7 м.	894. /2 = 13 ламп.
890.	Е^	21,7 лк.	895. Б, — 10,2 лк; Е2 = 10,7 лк.
891.	^гпах ~ $2,4 лк’ ^min		= 66,5 лк. 896. Е « 30,3 лк.
897.	а =	30°, h = 0,5 м.	
898.	Ei --=	=	= 6,25 ЛК;	£„ =	« 8,86 лк.
		(2/?)1	2 V 2R*
899. Е « 51 лк.
900. /2	46 с.
iR2
902. I =	= 80 кд.
903. I « 140 мкА, Ф = 4л/ = 3014,4 лм.
W-F"₽®’=" «•
905.	а) Е =-----— . б) В центре стола г = 0. Тогда Е = Д = 50 лк-
(й2 + г2)3/2	h-
На краю стола г — R. Тогда Е =-——« 36 лк.
(^ + /?2)3/2
906.	Ф = 95 лм.
$ 20. Прямолинейное распространение света. Отражение света
907.	I = | d = 18,6 м.
908.	На одной прямой. Плоскости предмета и экрана должны быть перпендикулярны к этой прямой.
909.	/ «з 1,4 • 10е км.
Рис. 378
910.	Через отверстие пройдет узкий пучок света, который осветит заднюю стенку камеры (рис. 378). Светлый кружочек диаметром d2 и будет изображением точки предмета, а совокупность изображений всех точек предмета образует его изображение. На изображении можно различать такие детали, изображения точек которых в виде светлых кружочков не будут перекрываться. Диаметр
.	Z-	U Л (СО-\-ОЕ\	„	-
изображения	точки С	а2	= а1 I ——— I	= 3	мм. Следовательно,	различать
\ С С* /
346
детали предмета размером меньше 3 мм на его изображении при заданном расположении невозможно.
911.	i = 1 а = 60°.
912.	На: а) прямолинейном распространении света в однородной среде; б) независимости действия световых пучков и в) отражении света. Угол падения лучей на плоское зеркало i = = 90°/2 = 45°.
913.	v = 4nRv = 157 м/с.
914.	Не будет.
915.	ft=//S.ing + ”) sin (р — а)
917.	1/6.
918.	Можно построить три изображения.
919.	Начать с построения изображения точки N с помощью одной из зеркальных стенок.
921.	Человек не увидит своего изображения.
922.	Равна половине роста человека. Расстояние нижнего края зеркала от пола должно быть равно половине рассто ния от глаза человека до пола.
923.	а = 45°.
924.	Изображения источника света наблюдатель сможет увидеть (рис. 379), если его глаз расположен внутри полосы, ограниченной лучами ОА и ОС.
полосы, ограниченной лучами ОА и ОС.
§ 21.	Преломление света
925.	t~54°4'.	928. t ® 24°31'.
926.	« 23°8'; а2 « 11°41'.	929. с1 « 1,67 • 108 м/с.
927.	п = 1,8.	930. с « 2  108 м/с.
931.	i = 2г = 78°. Второе условие выполняться не будет, поскольку максимальный угол падения света 90° (i = 2г = 108°).	_________
____ , d l/”n2 — sin2 i ,
932.	h =	—-------- «= 13,4 см.
у л2 — sin2 г—sin i
Воды необходимо налить V = а2Л =
935.	Водолазу будет казаться, что hx = nh = 4 м.
936.	а) Да. б) Нет.
. h sin (i — г) п ..
937.	d =-------------- ъ 2,44 см.
cos г
= 12,05 л.
933.	Водолаз видит Солнце в направлении продолжения преломленного солнечного луча ОА (рис. 380). Видимая высота Солнца над горизонтом ZS'O/Vl =<₽i, а действительная высота ZS0M = <р2. Из рисунка 380 видно, что <р2 = 90°—i, из А ОВА следует, что г — 90° — <pi = — 45°. Тогда arcsin i = arcsin (п sin г) = = 70°71' и ф2= 19°53'.
934-	Л«=Л/лй^+,==2м-
светящийся предмет находится на высоте
938.	d — 4 см.
939.	п = 1,57.
940.	dj = 6,8 см.
347
941.	Сосуд с жидкостями можно рассматривать как систему трех плоскопараллельных пластинок: масло—вода—стекло (рис. 381). r8 = arcsin^^-^j =33°28‘. 942. <р==36°.
948. Отраженный от посеребренной грани луч АВ пойдет в обратном направлении только тогда, когда угол его падения на грань будет равен нулю /	оооч т	sin а 2 У2 л г5 , ..
(рис. 382). Тогда п = -—- = —= у 2 ~= 1.41.
sin гр 2
Рис. 381	Рис. 382
945.	Лучи выходят из призмы параллельно прежнему направлению. Расстояния между лучами, вошедшими в призму и вышедшими из призмы, одинаковые. Пути первого и второго лучей внутри призмы одинаковые.
946.	п — 1,8.	949. др « 81°4'.
947.	а « 57°	950. Я1 « 10°53'.
948.	а « 16°.
§ 22.	Линзы
953.	п = 1 +	- 1,67.
*4 “Г К 2 р
954.	F == —Ц эд 33,3 см.
п — 1
\ М /	1
955.	£>2 = ----—--—- = — 0,525 дптр. Фокусное расстояние /2 — =- =—1,9 м.
п, — 1	ь'а
Знак минус указывает, что линза в данной жидкости является рассеивающей.
956.	/, « 39 см.
957.	пж « 1,62.
958.	D •!= 3,33 дптр.
959.
п	^r/?2(fe+l)a
' МЯг+ЯгИ'1- 1)
0,9
960.	16 см.
961. (х « 14,7 см и
962. 6 = 0,1 м; k ~ 1.
12 =г 85,4 см.
963. Поскольку линза образует увеличенное изображена лампочки, то лампочка находится от линзы на расстоянии а, лежащем в интервале / a .
Фокусное расстояние линзы: f —
ab а + b
= 11,25 см.
964. D « 4,2 дптр.
965. Переместится на 5 см ближе к экрану.
•34'8
066. 6Х = —30 см; Ьг — —18 см.
972. k = А = 40 раз.
967.	6 = 4 см.
968.	а) Ь3 = —20 см; б) Ьг = —6,7 см.
969.	b — —7,5 см.
970.	f = /?/2л « 16 см.
971.	I = 100 см.
973.	b = 4 м.
974.	а = 24 м.
975.	а = 72 см.
976.	а) 81 см; б) 16,2 см.
977.	а) 22,22 м; б) 22,22 см.
978. а, = „ а , , = 27 см.
D2a 4- 1
079. a) k = 5 раз; б) 30 см; в) Dt = 4дптр.
980.	fe = 4 раза.	983. k = 12 раз.
981.	/ок = 2 см.	984. fe2 = 125.
982.	k = 400 раз.
985.	В направлении от объектива. На 1 см.
986.	k = 	= 60 раз.
\'2~ II) /ок
§ 23.	Световые волны
987.	v = 2,83 • 1013 с"1; X, = 7,97 мкм; « 2,83 • 1013 с"1.
988.	а) лх « 943,4; б) п2 » 1254,7; в) п3 яз 1509,4.
989.	1}	0,83 м.
990.	В вакууме.
991.	Хх яз 0,625 мкм; Х2 « 0,500 мкм; Х3 яз 0,417 мкм.
1
992. а) Д = 2dn2 + (/ — 2d) п3 — Znx=3,123 мм; б) X = -5 = 0,45 мкм; в) Д<р =
993. Д = 500 мкм.	995. п = 4.
994. d = 333 мкм.	996. d яз 8,7 мм.
997. а) Максимум освещенности; б) п = 1,5.
998. d яз 3 мм. 999. d яз 0,4 мм. 1000. d «з 0,13 мкм.
1001.	а) Красный (X = 0,640 мкм); б) зеленый (X = 0,538 мкм).
1002.	d « 0,28 мкм.	1010. Будут; X = 0,48 мкм.
1003.	<7 = 0,11 мкм.	1011. Д<р = 24°48'.
1004.	a =20".	1012.	d = 5 мкм.
1005.	М = 22,4 см"1.	1013.	Д<р = 6'.
1006.	<р, = 1°40'; <р2 яз 3°24'.	1014. <р = 49°30'; k = 3.
1007.	п = 200 мм'1.	1015.	а) femax = 2; б) /?тах = 3.
1008.	X — 0,546 мкм.	1016.	X = 0,5 мкм.
1009.	X = 0,478 мкм.
§ 24. Основы теории относительности
1017. з = 2,6. 10е м/с.
1018. v = 0,995 с = 2,985 • 108 м/с. 1019. и.' = 0,994 с.
1020. р = 5,6 • 10~22 кг • м/с.; 1ГК = 1,06 • 10"13 Дж = 0,66 МэВ.
1021. v = 0,866 с = 2,6  10* м/с, 1022. р = 2,93 • 10”21 кг -м/с.
349
1023.	— = 1,156; 11,3 %.
1024.	Ди =0,77 с = 2,31-108 м/с; Др = 4,5-10’22 кг-м/с; ДП?К = 8,24-10'44 Дж.
1025.	Дт = 3,7 • 10й тонн.
§ 25.	Световые кванты
1026.	Е = 1,96  10'16 Дж, т = 2,2  10-34 кг; р = 6,6 . 10”24 кг • м/с.
1031. Будет.
1032. п = 61 %.
1027. л = 5 • 10’.
1028. 4  10».
1029.	А « 7,58 • 10-19 Дж « 4,74 эВ. 1033. т) = 36 %.
1030.	Ао = 0,52 • 10-8 мкм. Будет,
1034.	а) А = 4,5 эВ; б) й?к = 3,9 . 10"48 Дж; в) v ~ 9,2 • 106 м/с.
1035.	а) Й7К=О,О2 эВ; б) v = 2,7 • 10» м/с.
1036.	с = 1*7- 10е м/с.	1039. v = 6,5- 10» м/с.
1037.	и = 7,7- 10» м/с.	1040. А = 2 эВ.
1038.	v = 1,45 • 1046 Гц.
1041.	А = 5,2 • 10~48 Дж; v0 = 3,83 - 1044 Гц.
1042.	v= 1,43 • 104» Гц. 1043. U = 10,8 В.
he
1044.	a) E = eU = 6.104 эВ = 9,6. Ю"46 Дж; б) А, = -£ = 2,1 • 10-41 м; в) р =
= Y = 3,2 . 10~23 кг • м/с.
Л
VI. АТОМНАЯ И ЯДЕРНАЯ ФИЗИКА
$ 26. Физика атома и атомного ядра
1045.	Е « 3  IO"48 Дж.
1046.	На третий. Три линии.
1047.	ДЕ = he (»— +	~ 13 эВ.
\Л1	Л2/
О hr
1048.	?i = — « 1,24 • 10-7 м, mvi
1049.	о. 2 = тт ’-'о \ 1 ± 1/ 1-“V") -	« 5 • 104 м/с; о, «а 2  104 м/с
’	2	\	' теок/
931 [Zm + (Z - А) т -т]
1050.	Е  --------£--Ц=----------2--- =в 8,1 МэВ « 12,96 • 10“43 Дж.
1051.	J3AI+*He-> ijP+Jn; Е = с2 (тд| + тНе — тр — тп) а= 7,5 • 10-43 Дж.
1052.	ДЕ=931(т,, —т—mlt ) » 18,6 МэВ. ,С п ,d
1053.	Считая плотность ядерного вещества постоянной, найдем, что интересующее нас ядро — это ядро fBe. Его энергия связи Есв равна удельной энергии связи £ , которая примерно одинакова для всех ядер, умноженной на число нуклонов N-. Есв = EyaN = 8 МэВ  8 = 64 МэВ.
1054.	^Ве + jHe^С + Jn.
350
1055.	Кроме нейтрона образуется еще ядро атома гелия гНе. Полная энер-
гия Е =	8,5 • 1011 Дж, где Л4 = 2 * 10~з кг/моль — молярная масса
и А Л4
дейтерия. а 236г ; > 1	130п	< 05тг 
1056.	e2U -f- on —► 6eBa + 30Kr -f- 20п.
1057.	Нейтрон, протон и альфа-частица.
1058.	“N + Jn -> “С + 1Н; \*С ->+ “N.
1060.	1) НА' + J" i^Na + ^Не; 2)	+ 2Не -> ”О + Jn;
3) “Li + JH ->-^Не + 42Не; 4) “Са + JH -> JJK + 2Не.
Pi
1061.	т = М. —„	«=« 31 г.
Т)£<Ла
Pi
1062.	т = М хъкг-г 9000 кг*
Pi
1063.	т == М -- "т. « 166 г,
ПРИЛОЖЕНИЯ
1. Некоторые фундаментальные физические постоянные
Постоянная	Обозначения	Числовое значение
Гравитационная постоянная Ускорение свободного падения (нормальное) Скорость света в вакууме Средний радиус Земли Магнитная постоянная Электрическая постоянная Масса покоя электрона Масса покоя протона Заряд электрона Постоянная Планка Постоянная Авогадро Постоянная Больцмана Постоянная Фарадея Атомная единица массы Молярная газовая постоянная Нормальный (молярный) объем идеального газа при нормальных условиях (То =273,15 К, р„ = 101 325 Па)	G g с R Ро во те тр е h Л'а k F а. е. м. R Vo	6,67 • IO-11 Н • м2/кг2 9,80665 м/с2 299792,46 км/с«3-10’ м/с 6,37  10» м 12,6 • 10-’ Гн/м 8,85 - I0-12 Ф/м 9,1 • 10~31 кг 1836 те = 1,67 • 10 37 кг 1,6 • 10-10 Кл 6,63 • Ю-з* дж . с 6,02 • 1023 моль-1 1,38 • 10"23 Дж/К 96,5 • 103 Кл/моль 1,66 • 10-27 кг 8,31 Дж/(моль • К) 22 ,4 • 10-3 м3/моль
2. Значения некоторых физических величин
	Молярная масса, кг/моль		
Азот	28 • Ю-з	Гелий	4  Ю-з
Воздух	29 • IO-3	Кислород	32 • IO-3
Водород	2 • 10-3	Неон	20 • 10-з
	Плотность	, кг/м3	
Азот	1,25	Лед	0,9  103
Алюминий	2,7 • 103	Медь	8,9 • Ю3
Бензин	7 • Ю2	Олово	7,3 • 103
Бетон	2,2  Ю3	Парафин	9  Ю2
Водород	0,09	Пробка	2,4 • 102
Вода	1  103	Ртуть	13,6  Ю3
Воздух	1,29	Свинец	1,14 • Ю4
Глицерин	1,26  I03	Серебро	1,05 • 104
Железо	7,8 • 103	Цинк	7,1 • Ю3
Керосин	8 • 102		
352
	Удельная теплоемкость, Дж/(кг • К)	
Вода	4,2 • Юз	Медь	3,8 • 10«
Железо	4,6 • 102	Свинец	1,2 • 10»
Латунь	3,8- 102	Сталь	4,6 • 102
Лед	2,1 • 103	
	Удельная теплота плавления,	Дж/кг
Лед	0,32 • 10е	Свинец	2,5 • 10*
	Удельная теплота испарения.	Дж/кг
Вода	2,26 • 10е	
	Удельная теплота сгорания,	Дж/кг
Бензин	4,6 • 10’	
Керосин	4,3 • 10’	
Уголь	2,87 • 10’	
	Температура плавления.	К
Свинец	600 Модуль Юнга, Па	
Железобетон	5 • 1010	Медь		8  Ю1»
Железо	1,96 • 10“	Сталь	2,1  10“
Латунь	1,1  1010	
	Коэффициент линейного расширения, К-1	
Алюминий	2,4 • 10“4	Сталь	1,2 • IO-»
Железо	1.2 • 10-6	Стекло	1,2- IO"6
Латунь	1,7  IO"6	
	Коэффициент объемного расширения. К-1	
Вода	1,5 • 10-1	Керосин	9,6  10-*
Удельное сопротивление р и температурный коэффициент
сопротивления а
Металл, сплав	1, °C	р, 10-* См • м	а, 10-а К”*
Алюминий	20	2,69	4,2
Висмут	20	116	4,2
Вольфрам	20	5,5	4,6
	3500	131,4	——
Германий	—	46  10В	—
Г рафит	0	800—1400	—0,5
Железо	20	9,71	6,51
Золото	0	2,065	4,5
Кобальт	20	6,24	6,04
Константан	20	45—50	0,01
Медь	20	1,673	4,30
Молибден	0	5,03	4,7
Никеле	20	6,844	6,00
Никелин	20	42	0,02
Олово	20	12,8	4,2
Платина	0	9,81	3,927
Ртуть	20	95,8	0,89
Свинец	20	20,6	з,зб
Серебро	0	1,468	4,033
Сталь нержавеющая	20	7, Ь	—
Фехраль	20	110—130	0,1
Хром	20	18,9	5,88
Цинк	20	5,92	4,2
353
Показатели преломления света п при 20 °C относительно воздуха (для длины волны Л = 589,3 нм)
Вещество	п	Вещество	п
Твердые тела		Жидкости	
Алмаз	2,417	Ацетон	1,36
Изумруд	1,581	Бензол	1,501
Каменная соль	1,544	Вода	1,333
Кварц		Глицерин	1,470
кристаллический	1,544	Канадский бальзам	1,530
плавленый	1,458	Сероводород	1,885
Корунд (сапфир, рубин)	1,769	Сероуглерод	1,63
Лед	1,31	Скипидар	1,470
Плексиглас	1,50	Спирт	
Слюда	1,56-1,60	метиловый	1,33
Стекло		ЭТИЛОВЫЙ	1,362
флинт	1,60—1,80	Толуол	1,497
крои	1,56—1,60	Углерод четыреххлорис-	1,460
Шпат исландский	1,659	ТЫЙ	
Янтарь	1,540		
Предельные углы <р полного отражения
Вещество	ф. 0	Вещество	ч>. 0
Вода Глицерин Стекло (легкий крон) Спирт	49 43 40 47	Сероуглерод Стекло (тяжелый флинт) Алмаз	38 34 24
Работа выхода А электронов
Вещество	А, эВ	Вещество	А, зВ
Алюминий	4,25	Никель	4,50
Бериллии	3,92	Олово	4,38
Висмут	4,4	Плати на	5,32
Вольфрам	4,54	Ртуть	4,52
Германий	4,76	Свинец	4,0
Железо	4,31	Серебро	4,3
Золото	4,30	Сурьма	4,08
Кобальт	4,41	Углерод	4,7
Кремний	4,8	Уран	3,3
Литий	2,38	Хром	4,58
Медь	4,40	Цезий	2,7
Молибден	4,3	Цинк	3,9
354
Атомные массы М элементов
Атомный номер	Элемент	Mt a. e. m.	Атомный номер	Элемент	Mt a. e. m«
1	н	1,00797	48	Cd	112,40
2	Не	4,0026	49	In	114,82
3	Li	6,639	50	Sn	118,69
4	Be	9,0122	51	Sb	121,75
5	В	10,811	52	Те	127,60
6	С	12,01115	53	I	126,9044
7	N	14,0067	54	Xe	131,30
8	О	15,9994	55	Cs	132,905
9	F	18,9984	56	Ba	137,34
10	Ne	20,183	57	La	138,91
11	Na	22,9818	58	Ce	140,12
12	Mg	24,312	59	Pr	140,907
13	Al	26,9815	60	Nd	144,24
14	Si	28,086	61	Pm	147
15	P	30,9738	62	Sm	150,35
16	s	32,064	63	Eu	151,96
17	Cl	35,453	64	Gd	157,25
18	Ar	39,948	65	Tb	158,924
19	К	39,102	66	Dy	162,50
20	Ca	40,08	67	Ho	164,930
21	Sc	44,956	68	Er	167,26
22	Ti	47,90	69	Tm	168,934
23	V	50,942	70	Yb	173,04
24	Cr	51,996	71	Lu	174,97
25	Mn	54,9380	72	Hf	178,49
26	Fe	55,847	73	Ta	180,948
27	Co	58,9332	74	VV	183,85
28	Ni	58,71	75	Re	186,2
29	Cu	63,54	76	Os	190,2
30	Zn	65,37	77	Ir	192,2
31	Ga	69,72	78	Pt	195,09
32	Ge	72,59	79	Au	196,967
33	As	74,9216	80	Hg	200,59
34	Se	78,96	81	Tl	204,37
35	Br	79,909	82	Pb	207,19
36	Kr	83,80	83	Bi	208,980
37	Rb	85,47	84	Po	210
38	Sr	87,62	85	At	211
39	Y	88,905	86	Rn	222
40	Zr	91,22	87	Fr	223
41	Nb	92,906	88	Ra	226,12
42	Mo	95,94	89	Ac	227
43	Tc	99	90	Th	232,038
44	Ru	101,07	91	Pa	231
45	Rh	102,905	92	U	238,03
46	Pd	106,4	93	Np	237
47	Ag	107,870	94	Pu	242
3. Основные формулы Механика
Равноускоренное прямолинейное движение —►-	—
”* и As “* До	1. Дх dx
v = lim — , а = lim — , vx = lim — = — , д<-0 At дг-O At д/-о At dt
355
VX — V ^aXSXt aX — Я— • 2SX
+"о/. fx = fox + fxC S = vot + 1 a/2;
&i-,0 Д/ at at1	i	£
1	°x
fox = °, * = 2 ax*2 = — .
Равномерное движение по окружности ш = -у- , v = со/? = 2nv/?, а = Второй закон Ньютона F = та.
Закон всемирного тяготения тгт2 /?2 Сила и импульс
Ft = mv — mv0. Механическая работа А = Fs cos а. Кинетическая энергия _ту* Ек 2 • Потенциальная энергия А = — ЛЕ- Е = mgh, Е = —.
П ’ п « ’ п 2 Уравнение Бернулли
Р + pgh + р - =const.
r-^r-
Третий закон Ньютона
X = -K
Закон Гука	Сила трения
Fy=— kx. FT = ptf.
Закон сохранения импульса
Mj 01 4- m2t>2 = mjUt + m2u2.
Мощность
N = t
Теорема о кинетической энергии Л = ^к2 — ^Kl.
Закон сохранения энергии в механических процессах
^к1 + ^ni =	+ Еп2'
теории
_ m __ m _ М m° ~ N ~ vN~a~ 1ГА
2 _
р = -=• пЕ, р = nkT, О
Молекулярная физика и термодинамика
Основы молекулярно-кинетической N ч т v = — , М = — ,
Давление идеального газа 1	~2	1	-2
Р = -j nmov2 = - рп2, Энергия молекул и температура
Т = е + 273, Е = | kT.
Уравнение состояния идеального газа pV = vRT, pV^^RT, R=kNA.
Закон Бойля—Мариотта	Закон Гей-Люссака
V Т pV = const (при Т = const).	—1 = —1 (при р = const).
'2	*2
Закон Шарля	Закон Дальтона
п ~ (при V = const).	Р ~ Р1 + Рг + ‘ ' + Рп = J] Pt-
Pi	1-1
356
Внутренняя энергия идеального газа
U—^kTN = |™A=|v/?T=|pV;
At/ = -| vtfAT = у A (pV).
Первый закон термодинамики
Д77 = А + Q, At/ = Q — Лк Q = At/ + Av
К. п. д. теплового двигателя
_ 4.1 А — 0 — О п = ~ п = 'Г* ~ 'Гд
’I ~	^2’ n Q) ' lmax Ti
Теплообмен
Q = cm&t°; Q,, = rm, Q,,„ — Xm.
Относительная влажность воздуха
<р= -  100%. Ро
Поверхностное натяжение F А I ~ AS'
Высота поднятия смачивающей жидкости в капилляре
. 2о cos 0 h —---------
Pgr
Электродинамика
Закон сохранения электрического заряда
<71 + <7г + ‘ • ’ + <7л = const.
Закон Кулона
f
4ле0ег2
Напряженность электрического поля точечного заряда
E^-L, Е=—В—. £ = Х+^2+•••+Х-с?! 4ле0ега
Электроемкость с = . и
Напряжение
а = --
<7
77 = Ed-,
Э. д. с.
Д
_ '‘стар
<7
Тепловое
Электроемкость конденсатора „ eoeS
Закон Ома
действие тока
Q=Z2W.
Мощность тепловых потерь Мт= VR.
£ =
а
d

Сила тока
/ = 1ПП^ = ^.
Д/ dt
Работа электрического тока
А = IUt.
Мощность электрического тока N = IU.
Законы электролиза т = k&q = kl&t-,
Е .	__М_
Мд ’ neNA
е =
Магнитная индукция
F п max
357
Сила Ампера В — IBl sin а. Магнитный поток Ф — BS cos а.
Индуктивность
Сила ЛоренЦй
F — qvB sin а.
Закон электромагнитной индукции
/ ДФ\
’,=А"Г «)•
Э. д. с. самоиндукции
ДГ / Ы dt
Колебания и волны
Механические гармонические колебания 2л 1 х = хт cos (ш/ + ф), ш = 2л v = у , v = у
да—да; »’-/ «- /да
Электромагнитные гармонические колебания q di d2q d2q 1
UC = —UL, ~c = ~ L Tt = ~ L dt2 ' dt2 = — LC q’ 9 9o cos(o/*
T=^’
Виток в однородном магнитном поле d®
е = — у , Ф — BS cos cot, е = BS a sin со/.
Индуктивность и емкость в цепи переменного тока
c°s (ш/+ у У Cm = ZmLw, Х;_= 7^=С<в, l/c = <7mcos(wZ-y), ит=-^’ XC = Uj^ = ^C-
Полная электромагнитная энергия контура LI2 q2
Е ~ 2 + 2С '
Средняя мощность переменного тока
Мср = 71/ cos <р.
Коэффициент трансформации
„ _ ni__1 ~ Ui
Л~пг~^г~ U2-
Активная мощность
Ма = /С/ cos <р = I2R.
К. п. д. трансформатора М2
ОГЛАВЛЕНИЕ
Предисловие............................................3
I.	Механика
§ 1.	Кинематика......................................
§ 2.	Динамика.........................................27
§ 3.	Закон всемирного	тяготения......................50
§ 4.	Статика..........................................5°
§ 5.	Закон сохранения	импульса.......................71
§ 6.	Работа и энергия.	Закон сохранения энергии .... 82
§ 7.	Механика жидкостей и газов.......................99
II.	Молекулярная физика. Тепловые явления
§ 8.	Основы молекулярно кинетической теории. Законы идеального газа.......................................Ю7
§ 9.	Внутренняя энергия и механическая работа .... 126
§ 10.	Упругие и тепловые свойства твердых тел и жидкостей ...............................................140
§ 11.	Свойства паров.................................144
Ill.	Электродинамика
§ 12.	Электрические заряды и электрическое поле . . . 148
§ 13.	Постоянный электрический ток...................169
§ 14.	Электрический ток в вакууме, металлах, электролитах и газах . . . ..................................183
§ 15.	Работа и мощность электрического тока..........187
§ 16.	Магнитное поле. Электромагнитная индукция , . . 196
IV.	Колебания и волны
§ 17. Механические колебания и волны.................220
§ 18. Электромагнитные волны.........................233
V. Оптика
§ 19.	Фотометрия....................................238
§ 20.	Прямолинейное распространение света. Отражение
света..........................................245
§ 21.	Преломление света.............................252
§ 22.	Линзы.........................................262
§ 23.	Световые волны................................274
§ 24.	Основы теории относительности.................288
§ 25.	Световые кванты ..............................291
VI. Атомная и ядерная физика
§ 26.	Физика атома и атомного	ядра..................297
Ответы и решения...............................304
Приложения.....................................352